All Related Questions of Corporate Finance

Q: Your company is deciding whether to invest in a new machine.

Your company is deciding whether to invest in a new machine. The new machine will increase cash flow by $530,000 per year. You believe the technology used in the machine has a 10-year life; in other w...

See Answer

Q: Suppose a stock had an initial price of $64 per share

Suppose a stock had an initial price of $64 per share, paid a dividend of $1.20 per share during the year, and had an ending share price of $73. Compute the percentage total return.

See Answer

Q: With regard to bid and ask prices on a Treasury bond,

With regard to bid and ask prices on a Treasury bond, is it possible for the bid price to be higher? Why or why not?

See Answer

Q: Even though most corporate bonds in the United States make coupon payments

Even though most corporate bonds in the United States make coupon payments semiannually, bonds issued elsewhere often have annual coupon payments. Suppose a German company issues a bond with a par val...

See Answer

Q: Based on the following information, calculate the expected return and standard

Based on the following information, calculate the expected return and standard deviation for the two stocks:

See Answer

Q: Suppose stock returns can be explained by a two-factor model

Suppose stock returns can be explained by a two-factor model. The firm-specific risks for all stocks are independent. The following table shows the information for two diversified portfolios: If the...

See Answer

Q: Kiedis, Corp., has interest-bearing debt with a market

Kiedis, Corp., has interest-bearing debt with a market value of $65 million. The company also has 2 million shares that sell for $25 per share. What is the debt–equity ratio for this company based on...

See Answer

Q: In Problem 4, use MM Proposition I to find the price

In Problem 4, use MM Proposition I to find the price per share of equity under each of the two proposed plans. What is the value of the firm? Problem 4: Franklin Corporation is comparing two differen...

See Answer

Q: Edwards Construction currently has debt outstanding with a market value of $

Edwards Construction currently has debt outstanding with a market value of $75,000 and a cost of 9 percent. The company has EBIT of $6,750 that is expected to continue in perpetuity. Assume there are...

See Answer

Q: North Pole Fishing Equipment Corporation and South Pole Fishing Equipment Corporation would

North Pole Fishing Equipment Corporation and South Pole Fishing Equipment Corporation would have identical equity betas of 1.10 if both were all equity financed. The market value information for each...

See Answer

Q: The balance sheet for Levy Corp. is shown here in market

The balance sheet for Levy Corp. is shown here in market value terms. There are 14,000 shares of stock outstanding. The company has declared a dividend of $1.60 per share. The stock goes ex dividend...

See Answer

Q: Vital Silence, Inc., has a project with the following cash

Vital Silence, Inc., has a project with the following cash flows: The company evaluates all projects by applying the IRR rule. If the appropriate interest rate is 9 percent, should the company accep...

See Answer

Q: If a portfolio has a positive investment in every asset, can

If a portfolio has a positive investment in every asset, can the standard deviation on the portfolio be less than that on every asset in the portfolio? What about the portfolio beta?

See Answer

Q: We have seen that over long periods stock investments have tended to

We have seen that over long periods stock investments have tended to substantially outperform bond investments. However, it is not at all uncommon to observe investors with long horizons holding their...

See Answer

Q: What are the portfolio weights for a portfolio that has 165 shares

What are the portfolio weights for a portfolio that has 165 shares of Stock A that sell for $43 per share and 120 shares of Stock B that sell for $74 per share?

See Answer

Q: What are the differences between a k-factor model and the

What are the differences between a k-factor model and the market model?

See Answer

Q: A stock market analyst is able to identify mispriced stocks by comparing

A stock market analyst is able to identify mispriced stocks by comparing the average price for the last 10 days to the average price for the last 60 days. If this is true, what do you know about the m...

See Answer

Q: Some corporations, like one British company that offers its large shareholders

Some corporations, like one British company that offers its large shareholders free crematorium use, pay dividends in kind (i.e., offer their services to shareholders at below-market cost). Should mut...

See Answer

Q: Zipcar, the car sharing company, went public in April 2011

Zipcar, the car sharing company, went public in April 2011. Assisted by the investment bank Goldman Sachs, Zipcar sold 9.68 million shares at $18 each, thereby raising a total of $174.24 million. By t...

See Answer

Q: You notice that shares of stock in the Patel Corporation are going

You notice that shares of stock in the Patel Corporation are going for $50 per share. Call options with an exercise price of $35 per share are selling for $10. What’s wrong here? Describe how you can...

See Answer

Q: Your company owns a vacant lot in a suburban area. What

Your company owns a vacant lot in a suburban area. What is the advantage of waiting to develop the lot?

See Answer

Q: Look at Table 10.1 and Figure 10.7(

Look at Table 10.1 and Figure 10.7(Given Below) in the text. When were T-bill rates at their highest over the period from 1926 through 2014? Why do you think they were so high during this period? What...

See Answer

Q: Suppose a company has a preferred stock issue and a common stock

Suppose a company has a preferred stock issue and a common stock issue. Both have just paid a $2 dividend. Which do you think will have a higher price, a share of the preferred or a share of the commo...

See Answer

Q: In March 2014, BMW announced plans to spend $1 billion

In March 2014, BMW announced plans to spend $1 billion to expand production at its South Carolina plant. The plant produced the second-generation BMW X3 as well as the company’s X5 and X6 models. The...

See Answer

Q: The Dybvig Corporation’s common stock has a beta of 1.17

The Dybvig Corporation’s common stock has a beta of 1.17. If the risk-free rate is 3.8 percent and the expected return on the market is 11 percent, what is Dybvig’s cost of equity capital?

See Answer

Q: Scarlett Corp. uses no debt. The weighted average cost of

Scarlett Corp. uses no debt. The weighted average cost of capital is 8.4 percent. If the current market value of the equity is $43 million and there are no taxes, what is EBIT?

See Answer

Q: Suppose the real rate is 2.4 percent and the inflation

Suppose the real rate is 2.4 percent and the inflation rate is 3.7 percent. What rate would you expect to see on a Treasury bill?

See Answer

Q: Lee Ann, Inc., has declared a $7.50

Lee Ann, Inc., has declared a $7.50 per-share dividend. Suppose capital gains are not taxed, but dividends are taxed at 15 percent. New IRS regulations require that taxes be withheld when the dividend...

See Answer

Q: A stock has a beta of 1.15, the expected

A stock has a beta of 1.15, the expected return on the market is 10.6 percent, and the risk-free rate is 4.5 percent. What must the expected return on this stock be?

See Answer

Q: An investment offers a total return of 13 percent over the coming

An investment offers a total return of 13 percent over the coming year. Alan Wingspan thinks the total real return on this investment will be only 8 percent. What does Alan believe the inflation rate...

See Answer

Q: Say you own an asset that had a total return last year

Say you own an asset that had a total return last year of 11.6 percent. If the inflation rate last year was 5.3 percent, what was your real return?

See Answer

Q: Bruce & Co. expects its EBIT to be $145,

Bruce & Co. expects its EBIT to be $145,000 every year forever. The company can borrow at 8 percent. The company currently has no debt, and its cost of equity is 14 percent. If the tax rate is 35 perc...

See Answer

Q: A stock has an expected return of 13.4 percent,

A stock has an expected return of 13.4 percent, its beta is 1.20, and the risk-free rate is 4.4 percent. What must the expected return on the market be?

See Answer

Q: A stock has an expected return of 11.2 percent,

A stock has an expected return of 11.2 percent, a beta of 1.15, and the expected return on the market is 10.4 percent. What must the risk-free rate be?

See Answer

Q: A call option has an exercise price of $80 and matures

A call option has an exercise price of $80 and matures in six months. The current stock price is $84, and the risk-free rate is 5 percent per year, compounded continuously. What is the price of the ca...

See Answer

Q: What is the historical real return on long-term government bonds

What is the historical real return on long-term government bonds? On long-term corporate bonds?

See Answer

Q: Using information from the previous chapter about capital market history, determine

Using information from the previous chapter about capital market history, determine the return on a portfolio that is equally invested in large-company stocks and long-term government bonds. What is t...

See Answer

Q: Suppose the returns on large-company stocks are normally distributed.

Suppose the returns on large-company stocks are normally distributed. Based on the historical record, use the NORMDIST function in Excel ® to determine the probability that in any given year you will...

See Answer

Q: A researcher has determined that a two-factor model is appropriate

A researcher has determined that a two-factor model is appropriate to determine the return on a stock. The factors are the percentage change in GNP and an interest rate. GNP is expected to grow by 3.6...

See Answer

Q: You own a portfolio that is 20 percent invested in Stock X

You own a portfolio that is 20 percent invested in Stock X, 45 percent in Stock Y, and 35 percent in Stock Z. The expected returns on these three stocks are 11 percent, 17 percent, and 14 percent, res...

See Answer

Q: Shiller Corporation will pay a $2.75 per share dividend

Shiller Corporation will pay a $2.75 per share dividend next year. The company pledges to increase its dividend by 5 percent per year, indefinitely. If you require a return of 11 percent on your inves...

See Answer

Q: Mullineaux Corporation has a target capital structure of 70 percent common stock

Mullineaux Corporation has a target capital structure of 70 percent common stock and 30 percent debt. Its cost of equity is 11.5 percent, and the cost of debt is 5.9 percent. The relevant tax rate is...

See Answer

Q: What was the arithmetic average annual return on large-company stocks

What was the arithmetic average annual return on large-company stocks from 1926 through 2014? a. In nominal terms? b. In real terms?

See Answer

Q: Siblings, Inc., is expected to maintain a constant 5.

Siblings, Inc., is expected to maintain a constant 5.7 percent growth rate in its dividends, indefinitely. If the company has a dividend yield of 4.6 percent, what is the required return on the compan...

See Answer

Q: A stock is currently selling for $47 per share. A

A stock is currently selling for $47 per share. A call option with an exercise price of $50 sells for $3.80 and expires in three months. If the risk-free rate of interest is 2.6 percent per year, comp...

See Answer

Q: Miller Manufacturing has a target debt–equity ratio of .55

Miller Manufacturing has a target debt–equity ratio of .55. Its cost of equity is 12.5 percent, and its cost of debt is 7 percent. If the tax rate is 35 percent, what is the company’s WACC?

See Answer

Q: A put option that expires in six months with an exercise price

A put option that expires in six months with an exercise price of $75 sells for $4.89. The stock is currently priced at $72, and the risk-free rate is 3.6 percent per year, compounded continuously. Wh...

See Answer

Q: Gruber Corp. pays a constant $9 dividend on its stock

Gruber Corp. pays a constant $9 dividend on its stock. The company will maintain this dividend for the next 13 years and will then cease paying dividends forever. If the required return on this stock...

See Answer

Q: A put option and a call option with an exercise price of

A put option and a call option with an exercise price of $85 and three months to expiration sell for $6.18 and $5.09, respectively. If the risk-free rate is 4.8 percent per year, compounded continuous...

See Answer

Q: Yan Yan Corp. has a $2,000 par value

Yan Yan Corp. has a $2,000 par value bond outstanding with a coupon rate of 4.9 percent paid semiannually and 13 years to maturity. The yield to maturity of the bond is 3.8 percent. What is the dollar...

See Answer

Q: A put option and a call option with an exercise price of

A put option and a call option with an exercise price of $55 expire in two months and sell for $2.65 and $5.32, respectively. If the stock is currently priced at $57.30, what is the annual continuousl...

See Answer

Q: Union Local School District has bonds outstanding with a coupon rate of

Union Local School District has bonds outstanding with a coupon rate of 3.7 percent paid semiannually and 16 years to maturity. The yield to maturity on these bonds is 3.9 percent, and the bonds have...

See Answer

Q: Dan Ervin was recently hired by East Coast Yachts to assist the

Dan Ervin was recently hired by East Coast Yachts to assist the company with its short-term financial planning and also to evaluate the company’s financial performance. Dan graduated...

See Answer

Q: Indicate the impact of the following corporate actions on cash, using

Indicate the impact of the following corporate actions on cash, using the letter I for an increase, D for a decrease, or N when no change occurs. a. A dividend is paid with funds received from a sale...

See Answer

Q: What types of actions might the management of a firm take to

What types of actions might the management of a firm take to fight a hostile acquisition bid from an unwanted suitor? How do the target firm shareholders benefit from the defensive tactics of their ma...

See Answer

Q: Ethics Firms sometimes use the threat of a bankruptcy filing to force

Ethics Firms sometimes use the threat of a bankruptcy filing to force creditors to renegotiate terms. Critics argue that in such cases the firm is using bankruptcy laws “as a sword rather than a shiel...

See Answer

Q: Some countries encourage movements in their exchange rate relative to those of

Some countries encourage movements in their exchange rate relative to those of some other country as a short-term means of addressing foreign trade imbalances. For each of the following scenarios, eva...

See Answer

Q: For each of the short-term marketable securities given here,

For each of the short-term marketable securities given here, provide an example of the potential disadvantages the investment has for meeting a corporation’s cash management goals: a. U.S. Treasury bi...

See Answer

Q: It takes Cookie Cutter Modular Homes, Inc., about five days

It takes Cookie Cutter Modular Homes, Inc., about five days to receive and deposit checks from customers. Cookie Cutter’s management is considering a lockbox system to reduce the firm’s collection tim...

See Answer

Q: The Thakor Corporation’s purchases from suppliers in a quarter are equal to

The Thakor Corporation’s purchases from suppliers in a quarter are equal to 75 percent of the next quarter’s forecast sales. The payables period is 60 days. Wages,...

See Answer

Q: Consider the following premerger information about a bidding firm (Firm B

Consider the following premerger information about a bidding firm (Firm B) and a target firm (Firm T). Assume that both firms have no debt outstanding. Firm B has estimated that the value of the syn...

See Answer

Q: The Arizona Bay Corporation sells on credit terms of net 30.

The Arizona Bay Corporation sells on credit terms of net 30. Its accounts are, on average, 5 days past due. If annual credit sales are $8.95 million, what is the company’s balance sheet amount in acco...

See Answer

Q: Suppose the current exchange rate for the Polish zloty is Z 3

Suppose the current exchange rate for the Polish zloty is Z 3.29. The expected exchange rate in three years is Z 3.41. What is the difference in the annual inflation rates for the United States and Po...

See Answer

Q: Blue Steel Community Bank has the following market value balance sheet:

Blue Steel Community Bank has the following market value balance sheet: a. What is the duration of the assets? b. What is the duration of the liabilities? c. Is the bank immune to interest rate risk...

See Answer

Q: Holmes, Inc., has offered $295 million cash for all

Holmes, Inc., has offered $295 million cash for all of the common stock in Watson Corporation. Based on recent market information, Watson is worth $278 million as an independent operation. If the merg...

See Answer

Q: If a company moves to a JIT inventory management system, what

If a company moves to a JIT inventory management system, what will happen to inventory turnover? What will happen to total asset turnover? What will happen to return on equity (ROE)? (Hint: Remember t...

See Answer

Q: Last month, BlueSky Airline announced that it would stretch out its

Last month, BlueSky Airline announced that it would stretch out its bill payments to 45 days from 30 days. The reason given was that the company wanted to “control costs and optimize cash flow.” The i...

See Answer

Q: A company has a large bond issue maturing in one year.

A company has a large bond issue maturing in one year. When it matures, the company will float a new issue. Current interest rates are attractive, and the company is concerned that rates next year wil...

See Answer

Q: Ethics Several firms have entered bankruptcy, or threatened to enter bankruptcy

Ethics Several firms have entered bankruptcy, or threatened to enter bankruptcy, at least in part as a means of reducing labor costs. Whether this move is ethical, or proper, is hotly debated. Is this...

See Answer

Q: In the previous problem, are the shareholders of Firm T better

In the previous problem, are the shareholders of Firm T better off with the cash offer or the stock offer? At what exchange ratio of B shares to T shares would the shareholders in T be indifferent bet...

See Answer

Q: Air Spares is a wholesaler that stocks engine components and test equipment

Air Spares is a wholesaler that stocks engine components and test equipment for the commercial aircraft industry. A new customer has placed an order for eight high-bypass turbine engines, which increa...

See Answer

Q: Suppose your company imports computer motherboards from Singapore. The exchange rate

Suppose your company imports computer motherboards from Singapore. The exchange rate is given in Figure 31.1. You have just placed an order for 30,000 motherboards at a cost to you of 141.30 Singapore...

See Answer

Q: No More Pencils, Inc., disburses checks every two weeks that

No More Pencils, Inc., disburses checks every two weeks that average $61,700 and take seven days to clear. How much interest can the company earn annually if it delays transfer of funds from an intere...

See Answer

Q: The following is the sales budget for Shleifer, Inc., for

The following is the sales budget for Shleifer, Inc., for the first quarter of 2016: Credit sales are collected as follows: 65 percent in the month of the sale. 20 percent in the month after the sal...

See Answer

Q: Refer to Table 25.2 in the text to answer this

Refer to Table 25.2 in the text to answer this question. Suppose today is January 8, 2015, and your firm produces breakfast cereal and needs 140,000 bushels of corn in March 2015 for an upcoming pro...

See Answer

Q: You place an order for 400 units of inventory at a unit

You place an order for 400 units of inventory at a unit price of $115. The supplier offers terms of 1/10, net 30. a. How long do you have to pay before the account is overdue? If you take the full per...

See Answer

Q: Suppose a company in which you own stock has attracted two takeover

Suppose a company in which you own stock has attracted two takeover offers. Would it ever make sense for your company’s management to favor the lower offer? Does the form of payment affect your answer...

See Answer

Q: If a company’s inventory carrying costs are $5 million per year

If a company’s inventory carrying costs are $5 million per year and its fixed order costs are $8 million per year, do you think the firm keeps too much inventory on hand or too little? Why?

See Answer

Q: We discussed five international capital market relationships: Relative PPP, IRP

We discussed five international capital market relationships: Relative PPP, IRP, UFR, UIP, and the international Fisher effect. Which of these would you expect to hold most closely? Which do you think...

See Answer

Q: It is sometimes argued that excess cash held by a firm can

It is sometimes argued that excess cash held by a firm can aggravate agency problems (discussed in Chapter 1) and, more generally, reduce incentives for shareholder wealth maximization. How would you...

See Answer

Q: Last month, BlueSky Airline announced that it would stretch out its

Last month, BlueSky Airline announced that it would stretch out its bill payments to 45 days from 30 days. The reason given was that the company wanted to “control costs and optimize cash flow.” The i...

See Answer

Q: Explain why a swap is effectively a series of forward contracts.

Explain why a swap is effectively a series of forward contracts. Suppose a firm enters a swap agreement with a swap dealer. Describe the nature of the default risk faced by both parties.

See Answer

Q: Acquiring firm stockholders seem to benefit little from takeovers. Why is

Acquiring firm stockholders seem to benefit little from takeovers. Why is this finding a puzzle? What are some of the reasons offered for it?

See Answer

Q: Why do so many firms file for legal bankruptcy when private workouts

Why do so many firms file for legal bankruptcy when private workouts are so much less expensive?

See Answer

Q: Leeloo, Inc., is considering a change in its cash-

Leeloo, Inc., is considering a change in its cash-only sales policy. The new terms of sale would be net one month. Based on the following information, determine if the company should proceed or not. D...

See Answer

Q: Suppose the spot and six-month forward rates on the Norwegian

Suppose the spot and six-month forward rates on the Norwegian krone are Kr 6.97 and Kr 7.06, respectively. The annual risk-free rate in the United States is 3 percent, and the annual risk-free rate in...

See Answer

Q: Take a look back at Figure 31.1 to answer the

Take a look back at Figure 31.1 to answer the following questions: a. If you have $100, how many euros can you get? b. How much is one euro worth in dollars? c. If you have 5 million euros, how many d...

See Answer

Q: No More Books Corporation has an agreement with Floyd Bank, whereby

No More Books Corporation has an agreement with Floyd Bank, whereby the bank handles $2.9 million in collections a day and requires a $350,000 compensating balance. No More Books is contemplating canc...

See Answer

Q: Here are some important figures from the budget of Cornell, Inc

Here are some important figures from the budget of Cornell, Inc., for the second quarter of 2016: The company predicts that 5 percent of its credit sales will never be collected, 35 perce...

See Answer

Q: Consider the following premerger information about Firm A and Firm B:

Consider the following premerger information about Firm A and Firm B: Assume that Firm A acquires Firm B via an exchange of stock at a price of $13 for each share of B’s stock. Bot...

See Answer

Q: ABC Company and XYZ Company need to raise funds to pay for

ABC Company and XYZ Company need to raise funds to pay for capital improvements at their manufacturing plants. ABC Company is a well-established firm with an excellent credit rating in the debt market...

See Answer

Q: Last month, BlueSky Airline announced that it would stretch out its

Last month, BlueSky Airline announced that it would stretch out its bill payments to 45 days from 30 days. The reason given was that the company wanted to “control costs and optimize cash flow.” The i...

See Answer

Q: If you are an exporter who must make payments in foreign currency

If you are an exporter who must make payments in foreign currency three months after receiving each shipment and you predict that the domestic currency will appreciate in value over this period, is th...

See Answer

Q: One option a firm usually has with any excess cash is to

One option a firm usually has with any excess cash is to pay its suppliers more quickly. What are the advantages and disadvantages of this use of excess cash?

See Answer

Q: Suppose a firm enters a fixed for floating interest rate swap with

Suppose a firm enters a fixed for floating interest rate swap with a swap dealer. Describe the cash flows that will occur as a result of the swap.

See Answer

Q: At least part of Dell’s corporate profits can be traced to its

At least part of Dell’s corporate profits can be traced to its inventory management. Using just-in-time inventory, Dell typically maintains an inventory of three to four days’ sales. Competitors such...

See Answer

Q: Last month, BlueSky Airline announced that it would stretch out its

Last month, BlueSky Airline announced that it would stretch out its bill payments to 45 days from 30 days. The reason given was that the company wanted to “control costs and optimize cash flow.” The i...

See Answer

Q: In a typical month, the Warren Corporation receives 140 checks totaling

In a typical month, the Warren Corporation receives 140 checks totaling $113,500. These are delayed four days on average. What is the average daily float?

See Answer

Q: Suppose it is your task to evaluate two different investments in new

Suppose it is your task to evaluate two different investments in new subsidiaries for your company, one in your own country and the other in a foreign country. You calculate the cash flows of both pro...

See Answer

Q: Fhloston Manufacturing uses 1,860 switch assemblies per week and then

Fhloston Manufacturing uses 1,860 switch assemblies per week and then reorders another 1,860. If the relevant carrying cost per switch assembly is $6.25, and the fixed order cost is $730, is the compa...

See Answer

Q: You observe that the inflation rate in the United States is 1

You observe that the inflation rate in the United States is 1.8 percent per year and that T-bills currently yield 1.95 percent annually. What do you estimate the inflation rate to be in: a. Australia...

See Answer

Q: Time Bird’s Eye Treehouses, Inc., a Kentucky company, has

Time Bird’s Eye Treehouses, Inc., a Kentucky company, has determined that a majority of its customers are located in the Pennsylvania area. It therefore is considering using a lockbo...

See Answer

Q: Here are the most recent balance sheets for Country Kettles, Inc

Here are the most recent balance sheets for Country Kettles, Inc. Excluding accumulated depreciation, determine whether each item is a source or a use of cash, and the amount:

See Answer

Q: Show that the NPV of a merger can be expressed as the

Show that the NPV of a merger can be expressed as the value of the synergistic benefits, DV, less the merger premium.

See Answer

Q: Ted and Alice Hansel have a son who will begin college 10

Ted and Alice Hansel have a son who will begin college 10 years from today. School expenses of $30,000 will need to be paid at the beginning of each of the four years that their son plans to attend co...

See Answer

Q: Another option usually available is to reduce the firm’s outstanding debt.

Another option usually available is to reduce the firm’s outstanding debt. What are the advantages and disadvantages of this use of excess cash?

See Answer

Q: What is the difference between transactions and economic exposure? Which can

What is the difference between transactions and economic exposure? Which can be hedged more easily? Why?

See Answer

Q: An investment in a foreign subsidiary is estimated to have a positive

An investment in a foreign subsidiary is estimated to have a positive NPV after the discount rate used in the calculations is adjusted for political risk and any advantages from diversification. Does...

See Answer

Q: Refer to Table 25.2 in the text to answer this

Refer to Table 25.2 in the text to answer this question. Suppose you purchase a March 2015 cocoa futures contract on January 8, 2015, at the last price of the day. What will your profit or loss be if...

See Answer

Q: Fly-By-Night Couriers is analyzing the possible acquisition of

Fly-By-Night Couriers is analyzing the possible acquisition of Flash-in-the-Pan Restaurants. Neither firm has debt. The forecasts of Fly-By-Night show that the purchase would increase its annual after...

See Answer

Q: The Trektronics store begins each week with 675 phasers in stock.

The Trektronics store begins each week with 675 phasers in stock. This stock is depleted each week and reordered. If the carrying cost per phaser is $73 per year and the fixed order cost is $340, what...

See Answer

Q: Suppose the spot and three-month forward rates for the yen

Suppose the spot and three-month forward rates for the yen are ¥115.13 and ¥114.35, respectively. a. Is the yen expected to get stronger or weaker? b. What would you estimate is the difference between...

See Answer

Q: Cow Chips, Inc., a large fertilizer distributor based in California

Cow Chips, Inc., a large fertilizer distributor based in California, is planning to use a lockbox system to speed up collections from its customers located on the East Coast. A Philadelphia-area bank...

See Answer

Q: The sales budget for your company in the coming year is based

The sales budget for your company in the coming year is based on a quarterly growth rate of 10 percent, with the first-quarter sales projection at $185 million. In addition to this basic trend, the se...

See Answer

Q: What is the duration of a bond with two years to maturity

What is the duration of a bond with two years to maturity if the bond has a coupon rate of 6.4 percent paid semiannually, and the market interest rate is 4.9 percent?

See Answer

Q: An unfortunately common practice goes like this (Warning: Don’t try

An unfortunately common practice goes like this (Warning: Don’t try this at home): Suppose you are out of money in your checking account; however, your local grocery store will, as a convenience to yo...

See Answer

Q: Last month, BlueSky Airline announced that it would stretch out its

Last month, BlueSky Airline announced that it would stretch out its bill payments to 45 days from 30 days. The reason given was that the company wanted to “control costs and optimize cash flow.” The i...

See Answer

Q: If a U.S. company exports its goods to Japan

If a U.S. company exports its goods to Japan, how would it use a futures contract on Japanese yen to hedge its exchange rate risk? Would it buy or sell yen futures? Does the way the exchange rate is q...

See Answer

Q: If a U.S. firm raises funds for a foreign

If a U.S. firm raises funds for a foreign subsidiary, what are the disadvantages to borrowing in the United States? How would you overcome them?

See Answer

Q: Define financial distress using the stock-based and flow-based

Define financial distress using the stock-based and flow-based approaches.

See Answer

Q: Prove that when carrying costs and restocking costs are as described in

Prove that when carrying costs and restocking costs are as described in the chapter, the EOQ must occur at the point where the carrying costs and restocking costs are equal.

See Answer

Q: Harrods PLC has a market value of £360 million and 30 

Harrods PLC has a market value of £360 million and 30 million shares outstanding. Selfridge Department Store has a market value of £144 million and 18 million shares outstanding. Harrods is contemplat...

See Answer

Q: Suppose the spot exchange rate for the Hungarian forint is HUF 251

Suppose the spot exchange rate for the Hungarian forint is HUF 251. The inflation rate in the United States is 2.8 percent per year and is 3.7 percent in Hungary. What do you predict the exchange rate...

See Answer

Q: Wildcat, Inc., has estimated sales (in millions) for

Wildcat, Inc., has estimated sales (in millions) for the next four quarters as follows: Sales for the first quarter of the year after this one are projected at $120 million. Accounts receivable at t...

See Answer

Q: The forward price (F) of a contract on an asset

The forward price (F) of a contract on an asset with neither carrying costs nor convenience yield is the current spot price of the asset (S 0) multiplied by 1, plus the appropriate interest rate betwe...

See Answer

Q: For the following scenarios, describe a hedging strategy using futures contracts

For the following scenarios, describe a hedging strategy using futures contracts that might be considered. a. A public utility is concerned about rising costs. b. A candy manufacturer is concerned abo...

See Answer

Q: Rework Problem 13 assuming the following: a. Wildcat maintains

Rework Problem 13 assuming the following: a. Wildcat maintains a minimum cash balance of $20 million. b. Wildcat maintains a minimum cash balance of $10 million. Based on your answers in (a) and (b),...

See Answer

Q: The Harrington Corporation is considering a change in its cash-only

The Harrington Corporation is considering a change in its cash-only policy. The new terms would be net one period. Based on the following information, determine if Harrington should proceed or not. Th...

See Answer

Q: Bentley Corp. and Rolls Manufacturing are considering a merger. The

Bentley Corp. and Rolls Manufacturing are considering a merger. The possible states of the economy and each company’s value in that state are shown here: Bentley currently has a bo...

See Answer

Q: Lakonishok Equipment has an investment opportunity in Europe. The project costs

Lakonishok Equipment has an investment opportunity in Europe. The project costs €19 million and is expected to produce cash flows of €3.6 million in Year 1,€4.1 million in Year 2, and €5.1 million in...

See Answer

Q: Is it possible for a firm to have too much cash?

Is it possible for a firm to have too much cash? Why would shareholders care if a firm accumulates large amounts of cash?

See Answer

Q: If financial markets are perfectly competitive and the Eurodollar rate is above

If financial markets are perfectly competitive and the Eurodollar rate is above that offered in the U.S. loan market, you would immediately want to borrow money in the United States and invest it in E...

See Answer

Q: In May 2004, Sysco Corporation, the distributor of food and

In May 2004, Sysco Corporation, the distributor of food and food-related products (not to be confused with Cisco Systems), announced it had signed an interest rate swap. The interest rate swap effecti...

See Answer

Q: You enter into a forward contract to buy a 10-year

You enter into a forward contract to buy a 10-year, zero coupon bond that will be issued in one year. The face value of the bond is $1,000, and the 1-year and 11-year spot interest rates are 5 percent...

See Answer

Q: Cleveland Compressor and Pnew York Pneumatic are competing manufacturing firms. Their

Cleveland Compressor and Pnew York Pneumatic are competing manufacturing firms. Their financial statements are printed here. a. How are the current assets of each firm financed? b. Which firm has the...

See Answer

Q: Happy Times currently has an all-cash credit policy. It

Happy Times currently has an all-cash credit policy. It is considering making a change in the credit policy by going to terms of net 30 days. Based on the following information, what do you recommend?...

See Answer

Q: Plant, Inc., is considering making an offer to purchase Palmer

Plant, Inc., is considering making an offer to purchase Palmer Corp. Plant’s vice president of finance has collected the following information: Plant also knows that securities ana...

See Answer

Q: You are evaluating a proposed expansion of an existing subsidiary located in

You are evaluating a proposed expansion of an existing subsidiary located in Switzerland. The cost of the expansion would be SF 25 million. The cash flows from the project would be SF 6.9 million per...

See Answer

Q: This morning you agreed to buy a one-year Treasury bond

This morning you agreed to buy a one-year Treasury bond in six months. The bond has a face value of $1,000. Use the spot interest rates listed here to answer the following questions: a. What is the...

See Answer

Q: William Santiago is interested in entering the import/export business.

William Santiago is interested in entering the import/export business. During a recent visit with his financial advisers, he said, “If we play the game right, this is the safest business in the world....

See Answer

Q: The Silver Spokes Bicycle Shop has decided to offer credit to its

The Silver Spokes Bicycle Shop has decided to offer credit to its customers during the spring selling season. Sales are expected to be 700 bicycles. The average cost to the shop of a bicycle is $650....

See Answer

Q: What are some of the characteristics of a firm with a long

What are some of the characteristics of a firm with a long operating cycle?

See Answer

Q: The Chocolate Ice Cream Company and the Vanilla Ice Cream Company have

The Chocolate Ice Cream Company and the Vanilla Ice Cream Company have agreed to merge and form Fudge Swirl Consolidated. Both companies are exactly alike except that they are located in different tow...

See Answer

Q: Atreides International has operations in Arrakis. The balance sheet for this

Atreides International has operations in Arrakis. The balance sheet for this division in Arrakeen solaris shows assets of 43,000 solaris, debt in the amount of 14,000 solaris, and equity of 29,000 sol...

See Answer

Q: Suppose there were call options and forward contracts available on coal,

Suppose there were call options and forward contracts available on coal, but no put options. Show how a financial engineer could synthesize a put option using the available contracts. What does your a...

See Answer

Q: Kevin Nomura is a Japanese student who is planning a one-

Kevin Nomura is a Japanese student who is planning a one-year stay in the United States. He expects to arrive in the United States in eight months. He is worried about depreciation of the yen relative...

See Answer

Q: In the previous problem, assume the equity increases by 1,

In the previous problem, assume the equity increases by 1,750 solaris due to retained earnings. If the exchange rate at the end of the year is 1.24 solaris per dollar, what does the balance sheet look...

See Answer

Q: In Problem 14, what is the break-even quantity for

In Problem 14, what is the break-even quantity for the new credit policy?

See Answer

Q: From our discussion of the Fisher effect in Chapter 8, we

From our discussion of the Fisher effect in Chapter 8, we know that the actual relationship between a nominal rate, R, a real rate, r, and an inflation rate, h, can be written as follows: 1 + r = (1 +...

See Answer

Q: In Problem 14, what is the break-even price per

In Problem 14, what is the break-even price per unit that should be charged under the new credit policy? Assume that the sales figure under the new policy is 3,150 units and all other values remain th...

See Answer

Q: In Problem 15, what is the break-even price per

In Problem 15, what is the break-even price per unit under the new credit policy? Assume all other values remain the same.

See Answer

Q: Saché, Inc., expects to sell 700 of its designer suits

Saché, Inc., expects to sell 700 of its designer suits every week. The store is open seven days a week and expects to sell the same number of suits every day. The company has an EOQ of 500 suits and a...

See Answer

Q: Warf Computers, Inc., was founded 15 years ago by Nick

Warf Computers, Inc., was founded 15 years ago by Nick Warf, a computer programmer. The small initial investment to start the company was made by Nick and his friends. Over the years, this same group...

See Answer

Q: Solar Engines manufactures solar engines for tractor-trailers. Given the

Solar Engines manufactures solar engines for tractor-trailers. Given the fuel savings available, new orders for 125 units have been made by customers requesting credit. The variable cost is $11,400 pe...

See Answer

Q: In the previous problem, assume that the probability of default is

In the previous problem, assume that the probability of default is 15 percent. Should the orders be filled now? Assume the number of repeat customers is affected by the defaults. In other words, 30 pe...

See Answer

Q: Ben Bates graduated from college six years ago with a finance undergraduate

Ben Bates graduated from college six years ago with a finance undergraduate degree. Although he is satisfied with his current job, his goal is to become an investment banker. He feels that an MBA degr...

See Answer

Q: Indicate whether you think the following claims regarding takeovers are true or

Indicate whether you think the following claims regarding takeovers are true or false. In each case, provide a brief explanation for your answer. a. By merging competitors, takeovers have created mono...

See Answer

Q: If a firm is buying call options on pork belly futures as

If a firm is buying call options on pork belly futures as a hedging strategy, what must be true about the firm’s exposure to pork belly prices?

See Answer

Q: The Paden Corporation has annual sales of $29.5 million

The Paden Corporation has annual sales of $29.5 million. The average collection period is 27 days. What is the average investment in accounts receivable as shown on the balance sheet?

See Answer

Q: Use the information in Figure 31.1 to answer the following

Use the information in Figure 31.1 to answer the following questions: a. Which would you rather have, $100 or £100? Why? b. Which would you rather have, 100 Swiss francs (SF) or £100? Why? c. What is...

See Answer

Q: Each business day, on average, a company writes checks totaling

Each business day, on average, a company writes checks totaling $14,400 to pay its suppliers. The usual clearing time for the checks is four days. Meanwhile, the company is receiving payments from its...

See Answer

Q: Blizzard Corp. has a book value of equity of $14

Blizzard Corp. has a book value of equity of $14,750. Long-term debt is $8,300. Net working capital, other than cash, is $1,950. Fixed assets are $20,730 and current liabilities are $1,930. How much c...

See Answer

Q: Consider the following premerger information about firm X and firm Y:

Consider the following premerger information about firm X and firm Y: Assume that Firm X acquires Firm Y by paying cash for all the shares outstanding at a merger premium of $5 per share. Assuming t...

See Answer

Q: Refer to Table 25.2 in the text to answer this

Refer to Table 25.2 in the text to answer this question. Suppose you sell five March 2015 silver futures contracts on January 8, 2015, at the last price of the day. What will your profit or loss be if...

See Answer

Q: What are some benefits of financial distress?

What are some benefits of financial distress?

See Answer

Q: In what form is trade credit most commonly offered? What is

In what form is trade credit most commonly offered? What is the credit instrument in this case?

See Answer

Q: Seth Bullock, the owner of Bullock Gold Mining, is evaluating

Seth Bullock, the owner of Bullock Gold Mining, is evaluating a new gold mine in South Dakota. Dan Dority, the company’s geologist, has just finished his analysis of the mine site. H...

See Answer

Q: Suppose the rate of inflation in Mexico will run about 3 percent

Suppose the rate of inflation in Mexico will run about 3 percent higher than the U.S. inflation rate over the next several years. All other things being the same, what will happen to the Mexican peso...

See Answer

Q: What options are available to a firm if it believes it has

What options are available to a firm if it believes it has too much cash? How about too little?

See Answer

Q: What are some of the characteristics of a firm with a long

What are some of the characteristics of a firm with a long cash cycle?

See Answer

Q: After extensive research and development, Goodweek Tires, Inc., has

After extensive research and development, Goodweek Tires, Inc., has recently developed a new tire, the SuperTread, and must decide whether to make the investment necessary to produce and market it. Th...

See Answer

Q: For the year just ended, you have gathered the following information

For the year just ended, you have gathered the following information about the Holly Corporation: a. A $200 dividend was paid. b. Accounts payable increased by $500. c. Fixed asset purchases were $900...

See Answer

Q: Explain why diversification per se is probably not a good reason for

Explain why diversification per se is probably not a good reason for merger.

See Answer

Q: The exchange rate for the Australian dollar is currently A$1

The exchange rate for the Australian dollar is currently A$1.40. This exchange rate is expected to rise by 10 percent over the next year. a. Is the Australian dollar expected to get stronger or weaker...

See Answer

Q: Fair-to-Midland Manufacturing, Inc. (FMM),

Fair-to-Midland Manufacturing, Inc. (FMM), has applied for a loan at True Credit Bank. Jon Fulkerson, the credit analyst at the bank, has gathered the following information from the companyâ ...

See Answer

Q: Use the information in Figure 31.1 to answer the following

Use the information in Figure 31.1 to answer the following questions: a. What is the six-month forward rate for the Japanese yen in yen per U.S. dollar? Is the yen selling at a premium or a discount?...

See Answer

Q: Purple Feet Wine, Inc., receives an average of $13

Purple Feet Wine, Inc., receives an average of $13,800 in checks per day. The delay in clearing is typically three days. The current interest rate is .018 percent per day. a. What is the company’s flo...

See Answer

Q: Bethesda Mining is a midsized coal mining company with 20 mines located

Bethesda Mining is a midsized coal mining company with 20 mines located in Ohio, Pennsylvania, West Virginia, and Kentucky. The company operates deep mines as well as strip mines. Most of the coal min...

See Answer

Q: Indicate the effect that the following will have on the operating cycle

Indicate the effect that the following will have on the operating cycle. Use the letter I to indicate an increase, the letter D for a decrease, and the letter N for no change. a. Receivables average g...

See Answer

Q: Assume that the following balance sheets are stated at book value.

Assume that the following balance sheets are stated at book value. Suppose that Jurion Co. purchases James, Inc. Then suppose the fair market value of James’s fixed assets is $23,000...

See Answer

Q: Kyoto Joe, Inc., sells earnings forecasts for Japanese securities.

Kyoto Joe, Inc., sells earnings forecasts for Japanese securities. Its credit terms are 1/15, net 30. Based on experience, 70 percent of all customers will take the discount. a. What is the average co...

See Answer

Q: Suppose a financial manager buys call options on 50,000 barrels

Suppose a financial manager buys call options on 50,000 barrels of oil with an exercise price of $65 per barrel. She simultaneously sells a put option on 50,000 barrels of oil with the same exercise p...

See Answer

Q: What costs are associated with carrying receivables? What costs are associated

What costs are associated with carrying receivables? What costs are associated with not granting credit? What do we call the sum of the costs for different levels of receivables?

See Answer

Q: Are stockholders and creditors likely to agree on how much cash a

Are stockholders and creditors likely to agree on how much cash a firm should keep on hand?

See Answer

Q: What is prepackaged bankruptcy? What is the main benefit of prepackaged

What is prepackaged bankruptcy? What is the main benefit of prepackaged bankruptcy?

See Answer

Q: What is the difference between a forward contract and a futures contract

What is the difference between a forward contract and a futures contract? Why do you think that futures contracts are much more common? Are there any circumstances under which you might prefer to use...

See Answer

Q: What are the five Cs of credit? Explain why each is

What are the five Cs of credit? Explain why each is important.

See Answer

Q: No matter how you look at it, Dow Chemical had busy

No matter how you look at it, Dow Chemical had busy years in 2014 and 2015. In 2014, Dow announced that it was planning to sell its subsidiaries Angus Chemical Company, Sodium Borohydride, and AgroFre...

See Answer

Q: Bunyan Lumber, LLC, harvests timber and delivers logs to timber

Bunyan Lumber, LLC, harvests timber and delivers logs to timber mills for sale. The company was founded 70 years ago by Pete Bunyan. The current CEO is Paula Bunyan, the granddaughter of the founder....

See Answer

Q: Tidwell, Inc., has weekly credit sales of $31,

Tidwell, Inc., has weekly credit sales of $31,400, and the average collection period is 29 days. The cost of production is 75 percent of the selling price. What is the average accounts receivable figu...

See Answer

Q: Suppose the spot exchange rate for the Canadian dollar is Can $

Suppose the spot exchange rate for the Canadian dollar is Can $1.13 and the six-month forward rate is Can$1.16. a. Which is worth more, a U.S. dollar or a Canadian dollar? b. Assuming absolute PPP hol...

See Answer

Q: Delay Your neighbor goes to the post office once a month and

Delay Your neighbor goes to the post office once a month and picks up two checks, one for $9,700 and one for $2,600. The larger check takes four days to clear after it is deposited; the smaller one ta...

See Answer

Q: Indicate the impact of the following on the cash and operating cycles

Indicate the impact of the following on the cash and operating cycles, respectively. Use the letter I to indicate an increase, the letter D for a decrease, and the letter N for no change. a. The terms...

See Answer

Q: Silver Enterprises has acquired All Gold Mining in a merger transaction.

Silver Enterprises has acquired All Gold Mining in a merger transaction. Construct the balance sheet for the new corporation if the merger is treated as a purchase for accounting purposes....

See Answer

Q: You are long 10 gold futures contracts, established at an initial

You are long 10 gold futures contracts, established at an initial settle price of $1,210 per ounce, where each contract represents 100 ounces. Over the subsequent four trading days, gold settles at $1...

See Answer

Q: Why doesn’t financial distress always cause firms to die?

Why doesn’t financial distress always cause firms to die?

See Answer

Q: Are exchange rate changes necessarily good or bad for a particular company

Are exchange rate changes necessarily good or bad for a particular company?

See Answer

Q: What is the difference between cash management and liquidity management?

What is the difference between cash management and liquidity management?

See Answer

Q: Grohl Manufacturing, Inc., has recently installed a just-in

Grohl Manufacturing, Inc., has recently installed a just-in-time (JIT) inventory system. Describe the effect this is likely to have on the company’s carrying costs, shortage costs, and operating cycle...

See Answer

Q: Describe each of the following: a. Sight draft.

Describe each of the following: a. Sight draft. b. Time draft. c. Banker’s acceptance. d. Promissory note. e. Trade acceptance.

See Answer

Q: Bubbling Crude Corporation, a large Texas oil producer, would like

Bubbling Crude Corporation, a large Texas oil producer, would like to hedge against adverse movements in the price of oil because this is the firm’s primary source of revenue. What should the firm do?...

See Answer

Q: Is it possible for a firm’s cash cycle to be longer than

Is it possible for a firm’s cash cycle to be longer than its operating cycle? Explain why or why not.

See Answer

Q: What are some of the factors that determine the length of the

What are some of the factors that determine the length of the credit period? Why is the length of the buyer’s operating cycle often considered an upper bound on the length of the credit period?

See Answer

Q: A firm offers terms of 1/10, net 30.

A firm offers terms of 1/10, net 30. What effective annual interest rate does the firm earn when a customer does not take the discount? Without doing any calculations, explain what will happen to this...

See Answer

Q: Suppose the Japanese yen exchange rate is ¥126 5 $1

Suppose the Japanese yen exchange rate is ¥126 5 $1, and the British pound exchange rate is £1 5 $1.53. a. What is the cross-rate in terms of yen per pound? b. Suppose the cross-rate is ¥195.8 5 £1. I...

See Answer

Q: Your firm has an average receipt size of $119. A

Your firm has an average receipt size of $119. A bank has approached you concerning a lockbox service that will decrease your total collection time by two days. You typically receive 5,650 checks per...

See Answer

Q: The Litzenberger Company has projected the following quarterly sales amounts for the

The Litzenberger Company has projected the following quarterly sales amounts for the coming year: a. Accounts receivable at the beginning of the year are $335. The company has a 45-day collection per...

See Answer

Q: Penn Corp. is analyzing the possible acquisition of Teller Company.

Penn Corp. is analyzing the possible acquisition of Teller Company. Both firms have no debt. Penn believes the acquisition will increase its total aftertax annual cash flow by $1.3 million indefinitel...

See Answer

Q: You are short 25 gasoline futures contracts, established at an initial

You are short 25 gasoline futures contracts, established at an initial settle price of $1.36 per gallon, where each contract represents 42,000 gallons. Over the subsequent four trading days, gasoline...

See Answer

Q: What is the difference between liquidation and reorganization?

What is the difference between liquidation and reorganization?

See Answer

Q: Explain the purchase accounting method for mergers. What is the effect

Explain the purchase accounting method for mergers. What is the effect on cash flows? On EPS?

See Answer

Q: At one point, Duracell International confirmed that it was planning to

At one point, Duracell International confirmed that it was planning to open battery manufacturing plants in China and India. Manufacturing in these countries allows Duracell to avoid import duties of...

See Answer

Q: Why is a preferred stock with a dividend tied to short-

Why is a preferred stock with a dividend tied to short-term interest rates an attractive short-term investment for corporations with excess cash?

See Answer

Q: Are poison pills good or bad for stockholders? How do you

Are poison pills good or bad for stockholders? How do you think acquiring firms are able to get around poison pills?

See Answer

Q: A company produces an energy-intensive product and uses natural gas

A company produces an energy-intensive product and uses natural gas as the energy source. The competition primarily uses oil. Explain why this company is exposed to fluctuations in both oil and natura...

See Answer

Q: What are the costs of shortages? Describe them.

What are the costs of shortages? Describe them.

See Answer

Q: In each of the following pairings, indicate which firm would probably

In each of the following pairings, indicate which firm would probably have a longer credit period and explain your reasoning. a. Firm A sells a miracle cure for baldness; Firm B sells toupees. b. Firm...

See Answer

Q: Describe the advantages and disadvantages of a taxable merger as opposed to

Describe the advantages and disadvantages of a taxable merger as opposed to a tax-free exchange. What is the basic determinant of tax status in a merger? Would an LBO be taxable or nontaxable? Explain...

See Answer

Q: What is the absolute priority rule?

What is the absolute priority rule?

See Answer

Q: Given that many multinationals based in many countries have much greater sales

Given that many multinationals based in many countries have much greater sales outside their domestic markets than within them, what is the particular relevance of their domestic currency?

See Answer

Q: Consider the following financial statement information for the Rivers Corporation:

Consider the following financial statement information for the Rivers Corporation: Calculate the operating and cash cycles. How do you interpret your answer?

See Answer

Q: Suppose the exchange rate for the Swiss franc is quoted as SF

Suppose the exchange rate for the Swiss franc is quoted as SF 1.09 in the spot market and SF 1.11 in the 90-day forward market. a. Is the dollar selling at a premium or a discount relative to the fran...

See Answer

Q: The shareholders of Flannery Company have voted in favor of a buyout

The shareholders of Flannery Company have voted in favor of a buyout offer from Stultz Corporation. Information about each firm is given here: Flannery’s shareholders will receive...

See Answer

Q: Turnover Chen, Inc., has an average collection period of 34

Turnover Chen, Inc., has an average collection period of 34 days. Its average daily investment in receivables is $61,300. What are annual credit sales? What is the receivables turnover?

See Answer

Q: Use Figure 31.1 to answer the following questions. Suppose

Use Figure 31.1 to answer the following questions. Suppose interest rate parity holds, and the current six-month risk-free rate in the United States is 1.9 percent. What must the six-month risk-free r...

See Answer

Q: A mail-order firm processes 5,450 checks per month

A mail-order firm processes 5,450 checks per month. Of these, 70 percent are for $55 and 30 percent are for $80. The $55 checks are delayed two days on average; the $80 checks are delayed three days o...

See Answer

Q: What is the duration of a bond with three years to maturity

What is the duration of a bond with three years to maturity and a coupon of 6.1 percent paid annually if the bond sells at par?

See Answer

Q: Which would a firm prefer: A net collection float or a

Which would a firm prefer: A net collection float or a net disbursement float? Why?

See Answer

Q: If a textile manufacturer wanted to hedge against adverse movements in cotton

If a textile manufacturer wanted to hedge against adverse movements in cotton prices, it could buy cotton futures contracts or buy call options on cotton futures contracts. What would be the pros and...

See Answer

Q: In an ideal economy, net working capital is always zero.

In an ideal economy, net working capital is always zero. Why might net working capital be positive in a real economy?

See Answer

Q: What are the different inventory types? How do the types differ

What are the different inventory types? How do the types differ? Why are some types said to have dependent demand, whereas other types are said to have independent demand?

See Answer

Q: What are DIP loans? Where do DIP loans fall in the

What are DIP loans? Where do DIP loans fall in the APR?

See Answer

Q: If a firm is selling futures contracts on lumber as a hedging

If a firm is selling futures contracts on lumber as a hedging strategy, what must be true about the firm’s exposure to lumber prices?

See Answer

Q: Are the following statements true or false? Explain why.

Are the following statements true or false? Explain why. a. If the general price index in Great Britain rises faster than that in the United States, we would expect the pound to appreciate relative to...

See Answer

Q: Submarine Manufacturing is investigating a lockbox system to reduce its collection time

Submarine Manufacturing is investigating a lockbox system to reduce its collection time. It has determined the following: The total collection time will be reduced by three days if the lockbox syste...

See Answer

Q: Lewellen Products has projected the following sales for the coming year:

Lewellen Products has projected the following sales for the coming year: Sales in the year following this one are projected to be 15 percent greater in each quarter. a. Calculate payments to supplie...

See Answer

Q: Cholern Electric Company (CEC) is a public utility that provides

Cholern Electric Company (CEC) is a public utility that provides electricity to the central Colorado area. Recent events at its Mile-High Nuclear Station have been discouraging. Several shareholders h...

See Answer

Q: Essence of Skunk Fragrances, Ltd., sells 5,450 units

Essence of Skunk Fragrances, Ltd., sells 5,450 units of its perfume collection each year at a price per unit of $480. All sales are on credit with terms of 1/10, net 40. The discount is taken by 35 pe...

See Answer

Q: The treasurer of a major U.S. firm has $

The treasurer of a major U.S. firm has $30 million to invest for three months. The annual interest rate in the United States is .17 percent per month. The interest rate in Great Britain is .61 percent...

See Answer

Q: What is the duration of a bond with four years to maturity

What is the duration of a bond with four years to maturity and a coupon of 8.6 percent paid annually if the bond sells at par?

See Answer

Q: Suppose a firm has a book balance of $2 million.

Suppose a firm has a book balance of $2 million. At the automatic teller machine (ATM), the cash manager finds out that the bank balance is $2.5 million. What is the situation here? If this is an ongo...

See Answer

Q: What does it mean to say that a proposed merger will take

What does it mean to say that a proposed merger will take advantage of available economies of scale? Suppose Eastern Power Co. and Western Power Co. are located in different time zones. Both operate a...

See Answer

Q: Explain why a put option on a bond is conceptually the same

Explain why a put option on a bond is conceptually the same as a call option on interest rates.

See Answer

Q: Larissa has been talking with the company’s directors about the future of

Larissa has been talking with the company’s directors about the future of East Coast Yachts. To this point, the company has used outside suppliers for various key components of the c...

See Answer

Q: You recently graduated from college, and your job search led you

You recently graduated from college, and your job search led you to East Coast Yachts. Because you felt the company’s business was seaworthy, you accepted a job offer. The first day...

See Answer

Q: You are discussing your 401(k) with Dan Ervin when

You are discussing your 401(k) with Dan Ervin when he mentions that Sarah Brown, a representative from Bledsoe Financial Services, is visiting East Coast Yachts today. You decide that you should meet...

See Answer

Q: Dawn Browne, an investment broker, has been approached by client

Dawn Browne, an investment broker, has been approached by client Jack Thomas about the risk of his investments. Dawn has recently read several articles concerning the risk factors that can potentially...

See Answer

Q: You have recently been hired by Swan Motors, Inc. (

You have recently been hired by Swan Motors, Inc. (SMI), in its relatively new treasury management department. SMI was founded 8 years ago by Joe Swan. Joe found a method to manufacture a cheaper batt...

See Answer

Q: You have been at your job with East Coast Yachts for a

You have been at your job with East Coast Yachts for a week now and have decided you need to sign up for the company’s 401(k) plan. Even after your discussion with Sarah Brown, the B...

See Answer

Q: Stephenson Real Estate Company was founded 25 years ago by the current

Stephenson Real Estate Company was founded 25 years ago by the current CEO, Robert Stephenson. The company purchases real estate, including land and buildings, and rents the property to tenants. The c...

See Answer

Q: After Dan’s EFN analysis for East Coast Yachts (see the Mini

After Dan’s EFN analysis for East Coast Yachts (see the Mini Case in Chapter 3), Larissa has decided to expand the company’s operations. She has asked Dan to enlist an underwriter to help sell $50 mil...

See Answer

Q: The following table lists the stock prices for Microsoft from 1989 to

The following table lists the stock prices for Microsoft from 1989 to 1998. The company did not pay any dividends during the period. a. Estimate the average annual return you would have made on your i...

See Answer

Q: There is a conflict of interest between stockholders and managers. In

There is a conflict of interest between stockholders and managers. In theory, stockholders are expected to exercise control over managers through the annual meeting or the board of directors. In pract...

See Answer

Q: JLChem Corporation, a chemical manufacturing firm with changing investment opportunities,

JLChem Corporation, a chemical manufacturing firm with changing investment opportunities, is considering a major change in dividend policy. It currently has 50 million shares outstanding and pays an a...

See Answer

Q: You have been asked to value Alcoa and have come up with

You have been asked to value Alcoa and have come up with the following inputs. • The stock has a beta of 0.90, estimated over the last five years. During this period, the firm had an average debt/equi...

See Answer

Q: RJR Nabisco also had $10 billion in bonds outstanding at the

RJR Nabisco also had $10 billion in bonds outstanding at the time of the dividend increase in Problem 10. How would you expect the bonds to react to the announcement? Why?

See Answer

Q: Battle Mountain is a mining company that mines gold, silver,

Battle Mountain is a mining company that mines gold, silver, and copper in mines in South America, Africa, and Australia. The beta for the stock is estimated to be 0.30. Given the volatility in commod...

See Answer

Q: It is often argued that managers, when asked to maximize stock

It is often argued that managers, when asked to maximize stock price, have to choose between being socially responsible and carrying out their fiduciary duty. Do you agree? Can you provide an example...

See Answer

Q: You have collected returns on AnaDone , a large diversified manufacturing firm

You have collected returns on AnaDone , a large diversified manufacturing firm, and the NYSE index for five years: a. Estimate the intercept (alpha) and slope (beta) of the regression. b. If you bough...

See Answer

Q: You have been asked by JJ Corporation, a California- based

You have been asked by JJ Corporation, a California- based firm that manufacturers and services digital satellite TV systems, to evaluate its capital structure. They currently have 70 million shares o...

See Answer

Q: When firms increase dividends, stock prices tend to increase. One

When firms increase dividends, stock prices tend to increase. One reason given for this price reaction is that dividends operate as a positive signal. What is the increase in dividends signaling to ma...

See Answer

Q: GL Corporation, a retail firm, is making a decision on

GL Corporation, a retail firm, is making a decision on how much it should pay out to its stockholders. It has $100 million in investible funds. The following information is provided about the firm: &a...

See Answer

Q: You are analyzing a valuation done on a stable firm by a

You are analyzing a valuation done on a stable firm by a well-known analyst. Based on the expected FCFF next year of $30 million, and an expected growth rate of 5%, the analyst has estimated a value o...

See Answer

Q: Unicom is a regulated utility serving Northern Illinois. The following table

Unicom is a regulated utility serving Northern Illinois. The following table lists the stock prices and dividends on Unicom from 1989 to 1998. a. Estimate the average annual return you would have made...

See Answer

Q: U.S. steel companies have generally been considered mature in

U.S. steel companies have generally been considered mature in terms of growth and often take on high leverage to finance their plant and equipment. Steel companies in some emerging markets often have...

See Answer

Q: You have a project that does not require an initial in-

You have a project that does not require an initial in- vestment but has its expenses spread over the life of the project. Can the IRR be estimated for this project? Why or why not?

See Answer

Q: Assume that you are advising a Turkish firm on corporate financial questions

Assume that you are advising a Turkish firm on corporate financial questions and that you do not believe that the Turkish stock market is efficient. Would you recommend stock price maximization as the...

See Answer

Q: You are the supervisor of a town where the roads are in

You are the supervisor of a town where the roads are in need of repair. You have a limited budget and are considering two options: • You can patch up the roads for $100,000, but you will have to repea...

See Answer

Q: IPOs are difficult to value because firms going public tend to be

IPOs are difficult to value because firms going public tend to be small, and little information is available about them. Investment bankers have to underprice IPOs because they bear substantial pricin...

See Answer

Q: Pfizer, one of the largest pharmaceutical companies in the United States

Pfizer, one of the largest pharmaceutical companies in the United States, is considering what its debt capacity is. In March 1995, Pfizer had an outstanding market value of equity of $24.27 billion, d...

See Answer

Q: You are trying to decide whether the debt structure that Bethlehem Steel

You are trying to decide whether the debt structure that Bethlehem Steel has currently is appropriate, given its assets. You regress the changes in firm value against changes in interest rates and arr...

See Answer

Q: InTech, a computer software firm that has never paid dividends before

InTech, a computer software firm that has never paid dividends before, is considering whether it should start doing so. This firm has a cost of equity of 22% and a cost of debt of 10% (the tax rate is...

See Answer

Q: You have been asked to value Office Help, a private firm

You have been asked to value Office Help, a private firm providing office support services in the New York area. The firm reported pretax operating income of $10 million in its most recent financial y...

See Answer

Q: Businesses with severe capital rationing constraints should use IRR more than NPV

Businesses with severe capital rationing constraints should use IRR more than NPV. Do you agree? Explain.

See Answer

Q: Boise Cascade also had debt outstanding of $1.7 billion

Boise Cascade also had debt outstanding of $1.7 billion and a market value of equity of $1.5 billion; the corporate marginal tax rate was 36%. a. Assuming that the current beta of 0.95 for the stock i...

See Answer

Q: It has been argued by some that convertible bonds (i.

It has been argued by some that convertible bonds (i.e., bonds that are convertible into stock at the option of the bondholders) provide one form of protection against expropriation by stockholders. O...

See Answer

Q: You run a regression of monthly returns of Mapco, an oil

You run a regression of monthly returns of Mapco, an oil- and gas-producing firm, on the S&P 500 Index and come up with the following output for the period 1991–1995. Intercept of the regression = 0.0...

See Answer

Q: You are using the arbitrage pricing model to estimate the expected return

You are using the arbitrage pricing model to estimate the expected return on Bethlehem Steel and have derived the following estimates for the factor betas and risk premia: a. Which risk factor is Beth...

See Answer

Q: You have just run a regression of monthly returns of American Airlines

You have just run a regression of monthly returns of American Airlines (AMR) against the S&P 500 over the past five years. You have misplaced some of the output and are trying to derive it from what y...

See Answer

Q: You have to pick between three mutually exclusive projects with the following

You have to pick between three mutually exclusive projects with the following cash flows to the firm: The cost of capital is 12%. a. Which project would you pick using the NPV rule? b. Which project w...

See Answer

Q: You are the manager of a specialty retailing firm that is considering

You are the manager of a specialty retailing firm that is considering two strategies for getting into the Malaysian retail market. Under the first strategy, the firm will make an initial investment of...

See Answer

Q: You are the owner of a small and successful firm with an

You are the owner of a small and successful firm with an estimated market value of $50 million. You are considering going public. a. What are the considerations you would have in choosing an investmen...

See Answer

Q: Upjohn, another major pharmaceutical company, is also considering whether it

Upjohn, another major pharmaceutical company, is also considering whether it should borrow more. It has $664 million in book value of debt outstanding and 173 million shares outstanding at $30.75 per...

See Answer

Q: Railroad companies in the United States tend to have long-term

Railroad companies in the United States tend to have long-term, fixed rate, dollar denominated debt. Explain why.

See Answer

Q: LimeAde, a large soft drink manufacturing firm, is faced with

LimeAde, a large soft drink manufacturing firm, is faced with the decision of how much to pay out as dividends to its stockholders. It expects to have a net income of $1,000 (after depreciation of $50...

See Answer

Q: Now assume that the facts in Problem 1 remain unchanged except for

Now assume that the facts in Problem 1 remain unchanged except for the depreciation method, which is switched to an accelerated method with the following depreciation schedule: Depreciable asset = Ini...

See Answer

Q: Societies attempt to keep private interests in line by legislating against behavior

Societies attempt to keep private interests in line by legislating against behavior that might create social costs (such as polluting the water). If the legislation is comprehensive enough, does the p...

See Answer

Q: You have run a regression of monthly returns on Amgen, a

You have run a regression of monthly returns on Amgen, a large biotechnology firm, against monthly returns on the S&P 500 Index, and come up with the following output: The current one-year Treasur...

See Answer

Q: You work for a firm that has limited access to capital markets

You work for a firm that has limited access to capital markets. As a consequence, it has only $20 million available for new investments this year. The firm does have a ready supply of good projects, a...

See Answer

Q: You have been asked for advice on a rights offering by a

You have been asked for advice on a rights offering by a firm with 10 million shares outstanding trading at $50 per share. The firm needs to raise $100 million in new equity. Assuming that the rights...

See Answer

Q: Bethlehem Steel, one of the oldest and largest steel companies in

Bethlehem Steel, one of the oldest and largest steel companies in the United States, is considering the question of whether it has any excess debt capacity. The firm has $527 million in market value o...

See Answer

Q: The following table summarizes the results of regressing changes in firm value

The following table summarizes the results of regressing changes in firm value against changes in interest rates for six major footwear companies: Change in Firm Value = a + b(Change in Long â&#...

See Answer

Q: NoLone, an all-equity manufacturing firm, has net in

NoLone, an all-equity manufacturing firm, has net in- come of $100 million currently and expects this number to grow at 10% a year for the next three years. The firm’s working capital increased by $10...

See Answer

Q: The following were the P/E ratios of firms in the

The following were the P/E ratios of firms in the aerospace/defense industry with additional data on expected growth and risk: a. Estimate the average and median P/E ratios. What, if anything, would t...

See Answer

Q: You are analyzing an investment decision, in which you will have

You are analyzing an investment decision, in which you will have to make an initial investment of $10 million and you will be generating annual cash flows to the firm of$2 million every year, growing...

See Answer

Q: One of the arguments made for having legislation restricting hostile takeovers is

One of the arguments made for having legislation restricting hostile takeovers is that unscrupulous speculators may take over well-run firms and destroy them for personal gain. Allowing for the possib...

See Answer

Q: A closely held, publicly traded firm faces self-imposed capital

A closely held, publicly traded firm faces self-imposed capital rationing constraints of $100 million in this period and $75 million in the next period. It has to choose among the following projects (...

See Answer

Q: You have just run a regression of monthly returns on MAD,

You have just run a regression of monthly returns on MAD, a newspaper and magazine publisher, against returns on the S&P 500, and arrived at the following result: The regression has an R 2 of 22%....

See Answer

Q: You are analyzing a project with a thirty-year lifetime,

You are analyzing a project with a thirty-year lifetime, with the following characteristics: • The project will require an initial investment of $20 million and additional investments of $5 million in...

See Answer

Q: Kansas City Southern, a railroad company, had debt out-

Kansas City Southern, a railroad company, had debt out- standing of $985 million and 40 million shares trading at $46.25 per share in March 1995. It earned $203 million in EBIT, and faced a marginal t...

See Answer

Q: You have run a series of regressions of firm value changes at

You have run a series of regressions of firm value changes at Motorola, the semiconductor company, against changes in a number of macroeconomic variables. The results are summarized here: Change in Fi...

See Answer

Q: Boston Turkey is a publicly traded firm, with the following income

Boston Turkey is a publicly traded firm, with the following income statement and balance sheet from its most recent financial year: Boston Turkey expects its revenues to grow 10% next year and its exp...

See Answer

Q: NCH, which markets cleaning chemicals, insecticides, and other products

NCH, which markets cleaning chemicals, insecticides, and other products, paid dividends of $2.00 per share on earnings of $4.00 per share. The book value of equity per share was $40.00, and earnings a...

See Answer

Q: The following equation is reproduced from the study by Fama and French

The following equation is reproduced from the study by Fama and French of returns between 1963 and 1990. where MV is the market value of equity in hundreds of millions of dollar and BV is the book val...

See Answer

Q: You are the manager of a grocery store, and you are

You are the manager of a grocery store, and you are considering offering baby-sitting services to your customers. You estimate that the licensing and set up costs will amount to $150,000 initially and...

See Answer

Q: Assume that SmallTech has net income of $1 million and that

Assume that SmallTech has net income of $1 million and that the earnings will increase in proportion with the additional capital raised. a. Estimate the earning per share that SmallTech will have afte...

See Answer

Q: In 1995, an analysis of the capital structure of Reebok provided

In 1995, an analysis of the capital structure of Reebok provided the following results on the cost of capital and firm value. This analysis was based on the 1995 EBIT of $420 million and a tax rate of...

See Answer

Q: A commodity bond links interest and principal payments to the price of

A commodity bond links interest and principal payments to the price of a commodity. Differentiate a commodity bond from a straight bond and then from equity. How would you factor these differences int...

See Answer

Q: Z-Tec, a firm providing Internet services, reported net

Z-Tec, a firm providing Internet services, reported net income of $10 million in the most recent year, while making $25 million in capital expenditures (depreciation was $5 million). The firm had no w...

See Answer

Q: You are trying to estimate a price per share on an IPO

You are trying to estimate a price per share on an IPO of a company involved in environmental waste disposal. The company has a book value per share of $20 and earned $3.50 per share in the most recen...

See Answer

Q: Time Warner, the entertainment conglomerate, has a beta of 1

Time Warner, the entertainment conglomerate, has a beta of 1.61. Part of the reason for the high beta is the debt left over from the leveraged buyout of Time by Warner in 1989, which amounted to $10 b...

See Answer

Q: You are trying to estimate the NPV of a three-year

You are trying to estimate the NPV of a three-year project, where the discount rate is expected to change over time. a. Estimate the NPV of this project. Would you take this project? b. Estimate the I...

See Answer

Q: You run a financial service firm where you replace your employee’s computers

You run a financial service firm where you replace your employee’s computers every three years. You have 5,000 employees, and each computer costs $2,500 currently—the old computers can be sold for $50...

See Answer

Q: MVP, a manufacturing firm with no debt outstanding and a market

MVP, a manufacturing firm with no debt outstanding and a market value of $100 million, is considering borrowing $40 million and buying back stock. Assuming that the interest rate on the debt is 9% and...

See Answer

Q: You are trying to evaluate whether United Airlines (UAL) has

You are trying to evaluate whether United Airlines (UAL) has any excess debt capacity. In 1995, UAL had 12.2 million shares outstanding at $210 per share and debt outstanding of approximately $3 billi...

See Answer

Q: You are analyzing the dividend policy of Conrail, a major railroad

You are analyzing the dividend policy of Conrail, a major railroad, and you have collected the following information from the past five years The average debt ratio during this period was 40%, and the...

See Answer

Q: Longs Drug, a large U.S. drugstore chain operating

Longs Drug, a large U.S. drugstore chain operating primarily in northern California, had sales per share of $122 on which it reported earnings per share of $2.45 and paid a dividend per share of $1.12...

See Answer

Q: Barring the case of multiple IRRs, is it possible for the

Barring the case of multiple IRRs, is it possible for the NPV of a project to be positive while the IRR is less than the discount rate? Explain.

See Answer

Q: Now assume that Plastico is considering a project that re- quires

Now assume that Plastico is considering a project that re- quires an initial investment of $100 million and has the following projected income statement (depreciation for the project is expected to be...

See Answer

Q: Chrysler, the automotive manufacturer, had a beta of 1.

Chrysler, the automotive manufacturer, had a beta of 1.05 in 1995. It had $13 billion in debt outstanding in that year and 355 million shares trading at $50 per share. The firm had a cash balance of $...

See Answer

Q: You are helping a manufacturing firm decide whether it should invest in

You are helping a manufacturing firm decide whether it should invest in a new plant. The initial investment is expected to be $50 million, and the plant is expected to generate after-tax cash flows of...

See Answer

Q: You are examining the viability of a capital investment in which your

You are examining the viability of a capital investment in which your firm is interested. The project will require an initial investment of $500,000 and the projected revenues are $400,000 a year for...

See Answer

Q: A business in the 45% tax bracket is considering borrowing money

A business in the 45% tax bracket is considering borrowing money at 10%. a. What is the after-tax interest rate on the debt? b. What is the after-tax interest rate if only half of the interest expense...

See Answer

Q: Intel has an EBIT of $3.4 billion and faces

Intel has an EBIT of $3.4 billion and faces a marginal tax rate of 36.50%. It currently has $1.5 billion in debt out- standing, and a market value of equity of $51 billion. The beta for the stock is 1...

See Answer

Q: Assume now that you have been asked to forecast cash flows that

Assume now that you have been asked to forecast cash flows that you will have available to repurchase stock and pay dividends during the next five years for Conrail (from Problem 18). In making these...

See Answer

Q: You have been asked to assess whether Walgreen, a drugstore chain

You have been asked to assess whether Walgreen, a drugstore chain, is correctly priced relative to its competitors in the drugstore industry. The following are the price/sales ratios, profit margins,...

See Answer

Q: You are trying to estimate the beta of a private firm that

You are trying to estimate the beta of a private firm that manufactures home appliances. You have managed to obtain betas for publicly traded firms that also manufacture home appliances. The private f...

See Answer

Q: Your company is considering producing a new product. You have a

Your company is considering producing a new product. You have a production facility that is currently used to only 50% of capacity, and you plan to use some of the excess capacity for the new product....

See Answer

Q: WestingHome is a manufacturing company that has accumulated a net operating loss

WestingHome is a manufacturing company that has accumulated a net operating loss of $2 billion over time. It is considering borrowing $5 billion to acquire another company. a. Based on the corporate t...

See Answer

Q: MiniSink is a manufacturing company that has $100 million in debt

MiniSink is a manufacturing company that has $100 million in debt outstanding and 9 million shares trading at $100 per share. The current beta is 1.10, and the interest rate on the debt is 8%. In the...

See Answer

Q: NYNEX, the phone utility for the New York City area,

NYNEX, the phone utility for the New York City area, has approached you for advice on its capital structure. In 1995, NYNEX had debt outstanding of $12.14 billion and equity outstanding of $20.55 bill...

See Answer

Q: Cracker Barrel, which operates restaurants and gift stores, is reexamining

Cracker Barrel, which operates restaurants and gift stores, is reexamining its policy of paying minimal dividends. In 1995, Cracker Barrel reported net income of $66 million; it had capital expenditur...

See Answer

Q: Time Warner is considering a sale of its publishing division. The

Time Warner is considering a sale of its publishing division. The division had earnings EBITDA of $550 million in the most recent year (depreciation was $150 million), growing at an estimated 5% a yea...

See Answer

Q: You have been asked to analyze a project where the analyst has

You have been asked to analyze a project where the analyst has estimated the return on capital to be 37% over the ten-year lifetime of the project. The cost of capital is only 12%, but you have concer...

See Answer

Q: As the result of stockholder pressure, RJR Nabisco is considering spinning

As the result of stockholder pressure, RJR Nabisco is considering spinning off its food division. You have been asked to estimate the beta for the division and decide to do so by obtaining the beta of...

See Answer

Q: You are an analyst for a sporting goods corporation that is considering

You are an analyst for a sporting goods corporation that is considering a new project that will take advantage of excess capacity in an existing plant. The plant has a capacity to produce 50,000 tenni...

See Answer

Q: Answer true or false to the following questions relating to the free

Answer true or false to the following questions relating to the free cash flow hypothesis (as developed by Jensen). a. Companies with high operating earnings have high free cash flows. b. Companies wi...

See Answer

Q: 21. A small, private firm has approached you for advice

21. A small, private firm has approached you for advice on its capital structure decision. It is in the specialty retailing business, and it had an EBIT last year of $500,000. • The book value of equi...

See Answer

Q: Assume that Cracker Barrel, from Problem 20, wants to continue

Assume that Cracker Barrel, from Problem 20, wants to continue with its policy of not paying dividends. You are the CEO of Cracker Barrel and have been confronted by dissident stockholders, demanding...

See Answer

Q: Accounting rates of return are based on accounting income and book value

Accounting rates of return are based on accounting income and book value of investment, whereas internal rates of return are based on cash flows and take into account the time value of money. Under wh...

See Answer

Q: You are comparing the dividend policies of three dividend-paying utilities

You are comparing the dividend policies of three dividend-paying utilities. You have collected the following information on the ex-dividend behavior of these firms. If you were a tax-exempt investor,...

See Answer

Q: Southwestern Bell, a phone company, is considering expanding its operations

Southwestern Bell, a phone company, is considering expanding its operations into the media business. The beta for the company at the end of 1995 was 0.90, and the debt-to-equity ratio was 1. The media...

See Answer

Q: XCV, Inc., which manufactures automobile parts for assembly, is

XCV, Inc., which manufactures automobile parts for assembly, is considering the costs and the benefits of leverage. The CFO notes that the return on equity of the firm, which is only 12.75% now based...

See Answer

Q: Manpower, which provides nongovernment employment services in the United States,

Manpower, which provides nongovernment employment services in the United States, reported net income of $128 million in 1995. It had capital expenditures of $50 million and depreciation of $24 million...

See Answer

Q: The chief financial officer of Adobe Systems, a software manufacturing firm

The chief financial officer of Adobe Systems, a software manufacturing firm, has approached you for some advice regarding the beta of his company. He subscribes to a service that estimates Adobe Syste...

See Answer

Q: Assess the likelihood that the following firms will be taken over,

Assess the likelihood that the following firms will be taken over, based on your understanding of the free cash flow hypothesis. You can assume that earnings and free cash flows are highly correlated....

See Answer

Q: Nadir, an unlevered firm, has expected earnings before interest and

Nadir, an unlevered firm, has expected earnings before interest and taxes of $2 million per year. Nadir’s tax rate is 40%, and the market value is V = E = $12 million. The stock has...

See Answer

Q: How would your answers to the previous problem change if Manpower is

How would your answers to the previous problem change if Manpower is in plans to pay off its outstanding debt of $100 million next year and become a debt-free company?

See Answer

Q: You are analyzing Tiffany’s, an upscale retailer, and find that

You are analyzing Tiffany’s, an upscale retailer, and find that the regression estimate of the firm’s beta is 0.75; the standard error for the beta estimate is 0.50. You also note that the average unl...

See Answer

Q: You are an institutional investor and have collected the following information on

You are an institutional investor and have collected the following information on five maritime firms to assess their dividend policies The average risk-free rate during the period was 7%, and the ave...

See Answer

Q: A firm that has no debt has a market value of $

A firm that has no debt has a market value of $100 million and a cost of equity of 11%. In the Miller–Modigliani world. a. what happens to the value of the firm as the leverage is changed (assume no t...

See Answer

Q: Between 1988 and 2013, we saw an increase in the percentage

Between 1988 and 2013, we saw an increase in the percentage of cash returned to stockholders in the form of dividends. Why?

See Answer

Q: You are analyzing the dividend policy of Black and Decker, a

You are analyzing the dividend policy of Black and Decker, a manufacturer of tools and appliances. The following table summarizes the dividend payout ratios, yields, and expected growth rates of other...

See Answer

Q: Assume that personal investors pay a 40% tax rate on interest

Assume that personal investors pay a 40% tax rate on interest income and only a 20% tax rate on equity income. If the corporate tax rate is 30%, estimate whether debt has a tax benefit, relative to eq...

See Answer

Q: Handy and Harman, a leading fabricator of precious metal alloys,

Handy and Harman, a leading fabricator of precious metal alloys, pays out only 23% of its earnings as dividends. The average dividend payout ratio for metal fabricating firms is 45%. The average growt...

See Answer

Q: XYZ Pharma is a pharmaceutical company that traditionally has not used debt

XYZ Pharma is a pharmaceutical company that traditionally has not used debt to finance its projects. Over the past 10 years, it has also reported high returns on its projects and growth and made subst...

See Answer

Q: Unitrode, which makes analog/linear integrated circuits for power management

Unitrode, which makes analog/linear integrated circuits for power management, is a firm that has not used debt in the financing of its projects. The managers of the firm contend that they do not borro...

See Answer

Q: Consolidated Power is a regulated electric utility that has equity with a

Consolidated Power is a regulated electric utility that has equity with a market value of $1.5 billion and debt outstanding of $3 billion. A consultant notes that this is a high debt ratio relative to...

See Answer

Q: In December 1995, Boise Cascade’s stock had a beta of 0

In December 1995, Boise Cascade’s stock had a beta of 0.95. The Treasury bill rate at the time was 5.8%, and the Treasury bond rate was 6.4%. The firm had debt out- standing of $1.7 billion and a mark...

See Answer

Q: Stockholders can transfer wealth from bondholders through a variety of actions.

Stockholders can transfer wealth from bondholders through a variety of actions. How would the following actions by stockholders transfer wealth from bondholders? a. An increase in dividends b. A lever...

See Answer

Q: The following table summarizes the annual returns you would have made on

The following table summarizes the annual returns you would have made on two companies—Scientific Atlanta, a satellite and data equipment manufacturer, and AT&T, the telecomm gia...

See Answer

Q: Biogen, a biotechnology firm, had a beta of 1.

Biogen, a biotechnology firm, had a beta of 1.70 in 1995. It had no debt outstanding at the end of that year. a. Estimate the cost of equity for Biogen, if the Treasury bond rate is 6.4%. b. What effe...

See Answer

Q: Consider again the project described in Problem 1 (assume that the

Consider again the project described in Problem 1 (assume that the depreciation reverts to a straight line). Assume that 40% of the initial investment for the project will be financed with debt, with...

See Answer

Q: You own a rental building in the city and are interested in

You own a rental building in the city and are interested in replacing the heating system. You are faced with the following alternatives: a. A solar heating system, which will cost $12,000 to install a...

See Answer

Q: You are analyzing a new security that has been promoted as equity

You are analyzing a new security that has been promoted as equity, with the following features: • The dividend on the security is fixed in dollar terms for the life of the security, which is 20 years....

See Answer

Q: Plastico is considering a major change in its capital structure. It

Plastico is considering a major change in its capital structure. It has three options: • Option 1: Issue $1 billion in new stock and repurchase half of its outstanding debt. This will make it an AAA-r...

See Answer

Q: IOU has $5 billion in debt outstanding (carrying an interest

IOU has $5 billion in debt outstanding (carrying an interest rate of 9%) and 10 million shares trading at $50 per share. Based on its current EBIT of $200 million, its optimal debt ratio is only 30%....

See Answer

Q: Stock prices are much too volatile for financial markets to be efficient

Stock prices are much too volatile for financial markets to be efficient. Comment.

See Answer

Q: You are in a world where there are only two assets,

You are in a world where there are only two assets, gold and stocks. You are interested in investing your money in one, the other, or both assets. Consequently, you collect the following data on the r...

See Answer

Q: You have been given the following information on a project:

You have been given the following information on a project: • It has a five-year lifetime • The initial investment in the project will be $25 million, and the investment will be depreciated straight l...

See Answer

Q: Genting Berhad is a Malaysian conglomerate with holdings in plantations and tourist

Genting Berhad is a Malaysian conglomerate with holdings in plantations and tourist resorts. The beta estimated for the firm, relative to the Malaysian stock exchange, is 1.15, and the long-term local...

See Answer

Q: Answer true or false to the following statements: a.

Answer true or false to the following statements: a. The return on equity for a project will always be higher than the return on capital on the same project. b. If the return on capital is less than t...

See Answer

Q: Plastico is interested in how it compares with its competitors in the

Plastico is interested in how it compares with its competitors in the same industry. a. Taking each of these variables, explain at an intuitive level whether you would expect Plastico to have more or...

See Answer

Q: DGF Corporation has come to you for some advice on how best

DGF Corporation has come to you for some advice on how best to increase their leverage over time. In the most recent year, DGF had an EBITDA of $300 million, owed $1 billion in both book value and mar...

See Answer

Q: Cell Phone is a cellular firm that reported net income of $

Cell Phone is a cellular firm that reported net income of $50 million in the most recent financial year. The firm had $1 billion in debt, on which it reported interest expenses of $100 million in the...

See Answer

Q: Lube Oil, in Question 3, paid a dividend of $

Lube Oil, in Question 3, paid a dividend of $20 million and bought back $25 million in stock. Estimate how much the cash balance of the firm changed during the year.

See Answer

Q: You are analyzing a convertible preferred stock with the following characteristics for

You are analyzing a convertible preferred stock with the following characteristics for the security: • There are 50,000 preferred shares outstanding, with a face value of $100 and a 6% preferred divid...

See Answer

Q: Maximizing stock prices does not make sense because investors focus on short

Maximizing stock prices does not make sense because investors focus on short-term results and not on the long-term consequences. Comment.

See Answer

Q: You have just done a regression of monthly stock re- turns

You have just done a regression of monthly stock re- turns of HeavyTech, a manufacturer of heavy machinery, on monthly market returns over the past five years and come up with the following regression...

See Answer

Q: You have been asked to calculate the debt ratio for a firm

You have been asked to calculate the debt ratio for a firm that has the following components to its financing mix: • The firm has 1 million shares outstanding, trading at $50 per share. • The firm has...

See Answer

Q: A small manufacturing firm, which has limited access to capital,

A small manufacturing firm, which has limited access to capital, has a capital rationing constraint of $150 million and is faced with the following investment projects(numbers in millions): a. Which o...

See Answer

Q: As CEO of a major corporation, you have to make a

As CEO of a major corporation, you have to make a decision on how much you can afford to borrow. You currently have 10 million shares outstanding, and the market price per share is $50. You also curre...

See Answer

Q: STL has asked you for advice on putting together the details of

STL has asked you for advice on putting together the details of the new debt issues it is planning to make. What information would you need to obtain to provide this advice?

See Answer

Q: Netsoft is a company that manufactures networking software. In the current

Netsoft is a company that manufactures networking software. In the current year, the firm reported operating earnings before interest and taxes of $200 million (operating earnings does not include int...

See Answer

Q: Under what conditions will the return on equity on a project be

Under what conditions will the return on equity on a project be equal to the IRR, estimated from cash flows to equity investors, on the same project?

See Answer

Q: UJ Gas is a utility that has followed a policy of increasing

UJ Gas is a utility that has followed a policy of increasing dividends every quarter by 5% over dividends in the prior year. The company announces that it will increase quarterly dividends from $1.00...

See Answer

Q: How would your answers to the last two questions change if you

How would your answers to the last two questions change if you were told that Lube Oil started the year with $120 million in debt and ended the year with $135 million?

See Answer

Q: There are some corporate strategists who have suggested that firms focus on

There are some corporate strategists who have suggested that firms focus on maximizing market share rather than market prices. When might this strategy work, and when might it fail?

See Answer

Q: You are interested in creating a portfolio of two stocks—Coca

You are interested in creating a portfolio of two stocks—Coca Cola and Texas Utilities. Over the last decade, an investment in Coca Cola stock would have earned an average annual return of 25%, with a...

See Answer

Q: Assume that you have half of your money invested in Times Mirror

Assume that you have half of your money invested in Times Mirror, the media company, and the other half invested in Unilever, the consumer product giant. The expected returns and standard deviations o...

See Answer

Q: You are provided with the projected income statements for a project:

You are provided with the projected income statements for a project: • The tax rate is 40%. • The project required an initial investment of $15,000and an additional...

See Answer

Q: An income bondholder receives interest payments only if the firm makes income

An income bondholder receives interest payments only if the firm makes income. If the firm does not make interest payments in a year, the interest is cumulated and paid in the first year the firm make...

See Answer

Q: You have been hired as a capital budgeting analyst by a sporting

You have been hired as a capital budgeting analyst by a sporting goods firm that manufactures athletic shoes and has captured 10% of the overall shoe market (the total market is worth $100 million a y...

See Answer

Q: You have been asked to estimate the debt ratio for a firm

You have been asked to estimate the debt ratio for a firm with the following financing details: • The firm has two classes of shares outstanding: 50,000 shares of class A stock, with 2 voting rights p...

See Answer

Q: You have been hired as a management consultant by AD Corporation to

You have been hired as a management consultant by AD Corporation to evaluate whether it has an appropriate amount of debt (the company is worried about a leveraged buyout). You have collected the foll...

See Answer

Q: Assume now that you have uncovered the following facts about the types

Assume now that you have uncovered the following facts about the types of projects STL takes: • The projects are primarily infrastructure projects, requiring large initial investments and long gestati...

See Answer

Q: Now assume that Lube Oil has a return on equity of 5

Now assume that Lube Oil has a return on equity of 5% and a cost of equity of 10%. As a stockholder in Lube Oil, would you want the firm to change its dividend policy? Why or why not?

See Answer

Q: Gemco Jewelers earned $5 million in after-tax operating income

Gemco Jewelers earned $5 million in after-tax operating income in the most recent year. The firm also had capital expenditures of $4 million and depreciation of $2 million during the year, and the non...

See Answer

Q: Geotech Inc., which has had a history of high growth and

Geotech Inc., which has had a history of high growth and pays no dividends, announces that it will start paying dividends next quarter. How would you expect its stock price to react to the announcemen...

See Answer

Q: Antitakeover amendments can be in the best interests of stockholders. Under

Antitakeover amendments can be in the best interests of stockholders. Under what conditions is this likely to be true?

See Answer

Q: You are considering the possibility of replacing an existing machine that has

You are considering the possibility of replacing an existing machine that has a book value of $500,000, a remaining depreciable life of five years, and a salvage value of $300,000. The replacement mac...

See Answer

Q: UB is examining its capital structure with the intent of arriving at

UB is examining its capital structure with the intent of arriving at an optimal debt ratio. It currently has no debt and has a beta of 1.5. The riskless interest rate is 9%. Your research indicates th...

See Answer

Q: Plastico, a manufacturer of consumer plastic products, is evaluating its

Plastico, a manufacturer of consumer plastic products, is evaluating its capital structure. The balance sheet of the company is as follows (in millions): In addition, you are provided the following in...

See Answer

Q: You are attempting to structure a debt issue for Eaton Corporation,

You are attempting to structure a debt issue for Eaton Corporation, a manufacturer of automotive components. You have collected the following information on the market values of debt and equity for th...

See Answer

Q: JC Automobiles is a small auto parts manufacturing firm that has paid

JC Automobiles is a small auto parts manufacturing firm that has paid $1.00 in annual dividends each year for the past five years. It announces that dividends will increase to $1.25 next year. What wo...

See Answer

Q: Now assume that Gemco Jewelers has $10 million in cash and

Now assume that Gemco Jewelers has $10 million in cash and non-operating assets and that the firm has $15 million in outstanding debt. a. Estimate the value of equity in the firm. b. If the firm has 5...

See Answer

Q: Companies outside the United States often have two classes of stock outstanding

Companies outside the United States often have two classes of stock outstanding. One class of shares is voting and is held by the incumbent managers of the firm. The other class is nonvoting and repre...

See Answer

Q: Consider the project described in Problem 6. Assume that the firm

Consider the project described in Problem 6. Assume that the firm plans to finance 40% of its net capital expenditure and working capital needs with debt. a. Estimate the cash flow to equity for each...

See Answer

Q: You are analyzing the beta for Hewlett Packard and have broken down

You are analyzing the beta for Hewlett Packard and have broken down the company into four broad business groups, with market values and betas for each group. a. Estimate the beta for Hewlett Packard a...

See Answer

Q: You are provided with the following cash flows on a project:

You are provided with the following cash flows on a project: Plot the net present value (NPV) profile for this project. What is the IRR? If this firm had a cost of capital of 10% and a cost of equity...

See Answer

Q: You are helping a bookstore decide whether it should open a coffee

You are helping a bookstore decide whether it should open a coffee shop on the premises. The details of the investment are as follows: • The coffee shop will cost $50,000 to open; it will have a five-...

See Answer

Q: Office Helpers is a private firm that manufactures and sells office supplies

Office Helpers is a private firm that manufactures and sells office supplies. The firm has limited capital and is estimated to have a value of $80 million with the capital constraints. A venture capit...

See Answer

Q: GenCorp, an automotive parts manufacturer, currently has $25 million

GenCorp, an automotive parts manufacturer, currently has $25 million in outstanding debt and has 10 million shares outstanding. The book value per share is $10, and the market price per share is $25....

See Answer

Q: BMD is a firm with no debt on its books currently anda

BMD is a firm with no debt on its books currently anda market value of equity of $2 billion. On the basis of its EBITDA of $200 million, it can afford to have a debt ratio of 50%, at which level the f...

See Answer

Q: Repeat the analysis in Problem 7 for a private firm that has

Repeat the analysis in Problem 7 for a private firm that has provided you with the following estimates of operating income for the 10 years, for which you have the macroeconomic data:

See Answer

Q: Union Pacific Railroad reported net income of $770 million after interest

Union Pacific Railroad reported net income of $770 million after interest expenses of $320 million in a recent financial year. (The corporate tax rate was 36%.) It reported depreciation of $960 millio...

See Answer

Q: Tech Products, from Question 7, pays a dividend of $

Tech Products, from Question 7, pays a dividend of $40 million. Assuming that the firm started the period with no cash, how did it raise the funding for the dividend payment?

See Answer

Q: Would your answer be different for the previous problem if JC Automobiles

Would your answer be different for the previous problem if JC Automobiles were a large firm followed by thirty-five analysts? Why or why not?

See Answer

Q: In recent years, top managers have been given large packages of

In recent years, top managers have been given large packages of options, giving them the right to buy stock in the firm at a fixed price. Will these compensation schemes make managers more responsive...

See Answer

Q: Assume that the average variance of return for an individual security is

Assume that the average variance of return for an individual security is 50% and that the average covariance is 10%. What is the expected variance of a portfolio of 5, 10, 20, 50, and 100 securities....

See Answer

Q: The following table summarizes the percentage changes in operating income, percentage

The following table summarizes the percentage changes in operating income, percentage changes in revenue, and betas for four pharmaceutical firms. a. Calculate the degree of operating leverage for eac...

See Answer

Q: You have estimated the following cash flows on a project:

You have estimated the following cash flows on a project: Plot the NPV profile for this project. What is the IRR? If the cost of equity is 16%, would you accept this project?

See Answer

Q: The lining of a plating tank must be replaced every three years

The lining of a plating tank must be replaced every three years at the cost of approximately $2,000. A new lining material has been developed that is more resistant to the corrosive effects of the pla...

See Answer

Q: Assume now that Office Helpers decides to go public and would like

Assume now that Office Helpers decides to go public and would like to offer its shares at a target price of $10 per share. If the IPO is likely to be underpriced by 20%, how many shares should the fir...

See Answer

Q: Vernon Enterprises has current after-tax operating income of $100

Vernon Enterprises has current after-tax operating income of $100 million and a cost of capital of 10%. The firm earns a return on capital equal to its cost of capital. a. Assume that the firm is in s...

See Answer

Q: You have been called in as a consultant for Herbert’s a sporting

You have been called in as a consultant for Herbert’s a sporting goods retail firm, which is examining its debt policy. The firm currently has a balance sheet as follows: The firm&ac...

See Answer

Q: Assuming that you do the analysis in Problem 8 with both firm

Assuming that you do the analysis in Problem 8 with both firm value and operating income, what are the reasons for the differences you might find in the results, using each? When would you use one ove...

See Answer

Q: WeeMart, a retailer of children’s clothes, announces a cut in

WeeMart, a retailer of children’s clothes, announces a cut in dividends following a year in which both revenues and earnings dropped significantly. How would you expect its stock price to react? Expla...

See Answer

Q: Lockheed, one of the largest defense contractors in the United States

Lockheed, one of the largest defense contractors in the United States, reported EBITDA of $1,290 million in a recent financial year, prior to interest expenses of $215 million and depreciation charges...

See Answer

Q: New Age Telecomm is a young, high-growth telecommunications firm

New Age Telecomm is a young, high-growth telecommunications firm. It pays no dividends, although the average dividend payout for other firms in the telecommunications sector is 40%. Is New Age paying...

See Answer

Q: Reader’s Digest has voting and nonvoting shares. About 70% of

Reader’s Digest has voting and nonvoting shares. About 70% of the voting shares are held by charitable institutions, which are headed by the CEO of Reader’s Digest. Assume that you are a large holder...

See Answer

Q: Estimate the MIRR for the project described in Problem 8. Does

Estimate the MIRR for the project described in Problem 8. Does it change your decision on accepting this project?

See Answer

Q: You are a venture capitalist and have been approached by Cirrus Electronics

You are a venture capitalist and have been approached by Cirrus Electronics, a private firm. The firm has no debt outstanding and does not have earnings now but is expected to be earning $15 million i...

See Answer

Q: Terck, a leading pharmaceutical company, currently has a balance sheet

Terck, a leading pharmaceutical company, currently has a balance sheet that is as follows: The firm’s income statement looks as follows: The firm’s bonds are all 20...

See Answer

Q: Pfizer, a major pharmaceutical company, has a debt ratio of

Pfizer, a major pharmaceutical company, has a debt ratio of 10.30% and is considering increasing its debt ratio to 30%. Its cost of capital is expected to drop from 14.51% to 13.45%. Pfizer had an EBI...

See Answer

Q: Stock buybacks really do not return cash to stockholders because only those

Stock buybacks really do not return cash to stockholders because only those who sell back stock receive the cash. Is this statement true or false? Explain.

See Answer

Q: RJR Nabisco, in response to stockholder pressure in 1996, announced

RJR Nabisco, in response to stockholder pressure in 1996, announced a significant increase in dividends paid to stockholders financed by the sale of some of its assets. What would you expect the stock...

See Answer

Q: In the face of disappointing earnings results and increasingly assertive institutional stockholders

In the face of disappointing earnings results and increasingly assertive institutional stockholders, Eastman Kodak was considering the sale of its health division, which earned $560 million in EBIT in...

See Answer

Q: A prominent beta estimation service reports the beta of Comcast Corporation,

A prominent beta estimation service reports the beta of Comcast Corporation, a major cable TV operator, to be 1.45. The service claims to use weekly returns on the stock over the prior five years and...

See Answer

Q: In Germany, large banks are often large lenders and large equity

In Germany, large banks are often large lenders and large equity investors in the same firm. For instance, Deutsche Bank is the largest stockholder in Daimler Chrysler, as well as its largest lender....

See Answer

Q: Every investor in the capital asset pricing model owns a combination of

Every investor in the capital asset pricing model owns a combination of the market portfolio and a riskless asset. Assume that the standard deviation of the market portfolio is 30% and that the expect...

See Answer

Q: You are analyzing two mutually exclusive projects with the following cash flows

You are analyzing two mutually exclusive projects with the following cash flows: a. Estimate the NPV of each project, assuming a cost of capital of 10%. Which is the better project? b. Estimate the IR...

See Answer

Q: You have been asked to compare three alternative investments and make a

You have been asked to compare three alternative investments and make a recommendation. • Project A has an initial investment of $5 million and after-tax cash flows of $2.5 million a year for the next...

See Answer

Q: The unlevered beta of electronics firms, on average, is 1

The unlevered beta of electronics firms, on average, is 1.1. The riskless rate is 6.5%, and the market risk premium is 6%. a. Estimate the expected return, using the CAPM. b. If you are a venture capi...

See Answer

Q: You have been asked to analyze the capital structure of DASA,

You have been asked to analyze the capital structure of DASA, an environmental waste disposal firm, and make recommendations on a future course of action. DASA has 40 million shares outstanding, selli...

See Answer

Q: Upjohn, also a major pharmaceutical company, is considering increasing its

Upjohn, also a major pharmaceutical company, is considering increasing its debt ratio from 11% to 40%, which is its optimal debt ratio. Its beta is 1.17, and the current Treasury bond rate is 6.50%. T...

See Answer

Q: The following table lists returns on the market portfolio and on Microsoft

The following table lists returns on the market portfolio and on Microsoft, each year from 1989 to 1998. a. Estimate the covariance in returns between Microsoft and the market portfolio. b. Estimate t...

See Answer

Q: You are trying to choose a new siding for your house.

You are trying to choose a new siding for your house. A salesman offers you two choices: a. Wood siding, which will last 10 years and cost $5,000to install and $1,000/year to maintain. b. Aluminum sid...

See Answer

Q: You have just been approached by a magazine with an offer for

You have just been approached by a magazine with an offer for renewing your subscription. You can renew for one year at $20, two years for $36, or three years at $45. Assuming that you have an opportu...

See Answer

Q: Safecorp, which owns and operates grocery stores across the United States

Safecorp, which owns and operates grocery stores across the United States, currently has $50 million in debt and $100 million in equity outstanding. Its stock has a beta of 1.2. It is planning a lever...

See Answer

Q: Zycor Corporation obtains most of its funding internally. Assume that the

Zycor Corporation obtains most of its funding internally. Assume that the stock has a beta of 1.2, the riskless rate is 6.5%, and the market risk premium is 6%. a. Estimate the cost of internal equity...

See Answer

Q: Tech Products reported a net loss of $80 million for the

Tech Products reported a net loss of $80 million for the latest financial year. In addition, the firm reported a net capital expenditure of $70 million and a change in non- cash working capital of $10...

See Answer

Q: Novell, which had a market value of equity of $2

Novell, which had a market value of equity of $2 billion and a beta of 1.50, announced that it was acquiring Word- Perfect, which had a market value of equity of $1 billion and a beta of 1.30. Neither...

See Answer

Q: You have been asked to estimate a Markowitz portfolio across a universe

You have been asked to estimate a Markowitz portfolio across a universe of 1,250 assets. a. How many expected returns and variances would you need to compute? b. How many covariances would you nee...

See Answer

Q: The following is a regression of dividend payout ratios on the risk

The following is a regression of dividend payout ratios on the risk and ln(market capitalization: in millions) of chemical firms: Dividend Payout Ratio = 0.14 + 0.05 [ln(Market Capitalization in Mi...

See Answer

Q: You are a small business owner considering two alternatives for your phone

You are a small business owner considering two alternatives for your phone system. The discount rate is 8%. Which alternative would you pick?  

See Answer

Q: Sunshine Media has just completed an IPO, where 50 million shares

Sunshine Media has just completed an IPO, where 50 million shares of the 125 million shares outstanding were issued to the public at an offering price of $22 per share. On the offering date, the sto...

See Answer

Q: You have been asked to analyze the standard deviation of a portfolio

You have been asked to analyze the standard deviation of a portfolio composed of the following three assets: You have also been provided with the correlations across these three investments:...

See Answer

Q: You are the manager of a pharmaceutical company and are considering what

You are the manager of a pharmaceutical company and are considering what type of laptop computers to buy for your salespeople to take with them on their calls. • You can buy fairly inexpensive (and l...

See Answer

Q: United Airlines has a beta of 1.50. The standard

United Airlines has a beta of 1.50. The standard deviation in the market portfolio is 22% and United Airlines has a standard deviation of 66%. a. Estimate the correlation between United Airlines an...

See Answer

Q: National City, a bank holding company, reported earnings per share

National City, a bank holding company, reported earnings per share of $2.40 and paid dividends per share of $1.06. The earnings had grown 7.5% a year over the prior five years, and were expected to...

See Answer

Q: You are using the multifactor model to estimate the expected return on

You are using the multifactor model to estimate the expected return on Emerson Electric and have derived the following estimates for the factor betas and risk premia: With a riskless rate of 6%...

See Answer

Q: You are stockholder in a SmallTech, a company that is planning

You are stockholder in a SmallTech, a company that is planning to raise new equity. The stock is trading at $15 per share, and there are 1 million shares outstanding. The firm issues 500,000 rights...

See Answer

Q: You are the owner of a small hardware store, and you

You are the owner of a small hardware store, and you are considering opening a gardening store in a vacant area in the back of your present store. You estimate that it will cost you $50,000 to set u...

See Answer

Q: The following regression was run using all NYSE firms in 1995

The following regression was run using all NYSE firms in 1995 YIELD = 0.0478 − 0.0157 BETA + 0.0000008 MKTCAP + 0.006797 DBTRATIO + 0.0002 ROE − 0.09 NCEX∕TA R 2 = 12.88% Where BET...

See Answer

Q: In the illustration in Problem 25, what would the tax rate

In the illustration in Problem 25, what would the tax rate on equity income need to be for debt to not have an effect on value?  

See Answer

Q: Cello is a manufacturer of pianos. It earned an after-

Cello is a manufacturer of pianos. It earned an after-tax return on capital of 10% last year and expects to maintain this return next year. If the current year’s after-tax operating income is...

See Answer

Q: Assume you have all your wealth (a million dollars) invested

Assume you have all your wealth (a million dollars) invested in the Vanguard 500 index fund, and that you expect to earn an annual return of 12%, with a standard deviation in returns of 25%. Since y...

See Answer

Q: If Consolidated Power is priced at $50.00 with dividend

If Consolidated Power is priced at $50.00 with dividend, and its price falls to $46.50 when a dividend of $5.00 is paid, what is the implied marginal rate of personal taxes for its stockholders? Ass...

See Answer

Q: Assume in the previous question with Vernon Enterprises that the firm will

Assume in the previous question with Vernon Enterprises that the firm will earn a return on capital of 15% in perpetuity. a. Assume that the firm is in stable growth, growing 5% a year forever; est...

See Answer

Q: Lube Oil, a chain of automobile service stations, reports net

Lube Oil, a chain of automobile service stations, reports net income of $100 million after depreciation of $50 million. The firm has capital expenditures of $80 mil- lion, and the noncash working capi...

See Answer

Q: Southern Rail has just declared a dividend of $1. The

Southern Rail has just declared a dividend of $1. The average investor in Southern Rail faces an ordinary tax rate of 50%. Although the capital gains rate is also 50%, it is believed that the investor...

See Answer

Q: LMN Corporation, a real estate company, is planning to pay

LMN Corporation, a real estate company, is planning to pay a dividend of $0.50 per share. Most of the investors in LMN are other corporations that pay 40% of their ordinary income and 28% of their cap...

See Answer

Q: Imagine a decision task in which you are to choose between two

Imagine a decision task in which you are to choose between two alternatives that involve blindly drawing a single chip from one of two urns, labeled A and B respectively. Both urns contain colored bal...

See Answer

Q: Excel has a function, BINOM.DIST, for the binomial

Excel has a function, BINOM.DIST, for the binomial probability distribution. This function computes the probability of drawing x successes in n independent trials when the probability of drawing a suc...

See Answer

Q: Discuss the psychological aspects associated with how Groupon’s board and executives analyzed

Discuss the psychological aspects associated with how Groupon’s board and executives analyzed whether or not they should accept Google’s offer.

See Answer

Q: Consider the notion of “transaction utility.” How does this concept

Consider the notion of “transaction utility.” How does this concept apply to Group on’s business strategy?

See Answer

Q: Compare the IPOs of Groupon, Facebook, and Twitter in respect

Compare the IPOs of Groupon, Facebook, and Twitter in respect to hot issue market, initial under pricing, and long term underperformance.

See Answer

Q: Given the details in the minicase, to what extent did Groupon

Given the details in the minicase, to what extent did Groupon, Facebook, and Twitter engage in either catering behavior or market timing?

See Answer

Q: Discuss the chronology of Twitter through the lens of the Baker-

Discuss the chronology of Twitter through the lens of the Baker-Wurgler sentiment framework.

See Answer

Q: Based on the information presented in the minicase, and the discussion

Based on the information presented in the minicase, and the discussion in both Chapter 2 and this chapter, to what extent does Elon Musk fit the general characteristics of an entrepreneur and a CEO, e...

See Answer

Q: Consider the manner in which the readers of Aswath Damodaran’s blog responded

Consider the manner in which the readers of Aswath Damodaran’s blog responded to his $67 valuation for Tesla, as discussed in the minicase. Analyze the psychological basis for their responses.

See Answer

Q: From the MHHE web site for Chapter 6, download the free

From the MHHE web site for Chapter 6, download the free cash flow analysis file associated with this minicase. Compare the various cash flow analyses, for both the same point in time in 2013, to ident...

See Answer

Q: Consider the possibility that the volatility of Tesla’s stock has placed some

Consider the possibility that the volatility of Tesla’s stock has placed some investors in a situation where they are averse to a sure loss. On October 22, 2015, an article on the site seekingalpha.co...

See Answer

Q: On April 19, 2016, an article that appeared on the

On April 19, 2016, an article that appeared on the web site seekingalpha.com discussed the implications of selling Tesla stock short.33 The article warns short sellers to be cautious in believing that...

See Answer

Q: For the period 2010 through 2014, the Morgan Stanley team’s 2010

For the period 2010 through 2014, the Morgan Stanley team’s 2010 report on eBay forecasted that free cash flow would on average grow by 5.7 percent. Free cash flow actually grew by an average growth r...

See Answer

Q: Based on the information presented in the minicase, how would you

Based on the information presented in the minicase, how would you characterize PSINet CEO William Schrader’s susceptibility to psychological phenomena?

See Answer

Q: Discuss the extent to which PSINet’s financing and investment decisions conform to

Discuss the extent to which PSINet’s financing and investment decisions conform to the BPV framework presented in the chapter.

See Answer

Q: Discuss the extent to which Cogent’s financing and investment decisions conform to

Discuss the extent to which Cogent’s financing and investment decisions conform to the BPV framework presented in the chapter.

See Answer

Q: Analyze the ratio that Cogent Communication uses to measure leverage, and

Analyze the ratio that Cogent Communication uses to measure leverage, and compare it to common textbook ratios measuring leverage.

See Answer

Q: What are the main issues that the traditional approach emphasizes in respect

What are the main issues that the traditional approach emphasizes in respect to Apple’s dividend policy?

See Answer

Q: What are the main issues that the behavioral approach emphasizes in respect

What are the main issues that the behavioral approach emphasizes in respect to Apple’s dividend policy?

See Answer

Q: Drawing on the discussion in Chapters 5 and 7, as well

Drawing on the discussion in Chapters 5 and 7, as well as this chapter, analyze Apple’s policy in respect to debt, repurchasing shares, and splitting its stock.

See Answer

Q: In January 2004, a multifactor regression analysis of Apple’s stock return

In January 2004, a multifactor regression analysis of Apple’s stock return, based on five years of past monthly returns, indicated that only the excess return on the market was stati...

See Answer

Q: Discuss whether there are any behavioral issues in the minicase that have

Discuss whether there are any behavioral issues in the minicase that have not arisen in connection with any of the preceding questions.

See Answer

Q: How would you assess the review conducted by Hertz’s compensation committee in

How would you assess the review conducted by Hertz’s compensation committee in 2013?

See Answer

Q: In 2010, the Morgan Stanley analysts covering eBay had stopped using

In 2010, the Morgan Stanley analysts covering eBay had stopped using PEG to value eBay, relying on P/E, P/S, and discounted values of free cash flows. In 2013, they dropped P/E and P/S, and only used...

See Answer

Q: With reference to the Hertz minicase, what psychological phenomenon or phenomena

With reference to the Hertz minicase, what psychological phenomenon or phenomena were involved in the fraud triangle?

See Answer

Q: How would you use the COSO internal control framework to assess the

How would you use the COSO internal control framework to assess the internal control environment at Hertz?

See Answer

Q: In what ways was the situation at Hertz, as described in

In what ways was the situation at Hertz, as described in the minicase, similar to that of HealthSouth, and in what ways was it different?

See Answer

Q: Which psychological phenomena might have been at play in leading Microsoft to

Which psychological phenomena might have been at play in leading Microsoft to make the offer it did to acquire Yahoo!?

See Answer

Q: The minicase makes reference to some of the examples discussed in the

The minicase makes reference to some of the examples discussed in the chapter. Discuss the connection between the minicase and the other examples with reference to how the examples are mentioned in th...

See Answer

Q: In September 2016, between the time that Yahoo! and Verizon

In September 2016, between the time that Yahoo! and Verizon negotiated the terms of their deal and the completion of the deal, Yahoo! announced its discovery that in 2014 a “state-sponsored actor” had...

See Answer

Q: Which psychological phenomena might have been at play in leading Yahoo!

Which psychological phenomena might have been at play in leading Yahoo!’s board to reject Microsoft’s offer?

See Answer

Q: Discuss the discussion of reference point heuristics in the press coverage of

Discuss the discussion of reference point heuristics in the press coverage of the Microsoft offer, and indicate whether the comments in the press are consistent with the actual merger talks between th...

See Answer

Q: Estimate the value of the synergy associated with the projected cost savings

Estimate the value of the synergy associated with the projected cost savings of $1.5 billion per year, assuming that the stream of cost savings would begin a year after the acquisition announcement, a...

See Answer

Q: Had Microsoft acquired Yahoo!, would the acquisition have been classified as

Had Microsoft acquired Yahoo!, would the acquisition have been classified as a bad merger? The technical definition of a bad acquisition can be found in the endnotes to this chapter.

See Answer

Q: The Morgan Stanley report on eBay, dated July 23, 2015

The Morgan Stanley report on eBay, dated July 23, 2015, established a one-year forward price target of $29 per share.19 At the time of the report, eBay’s stock price was $28.45, which the report noted...

See Answer

Q: Identify behavioral issues associated with Microsoft’s decisions about capital structure and capital

Identify behavioral issues associated with Microsoft’s decisions about capital structure and capital budgeting.

See Answer

Q: Discuss whether there are any behavioral issues associated with whether Microsoft pays

Discuss whether there are any behavioral issues associated with whether Microsoft pays for Yahoo! using 100 percent cash, or instead 50 percent cash and 50 percent stock.

See Answer

Q: Financial economist Aswath Damordaran was quoted in The New York Times about

Financial economist Aswath Damordaran was quoted in The New York Times about the dangers of the Steve Jobs effect, whereby CEOs seek to emulate Jobs’s success in taking Apple from being close to failu...

See Answer

Q: What does the information presented in the minicase suggest about goal setting

What does the information presented in the minicase suggest about goal setting at Toshiba?

See Answer

Q: Discuss the issue of process loss in the events described in the

Discuss the issue of process loss in the events described in the minicase.

See Answer

Q: What are the major lessons to be learned from the minicase about

What are the major lessons to be learned from the minicase about corporate governance?

See Answer

Q: Discuss any psychological phenomena that you perceive to have been at work

Discuss any psychological phenomena that you perceive to have been at work in the S&L crisis, emphasizing the implicit option features described in the case. Relate these phenomena to the issues descr...

See Answer

Q: Describe the most important factors that impeded the Packard Foundation from diversifying

Describe the most important factors that impeded the Packard Foundation from diversifying, but not the Hewlett Foundation.

See Answer

Q: George Vera, CFO of the Packard Foundation, was quoted as

George Vera, CFO of the Packard Foundation, was quoted as saying: “This is a family foundation. The family makes these kinds of decisions, and the board is comfortable with this as a starting point.”...

See Answer

Q: Was there an endowment effect at work at the Packard Foundation?

Was there an endowment effect at work at the Packard Foundation?

See Answer

Q: In its April 2003 report, the Morgan Stanley team computed EBITDA

In its April 2003 report, the Morgan Stanley team computed EBITDA as “Adjusted Operating Income plus Depreciation Add back.” The adjustment in question is to “Reported Operating Income.” The report ex...

See Answer

Q: Does the chapter present any information that would lead you to conclude

Does the chapter present any information that would lead you to conclude that managers at Sun Microsystems were averse to a sure loss? Discuss.

See Answer

Q: During its most profitable years in the late 1990s, Sun did

During its most profitable years in the late 1990s, Sun did not carry any debt. In 1999, Sun paid $138.6 million in taxes. Compare the situation at Sun with the situation at Merck in respect to their...

See Answer

Q: Discuss the comment Scott McNealy made in his interview about not having

Discuss the comment Scott McNealy made in his interview about not having hired a chief operating officer.

See Answer

Q: You can find Sun’s statement of cash flows on the book web

You can find Sun’s statement of cash flows on the book web site. Do these statements suggest that Sun engaged in market timing during the technology bubble?

See Answer

Q: Did Merck’s managers exhibit confirmation bias in their assessment of Vioxx?

Did Merck’s managers exhibit confirmation bias in their assessment of Vioxx? Discuss this question.

See Answer

Q: At the beginning of 2001, Merck’s CFO Judy Lewent predicted that

At the beginning of 2001, Merck’s CFO Judy Lewent predicted that Vioxx sales for the year would be between $3 and $3.5 billion. In June she qualified her prediction to say that altho...

See Answer

Q: Merck’s VIGOR study used 8,000 subjects. Notably, Merck

Merck’s VIGOR study used 8,000 subjects. Notably, Merck chose to include only subjects whose risk of experiencing a heart attack was low. Half the subjects in the study received Vioxx, and the other h...

See Answer

Q: On November 2, 2004, The Wall Street Journal published an

On November 2, 2004, The Wall Street Journal published an article describing the manner in which Merck’s executives made decisions about Vioxx. The Wall Street Journal article pointed out that the Nov...

See Answer

Q: Compare two positions about Merck’s handling of Vioxx. The first position

Compare two positions about Merck’s handling of Vioxx. The first position is that Merck should have warned patients about the enhanced risk of heart attack or stroke (after prolonged use of the drug)...

See Answer

Q: On May 20, 2004, The Wall Street Journal ran a

On May 20, 2004, The Wall Street Journal ran a front page story entitled “Biotech’s Dismal Bottom Line: More Than $40 Billion in Losses.” The article makes several points. First, the majority of biote...

See Answer

Q: In the section of the book web site for this chapter,

In the section of the book web site for this chapter, you will find an Excel file containing counterparts to the free cash flow table in Exhibit 3-3, but for forecasts and valuations made in 2010 and...

See Answer

Q: Despite the growing popularity of cellular phones during the middle and late

Despite the growing popularity of cellular phones during the middle and late 1990s, Iridium undertook a $180 million promotional campaign to launch its product. It ran advertisements in The Wall Stree...

See Answer

Q: The column “Ahead of the Tape” that appeared in the

The column “Ahead of the Tape” that appeared in the February 13, 2004, issue of The Wall Street Journal states that prudent investors prefer to value firms using free cash flow instead of EBITDA. The...

See Answer

Q: Analyst Safa Rashtchy developed his 2010 forecast for eBay’s revenue by assuming

Analyst Safa Rashtchy developed his 2010 forecast for eBay’s revenue by assuming that its annual growth would be about a 30 percent compounded annual growth rate between 2002 and 2010. In the previous...

See Answer

Q: The March 11, 2004, issue of Businessweek magazine reported that

The March 11, 2004, issue of Businessweek magazine reported that Standard & Poor’s had reduced Sun Microsystem’s debt rating to junk status. Over the course of the next two days, Sun’s stock price fel...

See Answer

Q: An episode of the BBC program Horizons featured the following experiment.

An episode of the BBC program Horizons featured the following experiment. A researcher in a park with a lot of pedestrians invited passersby to examine an unopened bottle of wine, which the researcher...

See Answer

Q: Consider the responses to a survey conducted of geologists working in the

Consider the responses to a survey conducted of geologists working in the mining industry.46 The survey put the following question to the geologists: “If an economic deposit was discovered tomorrow, h...

See Answer

Q: On April 23, 2003, The Wall Street Journal published an

On April 23, 2003, The Wall Street Journal published an article suggesting that the analysts following eBay were excessively optimistic about eBay’s future revenue stream. The article pointed out that...

See Answer

Q: In the chapter discussion of the merger between HP and Compaq,

In the chapter discussion of the merger between HP and Compaq, HP director Sam Ginn initially voiced doubts about the deal. However, the McKinsey experts retorted that even a slim profit in PCs would...

See Answer

Q: HP director Patricia Dunn works in the banking industry, where consolidation

HP director Patricia Dunn works in the banking industry, where consolidation mergers have worked out well in the long run but are vulnerable in the first two years. She responded positively to the McK...

See Answer

Q: Consider a statement that begins, “I see myself as someone

Consider a statement that begins, “I see myself as someone who” Below are five phrases that might conclude this statement. Associate exactly one of the big five psychological characteristics to each p...

See Answer

Q: In the spring of 2003, analysts at Prudential established a target

In the spring of 2003, analysts at Prudential established a target price for eBay, justifying the P/E values in their analysis by appeal to PEG. Their report stated that their assumptions of a P/E of...

See Answer

Q: Answer the following six sub questions: 7.1.

Answer the following six sub questions: 7.1. Suppose you had $100 in a savings account and the interest rate was 2 percent per year. After five years, how much do you think you would have in your acco...

See Answer

Q: The Times article notes that three days after the ACP awarded a

The Times article notes that three days after the ACP awarded a $158 million contract to the Spanish firm that manufactures the tugboats, the son of the canal administrator joined the law firm represe...

See Answer

Q: In your view, how similar is the personality of Cogent Communications

In your view, how similar is the personality of Cogent Communications CEO Dave Schaeffer to the personality of PSINet CEO William Schrader?

See Answer

Q: With five family members and the former CEO of HP comprising half

With five family members and the former CEO of HP comprising half the Packard Foundation board, was the Packard board largely a homogenous group or a heterogeneous group?

See Answer

Q: Did the composition of the Packard Foundation finance committee predispose it to

Did the composition of the Packard Foundation finance committee predispose it to groupthink?

See Answer

Q: Was there an endowment effect at work at the Hewlett Foundation?

Was there an endowment effect at work at the Hewlett Foundation?

See Answer

Q: Which psychological phenomena might have been at play in the negotiations between

Which psychological phenomena might have been at play in the negotiations between Carol Bartz and Steve Ballmer?

See Answer

Q: Discuss Scott McNealy’s comments about sentiment in his interview with Fortune magazine

Discuss Scott McNealy’s comments about sentiment in his interview with Fortune magazine. In your answer, describe any valuation metric Scott McNealy mentions. Do McNealy’s comments suggest that Sun en...

See Answer

Q: In 2005, eBay CFO Rajiv Dutta stated that eBay’s investments were

In 2005, eBay CFO Rajiv Dutta stated that eBay’s investments were reflected in the firm’s income statement rather than its balance sheet. Discuss the extent to which Dutta’s perspective, in foresight,...

See Answer

Q: Consider the following excerpt from a Prudential report on Wal-Mart

Consider the following excerpt from a Prudential report on Wal-Mart, dated May 13, 2003.16 The report states: We are maintaining our Hold rating on Wal-Mart as we believe the stock’s current valuation...

See Answer

Q: In respect to the Morgan Stanley 2003 report on eBay, eBay’s

In respect to the Morgan Stanley 2003 report on eBay, eBay’s annualized geometric return between May 2003 and December 2014 was 7.9 percent, its annualized arithmetic return was 14.2 percent, and its...

See Answer

Q: Imagine yourself at a conference center, where there are two groups

Imagine yourself at a conference center, where there are two groups meeting in adjoining rooms, one a group of lawyers and the other a group of engineers. In fact, the room with lawyers contains 30 la...

See Answer

Q: In what way were the situations at Boeing (with the 787

In what way were the situations at Boeing (with the 787) and Airbus (with the A380) similar, and in what way were they different? Discuss this question through the lens of the planning fallacy.

See Answer

Q: In what way were the situations at Boeing (with the 787

In what way were the situations at Boeing (with the 787) and Motorola (with the Iridium project) the same, and in what way were they different? Discuss this question through the lens of the planning f...

See Answer

Q: In 1999 Iridium declared bankruptcy and was sold to private investors.

In 1999 Iridium declared bankruptcy and was sold to private investors. Suppose that you were to learn that by the end of 2003, Iridium had gone out of business. How surprised would you be? How would y...

See Answer

Q: Consider Robert Galvin’s approach to evaluating the satellite project proposal. The

Consider Robert Galvin’s approach to evaluating the satellite project proposal. The text suggests that in not developing discounted cash flow analysis, Galvin’s approach was flawed. In hindsight, Irid...

See Answer

Q: Consider the contention that excessive optimism and overconfidence are important characteristics of

Consider the contention that excessive optimism and overconfidence are important characteristics of leadership. Might these traits help managers initiate and complete daunting projects that they would...

See Answer

Q: One of the points made in the discussion about the construction of

One of the points made in the discussion about the construction of the Anglo-French tunnel is that engineers designed the tunnels before they designed the trains, so there were many design changes bec...

See Answer

Q: In April 2003, analysts at Morgan Stanley and Prudential had set

In April 2003, analysts at Morgan Stanley and Prudential had set 12-month target prices of $106 and $108 for the firm eBay. At that time, eBay’s stock price was $89.22 and the consensus forecast for e...

See Answer

Q: Consider the comments of Brian Walker, the president of Herman-

Consider the comments of Brian Walker, the president of Herman-Miller North America, who was quoted in the chapter as having said: “For dot.coms, it appears that the market has implicitly capitalized...

See Answer

Q: For many years, the large retail firm Walmart chose not to

For many years, the large retail firm Walmart chose not to provide guidance. The firm’s legendary founder, Sam Walton, wrote in his autobiography that he did not care what the market thought. Beginnin...

See Answer

Q: On August 19, 2004, the Internet search firm Google went

On August 19, 2004, the Internet search firm Google went public, at an offer price of $85 per share. The IPO was unconventional in that Google used an auction to determine its offer price and to sell...

See Answer

Q: Suppose that you are offered the opportunity to accept a risk involving

Suppose that you are offered the opportunity to accept a risk involving the toss of a fair coin, in which you will win $450 if heads comes up and lose $450 if tails comes up. a. Would you accept this...

See Answer

Q: In 2004, 55 percent of firms provided guidance to analysts,

In 2004, 55 percent of firms provided guidance to analysts, down from 72 percent in the prior year.30 Before Google’s IPO, the firm’s executives announced that they did not plan to issue earnings guid...

See Answer

Q: Discuss whether any of the three IPO phenomena apply in regard to

Discuss whether any of the three IPO phenomena apply in regard to the Palm IPO described in the Behavioral Pitfalls box.

See Answer

Q: Use the ideas developed in this chapter to assess whether Palm and

Use the ideas developed in this chapter to assess whether Palm and 3Com were efficiently priced on Palm’s first day of trading.

See Answer

Q: Analyze the assessment of Palm made by analyst Paul Sagawa.

Analyze the assessment of Palm made by analyst Paul Sagawa.

See Answer

Q: During a presentation in February 2001, the CFO of Palm Inc

During a presentation in February 2001, the CFO of Palm Inc. was asked how frequently her firm assesses and uses its cost of capital. In response, she stated that Palm computes its cost of capital “fr...

See Answer

Q: In 1999, the S&P 500 returned 21 percent,

In 1999, the S&P 500 returned 21 percent, closing out a streak of five consecutive stellar up-years. Then in 2000, the S&P 500 returned −9.1 percent. In 2001, the S&P 500 returned −16.1 percent. At th...

See Answer

Q: One behavioral school of thought holds that in complex situations where it

One behavioral school of thought holds that in complex situations where it is difficult to estimate probabilities, simple heuristics are generally better than complicated heuristics. Discuss this pers...

See Answer

Q: Chapter 3 contains a discussion about how the Morgan Stanley analyst team

Chapter 3 contains a discussion about how the Morgan Stanley analyst team computed a discount rate to use in its free cash flow valuation of eBay stock. Discuss any similarities between the procedure...

See Answer

Q: The Duke/CFO study on financial executives’ forecasts for the 10

The Duke/CFO study on financial executives’ forecasts for the 10-year horizon features a negative correlation between forecasts of return and forecasts of volatility. Discuss this property in light of...

See Answer

Q: In August 2004, Google went public at a price of $

In August 2004, Google went public at a price of $85 per share. One year later, its stock price reached $285, as the firm’s earnings consistently exceeded analysts’ consensus forecasts. At that time,...

See Answer

Q: Consider a trivia test consisting of 10 questions for you to answer

Consider a trivia test consisting of 10 questions for you to answer from memory alone. In addition to giving your best guess, consider a range: a low guess and a high guess so that you feel 90 percent...

See Answer

Q: From January 1990 through March 1993, the stock of Cypress Semiconductor

From January 1990 through March 1993, the stock of Cypress Semiconductor Corp. underperformed the S&P 500 by 26.5 percent on a cumulative basis. Then in April, the firm announced that its board author...

See Answer

Q: On December 19, 2000, an article appeared in The Wall

On December 19, 2000, an article appeared in The Wall Street Journal discussing stock price declines that followed share repurchases made by AT&T, Intel, Microsoft, and Hewlett-Packard.51 The article...

See Answer

Q: Imagine that AutoNation is contemplating a project that requires a $350

Imagine that AutoNation is contemplating a project that requires a $350 million initial outlay and features an NPV of $48 million. The firm is all-equity financed and has $150 million in cash that it...

See Answer

Q: Imagine that Adaptec is contemplating a project that requires a $3

Imagine that Adaptec is contemplating a project that requires a $3.75 billion initial outlay and features an NPV of $466 million. The firm is all-equity financed and has $1 billion in cash that it pla...

See Answer

Q: In 2011, Millard Drexler, the CEO of fashion retailing firm

In 2011, Millard Drexler, the CEO of fashion retailing firm J. Crew, together with two private equity firms, TPG and Leonard Green & Partners, did a $3 billion leveraged buyout. The strategy left J. C...

See Answer

Q: Solar energy firm Solyndra manufactured cylindrical solar panels that, while expensive

Solar energy firm Solyndra manufactured cylindrical solar panels that, while expensive to produce, were easy to install on the roofs of commercial buildings. The firm was founded in 2004. In March 200...

See Answer

Q: On July 23, 2002, an article entitled “Investors Appreciate

On July 23, 2002, an article entitled “Investors Appreciate Dividends Again, See Them as Safer Bets in Bear Market” appeared on Associated Press Newswires.21 The article described two reasons why fina...

See Answer

Q: Ashland Inc. is an oil services and diversified chemical company that

Ashland Inc. is an oil services and diversified chemical company that is located in Covington, Kentucky. In 2002 Ashland Inc. was paying an annual dividend of $1.10 per share and was planning to keep...

See Answer

Q: An article that appeared in The Wall Street Journal in February 2001

An article that appeared in The Wall Street Journal in February 2001 described the experiences of several investors who held dividend-paying stocks.22 The article mentions two investors, Wayne Denny a...

See Answer

Q: In 1996, Kodak paid a cash dividend of $1.

In 1996, Kodak paid a cash dividend of $1.60 per share. At year-end 1996, Kodak shares were trading at about $80 per share. Between 1997 and 2001, Kodak paid $1.76, and in 2002 raised its dividend to...

See Answer

Q: Imagine 100 book bags, each of which contains 1,000

Imagine 100 book bags, each of which contains 1,000 poker chips. Forty-five bags contain 700 black chips and 300 red chips. The other 55 bags contain 300 black chips and 700 red chips. You cannot see...

See Answer

Q: On January 31, 2005, an article appeared in The Wall

On January 31, 2005, an article appeared in The Wall Street Journal comparing the relative performance of stocks in the S&P 500 that pay dividends with stocks in the S&P 500 that do not pay dividends....

See Answer

Q: Analyze whether there are any behavioral issues in Cogent Communications’ dividend policy

Analyze whether there are any behavioral issues in Cogent Communications’ dividend policy, which is described in the minicase for Chapter 7.

See Answer

Q: Analyze whether there are any behavioral issues associated with CFO Judy Lewent’s

Analyze whether there are any behavioral issues associated with CFO Judy Lewent’s views about Merck’s dividend policy in respect to the Vioxx incident discussed in Chapter 2.

See Answer

Q: In December of 1998, energy firm British Petroleum acquired American-

In December of 1998, energy firm British Petroleum acquired American-based Amoco and became BP-Amoco. BP’s stock was listed on the London Stock Exchange and also traded on U.S markets through an Ameri...

See Answer

Q: In July 2003 Samuel Waksal, the founder of ImClone Systems Inc

In July 2003 Samuel Waksal, the founder of ImClone Systems Inc., began a seven-year prison term. In December 2001 Waksal received word that the Federal Drug Administration (FDA) was about to issue a n...

See Answer

Q: Discuss any agency conflicts associated with HealthSouth’s pristine-audit program.

Discuss any agency conflicts associated with HealthSouth’s pristine-audit program.

See Answer

Q: Eleanor Bloxham is the founder and CEO of the Value Alliance Company

Eleanor Bloxham is the founder and CEO of the Value Alliance Company and the Corporate Governance Alliance, and is a respected authority on matters involving corporate governance and valuation. In 201...

See Answer

Q: When the Internet firm eToys went public in May 1999, its

When the Internet firm eToys went public in May 1999, its CEO Toby Lenk’s stockholdings were worth $850 million on the first day the company’s stock traded on the New York Stock Exchange. Lenk is quot...

See Answer

Q: Michael Jensen developed some of the seminal ideas underlying agency theory.

Michael Jensen developed some of the seminal ideas underlying agency theory. He suggests that at the time the market value of Enron peaked at approximately $70 billion, its intrinsic value was approxi...

See Answer

Q: In 2016, automobile manufacturer Volkswagen was charged in the United States

In 2016, automobile manufacturer Volkswagen was charged in the United States for having installed software that engaged antipollution technology in diesel cars not at all times as regulations required...

See Answer

Q: Suppose that a university is attempting to predict the grade point average

Suppose that a university is attempting to predict the grade point average (GPA) of some graduating students based upon their high school GPA levels. GPA scores lie between 0 and 4. Below are some dat...

See Answer

Q: Under the purchase accounting method, an acquirer that pays more than

Under the purchase accounting method, an acquirer that pays more than the fair value for a target amortizes the difference over time on its income statement. In the 1990s, mergers between equally size...

See Answer

Q: HP executives indicate that they use traditional discounted cash flow (DCF

HP executives indicate that they use traditional discounted cash flow (DCF) analysis to evaluate investment projects and that the firm’s cost of capital is about 12 percent. Consider...

See Answer

Q: Identify any psychological phenomena that were germane in HP’s acquisition of Autonomy

Identify any psychological phenomena that were germane in HP’s acquisition of Autonomy.

See Answer

Q: A traditional counterargument to the behavioral position described in the chapter is

A traditional counterargument to the behavioral position described in the chapter is that the chapter arguments only focus on problematic acquisitions. For example, consider consumer products firm Col...

See Answer

Q: What insights are to be gleaned from the comments Steve Case made

What insights are to be gleaned from the comments Steve Case made at various times about AOL’s acquisition of Time Warner?

See Answer

Q: On February 23, 2000 MGM Grand, Inc. announced its

On February 23, 2000 MGM Grand, Inc. announced its intention to acquire Mirage Resorts, Inc. Over a three day window beginning the day before the announcement, MGM Grand’s market value fell by more th...

See Answer

Q: In May 2002, Hewlett-Packard acquired Compaq Computer in a

In May 2002, Hewlett-Packard acquired Compaq Computer in a takeover that featured considerable drama. In deliberating the acquisition, HP director Walter Hewlett, son of founder William Hewlett, sugge...

See Answer

Q: In 2001 the chief executive of AOL Time Warner, Gerald Levin

In 2001 the chief executive of AOL Time Warner, Gerald Levin, sought to acquire AT&T’s cable business, the only cable business larger than the one already owned by AOL Time Warner. In doing so, he did...

See Answer

Q: What are the main psychological challenges that Ford CEO Alan Mulally faced

What are the main psychological challenges that Ford CEO Alan Mulally faced in respect to Ford’s regular Thursday managers’ meetings, and how do these relate to issues of process and culture at the fi...

See Answer

Q: In respect to the experiences of financial firms in the lead-

In respect to the experiences of financial firms in the lead-up to the global financial crisis, for each firm identify the most important psychological phenomena that destroyed value, and wherever pos...

See Answer

Q: Consider the assumptions in Exhibit 3-2 that underlie the valuations

Consider the assumptions in Exhibit 3-2 that underlie the valuations associated with P/E, PEG, and price-to-sales. Analyze the degree to which these assumptions are mutually consistent, and correspond...

See Answer

Q: The financial instability hypothesis holds that firms take on excessive debt during

The financial instability hypothesis holds that firms take on excessive debt during periods of euphoria. Discuss the behavioral basis for this perspective, drawing if necessary from discussions in pre...

See Answer

Q: Problem 7 in Chapter 9 pertains to Volkswagen’s having installed “defeat

Problem 7 in Chapter 9 pertains to Volkswagen’s having installed “defeat devices.” In describing the issue, media reports pointed to Volkswagen’s corporate culture, which according to the New York Sta...

See Answer

Q: Discuss whether the comments of John McCormack that were quoted in Section

Discuss whether the comments of John McCormack that were quoted in Section 12.2 involve any behavioral issues.

See Answer

Q: Imagine that the current date is t1. A year ago (

Imagine that the current date is t1. A year ago (t0), the firm invested $125 million in the project depicted in the illustrative example in Section 12.5 at a time when expected cash flows were $25 mil...

See Answer

Q: Suppose that the firm just had the one project (see Question

Suppose that the firm just had the one project (see Question 2) and you were thinking about acquiring the firm. What would the fair value of the acquisition be today, under the assumption that the fir...

See Answer

Q: Section 12.5 contains a real-option example involving a

Section 12.5 contains a real-option example involving a discount rate of 10 percent. Analyze how the real-option exercise policy is affected if the required return on the project is 15 percent instead...

See Answer

Q: Make three modifications to the example in Question 4 and then reanalyze

Make three modifications to the example in Question 4 and then reanalyze the problem. The three changes are: (1) Change the up-move to 15 percent from 25 percent and the down-move to –13 percent from...

See Answer

Q: An overconfident manager misjudges the risk associated with a project and uses

An overconfident manager misjudges the risk associated with a project and uses a discount rate that is too low. Suppose that the true situation is described by the situation in Question 4, but the man...

See Answer

Q: On January 26, 2005, James Stewart wrote about eBay in

On January 26, 2005, James Stewart wrote about eBay in his Wall Street Journal column “Common Sense.” Stewart indicated that he would consider purchasing eBay stock in the wake of its decline. While a...

See Answer

Q: On February 13, 2016, The New York Times published

On February 13, 2016, The New York Times published a story by journalist Jeff Sommer entitled “Dividends, Wall Street’s Battered Status Symbol.” Sommer discussed the fact that firms had been reducin...

See Answer

Q: In connection with chapter question 2, answer the following questions.

In connection with chapter question 2, answer the following questions. a. What probability would you assign to Jack being an engineer if the question instead stated that the room with lawyers has 70 l...

See Answer

Q: Warren Buffett described his investment philosophy as being greedy when others are

Warren Buffett described his investment philosophy as being greedy when others are fearful and being fearful when others are greedy. Bob Goldfarb is the chairman of Ruane, Cunniff & Goldfarb, the inve...

See Answer

Q: Impact investors make investments in firms, organizations, and funds in

Impact investors make investments in firms, organizations, and funds in order to generate positive social and environmental impacts alongside a financial return. Toniic is an organization dedicated to...

See Answer

Q: The minicase in Chapter 3 contains an excerpt from a ValuEngine analyst

The minicase in Chapter 3 contains an excerpt from a ValuEngine analyst report about the firm Aetna. The report states that ValuEngine’s forecasting models capture important features of stock price dy...

See Answer

Q: On the MHHE web site for this book, in the Chapter

On the MHHE web site for this book, in the Chapter 13 files, you will find an Excel file with return data on Valeant and other firms. Use regression analysis to assess the factor structure of the stoc...

See Answer

Q: Precept Capital Management is a hedge fund located in Dallas, Texas

Precept Capital Management is a hedge fund located in Dallas, Texas. The fund employs several investment strategies, one of which is based on comparing the trajectory of stock prices to the trajecto...

See Answer

Q: Given the information provided in the chapter, discuss the psychological phenomena

Given the information provided in the chapter, discuss the psychological phenomena associated with Martha Stewart’s investment decisions.

See Answer

Q: Discuss if the manner in which people’s answers to questions 7.

Discuss if the manner in which people’s answers to questions 7.1 through 7.6 above provides an indication of their financial literacy.90 Questions 7.1: Suppose you had $100 in a savings account and...

See Answer

Q: Consider the financial planning case study in Chapter 13 about William and

Consider the financial planning case study in Chapter 13 about William and Mira Bold. At the end of the case, financial planner Claire begins to prepare for her second meeting with the Bolts. How can...

See Answer

Q: Of the 10 psychological phenomena introduced in Chapter 1, identify which

Of the 10 psychological phenomena introduced in Chapter 1, identify which ones apply to the minicase, and give reasons to support your answer.

See Answer

Q: As of July 2015, 30 countries worldwide are operating 438 nuclear

As of July 2015, 30 countries worldwide are operating 438 nuclear reactors for electricity generation and 67 new nuclear plants are under construction in 15 countries. Nuclear power plants provided 10...

See Answer

Q: Imagine a decision task in which you are to choose between two

Imagine a decision task in which you are to choose between two alternatives that involve blindly drawing a single chip from one of two urns, labeled A and B respectively. Both urns contain colored bal...

See Answer

Q: The U.S. Army Corps of Engineers maintains locks and

The U.S. Army Corps of Engineers maintains locks and dams on U.S. waterways, and engages consultants to analyze the kinds of issues described in the minicase. One of these consultants points out that...

See Answer

Q: Identify the psychological phenomena in the minicase. Prioritize the phenomena from

Identify the psychological phenomena in the minicase. Prioritize the phenomena from most important to least important. Begin your answer by defining the phenomena, and then describing their role in th...

See Answer

Q: How would you use behavioral concepts to explain generally why different analysts

How would you use behavioral concepts to explain generally why different analysts arrive at different price targets?

See Answer

Q: What insights from the discussion of the Morgan Stanley 2003 report on

What insights from the discussion of the Morgan Stanley 2003 report on eBay apply to the 2013 report on Aetna by Jefferies?

See Answer

Q: What insights from the discussion of the Morgan Stanley 2003 report on

What insights from the discussion of the Morgan Stanley 2003 report on eBay apply to the 2013 report on Aetna by Cantor Fitzgerald?

See Answer

Q: What insights from the discussion of the Morgan Stanley 2003 report on

What insights from the discussion of the Morgan Stanley 2003 report on eBay apply to the 2013 report on Aetna by Leerink Swann?

See Answer

Q: What insights from the discussion of the Morgan Stanley 2003 report on

What insights from the discussion of the Morgan Stanley 2003 report on eBay apply to the 2013 report on Aetna by ValuEngine?

See Answer

Q: Consider the issues of project budget, scope, and project timetable

Consider the issues of project budget, scope, and project timetable. Discuss the extent to which the City Center project reflects survey evidence discussed in the chapter about capital budgeting biase...

See Answer

Q: Consider whether MGM faced issues in the City Center project that could

Consider whether MGM faced issues in the City Center project that could be characterized as sunk costs, and if so, whether they exhibited behavior consistent with “escalation of commitment.”

See Answer

Q: Compare the City Center project with specific projects discussed in the chapter

Compare the City Center project with specific projects discussed in the chapter, namely: Sony’s Chromatron, Syntex’s Enrprostil, Motorola’s Iridium, Eurotunnel’s Channel Tunnel, Boeing’s Dreamliner, a...

See Answer

Q: What are the portfolio weights for a portfolio that has

What are the portfolio weights for a portfolio that has 180 shares of Stock A that sell for $45 per share and 140 shares of Stock B that sell for $27 per share?

See Answer

Q: Consider the following information:

Consider the following information:a. Your portfolio is invested 30 percent each in A and C, and 40 percent in B. What is the expected return of the portfolio?b. What is the variance of this portfolio...

See Answer

Q: Suppose the spot exchange rate for the Hungarian forint is

Suppose the spot exchange rate for the Hungarian forint is HUF 152.93. The inflation rate in the United States will be 4.9 percent per year. It will be 8.6 percent in Hungary. What do you predict the...

See Answer

Q: Use the information in Figure 21.1 to answer the following

Use the information in Figure 21.1 to answer the following questions:Figure 21.1:a. Which would you rather have, $100 or £100? Why?b. Which would you rather have, 100 Swiss francs (SF) or...

See Answer

Q: Use the information in Figure 21.1 to answer the following

Use the information in Figure 21.1 to answer the following questions:Figure 21.1:a. What is the six-month forward rate for the Japanese yen in yen per U.S. dollar? Is the yen selling at a premium or a...

See Answer

Q: Suppose the spot exchange rate for the Canadian dollar is

Suppose the spot exchange rate for the Canadian dollar is Can$1.06 and the six-month forward rate is Can$1.11.a. Which is worth more, a U.S. dollar or a Canadian dollar?b. Assuming absolute PPP holds,...

See Answer

Q: Suppose the Japanese yen exchange rate is ¥112 =

Suppose the Japanese yen exchange rate is ¥112 = $1, and the British pound exchange rate is £1 = $1.93.a. What is the cross-rate in terms of yen per pound?b. Suppose the cross-rate is ¥209 = £1. Is t...

See Answer

Q: Use Figure 21.1 to answer the following questions: Suppose interest

Use Figure 21.1 to answer the following questions: Suppose interest rate parity holds, and the current six-month risk-free rate in the United States is 2.2 percent. What must the six-month risk-free r...

See Answer

Q: The treasurer of a major U.S. firm has $30 million

The treasurer of a major U.S. firm has $30 million to invest for three months. The interest rate in the United States is .37 percent per month. The interest rate in Great Britain is .51 percent per mo...

See Answer

Q: Suppose the current exchange rate for the Polish zloty is

Suppose the current exchange rate for the Polish zloty is Z 2.17. The expected exchange rate in three years is Z 2.26. What is the difference in the annual inflation rates for the United States and Po...

See Answer

Q: Suppose your company imports computer motherboards from Singapore. The exchange

Suppose your company imports computer motherboards from Singapore. The exchange rate is given in Figure 21.1 . You have just placed an order for 30,000 motherboards at a cost to you of 204.7 Singapore...

See Answer

Q: Indicate the likely impact of each of the following on

Indicate the likely impact of each of the following on a company’s target cash balance. Use the letter I to denote an increase and D to denote a decrease. Briefly explain your reasoning in each case:a...

See Answer

Q: You own a stock portfolio invested 25 percent in Stock

You own a stock portfolio invested 25 percent in Stock Q, 20 percent in Stock R, 15 percent in Stock S, and 40 percent in Stock T. The betas for these four stocks are .84, 1.17, 1.11, and 1.36, respec...

See Answer

Q: All Night Corporation has determined that its target cash balance

All Night Corporation has determined that its target cash balance if it uses the BAT model is $2,700. The total cash needed for the year is $28,000, and the order cost is $10. What interest rate must...

See Answer

Q: Given the following information, calculate the target cash balance using

Given the following information, calculate the target cash balance using the BAT model:Annual interest rate ……………………………….…………. 6%Fixed order cost ………………………………………..……… $25Total cash needed ………………………………...

See Answer

Q: White Whale Corporation has an average daily cash balance of

White Whale Corporation has an average daily cash balance of $1,300. Total cash needed for the year is $43,000. The interest rate is 5 percent, and replenishing the cash costs $8 each time. What are t...

See Answer

Q: Debit and Credit Bookkeepers needs a total of $16,000 in

Debit and Credit Bookkeepers needs a total of $16,000 in cash during the year for transactions and other purposes. Whenever cash runs low, it sells $1,500 in securities and transfers the cash in. The...

See Answer

Q: The All Day Company is currently holding $690,000 in cash.

The All Day Company is currently holding $690,000 in cash. It projects that over the next year its cash outflows will exceed cash inflows by $140,000 per month. How much of the current cash holdings s...

See Answer

Q: Econoline Crush, Inc., uses a Miller–Orr cash management approach with

Econoline Crush, Inc., uses a Miller–Orr cash management approach with a lower limit of $43,000, an upper limit of $125,000, and a target balance of $80,000. Explain what each of these points represen...

See Answer

Q: Slap Shot Corporation has a fixed cost associated with buying

Slap Shot Corporation has a fixed cost associated with buying and selling marketable securities of $40. The interest rate is currently .021 percent per day, and the firm has estimated that the standar...

See Answer

Q: Based on the Miller–Orr model, describe what will happen to

Based on the Miller–Orr model, describe what will happen to the lower limit, the upper limit, and the spread (the distance between the two) if the variation in net cash flow grows. Give an intuitive e...

See Answer

Q: The variance of the daily cash flows for the Pele

The variance of the daily cash flows for the Pele Bicycle Shop is $890,000. The opportunity cost to the firm of holding cash is 7 percent per year. What should the target cash level and the upper limi...

See Answer

Q: Bismark Co. is in the process of considering a change

Bismark Co. is in the process of considering a change in its terms of sale. The current policy is cash only; the new policy will involve one period’s credit. Sales are 40,000 units per period at a pri...

See Answer

Q: You own a portfolio equally invested in a risk free

You own a portfolio equally invested in a risk free asset and two stocks. If one of the stocks has a beta of 1.38 and the total portfolio is equally as risky as the market, what must the beta be for t...

See Answer

Q: The Johnson Company sells 3,300 pairs of running shoes per

The Johnson Company sells 3,300 pairs of running shoes per month at a cash price of $90 per pair. The firm is considering a new policy that involves 30 days’ credit and an increase in price to $91.84...

See Answer

Q: Silicon Wafers, Inc. (SWI), is debating whether or not to

Silicon Wafers, Inc. (SWI), is debating whether or not to extend credit to a particular customer. SWI’s products, primarily used in the manufacture of semiconductors, currently sell for $1,140 per uni...

See Answer

Q: Consider the following information about two alternative credit strategies:

Consider the following information about two alternative credit strategies:The higher cost per unit reflects the expense associated with credit orders, and the higher price per unit reflects the exist...

See Answer

Q: Suppose a corporation currently sells Q units per month for

Suppose a corporation currently sells Q units per month for a cash-only price of P . Under a new credit policy that allows one month’s credit, the quantity sold will be Q and the price per unit will b...

See Answer

Q: Night Shades Inc. (NSI) manufactures biotech sunglasses. The variable materials

Night Shades Inc. (NSI) manufactures biotech sunglasses. The variable materials cost is $5.43 per unit, and the variable labor cost is $3.13 per unit.a. What is the variable cost per unit?b. Suppose N...

See Answer

Q: Consider a project with the following data: accounting break-even quantity

Consider a project with the following data: accounting break-even quantity = 15,500 units; cash break-even quantity = 13,200 units; life = five years; fixed costs = $140,000; variable costs = $24 per...

See Answer

Q: At an output level of 65,000 units, you calculate that

At an output level of 65,000 units, you calculate that the degree of operating leverage is 3.40. If output rises to 70,000 units, what will the percentage change in operating cash flow be? Will the ne...

See Answer

Q: In the previous problem, suppose fixed costs are $130,000. What

In the previous problem, suppose fixed costs are $130,000. What is the operating cash flow at 58,000 units? The degree of operating leverage?Previous problem:At an output level of 65,000 units, you ca...

See Answer

Q: A proposed project has fixed costs of $73,000 per year.

A proposed project has fixed costs of $73,000 per year. The operating cash flow at 8,000 units is $87,500. Ignoring the effect of taxes, what is the degree of operating leverage? If units sold rise fr...

See Answer

Q: At an output level of 10,000 units, you have calculated

At an output level of 10,000 units, you have calculated that the degree of operating leverage is 2.35. The operating cash flow is $43,000 in this case. Ignoring the effect of taxes, what are fixed cos...

See Answer

Q: A stock has a beta of 1.05, the expected return

A stock has a beta of 1.05, the expected return on the market is 11 percent, and the risk-free rate is 5.2 percent. What must the expected return on this stock be?

See Answer

Q: In the previous problem, what will be the new degree

In the previous problem, what will be the new degree of operating leverage in each case?Previous problem:At an output level of 10,000 units, you have calculated that the degree of operating leverage i...

See Answer

Q: Titan Mining Corporation has 9 million shares of common stock

Titan Mining Corporation has 9 million shares of common stock outstanding, 250,000 shares of 6 percent preferred stock outstanding, and 105,000 7.5 percent semiannual bonds outstanding, par value $1,0...

See Answer

Q: An all-equity firm is considering the following projects:

An all-equity firm is considering the following projects:The T-bill rate is 5 percent, and the expected return on the market is 11 percent.a. Which projects have a higher expected return than the firm...

See Answer

Q: Suppose your company needs $20 million to build a new

Suppose your company needs $20 million to build a new assembly line. Your target debtequity ratio is .75. The flotation cost for new equity is 8 percent, but the flotation cost for debt is only 5 perc...

See Answer

Q: Southern Alliance Company needs to raise $45 million to start

Southern Alliance Company needs to raise $45 million to start a new project and will raise the money by selling new bonds. The company will generate no internal equity for the foreseeable future. The...

See Answer

Q: K-Too Ever wear Corporation can manufacture mountain climbing shoes for

K-Too Ever wear Corporation can manufacture mountain climbing shoes for $24.86 per pair in variable raw material costs and $14.08 per pair in variable labor expense. The shoes sell for $135 per pair....

See Answer

Q: Olin Transmissions, Inc. has the following estimates for its new

Olin Transmissions, Inc. has the following estimates for its new gear assembly project: price = $1,900 per unit; variable costs = $240 per unit; fixed costs = $4.8 million; quantity = 95,000 units. Su...

See Answer

Q: For the company in the previous problem, suppose management is

For the company in the previous problem, suppose management is most concerned about the impact of its price estimate on the project’s profitability. How could you address this concern? Describe how yo...

See Answer

Q: We are evaluating a project that costs $724,000, has an

We are evaluating a project that costs $724,000, has an eight-year life, and has no salvage value. Assume that depreciation is straight-line to zero over the life of the project. Sales are projected a...

See Answer

Q: In the previous problem, suppose the projections given for price,

In the previous problem, suppose the projections given for price, quantity, variable costs, and fixed costs are all accurate to within ± 10 percent. Calculate the best-case and worst-case NPV fi gures...

See Answer

Q: A stock has an expected return of 10.2 percent, the

A stock has an expected return of 10.2 percent, the risk-free rate is 4.5 percent, and the market risk premium is 8.5 percent. What must the beta of this stock be?

See Answer

Q: In each of the following cases, calculate the accounting break-even

In each of the following cases, calculate the accounting break-even and the cash break-even points. Ignore any tax effects in calculating the cash break-even.,,,

See Answer

Q: In each of the following cases, find the unknown variable:

In each of the following cases, find the unknown variable:,,,

See Answer

Q: A project has the following estimated data: price = $57

A project has the following estimated data: price = $57 per unit; variable costs = $32 per unit; fixed costs = $9,000; required return = 12 percent; initial investment = $18,000; life = four years. Ig...

See Answer

Q: The Gecko Company and the Gordon Company are two firms

The Gecko Company and the Gordon Company are two firms whose business risk is the same but that have different dividend policies. Gecko pays no dividend, whereas Gordon has an expected dividend yield...

See Answer

Q: As discussed in the text, in the absence of market

As discussed in the text, in the absence of market imperfections and tax effects, we would expect the share price to decline by the amount of the dividend payment when the stock goes ex dividend. Once...

See Answer

Q: National Business Machine Co. (NBM) has $2 million of extra

National Business Machine Co. (NBM) has $2 million of extra cash after taxes have been paid. NBM has two choices to make use of this cash. One alternative is to invest the cash in fi nancial assets. T...

See Answer

Q: After completing its capital spending for the year, Carlson Manufacturing

After completing its capital spending for the year, Carlson Manufacturing has $1,000 extra cash. Carlson’s managers must choose between investing the cash in Treasury bonds that yield 6 percent or pay...

See Answer

Q: In exchange for a $400 million fixed commitment line of

In exchange for a $400 million fixed commitment line of credit, your firm has agreed to do the following:1. Pay 1.9 percent per quarter on any funds actually borrowed.2. Maintain a 4 percent compensat...

See Answer

Q: Come and Go Bank offers your firm a 10 percent

Come and Go Bank offers your firm a 10 percent discount interest loan for up to $25 million, and in addition requires you to maintain a 5 percent compensating balance against the amount borrowed. What...

See Answer

Q: In a world of corporate taxes only, show that the

In a world of corporate taxes only, show that the WACC can be written as WACC = R U × [1 - T C ( D / V )]

See Answer

Q: A stock has an expected return of 13.5 percent, its

A stock has an expected return of 13.5 percent, its beta is 1.17, and the risk-free rate is 5.5 percent. What must the expected return on the market be?

See Answer

Q: Saché, Inc., expects to sell 700 of its designer suits

Saché, Inc., expects to sell 700 of its designer suits every week. The store is open seven days a week and expects to sell the same number of suits every day. The company has an EOQ of 500 suits and a...

See Answer

Q: Assume a firm’s debt is risk-free, so that the cost

Assume a firm’s debt is risk-free, so that the cost of debt equals the risk-free rate, R f . Define

See Answer

Q: In Problem 14, what is the break-even quantity for the

In Problem 14, what is the break-even quantity for the new credit policy?Problem 14:The Harrington Corporation is considering a change in its cash-only policy. The new terms would be net one period. B...

See Answer

Q: In Problem 14, what is the break-even price per unit

In Problem 14, what is the break-even price per unit that should be charged under the new credit policy? Assume that the sales figure under the new policy is 4,100 units and all other values remain th...

See Answer

Q: Photo chronograph Corporation (PC) manufactures time series photographic equipment. It

Photo chronograph Corporation (PC) manufactures time series photographic equipment. It is currently at its target debt ­equity ratio of .70. It’s considering building a new $45 million manufacturing f...

See Answer

Q: You have $100,000 to invest in a portfolio containing Stock

You have $100,000 to invest in a portfolio containing Stock X and Stock Y. Your goal is to create a portfolio that has an expected return of 18.5 percent. If Stock X has an expected return of 17.2 per...

See Answer

Q: Consider the following information about Stocks I and II:

Consider the following information about Stocks I and II:The market risk premium is 8 percent, and the risk-free rate is 4 percent. Which stock has the most systematic risk? Which one has the most uns...

See Answer

Q: Suppose you observe the following situation:

Suppose you observe the following situation:Assume these securities are correctly priced. Based on the CAPM, what is the expected return on the market? What is the risk-free rate?

See Answer

Q: Suppose you observe the following situation:

Suppose you observe the following situation:a. Calculate the expected return on each stock.b. Assuming the capital asset pricing model holds and stock A’s beta is greater than stock...

See Answer

Q: Use the results of Problem 25 to find the accounting,

Use the results of Problem 25 to find the accounting, cash, and financial break-even quantities for the company in Problem 27

See Answer

Q: A stock has an expected return of 14 percent, its

A stock has an expected return of 14 percent, its beta is 1.45, and the expected return on the market is 11.5 percent. What must the risk-free rate be?

See Answer

Q: Use the results of Problem 26 to find the degree

Use the results of Problem 26 to find the degree of operating leverage for the company in Problem 27 at the base-case output level of 35,000 units. How does this number compare to the sensitivity figu...

See Answer

Q: Trower Corp. has a debtequity ratio of 1.20. The company

Trower Corp. has a debtequity ratio of 1.20. The company is considering a new plant that will cost $145 million to build. When the company issues new equity, it incurs a flotation cost of 8 percent. T...

See Answer

Q: This is a comprehensive project evaluation problem bringing together much

This is a comprehensive project evaluation problem bringing together much of what you have learned in this and previous chapters. Suppose you have been hired as a fi nancial consultant to Defense Elec...

See Answer

Q: In Problem 15, what is the break-even price per unit

In Problem 15, what is the break-even price per unit under the new credit policy? Assume all other values remain the same.Problem 15:Happy Times currently has an all-cash credit policy. It is consider...

See Answer

Q: In the previous problem, assume the equity increases by 1,250

In the previous problem, assume the equity increases by 1,250 solaris due to retained earnings. If the exchange rate at the end of the year is 1.24 solaris per dollar, what does the balance sheet look...

See Answer

Q: From our discussion of the Fisher effect in Chapter 7,

From our discussion of the Fisher effect in Chapter 7, we know that the actual relationship between a nominal rate, R, a real rate, r, and an inflation rate, h, can be written as:1 + r = (1 + R)/ (1 +...

See Answer

Q: This problem concerns the effect of taxes on the various

This problem concerns the effect of taxes on the various break-even measures.a. Show that, when we consider taxes, the general relationship between operating cash flow, OCF, and sales volume, Q , can...

See Answer

Q: Show that if we consider the effect of taxes, the

Show that if we consider the effect of taxes, the degree of operating leverage can be written as: DOL = 1 + [FC × (1 - T ) - T × D ] / OCF Notice that this reduces to our previous result if T = 0. Can...

See Answer

Q: Consider a project to supply Detroit with 35,000 tons of

Consider a project to supply Detroit with 35,000 tons of machine screws annually for automobile production. You will need an initial $3,200,000 investment in threading equipment to get the project sta...

See Answer

Q: In Problem 27, suppose you’re confident about your own projections,

In Problem 27, suppose you’re confident about your own projections, but you’re a little unsure about Detroit’s actual machine screw requirement. What is the sensitivity of the project OCF to changes i...

See Answer

Q: A stock has a beta of 1.35 and an expected

A stock has a beta of 1.35 and an expected return of 16 percent. A risk-free asset currently earns 4.8 percent.a. What is the expected return on a portfolio that is equally invested in the two assets?...

See Answer

Q: Teardrop, Inc., wishes to expand its facilities. The company currently

Teardrop, Inc., wishes to expand its facilities. The company currently has 8 million shares outstanding and no debt. The stock sells for $50 per share, but the book value per share is $18. Net income...

See Answer

Q: The Metallica Heavy Metal Mining (MHMM) Corporation wants to diversify

The Metallica Heavy Metal Mining (MHMM) Corporation wants to diversify its operations. Some recent fi nancial information for the company is shown here:MHMM is considering an investment that has the s...

See Answer

Q: In the previous problem, what would the ROE on the

In the previous problem, what would the ROE on the investment have to be if we wanted the price after the offering to be $98 per share? (Assume the PE ratio remains constant.) What is the NPV of this...

See Answer

Q: Keira Mfg. is considering a rights offer. The company has

Keira Mfg. is considering a rights offer. The company has determined that the ex-rights price would be $71. The current price is $76 per share, and there are 19 million shares outstanding. The rights...

See Answer

Q: You bought one of Great White Shark Repellant Co.’s 8

You bought one of Great White Shark Repellant Co.’s 8 percent coupon bonds one year ago for $1,030. These bonds make annual payments and mature six years from now. Suppose you decide to sell your bond...

See Answer

Q: You find a certain stock that had returns of 7

You find a certain stock that had returns of 7 percent, - 12 percent, 18 percent, and 19 percent for four of the last five years. If the average return of the stock over this period was 10.5 percent,...

See Answer

Q: A stock has had returns of 3 percent, 38 percent,

A stock has had returns of 3 percent, 38 percent, 21 percent, -15 percent, 29 percent, and - 13 percent over the last six years. What are the arithmetic and geometric returns for the stock?

See Answer

Q: A stock has had the following yearend prices and dividends:

A stock has had the following yearend prices and dividends:What are the arithmetic and geometric returns for the stock?

See Answer

Q: Consider a four-year project with the following information: initial fixed

Consider a four-year project with the following information: initial fixed asset investment = $490,000; straight-line depreciation to zero over the four-year life; zero salvage value; price = $32; var...

See Answer

Q: In the previous problem, what is the degree of operating

In the previous problem, what is the degree of operating leverage at the given level of output? What is the degree of operating leverage at the accounting break-even level of output?Previous problem:C...

See Answer

Q: Asset W has an expected return of 15.2 percent and

Asset W has an expected return of 15.2 percent and a beta of 1.25. If the risk-free rate is 5.3 percent, complete the following table for portfolios of Asset W and a risk-free asset. Illustrate the re...

See Answer

Q: You are considering a new product launch. The project will

You are considering a new product launch. The project will cost $1,700,000, have a four-year life, and have no salvage value; depreciation is straight-line to zero. Sales are projected at 190 units pe...

See Answer

Q: McGilla Golf has decided to sell a new line of

McGilla Golf has decided to sell a new line of golf clubs. The clubs will sell for $750 per set and have a variable cost of $330 per set. The company has spent $150,000 for a marketing study that dete...

See Answer

Q: In the previous problem, you feel that the values are

In the previous problem, you feel that the values are accurate to within only ±10 percent. What are the best-case and worst-case NPVs? (Hint: The price and variable costs for the two existing sets of...

See Answer

Q: Hybrid cars are touted as a “green” alternative; however, the

Hybrid cars are touted as a “green” alternative; however, the fi nancial aspects of hybrid ownership are not as clear. Consider the 2006 Honda Accord Hybrid, which had a list price of $5,450 (includin...

See Answer

Q: In an effort to capture the large jet market, Airbus

In an effort to capture the large jet market, Airbus invested $13 billion developing its A380, which is capable of carrying 800 passengers. The plane has a list price of $280 million. In discussing th...

See Answer

Q: Suppose the returns on long-term corporate bonds are normally distributed.

Suppose the returns on long-term corporate bonds are normally distributed. Based on the historical record, what is the approximate probability that your return on these bonds will be less than - 2.2 p...

See Answer

Q: In Problem 18, what is the probability that the return

In Problem 18, what is the probability that the return is less than -100 percent (think)? What are the implications for the distribution of returns?Problem 18:Assuming that the returns from holding sm...

See Answer

Q: Over a 40-year period an asset had an arithmetic return

Over a 40-year period an asset had an arithmetic return of 15.3 percent and a geometric return of 11.9 percent. Using Blume’s formula, what is your best estimate of the future annual returns over 5 ye...

See Answer

Q: Assume that the historical return on large-company stocks is a

Assume that the historical return on large-company stocks is a predictor of the future returns. What return would you estimate for large company stocks over the next year? The next 5 years? 20 years?...

See Answer

Q: Stock Y has a beta of 1.3 and an expected

Stock Y has a beta of 1.3 and an expected return of 18.5 percent. Stock Z has a beta of .70 and an expected return of 12.1 percent. If the risk-free rate is 8 percent and the market risk premium is 7....

See Answer

Q: Scanlin, Inc., is considering a project that will result in

Scanlin, Inc., is considering a project that will result in initial after tax cash savings of $2.7 million at the end of the first year, and these savings will grow at a rate of 4 percent per year ind...

See Answer

Q: Goodbye, Inc., recently issued new securities to finance a new

Goodbye, Inc., recently issued new securities to finance a new TV show. The project cost $15 million, and the company paid $850,000 in flotation costs. In addition, the equity issued had a flotation c...

See Answer

Q: Ying Import has several bond issues outstanding, each making semiannual

Ying Import has several bond issues outstanding, each making semiannual interest payments. The bonds are listed in the following table. If the corporate tax rate is 34 percent, what is the after tax c...

See Answer

Q: Floyd Industries stock has a beta of 1.50. The company

Floyd Industries stock has a beta of 1.50. The company just paid a dividend of $.80, and the dividends are expected to grow at 5 percent. The expected return of the market is 12 percent, and Treasury...

See Answer

Q: Show that the value of a right just prior to

Show that the value of a right just prior to expiration can be written as:Value of a right = PRO – PX = (PRO – PS)/(N + 1) Where P RO , P S , and P X stand for the rights-on price, the subscription pr...

See Answer

Q: Roth Corp. wants to raise $5.6 million via a rights

Roth Corp. wants to raise $5.6 million via a rights offering. The company currently has 650,000 shares of common stock outstanding that sell for $50 per share. Its underwriter has set a subscription p...

See Answer

Q: Knight Inventory Systems, Inc., has announced a rights offer. The

Knight Inventory Systems, Inc., has announced a rights offer. The company has announced that it will take four rights to buy a new share in the offering at a subscription price of $35. At the close of...

See Answer

Q: Tool Manufacturing has an expected EBIT of $64,000 in perpetuity

Tool Manufacturing has an expected EBIT of $64,000 in perpetuity and a tax rate of 35 percent. The firm has $95,000 in outstanding debt at an interest rate of 8.5 percent, and its unlevered cost of ca...

See Answer

Q: Until It Sleeps Corporation expects an EBIT of $14,000 every

Until It Sleeps Corporation expects an EBIT of $14,000 every year forever. Until It Sleeps currently has no debt, and its cost of equity is 16 percent. The firm can borrow at 9 percent. If the corpora...

See Answer

Q: The Veblen Company and the Knight Company are identical in

The Veblen Company and the Knight Company are identical in every respect except that Veblen is not levered. Financial information for the two firms appears in the following table. All earnings streams...

See Answer

Q: You own a portfolio that has $2,950 invested in Stock

You own a portfolio that has $2,950 invested in Stock A and $3,700 invested in Stock B. If the expected returns on these stocks are 11 percent and 15 percent, respectively, what is the expected return...

See Answer

Q: The Down and Out Co. just issued a dividend of

The Down and Out Co. just issued a dividend of $2.40 per share on its common stock. The company is expected to maintain a constant 5.5 percent growth rate in its dividends indefinitely. If the stock s...

See Answer

Q: You own 1,000 shares of stock in Avondale Corporation. You

You own 1,000 shares of stock in Avondale Corporation. You will receive a $2.30 per share dividend in one year. In two years, Avondale will pay a liquidating dividend of $53 per share. The required re...

See Answer

Q: In the previous problem, suppose you want only $750 total

In the previous problem, suppose you want only $750 total in dividends the first year. What will your homemade dividend be in two years?Previous problem:You own 1,000 shares of stock in Avondale Corpo...

See Answer

Q: Fly chucker Corporation is evaluating an extra dividend versus a

Fly chucker Corporation is evaluating an extra dividend versus a share repurchase. In either case, $9,000 would be spent. Current earnings are $1.30 per share, and the stock currently sells for $64 pe...

See Answer

Q: You’ve worked out a line of credit arrangement that allows

You’ve worked out a line of credit arrangement that allows you to borrow up to $50 million at any time. The interest rate is .64 percent per month. In addition, 5 percent of the amount that you borrow...

See Answer

Q: A bank offers your firm a revolving credit arrangement for

A bank offers your firm a revolving credit arrangement for up to $70 million at an interest rate of 2.3 percent per quarter. The bank also requires you to maintain a compensating balance of 4 percent...

See Answer

Q: Wildcat, Inc., has estimated sales (in millions) for the next

Wildcat, Inc., has estimated sales (in millions) for the next four quarters as follows:Sales for the first quarter of the year after this one are projected at $240 million. Accounts receivable at the...

See Answer

Q: Rework Problem 15 assuming:

Rework Problem 15 assuming:Problem 15:Wildcat, Inc., has estimated sales (in millions) for the next four quarters as follows:Wildcat’s purchases from suppliers in a quarter are equal...

See Answer

Q: Bird’s Eye Tree houses, Inc., a Kentucky company, has determined

Bird’s Eye Tree houses, Inc., a Kentucky company, has determined that a majority of its customers are located in the Pennsylvania area. It therefore is considering using a lockbox sy...

See Answer

Q: Cow Chips, Inc., a large fertilizer distributor based in California,

Cow Chips, Inc., a large fertilizer distributor based in California, is planning to use a lockbox system to speed up collections from its customers located on the East Coast. A Philadelphia-area bank...

See Answer

Q: The Harrington Corporation is considering a change in its cash-only

The Harrington Corporation is considering a change in its cash-only policy. The new terms would be net one period. Based on the following information, determine if Harrington should proceed or not. Th...

See Answer

Q: Sixx AM Manufacturing has a target debt equity ratio of

Sixx AM Manufacturing has a target debt equity ratio of .65. Its cost of equity is 15 percent, and its cost of debt is 9 percent. If the tax rate is 35 percent, what is the company’s WACC?

See Answer

Q: Happy Times currently has an all-cash credit policy. It is

Happy Times currently has an all-cash credit policy. It is considering making a change in the credit policy by going to terms of net 30 days. Based on the following information, what do you recommend?...

See Answer

Q: The Silver Spokes Bicycle Shop has decided to offer credit

The Silver Spokes Bicycle Shop has decided to offer credit to its customers during the spring selling season. Sales are expected to be 500 bicycles. The average cost to the shop of a bicycle is $490....

See Answer

Q: Lakonishok Equipment has an investment opportunity in Europe. The project

Lakonishok Equipment has an investment opportunity in Europe. The project costs €14 million and is expected to produce cash flows of €2.1 million in year 1, €3.4 million in year 2, and €4.3 million in...

See Answer

Q: You are evaluating a proposed expansion of an existing subsidiary

You are evaluating a proposed expansion of an existing subsidiary located in Switzerland. The cost of the expansion would be SF 24 million. The cash flows from the project would be SF 6.6 million per...

See Answer

Q: Atreides International has operations in Arrakis. The balance sheet for

Atreides International has operations in Arrakis. The balance sheet for this division in Arrakeen solaris shows assets of 23,000 solaris, debt in the amount of 9,000 solaris, and equity of 14,000 sola...

See Answer

Q: Suppose a stock had an initial price of $91 per

Suppose a stock had an initial price of $91 per share, paid a dividend of $2.40 per share during the year, and had an ending share price of $102. Compute the percentage total return

See Answer

Q: In Problem 1, what was the dividend yield? The capital

In Problem 1, what was the dividend yield? The capital gains yield?Problem 1:Suppose a stock had an initial price of $91 per share, paid a dividend of $2.40 per share during the year, and had an endin...

See Answer

Q: Rework Problems 1 and 2 assuming the ending share price

Rework Problems 1 and 2 assuming the ending share price is $83.Problems 1:what was the dividend yield? The capital gains yield?Problems 2:Suppose a stock had an initial price of $91 per share, paid a...

See Answer

Q: Suppose you bought a 7 percent coupon bond one year

Suppose you bought a 7 percent coupon bond one year ago for $1,040. The bond sells for $1,070 today.a. Assuming a $1,000 face value, what was your total dollar return on this investment over the past...

See Answer

Q: What was the average annual return on large-company stock from

What was the average annual return on large-company stock from 1926 through 2007:a. In nominal terms?b. In real terms?

See Answer

Q: Fama’s Llamas has a weighted average cost of capital of

Fama’s Llamas has a weighted average cost of capital of 8.9 percent. The company’s cost of equity is 12 percent, and its pretax cost of debt is 7.9 percent. The tax rate is 35 percent. What is the com...

See Answer

Q: Using the following returns, calculate the arithmetic average returns, the

Using the following returns, calculate the arithmetic average returns, the variances, and the standard deviations for X and Y.,,,

See Answer

Q: Refer to Table 12.1 in the text and look at

Refer to Table 12.1 in the text and look at the period from 1970 through 1975.Table 12.1:a. Calculate the arithmetic average returns for large-company stocks and T-bills over this period.b. Calculate...

See Answer

Q: You’ve observed the following returns on Crash-n-Burn Computer’s stock over

You’ve observed the following returns on Crash-n-Burn Computer’s stock over the past five years: 7 percent, - 12 percent, 11 percent, 38 percent, and 14 percent.a. What was the arithmetic average retu...

See Answer

Q: For Problem 9, suppose the average inflation rate over this

For Problem 9, suppose the average inflation rate over this period was 3.5 percent and the average T-bill rate over the period was 4.2 percent.Problem 9:You’ve observed the following returns on Crash-...

See Answer

Q: Given the information in Problem 10, what was the average

Given the information in Problem 10, what was the average real risk-free rate over this time period? What was the average real risk premium?Problem 10:suppose the average inflation rate over this peri...

See Answer

Q: Look at Table 12.1 and Figure 12.7 in the text.

Look at Table 12.1 and Figure 12.7 in the text. When were T-bill rates at their highest over the period from 1926 through 2007? Why do you think they were so high during this period? What relationship...

See Answer

Q: Consider the following information about three stocks:

Consider the following information about three stocks:a. If your portfolio is invested 40 percent each in A and B and 20 percent in C, what is the portfolio expected return? The variance? The standard...

See Answer

Q: Suppose the returns on large-company stocks are normally distributed. Based

Suppose the returns on large-company stocks are normally distributed. Based on the historical record, use the cumulative normal probability table (rounded to the nearest table value) in the appendix o...

See Answer

Q: You want to create a portfolio equally as risky as

You want to create a portfolio equally as risky as the market, and you have $1,000,000 to invest. Given this information, fi ll in the rest of the following table:,,,

See Answer

Q: Suppose the returns on long-term corporate bonds and T-bills are

Suppose the returns on long-term corporate bonds and T-bills are normally distributed. Based on the historical record, use the cumulative normal probability table (rounded to the nearest table value)...

See Answer

Q: Filer Manufacturing has 11 million shares of common stock outstanding.

Filer Manufacturing has 11 million shares of common stock outstanding. The current share price is $68, and the book value per share is $6. Filer Manufacturing also has two bond issues outstanding. The...

See Answer

Q: In Problem 12, suppose the most recent dividend was $4.10

In Problem 12, suppose the most recent dividend was $4.10 and the dividend growth rate is 6 percent. Assume that the overall cost of debt is the weighted average of that implied by the two outstanding...

See Answer

Q: Jungle, Inc., has a target debtequity ratio of 1.05. Its

Jungle, Inc., has a target debtequity ratio of 1.05. Its WACC is 9.4 percent, and the tax rate is 35 percent.a. If Jungle’s cost of equity is 14 percent, what is its pretax cost of debt?b. If instead...

See Answer

Q: Given the following information for even flow Power Co., find

Given the following information for even flow Power Co., find the WACC. Assume the company’s tax rate is 35 percent.Debt: 8,000 6.5 percent coupon bonds outstanding, $1,000 par value, 20 years to matu...

See Answer

Q: The Up and Coming Corporation’s common stock has a beta

The Up and Coming Corporation’s common stock has a beta of 1.05. If the risk-free rate is 5.3 percent and the expected return on the market is 12 percent, what is the company’s cost of equity capital?

See Answer

Q: Stock in Country Road Industries has a beta of .85.

Stock in Country Road Industries has a beta of .85. The market risk premium is 8 percent, and T-bills are currently yielding 5 percent. The company’s most recent dividend was $1.60 per share, and divi...

See Answer

Q: Suppose In a Found Ltd. just issued a dividend of

Suppose In a Found Ltd. just issued a dividend of $1.43 per share on its common stock. The company paid dividends of $1.05, $1.12, $1.19, and $1.30 per share in the last four years. If the stock curre...

See Answer

Q: You own a portfolio that is 60 percent invested in

You own a portfolio that is 60 percent invested in Stock X, 25 percent in Stock Y, and 15 percent in Stock Z. The expected returns on these three stocks are 9 percent, 17 percent, and 13 percent, resp...

See Answer

Q: Holdup Bank has an issue of preferred stock with a

Holdup Bank has an issue of preferred stock with a $6 stated dividend that just sold for $96 per share. What is the bank’s cost of preferred stock?

See Answer

Q: Waller, Inc., is trying to determine its cost of debt.

Waller, Inc., is trying to determine its cost of debt. The firm has a debt issue outstanding with 15 years to maturity that is quoted at 107 percent of face value. The issue makes semiannual payments...

See Answer

Q: Jiminy’s Cricket Farm issued a 30-year, 8 percent semiannual bond

Jiminy’s Cricket Farm issued a 30-year, 8 percent semiannual bond 7 years ago. The bond currently sells for 95 percent of its face value. The company’s tax rate is 35 percent.a. What is the pretax cos...

See Answer

Q: For the firm in Problem 7, suppose the book value

For the firm in Problem 7, suppose the book value of the debt issue is $80 million. In addition, the company has a second debt issue on the market, a zero coupon bond with seven years left to maturity...

See Answer

Q: Mullineaux Corporation has a target capital structure of 60 percent

Mullineaux Corporation has a target capital structure of 60 percent common stock, 5 percent preferred stock, and 35 percent debt. Its cost of equity is 14 percent, the cost of preferred stock is 6 per...

See Answer

Q: Big Time, Inc., is proposing a rights offering. Presently there

Big Time, Inc., is proposing a rights offering. Presently there are 500,000 shares outstanding at $81 each. There will be 60,000 new shares offered at $70 each.a. What is the new market value of the c...

See Answer

Q: The Clifford Corporation has announced a rights offer to raise

The Clifford Corporation has announced a rights offer to raise $40 million for a new journal, the Journal of Financial Excess. This journal will review potential articles after the author pays a nonre...

See Answer

Q: Red Shoe Co. has concluded that additional equity financing will

Red Shoe Co. has concluded that additional equity financing will be needed to expand operations and that the needed funds will be best obtained through a rights offering. It has correctly determined t...

See Answer

Q: The Woods Co. and the Mickelson Co. have both announced

The Woods Co. and the Mickelson Co. have both announced IPOs at $40 per share. One of these is undervalued by $7, and the other is overvalued by $5, but you have no way of knowing which is which. You...

See Answer

Q: The Educated Horses Corporation needs to raise $60 million to

The Educated Horses Corporation needs to raise $60 million to finance its expansion into new markets. The company will sell new shares of equity via a general cash offering to raise the needed funds....

See Answer

Q: You have $10,000 to invest in a stock portfolio. Your

You have $10,000 to invest in a stock portfolio. Your choices are Stock X with an expected return of 14 percent and Stock Y with an expected return of 10.5 percent. If your goal is to create a portfol...

See Answer

Q: In the previous problem, if the SEC fi ling fee

In the previous problem, if the SEC fi ling fee and associated administrative expenses of the offering are $900,000, how many shares need to be sold?Previous problem:The Educated Horses Corporation ne...

See Answer

Q: The Raven Co. has just gone public. Under a firm

The Raven Co. has just gone public. Under a firm commitment agreement, Raven received $18.20 for each of the 10 million shares sold. The initial offering price was $20 per share, and the stock rose to...

See Answer

Q: Left Turn, Inc., has 120,000 shares of stock outstanding. Each

Left Turn, Inc., has 120,000 shares of stock outstanding. Each share is worth $94, so the company’s market value of equity is $11,280,000. Suppose the firm issues 25,000 new shares at the following pr...

See Answer

Q: Maynard, Inc., has no debt outstanding and a total market

Maynard, Inc., has no debt outstanding and a total market value of $250,000. Earnings before interest and taxes, EBIT, are projected to be $28,000 if economic conditions are normal. If there is strong...

See Answer

Q: Wood Corp. uses no debt. The weighted average cost of

Wood Corp. uses no debt. The weighted average cost of capital is 9 percent. If the current market value of the equity is $23 million and there are no taxes, what is EBIT?

See Answer

Q: In the previous question, suppose the corporate tax rate is

In the previous question, suppose the corporate tax rate is 35 percent. What is EBIT in this case? What is the WACC? Explain.Previous question:Wood Corp. uses no debt. The weighted average cost of cap...

See Answer

Q: Maxwell Industries has a debt–equity ratio of 1.5. Its WACC

Maxwell Industries has a debt–equity ratio of 1.5. Its WACC is 10 percent, and its cost of debt is 7 percent. The corporate tax rate is 35 percent.a. What is the company’s cost of equity capital?b. Wh...

See Answer

Q: Empress Corp. has no debt but can borrow at 8.2

Empress Corp. has no debt but can borrow at 8.2 percent. The firm’s WACC is currently 11 percent, and the tax rate is 35 percent.a. What is the company’s cost of equity?b. If the firm converts to 25 p...

See Answer

Q: Frederick & Co. expects its EBIT to be $92,000 every

Frederick & Co. expects its EBIT to be $92,000 every year forever. The firm can borrow at 9 percent. Frederick currently has no debt, and its cost of equity is 15 percent. If the tax rate is 35 percen...

See Answer

Q: In Problem 14, what is the cost of equity after

In Problem 14, what is the cost of equity after recapitalization? What is the WACC? What are the implications for the firm’s capital structure decision?Problem 14:Frederick & Co. expects its EBIT to b...

See Answer

Q: Based on the following information, calculate the expected return:

Based on the following information, calculate the expected return:,,,

See Answer

Q: Repeat parts (a) and (b) in Problem 1 assuming Maynard

Repeat parts (a) and (b) in Problem 1 assuming Maynard has a tax rate of 35 percent.Parts (a) and (b) in Problem 1:a. Calculate earnings per share (EPS) under each of the three economic scenarios befo...

See Answer

Q: Suppose the company in Problem 1 has a market to-

Suppose the company in Problem 1 has a market to- book ratio of 1.0.Problem 1:Maynard, Inc., has no debt outstanding and a total market value of $250,000. Earnings before interest and taxes, EBIT, are...

See Answer

Q: James Corporation is comparing two different capital structures: an all-equity

James Corporation is comparing two different capital structures: an all-equity plan (Plan I) and a levered plan (Plan II). Under Plan I, the company would have 160,000 shares of stock outstanding. Und...

See Answer

Q: In Problem 4, use M&M Proposition I to find the

In Problem 4, use M&M Proposition I to find the price per share of equity under each of the two proposed plans. What is the value of the firm?Problem 4:James Corporation is comparing two different cap...

See Answer

Q: Keenan Corp. is comparing two different capital structures. Plan I

Keenan Corp. is comparing two different capital structures. Plan I would result in 7,000 shares of stock and $160,000 in debt. Plan II would result in 5,000 shares of stock and $240,000 in debt. The i...

See Answer

Q: Ignoring taxes in Problem 6, what is the price per

Ignoring taxes in Problem 6, what is the price per share of equity under Plan I? Plan II? What principle is illustrated by your answers?Problem 6:Keenan Corp. is comparing two different capital struct...

See Answer

Q: Seether, Inc., a prominent consumer products firm, is debating whether

Seether, Inc., a prominent consumer products firm, is debating whether to convert its all-equity capital structure to one that is 35 percent debt. Currently, there are 8,000 shares outstanding, and th...

See Answer

Q: ABC Co. and XYZ Co. are identical firms in all

ABC Co. and XYZ Co. are identical firms in all respects except for their capital structure. ABC is all equity financed with $600,000 in stock. XYZ uses both stock and perpetual debt; its stock is wort...

See Answer

Q: So Much, Inc., has declared a $4.60 per share dividend.

So Much, Inc., has declared a $4.60 per share dividend. Suppose capital gains are not taxed, but dividends are taxed at 15 percent. New IRS regulations require that taxes be withheld at the time the d...

See Answer

Q: The owners’ equity accounts for Quadrangle International are shown here:

The owners’ equity accounts for Quadrangle International are shown here:a. If Quadrangle stock currently sells for $30 per share and a 10 percent stock dividend is declared, how many...

See Answer

Q: Based on the following information, calculate the expected return:

Based on the following information, calculate the expected return:,,,

See Answer

Q: For the company in Problem 2, show how the equity

For the company in Problem 2, show how the equity accounts will change if:Problem 2:The owners’ equity accounts for Quadrangle International are shown here:a. Quadrangle declares a f...

See Answer

Q: Red Rocks Corporation (RRC) currently has 350,000 shares of stock

Red Rocks Corporation (RRC) currently has 350,000 shares of stock outstanding that sell for $90 per share. Assuming no market imperfections or tax effects exist, what will the share price be after:a....

See Answer

Q: The balance sheet for Chevelle Corp. is shown here in

The balance sheet for Chevelle Corp. is shown here in market value terms. There are 8,000 shares of stock outstanding.The company has declared a dividend of $1.30 per share. The stock goes ex dividend...

See Answer

Q: In the previous problem, suppose Chevelle has announced it is

In the previous problem, suppose Chevelle has announced it is going to repurchase $10,400 worth of stock. What effect will this transaction have on the equity of the firm? How many shares will be outs...

See Answer

Q: The market value balance sheet for Vena Sera Manufacturing is

The market value balance sheet for Vena Sera Manufacturing is shown here. Vena Sera has declared a 25 percent stock dividend. The stock goes ex dividend tomorrow (the chronology for a stock dividend i...

See Answer

Q: The company with the common equity accounts shown here has

The company with the common equity accounts shown here has declared a 15 percent stock dividend when the market value of its stock is $35 per share. What effects on the equity accounts will the distri...

See Answer

Q: In the previous problem, suppose the company instead decides on

In the previous problem, suppose the company instead decides on a four-for-one stock split. The firm’s 85-cent per share cash dividend on the new (post-split) shares represents an in...

See Answer

Q: The following is the sales budget for Trickle, Inc., for

The following is the sales budget for Trickle, Inc., for the first quarter of 2009:Credit sales are collected as follows:65 percent in the month of the sale20 percent in the month after the sale15 per...

See Answer

Q: Here are some important fi gures from the budget of

Here are some important fi gures from the budget of Nashville Nougats, Inc., for the second quarter of 2009:The company predicts that 5 percent of its credit sales will never be collected, 35 percent...

See Answer

Q: Below are the most recent balance sheets for Country Kettles,

Below are the most recent balance sheets for Country Kettles, Inc. Excluding accumulated depreciation, determine whether each item is a source or a use of cash, and the amount:,,,

See Answer

Q: Based on the following information, calculate the expected return and

Based on the following information, calculate the expected return and standard deviation for the two stocks:,,,

See Answer

Q: Rocco Corp. has a book net worth of $10,380. Long-term

Rocco Corp. has a book net worth of $10,380. Long-term debt is $7,500. Net working capital, other than cash, is $2,105. Fixed assets are $15,190. How much cash does the company have? If current liabil...

See Answer

Q: Indicate the effect that the following will have on the

Indicate the effect that the following will have on the operating cycle. Use the letter I to indicate an increase, the letter D for a decrease, and the letter N for no change:a. Average receivables go...

See Answer

Q: The Morning Jolt Coffee Company has projected the following quarterly

The Morning Jolt Coffee Company has projected the following quarterly sales amounts for the coming year:a. Accounts receivable at the beginning of the year are $360. Morning Jolt has a 45-day collecti...

See Answer

Q: Consider the following financial statement information for the Mediate Corporation:

Consider the following financial statement information for the Mediate Corporation:Calculate the operating and cash cycles. How do you interpret your answer?

See Answer

Q: Your firm has an average collection period of 32 days.

Your firm has an average collection period of 32 days. Current practice is to factor all receivables immediately at a 1.5 percent discount. What is the effective cost of borrowing in this case? Assume...

See Answer

Q: Iron Man Products has projected the following sales for the

Iron Man Products has projected the following sales for the coming year:Sales in the year following this one are projected to be 15 percent greater in each quarter.a. Calculate payments to suppliers a...

See Answer

Q: The Torrey Pine Corporation’s purchases from suppliers in a quarter

The Torrey Pine Corporation’s purchases from suppliers in a quarter are equal to 75 percent of the next quarter’s forecast sales. The payables period is 60 days. Wa...

See Answer

Q: In a typical month, the Jeremy Corporation receives 80 checks

In a typical month, the Jeremy Corporation receives 80 checks totaling $156,000. These are delayed four days on average. What is the average daily float?

See Answer

Q: No More Books Corporation has an agreement with Floyd Bank

No More Books Corporation has an agreement with Floyd Bank whereby the bank handles $4 million in collections a day and requires a $400,000 compensating balance. No More Books is contemplating canceli...

See Answer

Q: Each business day, on average, a company writes checks totaling

Each business day, on average, a company writes checks totaling $14,000 to pay its suppliers. The usual clearing time for the checks is four days. Meanwhile, the company is receiving payments from its...

See Answer

Q: A portfolio is invested 25 percent in Stock G, 55

A portfolio is invested 25 percent in Stock G, 55 percent in Stock J, and 20 percent in Stock K. The expected returns on these stocks are 8 percent, 15 percent, and 24 percent, respectively. What is t...

See Answer

Q: Your neighbor goes to the post office once a month

Your neighbor goes to the post office once a month and picks up two checks, one for $17,000 and one for $6,000. The larger check takes four days to clear after it is deposited; the smaller one takes f...

See Answer

Q: Your firm has an average receipt size of $108. A

Your firm has an average receipt size of $108. A bank has approached you concerning a lockbox service that will decrease your total collection time by two days. You typically receive 8,500 checks per...

See Answer

Q: Paper Submarine Manufacturing is investigating a lockbox system to reduce

Paper Submarine Manufacturing is investigating a lockbox system to reduce its collection time. It has determined the following:The total collection time will be reduced by three days if the lockbox sy...

See Answer

Q: It takes Cookie Cutter Modular Homes, Inc., about six days

It takes Cookie Cutter Modular Homes, Inc., about six days to receive and deposit checks from customers. Cookie Cutter’s management is considering a lockbox system to reduce the firm’s collection time...

See Answer

Q: No More Pencils, Inc., disburses checks every two weeks that

No More Pencils, Inc., disburses checks every two weeks that average $93,000 and take seven days to clear. How much interest can the company earn annually if it delays transfer of funds from an intere...

See Answer

Q: You place an order for 400 units of inventory at

You place an order for 400 units of inventory at a unit price of $125. The supplier offers terms of 1/10, net 30.a. How long do you have to pay before the account is overdue? If you take the full peri...

See Answer

Q: Devour, Inc., is considering a change in its cash only

Devour, Inc., is considering a change in its cash only sales policy. The new terms of sale would be net one month. Based on the following information, determine if Devour should proceed or not. Descri...

See Answer

Q: Redan Manufacturing uses 2,500 switch assemblies per week and then

Redan Manufacturing uses 2,500 switch assemblies per week and then reorders another 2,500. If the relevant carrying cost per switch assembly is $9, and the fixed order cost is $1,700, is Redan’s inven...

See Answer

Q: The Techtronic’s store begins each week with 300 phasers in

The Techtronic’s store begins each week with 300 phasers in stock. This stock is depleted each week and reordered. If the carrying cost per phaser is $41 per year and the fixed order cost is $95, what...

See Answer

Q: The Sand Surfer Corporation has annual sales of $47 million.

The Sand Surfer Corporation has annual sales of $47 million. The average collection period is 36 days. What is the average investment in accounts receivable as shown on the balance sheet?

See Answer

Q: Consider the following information:

Consider the following information:a. What is the expected return on an equally weighted portfolio of these three stocks?b. What is the variance of a portfolio invested 20 percent each in A and B and...

See Answer

Q: Kyoto Joe, Inc., sells earnings forecasts for Japanese securities. Its

Kyoto Joe, Inc., sells earnings forecasts for Japanese securities. Its credit terms are 210, net 30. Based on experience, 65 percent of all customers will take the discount.a. What is the average coll...

See Answer

Q: Skye Flyer, Inc., has weekly credit sales of $19,400, and

Skye Flyer, Inc., has weekly credit sales of $19,400, and the average collection period is 34 days. The cost of production is 75 percent of the selling price. What is the average accounts receivable f...

See Answer

Q: Rise Above This, Inc., has an average collection period of

Rise Above This, Inc., has an average collection period of 39 days. Its average daily investment in receivables is $47,500. What are annual credit sales? What is the receivables turnover?

See Answer

Q: Essence of Skunk Fragrances, Ltd., sells 5,600 units of its

Essence of Skunk Fragrances, Ltd., sells 5,600 units of its perfume collection each year at a price per unit of $425. All sales are on credit with terms of 1/10, net 40. The discount is taken by 60 pe...

See Answer

Q: The Arizona Bay Corporation sells on credit terms of net

The Arizona Bay Corporation sells on credit terms of net 30. Its accounts are, on average, 8 days past due. If annual credit sales are $8.4 million, what is the company’s balance sheet amount in accou...

See Answer

Q: Air Spares is a wholesaler that stocks engine components and

Air Spares is a wholesaler that stocks engine components and test equipment for the commercial aircraft industry. A new customer has placed an order for eight high-bypass turbine engines, which increa...

See Answer

Q: Take a look back at Figure 21.1 to answer the

Take a look back at Figure 21.1 to answer the following questions:Figure 21.1:a. If you have $100, how many euros can you get?b. How much is one euro worth?c. If you have 5 million euros, how many dol...

See Answer

Q: Suppose the spot and six-month forward rates on the Norwegian

Suppose the spot and six-month forward rates on the Norwegian krone are Kr 5.15 and Kr 5.22, respectively. The annual risk-free rate in the United States is 3.8 percent, and the annual risk-free rate...

See Answer

Q: You observe that the inflation rate in the United States

You observe that the inflation rate in the United States is 3.9 percent per year and that T-bills currently yield 5.8 percent annually. What do you estimate the inflation rate to be in:a. Australia, i...

See Answer

Q: Suppose the spot and three-month forward rates for the yen

Suppose the spot and three-month forward rates for the yen are ¥114.32 and ¥116.03, respectively.a. Is the yen expected to get stronger or weaker?b. What would you estimate is the difference between t...

See Answer

Q: Refer to Table 12.1 in the text and look at

Refer to Table 12.1 in the text and look at the period from 1973 through 1980:Table 12.1:a. Calculate the average return for Treasury bills and the average annual inflation rate (consumer price index)...

See Answer

Q: The Abercrombie Supply Company reported the following information for 2017. Prepare

The Abercrombie Supply Company reported the following information for 2017. Prepare a common-size income statement for the year ended June 30, 2017? Abercrombie Supply Company Income Statement for the...

See Answer

Q: Emily Smith deposits $1,200 in her bank today.

Emily Smith deposits $1,200 in her bank today. a. If the bank pays 4 percent simple interest, how much money will she have at the end of five years? b. What if the bank pays compound interest? c. H...

See Answer

Q: Caroline Weslin needs to decide whether to accept a bonus of $

Caroline Weslin needs to decide whether to accept a bonus of $1,820 today or wait two years and receive $2,100 then. She can invest at 6 percent. What should she do?

See Answer

Q: Your aunt is planning to invest in a bank CD that will

Your aunt is planning to invest in a bank CD that will pay 7.5 percent interest compounded semiannually. If she has $5,000 to invest, how much will she have at the end of four years?

See Answer

Q: Congress and the President have decided to increase the federal tax rate

Congress and the President have decided to increase the federal tax rate in an effort to reduce the budget deficit. Suppose that Caroline Weslin, from problem 5.29, will pay 35 percent of her bonus to...

See Answer

Q: You have $2,500 that you want to invest in

You have $2,500 that you want to invest in your classmate’s start-up business. You believe the business idea to be great and hope to get $3,700 back at the end of three years. If all goes according to...

See Answer

Q: Patrick Seeley has $2,400 to invest. His brother

Patrick Seeley has $2,400 to invest. His brother approached him with an investment opportunity that could double his money in four years. What interest rate would the investment have to yield in order...

See Answer

Q: You have $12,000 in cash. You can deposit

You have $12,000 in cash. You can deposit it today in a mutual fund earning 8.2 percent compounded semiannually, or you can wait, enjoy some of it, and invest $11,000 in your brother’s business in two...

See Answer

Q: When you were born your parents set up a bank account in

When you were born your parents set up a bank account in your name with an initial investment of $5,000. You are turning 21 in a few days and will have access to all your funds. The account was earnin...

See Answer

Q: Jared Goff, the number 1 draft pick of the NFL Los

Jared Goff, the number 1 draft pick of the NFL Los Angeles Rams in 2016, and his agent are evaluating three contract options. Each option offers a signing bonus and a series of payments over the life...

See Answer

Q: Surmec, Inc., reported sales of $2.1 million

Surmec, Inc., reported sales of $2.1 million last year. The company’s primary business is the manufacture of nuts and bolts. Since this is a mature industry, analysts are confident that sales will gro...

See Answer

Q: You will be graduating in two years and are thinking about your

You will be graduating in two years and are thinking about your future. You know that you will want to buy a house five years after you graduate and that you will want to put down $60,000. As of right...

See Answer

Q: Kelly Martin has $10,000 that she can deposit into

Kelly Martin has $10,000 that she can deposit into a savings account for five years. Bank A pays compounds interest annually, Bank B twice a year, and Bank C quarterly. Each bank has a stated interest...

See Answer

Q: Kate Eden received a graduation present of $2,000 that

Kate Eden received a graduation present of $2,000 that she is planning on investing in a mutual fund that earns 8.5 percent each year. How much money will she have in three years?

See Answer

Q: Your bank pays 5 percent interest semiannually on your savings account.

Your bank pays 5 percent interest semiannually on your savings account. You don’t expect to add to the current balance of $2,700 over the next four years. How much money can you expect to have at the...

See Answer

Q: Your birthday is coming up and instead of other presents, your

Your birthday is coming up and instead of other presents, your parents promised to give you $1,000 in cash. Since you have a part time job and thus don’t need the cash immediately, you decide to inves...

See Answer

Q: Find the future value of a five-year $100,

Find the future value of a five-year $100,000 investment that pays 8.75 percent and that has the following compounding periods: a. Quarterly. b. Monthly. c. Daily. d. Continuous.

See Answer

Q: Joe Mauer, a catcher for the Minnesota Twins, is expected

Joe Mauer, a catcher for the Minnesota Twins, is expected to hit 15 home runs in 2018. If his home-run-hitting ability is expected to grow by 12 percent every year for the following five years, how ma...

See Answer

Q: Roy Gross is considering an investment that pays 7.6 percent

Roy Gross is considering an investment that pays 7.6 percent, compounded annually. How much will he have to invest today so that the investment will be worth $25,000 in six years?

See Answer

Q: Konerko, Inc., management expects the company to earn cash flows

Konerko, Inc., management expects the company to earn cash flows of $13,227, $15,611, $18,970, and $19,114 over the next four years. If the company uses an 8 percent discount rate, what is the future...

See Answer

Q: Cecelia Thomas is a sales executive at a Baltimore firm. She

Cecelia Thomas is a sales executive at a Baltimore firm. She is 25 years old and plans to invest $3,000 every year in an IRA account, beginning at the end of this year until she reaches the age of 65....

See Answer

Q: Refer to Problem 6.10. If Cecelia Thomas invests at

Refer to Problem 6.10. If Cecelia Thomas invests at the beginning of each year, how much will she have at age 65? Refer to problem 6.10: Cecelia Thomas is a sales executive at a Baltimore firm. She i...

See Answer

Q: Kevin Winthrop is saving for an Australian vacation in three years.

Kevin Winthrop is saving for an Australian vacation in three years. He estimates that he will need $5,000 to cover his airfare and all other expenses for a week-long holiday in Australia. If he can in...

See Answer

Q: What effect does an increase in the demand for business goods and

What effect does an increase in the demand for business goods and services have on the real interest rate? What other factors can affect the real interest rate?

See Answer

Q: The Elkridge Bar & Grill has a seven-year loan of

The Elkridge Bar & Grill has a seven-year loan of $23,500 with Bank of America. It plans to repay the loan in seven equal installments starting today. If the rate of interest is 8.4 percent, how much...

See Answer

Q: Your grandfather is retiring at the end of next year. He

Your grandfather is retiring at the end of next year. He would like to ensure that his heirs receive payments of $10,000 a year forever, starting when he retires. If he can earn 6.5 percent annually,...

See Answer

Q: Calculate the annual cash flows for each of the following investments:

Calculate the annual cash flows for each of the following investments: a. $250,000 invested at 6% b. $50,000 invested at 12% c. $100,000 invested at 10%

See Answer

Q: Marshall Chavez bought a Honda Civic for $17,345.

Marshall Chavez bought a Honda Civic for $17,345. He put down $6,000 and financed the rest through the dealer at an APR of 4.9 percent for four years. What is the effective annual interest rate (EAR)...

See Answer

Q: Cyclone Rentals borrowed $15,550 from a bank for three

Cyclone Rentals borrowed $15,550 from a bank for three years. If the quoted rate (APR) is 6.75 percent, and the compounding is daily, what is the effective annual interest rate (EAR)?

See Answer

Q: You are evaluating a growing perpetuity investment from a large financial services

You are evaluating a growing perpetuity investment from a large financial services firm. The investment promises an initial payment of $20,000 at the end of this year and subsequent payments that will...

See Answer

Q: Trigen Corp. management will invest $331,000, $

Trigen Corp. management will invest $331,000, $616,450, $212,775, $818,400, $1,239,644, and $1,617,848 in research and development over the next six years. If the appropriate interest rate is 6.75 per...

See Answer

Q: Ben Woolmer has an investment that will pay him the following cash

Ben Woolmer has an investment that will pay him the following cash flows over the next five years: $2,350, $2,725, $3,128, $3,366, and $3,695. If his investments typically earn 7.65 percent, what is t...

See Answer

Q: Stephanie Watson is 20 years old and plans to make the following

Stephanie Watson is 20 years old and plans to make the following investments beginning next year. She will invest $3,125 in each of the next three years and will then make investments of $3,650, $3,72...

See Answer

Q: Carol Jenkins, a lottery winner, will receive the following payments

Carol Jenkins, a lottery winner, will receive the following payments over the next seven years. She has been approached by an investor who will pay Carol a lump sum today for the rights to those futur...

See Answer

Q: The Huntington Rain Gear Company had $633,125 in taxable

The Huntington Rain Gear Company had $633,125 in taxable income in the year ending September 30, 2017. Calculate the company’s tax using the tax schedule in Exhibit 3.6?

See Answer

Q: Gary Whitmore is a high school sophomore. He currently has $

Gary Whitmore is a high school sophomore. He currently has $7,500 in a savings account that pays5.65 percent annually. Gary plans to use his current savings plus what he can save over the next four ye...

See Answer

Q: Modern Energy Company owns several gas stations. Management is looking to

Modern Energy Company owns several gas stations. Management is looking to open a new station in the western suburbs of Baltimore. One possibility that managers at the company are evaluating is to take...

See Answer

Q: Jeremy Denham plans to save $5,000 every year for

Jeremy Denham plans to save $5,000 every year for the next eight years, starting today. At the end of eight years, Jeremy will turn 30 years old and plans to use his savings toward the down payment on...

See Answer

Q: Grant Productions borrowed some money from the California Finance Company at a

Grant Productions borrowed some money from the California Finance Company at a rate of 17.5 percent for a seven-year period. The loan calls for a payment of $1,540,862.19 each year beginning today. Ho...

See Answer

Q: Sharon Kabana has won a state lottery and will receive a payment

Sharon Kabana has won a state lottery and will receive a payment of $89,729.45 every year, starting today, for the next 20 years. If she invests the proceeds at a rate of 7.25 percent, what is the pre...

See Answer

Q: You wrote a piece of software that does a better job of

You wrote a piece of software that does a better job of allowing computers to network than any other program designed for this purpose. A large networking company wants to incorporate your software in...

See Answer

Q: Suppose that the networking company in problem 6.27 will not

Suppose that the networking company in problem 6.27 will not start paying you until the first of new systems that uses your software is sold in two years. What is the present value of that annuity? As...

See Answer

Q: Calculate the present value of the following perpetuities: a.

Calculate the present value of the following perpetuities: a. $1,250 discounted to the present at 7 percent b. $7,250 discounted to the present at 6.33 percent c. $850 discounted to the present at 20...

See Answer

Q: You are a freshman in college and are planning a trip to

You are a freshman in college and are planning a trip to Europe when you graduate from college at the end of four years. You plan to save the following amounts annually, starting today: $625, $700, $7...

See Answer

Q: Find the effective annual interest rate (EAR) on each of

Find the effective annual interest rate (EAR) on each of the following: a. 6 percent compounded quarterly. b. 4.99 percent compounded monthly. c. 7.25 percent compounded semiannually. d. 5.6 percent c...

See Answer

Q: You lent $100 to a friend for one year at a

You lent $100 to a friend for one year at a nominal rate of interest of 3 percent. Inflation during that year was 2 percent. Did you experience an increase or decrease in the purchasing power of you...

See Answer

Q: Which of the following investments has the highest effective annual interest rate

Which of the following investments has the highest effective annual interest rate (EAR)? a. A bank CD that pays 8.25 percent compounded quarterly. b. A bank CD that pays 8.25 percent compounded monthl...

See Answer

Q: You are considering three alternative investments: (1) A

You are considering three alternative investments: (1) A three-year bank CD paying 7.5 percent compounded quarterly; (2) A three-year bank CD paying 7.3 percent compounded monthly; and (3) A three-yea...

See Answer

Q: You have been offered the opportunity to invest in a project which

You have been offered the opportunity to invest in a project which is expected to provide you with the following cash flows: $4,000 in one year, $12,000 in two years, and $8,000 in three years. If the...

See Answer

Q: Tirade Owens, a professional athlete, currently has a contract that

Tirade Owens, a professional athlete, currently has a contract that will pay him a large amount in the first year of his contract and smaller amounts thereafter. He and his agent have asked the team t...

See Answer

Q: Gary Kornig is 30 years old and wants to retire when he

Gary Kornig is 30 years old and wants to retire when he is 65. So far he has saved (1) $6,950 in an IRA account in which his money is earning 8.3 percent annually and (2) $5,000 in a money market acco...

See Answer

Q: The top prize for the state lottery is $100,000

The top prize for the state lottery is $100,000,000. You have decided it is time for you to take a chance and purchase a ticket. Before you purchase the ticket, you must decide whether to choose the c...

See Answer

Q: At what interest rate would you be indifferent between the cash and

At what interest rate would you be indifferent between the cash and annual payment options in problem 6.36?

See Answer

Q: Babu Baradwaj is saving for his son’s college tuition. His son

Babu Baradwaj is saving for his son’s college tuition. His son is currently 11 years old and will begin college in seven years. Babu has an index fund investment of $7,500 that is earning 9.5 percent...

See Answer

Q: You are now 50 years old and plan to retire at age

You are now 50 years old and plan to retire at age 65. You currently have a stock portfolio worth $150,000, a 401(k) retirement plan worth $250,000, and a money market account worth $50,000. Your stoc...

See Answer

Q: Saul Cervantes has just purchased some equipment for his landscaping business.

Saul Cervantes has just purchased some equipment for his landscaping business. For this equipment he must pay the following amounts at the end of each of the next five years: $10,450, $8,500, $9,675,...

See Answer

Q: You are given the following information about Clarkesville Plumbing Company. Revenues

You are given the following information about Clarkesville Plumbing Company. Revenues in 2017 totaled $896, depreciation expenses $75, costs of goods sold $365, and interest expenses $54. At the end o...

See Answer

Q: Trevor Diaz wants to purchase a Maserati Qattroporte sedan, which has

Trevor Diaz wants to purchase a Maserati Qattroporte sedan, which has an invoice price of $121,737 and a total cost of $129,482. Trevor plans to put down $20,000 and will pay the rest by taking on a 5...

See Answer

Q: The Yan family buying a new 3,500-square-

The Yan family buying a new 3,500-square-foot house in Muncie, Indiana, and will borrow $237,000 from Bank One at a rate of 6.375 percent for 15 years. What will be their monthly loan payment? Prepare...

See Answer

Q: Assume you are now 21 years old and will start working as

Assume you are now 21 years old and will start working as soon as you graduate from college. You plan to start saving for your retirement on your 25th birthday and retire on your 65th birthday. After...

See Answer

Q: Jeremy Fenloch borrowed some money from his friend and promised to repay

Jeremy Fenloch borrowed some money from his friend and promised to repay him $1,225, $1,350, $1,500, $1,600, and $1,600 over the next five years. If the friend normally discounts investments at 8 perc...

See Answer

Q: Biogenesis Inc. management expects the following cash flow stream over the

Biogenesis Inc. management expects the following cash flow stream over the next five years. They discount all cash flows using a 23 percent discount rate. What is the present value of this cash flow s...

See Answer

Q: An investment opportunity requires a payment of $750 for 12 years

An investment opportunity requires a payment of $750 for 12 years, starting a year from today. If your required rate of return is 8 percent, what is the value of the investment to you today?

See Answer

Q: Dynamics Telecommunications Corp. has made an investment in another company that

Dynamics Telecommunications Corp. has made an investment in another company that will guarantee it a cash flow of $22,500 each year for the next five years. If the company uses a discount rate of 15 p...

See Answer

Q: Robert Hobbes plans to invest $25,000 a year at

Robert Hobbes plans to invest $25,000 a year at the end of each year for the next seven years in an investment that will pay him a rate of return of 11.4 percent. How much money will Robert have at th...

See Answer

Q: Describe the difference between a total holding period return and an expected

Describe the difference between a total holding period return and an expected return?

See Answer

Q: Susan is expecting the returns on the market portfolio to be negative

Susan is expecting the returns on the market portfolio to be negative in the near term. Since she is managing a stock mutual fund, she must remain invested in a portfolio of stocks. However, she is al...

See Answer

Q: Amit Patel is planning to invest $10,000 in a

Amit Patel is planning to invest $10,000 in a bank certificate of deposit (CD) for five years. The CD will pay interest of 9 percent. What is the future value of Amit’s investment?

See Answer

Q: Describe and justify what the value of the beta of a U

Describe and justify what the value of the beta of a U.S. Treasury bill should be?

See Answer

Q: If the expected rate of return for the market is not much

If the expected rate of return for the market is not much greater than the risk-free rate of return, what does this suggest about the general level of compensation for bearing systematic risk?

See Answer

Q: Describe the Capital Asset Pricing Model (CAPM) and what it

Describe the Capital Asset Pricing Model (CAPM) and what it tells us?

See Answer

Q: The distribution of grades in an introductory finance class is normally distributed

The distribution of grades in an introductory finance class is normally distributed, with an expected grade of 75. If the standard deviation of grades is 7, in what range would you expect 95 percent o...

See Answer

Q: Barbara is considering investing in a company’s stock and is aware that

Barbara is considering investing in a company’s stock and is aware that the return on that investment is particularly sensitive to how the economy is performing. Her analysis suggest...

See Answer

Q: Ben would like to invest in gold and is aware that the

Ben would like to invest in gold and is aware that the returns on such an investment can be quite volatile. Use the following table of states, probabilities, and returns to determine the expected retu...

See Answer

Q: John is watching an old game show rerun on television called Let’s

John is watching an old game show rerun on television called Let’s Make a Deal in which the contestant chooses a prize behind one of two curtains. Behind one of the curtains is a gag prize worth $150,...

See Answer

Q: Using the information from Problems 7.17, 7.18

Using the information from Problems 7.17, 7.18, and 7.19, calculate the coefficient of variation for each of the investments in those problems?

See Answer

Q: Emmy is analyzing a two-stock portfolio that consists of a

Emmy is analyzing a two-stock portfolio that consists of a utility stock and a commodity stock. She knows that the return on the utility stock has a standard deviation of 40 percent and the return on...

See Answer

Q: You have constructed a diversified portfolio of stocks such that there is

You have constructed a diversified portfolio of stocks such that there is no unsystematic risk. Explain why the expected return of that portfolio should be greater than the expected return of a risk-f...

See Answer

Q: The Ellicott City Ice Cream Company management has just completed an assessment

The Ellicott City Ice Cream Company management has just completed an assessment of the company’s assets and liabilities and has obtained the following information. The firm has total current assets wo...

See Answer

Q: Write out the equation for the covariance in the returns of two

Write out the equation for the covariance in the returns of two assets, Asset 1 and Asset 2. Using that equation, explain the easiest way for the two asset returns to have a covariance of zero?

See Answer

Q: Evaluate the following statement: “By fully diversifying a portfolio,

Evaluate the following statement: “By fully diversifying a portfolio, such as by buying every asset in the market, we can completely eliminate all types of risk, thereby creating a synthetic Treasury...

See Answer

Q: You have chosen biology as your college major because you would like

You have chosen biology as your college major because you would like to be a medical doctor. However, you find that the probability of being accepted to medical school is about 10 percent. If you are...

See Answer

Q: Peter knows that the covariance in the return on two assets is

Peter knows that the covariance in the return on two assets is –0.0025. Without knowing the expected return of the two assets, explain what that covariance means?

See Answer

Q: In order to expect that it will fund her retirement, Glenda

In order to expect that it will fund her retirement, Glenda needs her portfolio to have an expected return of 12 percent per year over the next 30 years. She has decided to invest in Stocks 1, 2, and...

See Answer

Q: Explain why investors who have diversified their portfolios will determine the price

Explain why investors who have diversified their portfolios will determine the price and, consequently, the expected return on an asset?

See Answer

Q: Brad is about to purchase an additional asset for his well-

Brad is about to purchase an additional asset for his well-diversified portfolio. He notices that when he plots the historical returns of the asset against those of the market portfolio, the line of b...

See Answer

Q: Draw the Security Market Line (SML) for the case where

Draw the Security Market Line (SML) for the case where the market risk premium is 5 percent and the risk-free rate is 7 percent. Now suppose an asset has a beta of –1.0 and an expected return of 4 per...

See Answer

Q: If the CAPM describes the relation between systematic risk and expected returns

If the CAPM describes the relation between systematic risk and expected returns, can both an individual asset and the market portfolio of all risky assets have negative expected real rates of return?...

See Answer

Q: You have been provided the following data on the securities of three

You have been provided the following data on the securities of three firms and the market: Assume the CAPM and SML are true and fill in the missing values in the table. Would you invest in the stock...

See Answer

Q: Mike White is planning to save up for a trip to Europe

Mike White is planning to save up for a trip to Europe in three years. He will need $7,500 when he is ready to make the trip. He plans to invest the same amount at the end of each of the next three ye...

See Answer

Q: Describe the general relation between risk and return that we observe in

Describe the general relation between risk and return that we observe in the historical bond and stock market data?

See Answer

Q: Stocks A, B, and C have expected returns of 15

Stocks A, B, and C have expected returns of 15 percent, 15 percent, and 12 percent, respectively, while their standard deviations are 45 percent, 30 percent, and 30 percent, respectively. If you were...

See Answer

Q: What was the average annual return that Tanner earned over the 2012

What was the average annual return that Tanner earned over the 2012 through 2015 period (see problem 7.6)?

See Answer

Q: Describe how investing in more than one asset can reduce risk through

Describe how investing in more than one asset can reduce risk through diversification?

See Answer

Q: BA Corp is issuing a 10-year bond with a coupon

BA Corp is issuing a 10-year bond with a coupon rate of 8 percent. The interest rate for similar bonds is currently 6 percent. Assuming annual payments, what is the value of the bond?

See Answer

Q: Rudy Sandberg wants to invest in four-year bonds that are

Rudy Sandberg wants to invest in four-year bonds that are currently priced at $868.43. These bonds have a coupon rate of 6 percent and make semiannual coupon payments. What is the current market yield...

See Answer

Q: Josh Kavern bought 10-year, 12 percent coupon bonds issued

Josh Kavern bought 10-year, 12 percent coupon bonds issued by the U.S. Treasury three years ago at $913.44. If he sells these bonds, for which he paid the face value of $1,000, at the current price of...

See Answer

Q: Four years ago, Lisa Stills bought six-year, 5

Four years ago, Lisa Stills bought six-year, 5.5 percent coupon bonds issued by the Fairways Corp. for $947.68. If she sells these bonds at the current price of $894.52, what will be her realized yiel...

See Answer

Q: The International Publishing Group is raising $10 million by issuing 15

The International Publishing Group is raising $10 million by issuing 15-year bonds with a coupon rate of 8.5 percent. Coupon payments will be made annually. Investors buying the bonds today will earn...

See Answer

Q: Lopez Information Systems management is planning to issue 10-year bonds

Lopez Information Systems management is planning to issue 10-year bonds. The going market yield for such bonds is 8.125 percent. Assume that coupon payments will be made semiannually. Management is tr...

See Answer

Q: Becky Scholes has $150,000 to invest. She wants

Becky Scholes has $150,000 to invest. She wants to be able to withdraw $12,500 every year forever without using up any of her principal. What interest rate would her investment have to earn in order f...

See Answer

Q: Marshall Company is issuing eight-year bonds with a coupon rate

Marshall Company is issuing eight-year bonds with a coupon rate of 6.5 percent and semiannual coupon payments. If the current market rate for similar bonds is 8 percent, what will be the bond price? I...

See Answer

Q: Rockne, Inc., has outstanding bonds that will mature in six

Rockne, Inc., has outstanding bonds that will mature in six years and pay an 8 percent coupon semiannually. If you paid $1036.65 today and your required rate of return was 6.6 percent, did you pay the...

See Answer

Q: Nanotech, Inc., has a bond issue maturing in seven years

Nanotech, Inc., has a bond issue maturing in seven years that is paying a coupon rate of 9.5 percent (semiannual payments). Management wants to retire a portion of the issue by buying the securities i...

See Answer

Q: Kintel, Inc., management wants to raise $1 million by

Kintel, Inc., management wants to raise $1 million by issuing six-year zero coupon bonds with a face value of $1,000. The company’s investment banker states that investors would use an 11.4 percent di...

See Answer

Q: Rockinghouse Corp. management plans to issue seven-year zero coupon

Rockinghouse Corp. management plans to issue seven-year zero coupon bonds. It has learned that these bonds will sell today at a price of $439.76. Assuming annual coupon payments, what is the yield to...

See Answer

Q: Pierre Dupont just received a cash gift from his grandfather. He

Pierre Dupont just received a cash gift from his grandfather. He plans to invest in a five-year bond issued by Venice Corp. that pays an annual coupon rate of 5.5 percent. If the current market rate i...

See Answer

Q: Electrolex, Inc., has four-year bonds outstanding that pay

Electrolex, Inc., has four-year bonds outstanding that pay a coupon rate of 6.6 percent and make coupon payments semiannually. If these bonds are currently selling at $914.89, what is the yield to mat...

See Answer

Q: Serengeti Corp. has five-year bonds outstanding that pay a

Serengeti Corp. has five-year bonds outstanding that pay a coupon of 8.8 percent. If these bonds are priced at $1,064.86, what is the yield to maturity on these bonds? Assume semiannual coupon payment...

See Answer

Q: Adrienne Dawson is planning to buy 10-year zero coupon bonds

Adrienne Dawson is planning to buy 10-year zero coupon bonds issued by the U.S. Treasury. If these bonds have a face value of $1,000 and are currently selling at $404.59, what is the expected return o...

See Answer

Q: Brown & Co. issued seven-year bonds two years ago

Brown & Co. issued seven-year bonds two years ago that can be called after two years. The bonds make semiannual coupon payments at a coupon rate of 7.875 percent. Each bond has a market value of $1,05...

See Answer

Q: Prepare a common-size balance sheet from the following information for

Prepare a common-size balance sheet from the following information for Abercrombie Supply Company.

See Answer

Q: Dynamo Corp. is expecting annual payments of $34,225

Dynamo Corp. is expecting annual payments of $34,225 for the next seven years from a customer. What is the present value of this annuity if the discount rate is 8.5 percent?

See Answer

Q: Trevor Price bought 10-year bonds issued by Harvest Foods five

Trevor Price bought 10-year bonds issued by Harvest Foods five years ago for $936.05. The bonds make semiannual coupon payments at a rate of 8.4 percent. If the current price of the bonds is $1,048.77...

See Answer

Q: You bought a six-year bond issued by Runaway Corp.

You bought a six-year bond issued by Runaway Corp. four years ago. At that time, you paid $974.33 for the bond. The bond pays a coupon rate of 7.375 percent, and coupon payments are made semiannually....

See Answer

Q: Pullman Corp issued 10-year bonds four years ago with a

Pullman Corp issued 10-year bonds four years ago with a coupon rate of 9.375 percent. At the time of issue, the bonds sold at par. Today bonds of similar risk and maturity must pay an annual coupon of...

See Answer

Q: Showbiz, Inc., has issued eight-year bonds with a

Showbiz, Inc., has issued eight-year bonds with a coupon of 6.375 percent and semiannual coupon payments. The market’s required rate of return on such bonds is 7.65 percent. a. What is the market pri...

See Answer

Q: Peabody Corp. has seven-year bonds outstanding. The bonds

Peabody Corp. has seven-year bonds outstanding. The bonds pay a coupon of 8.375 percent semiannually and are currently worth $1,063.49. The bonds can be called in three years at price of $1,075. a. Wh...

See Answer

Q: The Maryland Department of Transportation has issued 25-year bonds that

The Maryland Department of Transportation has issued 25-year bonds that make semiannual coupon payments at a rate of 9.875 percent. The current market rate for similar securities is 11 percent. a. Wh...

See Answer

Q: Knight, Inc., has issued a three-year bond that

Knight, Inc., has issued a three-year bond that pays a coupon rate of 6.10 percent. Coupon payments are made semiannually. Given the market rate of interest of 5.80 percent, what is the market value o...

See Answer

Q: Rachette Corp. has18-year bonds outstanding. These bonds,

Rachette Corp. has18-year bonds outstanding. These bonds, which pay interest semiannually, have a coupon rate of 9.735 percent and a yield to maturity of 7.95 percent. a. Compute the current price of...

See Answer

Q: Zippy Corporation just sold $30 million of convertible bonds with a

Zippy Corporation just sold $30 million of convertible bonds with a conversion ratio of 40. Each $1,000 bond is convertible into 25 shares of Zippy’s stock. a. What is the conversion price of Zippy’s...

See Answer

Q: Regatta, Inc., has seven-year bonds outstanding that pay

Regatta, Inc., has seven-year bonds outstanding that pay a 12 percent coupon rate. Investors buying these bonds today can expect to earn a yield to maturity of 8.875 percent. What is the current value...

See Answer

Q: Kaaran made a friendly wager with a colleague that involves the result

Kaaran made a friendly wager with a colleague that involves the result from flipping a coin. If heads comes up, Kaaran must pay her colleague $15; otherwise, her colleague will pay Kaaran $15. What is...

See Answer

Q: You are interested in investing in a five-year bond that

You are interested in investing in a five-year bond that pays a 7.8 percent coupon rate with interest to be received semiannually. Your required rate of return is 8.4 percent. What is the most you wou...

See Answer

Q: Diane Carter is interested in buying a five-year zero coupon

Diane Carter is interested in buying a five-year zero coupon bond with a face value of $1,000. She understands that the market interest rate for similar investments is 9 percent. Assume annual coupon...

See Answer

Q: Ten-year zero coupon bonds issued by the U.S

Ten-year zero coupon bonds issued by the U.S. Treasury have a face value of $1,000 and interest is compounded semiannually. If similar bonds in the market yield 10.5 percent, what is the value of thes...

See Answer

Q: Northrop Real Estate Company management is planning to fund a development project

Northrop Real Estate Company management is planning to fund a development project by issuing 10-year zero coupon bonds with a face value of $1,000. Assuming semiannual compounding, what will be the pr...

See Answer

Q: Ruth Hornsby is looking to invest in a three-year bond

Ruth Hornsby is looking to invest in a three-year bond that makes semiannual coupon payments at a rate of 5.875 percent. If these bonds have a market price of $981.13, what yield to maturity can she e...

See Answer

Q: What is the difference between saver–lenders and borrower–spenders

What is the difference between saver–lenders and borrower–spenders, and who are the major representatives of each group?

See Answer

Q: How does the use of market-value accounting help managers?

How does the use of market-value accounting help managers?

See Answer

Q: Suppose you own a security that you know can be easily sold

Suppose you own a security that you know can be easily sold in the secondary market, but the security will sell at a lower price than you paid for it. What does this imply for the security’s marketabi...

See Answer

Q: The financial information for Laurel Electronics referred to in Problem 3.

The financial information for Laurel Electronics referred to in Problem 3.5 is all at book value. Suppose marking to market reveals that the market value of the firm’s inventory is 20 percent below it...

See Answer

Q: Trader Inc. is a $300 million company, as measured

Trader Inc. is a $300 million company, as measured by asset value, and Horst Corp. is a $35 million company. Both are privately held corporations. Explain which firm more likely to go public and regis...

See Answer

Q: You know that the price of CFI, Inc., stock will

You know that the price of CFI, Inc., stock will be $12 exactly one year from today. Today the price of the stock is $11. Describe what must happen to the price of CFI, Inc., today in order for an inv...

See Answer

Q: Describe the organization of the statement of cash flows?

Describe the organization of the statement of cash flows?

See Answer

Q: During 2017, Towson Recording Company increased its investment in marketable securities

During 2017, Towson Recording Company increased its investment in marketable securities by $36,845, funded fixed-assets acquisitions of $109,455, and had marketable securities of $14,215 mature. What...

See Answer

Q: What does it mean to “underwrite” a new security issue

What does it mean to “underwrite” a new security issue? What compensation does an investment banker get from underwriting a security issue?

See Answer

Q: Identify and describe the noncash expenses that a firm may incur?

Identify and describe the noncash expenses that a firm may incur?

See Answer

Q: Caustic Chemicals management identified the following cash flows as significant in its

Caustic Chemicals management identified the following cash flows as significant in its year-end meeting with analysts. During the year Caustic had repaid existing debt of $312,080 and raised additiona...

See Answer

Q: Explain how the choice of FIFO versus LIFO can affect a firm’s

Explain how the choice of FIFO versus LIFO can affect a firm’s balance sheet and income statement?

See Answer

Q: Laurel Electronics reported the following information at its annual meetings: The

Laurel Electronics reported the following information at its annual meetings: The company had cash and marketable securities worth $1,235,455, accounts payables worth $4,159,357, inventory of $7,121,5...

See Answer

Q: The Oakland Mills Company has disclosed the following financial information in its

The Oakland Mills Company has disclosed the following financial information in its annual reports for the period ending March 31, 2017: sales of $1.45 million, cost of goods sold of $812,500, deprecia...

See Answer

Q: What is the primary role of money markets? Explain how the

What is the primary role of money markets? Explain how the money markets work?

See Answer

Q: Manz Property Management Company announced that in the year ended June 30

Manz Property Management Company announced that in the year ended June 30, 2017, its earnings before taxes amounted to $1,478,936. Calculate its taxes using Exhibit 3.6?

See Answer

Q: The expected value of a normal distribution of prices for a stock

The expected value of a normal distribution of prices for a stock is $50. If you are 90 percent sure that the price of the stock will be between $40 and $60, then what is the variance of the prices fo...

See Answer

Q: Tim Dye, the CFO of Blackwell Automotive, Inc., is

Tim Dye, the CFO of Blackwell Automotive, Inc., is putting together this year’s financial statements. He has gathered the following balance sheet information. The firm had a cash balance of $23,015, a...

See Answer

Q: Describe the informational differences that distinguish the three forms of market efficiency

Describe the informational differences that distinguish the three forms of market efficiency?

See Answer

Q: Zippy Computers announced strong fourth quarter results. Sales and earnings were

Zippy Computers announced strong fourth quarter results. Sales and earnings were both above analysts’ expectations. You notice in the newspaper that Zippy’s stock price went up sharply on the day of t...

See Answer

Q: In problem 2.20, if the market is efficient,

In problem 2.20, if the market is efficient, would it have been possible for Zippy’s stock price to go down on the day that the firm announced the strong fourth quarter results?

See Answer

Q: If the market is strong-form efficient, then trading on

If the market is strong-form efficient, then trading on tips you hear from Jim Cramer (the host of Mad Money on CNBC) will generate no excess returns (i.e., returns in excess of fair compensation for...

See Answer

Q: Fraser Corporation has announced that its net income for the year ended

Fraser Corporation has announced that its net income for the year ended June 30, 2017, was $1,353,412. The company had EBITDA of $ 4,967,855, and its depreciation and amortization expense was equal to...

See Answer

Q: For its most recent fiscal year, Carmichael Hobby Shop recorded EBITDA

For its most recent fiscal year, Carmichael Hobby Shop recorded EBITDA of $512,725.20, EBIT of $362,450.20, zero interest expense, and cash flow to investors from operating activity of $348,461.25. A...

See Answer

Q: What is the real rate of interest, and how is it

What is the real rate of interest, and how is it determined?

See Answer

Q: How does the nominal rate of interest vary over time?

How does the nominal rate of interest vary over time?

See Answer

Q: What is the Fisher equation, and how is it used?

What is the Fisher equation, and how is it used?

See Answer

Q: During the period from 2011 through 2015 the annual returns on small

During the period from 2011 through 2015 the annual returns on small U.S. stocks were -3.26 percent, 18.23 percent, 45.07 percent, 2.92 percent, and -3.60 percent, respectively. What would a $1 inves...

See Answer

Q: Imagine you borrow $500 from your roommate, agreeing to pay

Imagine you borrow $500 from your roommate, agreeing to pay her back $500 plus 7 percent nominal interest in one year. Assume inflation over the life of the contract is expected to be 4.25 percent. Wh...

See Answer

Q: Your parents have given you $1,000 a year before

Your parents have given you $1,000 a year before your graduation so that you can take a trip when you graduate. You wisely decide to invest the money in a bank CD that pays 6.75 percent interest. You...

See Answer

Q: Menomonie Casino Company earned $23,458,933 before interest

Menomonie Casino Company earned $23,458,933 before interest and taxes for the fiscal year ending March 31, 2017. If the casino had interest expenses of $1,645,123, calculate its tax obligation using E...

See Answer

Q: Vanderheiden Hog Products Corp. provided the following financial information for the

Vanderheiden Hog Products Corp. provided the following financial information for the quarter ending June 30, 2017: Net income: …………………….…….………. $189,425 Depreciation and amortization: ….……... $63,114...

See Answer

Q: Analysts following the Tomkovick Golf Company were given the following balance sheet

Analysts following the Tomkovick Golf Company were given the following balance sheet information for the years ended June 30, 2017 and June 30 2016: In addition, it was reported that the company had...

See Answer

Q: Based on the financial statements for Tomkovick Golf Company in Problem 3

Based on the financial statements for Tomkovick Golf Company in Problem 3.32, compute the cash flow invested in net working capital and the cash flow invested in long-term assets that you would use in...

See Answer

Q: Mukhopadhya Network Associates has a current ratio of 1.60,

Mukhopadhya Network Associates has a current ratio of 1.60, where the current ratio is defined as follows: current ratio = current assets/current liabilities. The firm’s current assets are equal to $1...

See Answer

Q: Reservoir Bottling Co. reported the following information at the end of

Reservoir Bottling Co. reported the following information at the end of the year. Total current assets are worth $237,513 at book value and $219,344 at market value. In addition, plant and equipment h...

See Answer

Q: Nimitz Rental Company provided the following information to its auditors. For

Nimitz Rental Company provided the following information to its auditors. For the year ended March 31, 2017, the company had revenues of $878,412, general and administrative expenses of $352,666, depr...

See Answer

Q: Sosa Corporation recently reported an EBITDA of $31.3 million

Sosa Corporation recently reported an EBITDA of $31.3 million and net income of $9.7 million. The company had $6.8 million in interest expense, and its average corporate tax rate was 35 percent. What...

See Answer

Q: You must choose between investing in stock A and stock B.

You must choose between investing in stock A and stock B. You have already used CAPM to calculate the rate of return you should expect to receive for each stock given their systematic risk and decided...

See Answer

Q: Columbia Construction Company earned $451,888 during the year ended

Columbia Construction Company earned $451,888 during the year ended June 30, 2017. After paying out $225,794 in dividends, the balance went into retained earnings. If the firm’s total retained earning...

See Answer

Q: Mount Hebron Electrical Company’s financial statements indicated that the company had earnings

Mount Hebron Electrical Company’s financial statements indicated that the company had earnings before interest and taxes of $718,323. The interest rate on its $850,000 debt was 8.95 percent. Calculate...

See Answer

Q: The Centennial Chemical Corporation announced that, for the period ending March

The Centennial Chemical Corporation announced that, for the period ending March 31, 2017, it had earned income after taxes of $2,768,028.25 on revenues of $13,144,680. The company’s costs (excluding d...

See Answer

Q: Eau Claire Paper Mill, Inc., had, at the beginning

Eau Claire Paper Mill, Inc., had, at the beginning of the current fiscal year, April 1, 2016, retained earnings of $323,325. During the year ended March 31, 2017, the company produced net income after...

See Answer

Q: Explain why the quick ratio or acid-test ratio is a

Explain why the quick ratio or acid-test ratio is a better measure of a firm’s liquidity than the current ratio?

See Answer

Q: Trademark Corp.’s financial manager collected the following information for its

Trademark Corp.’s financial manager collected the following information for its peer group to compare its performance against that of its peers. a. Explain how Trademark is perform...

See Answer

Q: Rockwell Jewelers management announced that the company had net earnings of $

Rockwell Jewelers management announced that the company had net earnings of $6,481,778 for this year. The company has 2,543,800 shares outstanding, and the year-end stock price is $54.21. What are Roc...

See Answer

Q: Chisel Corporation has 3 million shares outstanding at a price per share

Chisel Corporation has 3 million shares outstanding at a price per share of $3.25. If the debt-to-equity ratio is 1.7 and total book value of debt equals $12,400,000, what is the market-to-book ratio...

See Answer

Q: Laurel Electronics has a quick ratio of 1.15, current

Laurel Electronics has a quick ratio of 1.15, current liabilities of $5,311,020, and inventories of $7,121,599. What is the firm’s current ratio?

See Answer

Q: Lambda Corporation has current liabilities of $450,000, a

Lambda Corporation has current liabilities of $450,000, a quick ratio of 1.8, inventory turnover of 5.0, and a current ratio of 3.5. What is the cost of goods sold for Lambda Corporation?

See Answer

Q: Calculate the price of a five-year bond that has a

Calculate the price of a five-year bond that has a coupon of 6.5 percent paid annually. The current market rate is 5.75 percent?

See Answer

Q: Norwood Corp. currently has accounts receivable of $1,223

Norwood Corp. currently has accounts receivable of $1,223,675 on net sales of $6,216,900. What are its accounts receivable turnover and days’ sales outstanding (DSO)?

See Answer

Q: If Norwood Corp.’s management wants to reduce the DSO from

If Norwood Corp.’s management wants to reduce the DSO from that calculated in Problem 4.16 to an industry average of 56.3 days and its net sales are expected to decline by about 12 percent, what would...

See Answer

Q: Nimitz Rental Company had depreciation expenses of $108,905,

Nimitz Rental Company had depreciation expenses of $108,905, interest expenses of $78,112, and an EBIT of $1,254,338 for the year ended June 30, 2017. What are the times interest earned and cash cover...

See Answer

Q: Conseco, Inc., has a debt ratio of 0.56

Conseco, Inc., has a debt ratio of 0.56. What are the company’s debt-to-equity ratio and equity multiplier?

See Answer

Q: Flying Penguins Corp. has total current assets of $11,

Flying Penguins Corp. has total current assets of $11,845,175, current liabilities of $5,311,020, and a quick ratio of 0.89. How much inventory does it have?

See Answer

Q: Cisco Systems has total assets of $35.594 billion,

Cisco Systems has total assets of $35.594 billion, total debt of $9.678 billion, and net sales of $22.045 billion. Its net profit margin for the year is 20 percent, while the operating profit margin i...

See Answer

Q: Procter & Gamble reported the following information for its fiscal year end

Procter & Gamble reported the following information for its fiscal year end: On net sales of $51.407 billion, the company earned net income after taxes of $6.481 billion. It had cost of goods sold of...

See Answer

Q: Wal-Mart, Inc., has net income of $9

Wal-Mart, Inc., has net income of $9,054,000 on net sales of $256,329,812. The company has total assets of $104,912,112 and stockholders’ equity of $43,623,445. Use the extended DuPont identity to fin...

See Answer

Q: Xtreme Sports Innovations has disclosed the following information: EBIT =

Xtreme Sports Innovations has disclosed the following information: EBIT = $25,664,300 ……………………………. Net income = $13,054,000 …………………………………….…………………….…… Net sales = $83,125,336 Total debt = $20,885,753...

See Answer

Q: Cisco Systems had net income of $4.401 billion and

Cisco Systems had net income of $4.401 billion and at year end 6.735 billion shares outstanding. Calculate the earnings per share for the company?

See Answer

Q: Bigbie Corp. issued a five-year bond a year ago

Bigbie Corp. issued a five-year bond a year ago with a coupon of 8 percent. The bond pays interest semiannually. If the yield to maturity on this bond is 9 percent, what is the price of the bond?

See Answer

Q: Use the information for Cisco Systems in Problem 4.24.

Use the information for Cisco Systems in Problem 4.24. In addition, the company’s EBITDA was $6.834 billion and its share price was $22.36. Compute the firm’s price-earnings ratio and the price-EBITDA...

See Answer

Q: Carter, Inc., a manufacturer of electrical supplies, has an

Carter, Inc., a manufacturer of electrical supplies, has an ROE of 23.1 percent, a profit margin of 4.9 percent, and a total asset turnover ratio of 2.6 times. Its peer group also has an ROE of 23.1 p...

See Answer

Q: Grossman Enterprises has an equity multiplier of 2.6 times,

Grossman Enterprises has an equity multiplier of 2.6 times, total assets of $2,312,000, an ROE of 14.8 percent, and a total asset turnover ratio of 2.8 times. Calculate the firm’s sales and ROA?

See Answer

Q: Complete the balance sheet of Flying Roos Corporation. /

Complete the balance sheet of Flying Roos Corporation. You have the following information: Debt ratio = 40% ………..…&acir...

See Answer

Q: For the year ended June 30, 2017, Northern Clothing Company

For the year ended June 30, 2017, Northern Clothing Company has total assets of $87,631,181, ROA of 11.67 percent, ROE of 21.19 percent, and a net profit margin of 11.59 percent. What are the company’...

See Answer

Q: If Newton Manufacturers has an accounts receivable turnover of 4.8

If Newton Manufacturers has an accounts receivable turnover of 4.8 times and net sales of $7,812,379, what would its receivables be?

See Answer

Q: Blackwell Automotive’s balance sheet at the end of its most recent fiscal

Blackwell Automotive’s balance sheet at the end of its most recent fiscal year shows the following information? In addition, it was reported that the firm had a net income of $156,0...

See Answer

Q: The following are the financial statements for Nederland Consumer Products Company for

The following are the financial statements for Nederland Consumer Products Company for the fiscal year ended September 30, 2017. Nederland Consumer Products Company Income Statement for the Fiscal Yea...

See Answer

Q: Refer to the preceding information for Nederland Consumer Products Company. Compute

Refer to the preceding information for Nederland Consumer Products Company. Compute the firm’s ratios for the following categories and briefly evaluate the company’...

See Answer

Q: Refer to the earlier information for Nederland Consumer Products Company. Using

Refer to the earlier information for Nederland Consumer Products Company. Using the DuPont identity, calculate the return on equity for Nederland, after calculating the ratios that make up the DuPont...

See Answer

Q: Rockwell Industries has a three-year bond outstanding that pays a

Rockwell Industries has a three-year bond outstanding that pays a 7.25 percent coupon and is currently priced at $913.88. What is the yield to maturity of this bond? Assume annual coupon payments?

See Answer

Q: Nugent, Inc., has a gross profit margin of 31.

Nugent, Inc., has a gross profit margin of 31.7 percent on net sales of $9,865,214 and total assets of $7,125,852. The company has a current ratio of 2.7 times, accounts receivable of $1,715,363, cash...

See Answer

Q: Recreational Supplies Co. has net sales of $11,655

Recreational Supplies Co. has net sales of $11,655,000, an ROE of 17.64 percent, and a total asset turnover of 2.89 times. If the firm has a debt-to-equity ratio of 1.43, what is the company’s net inc...

See Answer

Q: Nutmeg Houseware, Inc., has an operating profit margin of 10

Nutmeg Houseware, Inc., has an operating profit margin of 10.3 percent on revenues of $24,547,125 and total assets of $8,652,352. a. Calculate the company’s total asset turnover ratio and its operati...

See Answer

Q: Modern Appliances Corporation has reported its financial results for the year ended

Modern Appliances Corporation has reported its financial results for the year ended December 31, 2017? Modern Appliances Corporation Income Statement for the Fiscal Year Ended December 31, 2017 Net Sa...

See Answer

Q: The TBI Company has a number of days of inventory of 50

The TBI Company has a number of days of inventory of 50. Therefore, the TBI Company’s inventory turnover is closest to: a. 4.8 times. b. 7.3 times. c. 8.4 times. d. 9.6 times.

See Answer

Q: Bummel and Strand Corp. has a gross profit margin of 33

Bummel and Strand Corp. has a gross profit margin of 33.7 percent, sales of $47,112,365, and inventory of $14,595,435. What is its inventory turnover ratio?

See Answer

Q: If a company’s net profit margin is –5 percent, its

If a company’s net profit margin is –5 percent, its total asset turnover is 1.5 times, and its equity multiplier is 1.2 times, its return on equity is closest to a. –9.0 percent. b. –7.5 percent. c. –...

See Answer

Q: Sorenson Inc. has sales of $3,112,489

Sorenson Inc. has sales of $3,112,489, a gross profit margin of 23.1 percent, and inventory of $833,145. What are the company’s inventory turnover ratio and days’ sales in inventory?

See Answer

Q: Breckenridge Ski Company has total assets of $422,235,

Breckenridge Ski Company has total assets of $422,235,811 and a debt ratio of 29.5 percent. Calculate the company’s debt-to-equity ratio and equity multiplier?

See Answer

Q: Norton Company has a debt-to-equity ratio of 1

Norton Company has a debt-to-equity ratio of 1.65, ROA of 11.3 percent, and total equity of $1,322,796. What are the company’s equity multiplier, debt ratio, and ROE?

See Answer

Q: Hindenberg, Inc., has a 10-year bond that is

Hindenberg, Inc., has a 10-year bond that is priced at $1,100.00. It has a coupon of 8 percent paid semiannually. What is the yield to maturity on this bond?

See Answer

Q: The Rangoon Timber Company has the following ratios: Net sales

The Rangoon Timber Company has the following ratios: Net sales/Total assets = 2.23; …………………. ROA = 9.69%; …………………. ROE = 16.4% What are Rangoon’s profit margin and debt ratios?

See Answer

Q: Lemmon Enterprises has a total asset turnover of 2.1 and

Lemmon Enterprises has a total asset turnover of 2.1 and a net profit margin of 7.5%. If its equity multiplier is 1.90, what is the ROE for Lemmon Enterprises?

See Answer

Q: Chuck Tomkovick is planning to invest $25,000 today in

Chuck Tomkovick is planning to invest $25,000 today in a mutual fund that will provide a return of 8 percent each year. What will be the value of the investment in 10 years?

See Answer

Q: Maria Addai has been offered a future payment of $750 two

Maria Addai has been offered a future payment of $750 two years from now. If she can earn an annual rate of 6.5 percent, compounded daily, on her investment, what should she pay for this investment to...

See Answer

Q: Your brother has asked you for a loan and has promised to

Your brother has asked you for a loan and has promised to pay you $7,750 at the end of three years. If you normally invest to earn 6 percent per year, how much will you be willing to lend to your brot...

See Answer

Q: Tracy Chapman is saving to buy a house in five years.

Tracy Chapman is saving to buy a house in five years. She plans to put 20 percent down at that time, and she believes that she will need $35,000 for the down payment. If Tracy can invest in a fund tha...

See Answer

Q: You want to buy some bonds that will have a value of

You want to buy some bonds that will have a value of $1,000 at the end of seven years. The bonds pay 4.5 percent interest annually. How much should you pay for them today?

See Answer

Q: Elizabeth Sweeney wants to accumulate $12,000 by the end

Elizabeth Sweeney wants to accumulate $12,000 by the end of 12 years. If the annual interest rate is 7 percent and interest compounds semiannually, how much will she have to invest today to achieve he...

See Answer

Q: You are in desperate need of cash and turn to your uncle

You are in desperate need of cash and turn to your uncle, who has offered to lend you some money. You decide to borrow $1,300 and agree to pay back $1,500 in two years. Alternatively, you could borrow...

See Answer

Q: You invest $150 in a mutual fund today that pays 9

You invest $150 in a mutual fund today that pays 9 percent interest annually. How long will it take to double your money?

See Answer

Q: Using the 2017 data above for Abercrombie Supply Company calculate the following

Using the 2017 data above for Abercrombie Supply Company calculate the following liquidity ratios: a. Current ratio b. Quick ratio

See Answer

Q: Highland Corp., a U.S. company, has a

Highland Corp., a U.S. company, has a five-year bond whose yield to maturity is 6.5 percent. The bond has no coupon payments. What is the price of this zero coupon bond?

See Answer

Q: Your finance textbook sold 53,250 copies in its first year

Your finance textbook sold 53,250 copies in its first year. The publishing company expects the sales to grow at a rate of 20 percent each year for the next three years and by 10 percent in the fourth...

See Answer

Q: CelebNav, Inc., had sales last year of $700,

CelebNav, Inc., had sales last year of $700,000, and the analysts are predicting a good year for the start-up, with sales growing 20 percent a year for the next three years. After that, the sales shou...

See Answer

Q: You decide to take advantage of the current online dating craze and

You decide to take advantage of the current online dating craze and start your own Web site. You know that you have 450 people who will sign up immediately and, through a careful marketing research an...

See Answer

Q: Ted Rogers is investing $7,500 in a bank CD

Ted Rogers is investing $7,500 in a bank CD that pays a 6 percent annual interest. How much will the CD be worth at the end of five years?

See Answer

Q: Find the future value of an investment of $2,500

Find the future value of an investment of $2,500 made today for the following rates and periods: a. 6.25 percent compounded semiannually for 12 years b. 7.63 percent compounded quarterly for 6 years c...

See Answer

Q: Find the present value of $3,500 under each of

Find the present value of $3,500 under each of the following rates and periods. a. 8.9% compounded monthly for five years. b. 6.6% compounded quarterly for eight years. c. 4.3% compounded daily for fo...

See Answer

Q: Samantha plans to invest some money so that she has $5

Samantha plans to invest some money so that she has $5,500 at the end of three years. How much should she invest today given the following choices: a. 4.2 percent compounded daily. b. 4.9 percent com...

See Answer

Q: You are able to deposit $850 in a bank CD today

You are able to deposit $850 in a bank CD today, and you will withdraw the money only once the balance is $1,000. If the bank pays 5 percent interest, how long will it take for the balance to reach $1...

See Answer

Q: Neon Lights Company is a private company with sales of $1

Neon Lights Company is a private company with sales of $1.3 million a year. Management wants to take the company public but has to wait until the sales reach $2 million. If the sales are expected to g...

See Answer

Q: You have just inherited $550,000. You plan to

You have just inherited $550,000. You plan to save this money and continue to live off the money that you are earning in your current job. If you can invest the money in a bond that pays 4.6 percent i...

See Answer

Q: Kronka, Inc., is expecting cash inflows of $13,

Kronka, Inc., is expecting cash inflows of $13,000, $11,500, $12,750, and $9,635 over the next four years. What is the present value of these cash flows if the appropriate discount rate is 8 percent?...

See Answer

Q: Xenix Corp had sales of $353,866 in 2017.

Xenix Corp had sales of $353,866 in 2017. If management expects its sales to be $476,450 in three years, what is the rate at which the company’s sales are expected to grow?

See Answer

Q: Given the following information about Elkridge Sporting Goods, Inc., construct

Given the following information about Elkridge Sporting Goods, Inc., construct a balance sheet for June 30, 2017. On that date the firm had cash and marketable securities of $25,135, accounts receivab...

See Answer

Q: What is the difference between FIFO and LIFO accounting?

What is the difference between FIFO and LIFO accounting?

See Answer

Q: Identify seven mechanisms that can help better align the goals of managers

Identify seven mechanisms that can help better align the goals of managers with those of stockholders?

See Answer

Q: How are brokers different from dealers?

How are brokers different from dealers?

See Answer

Q: What is the matching principle, and how can it cause accounting

What is the matching principle, and how can it cause accounting expenses to differ from actual cash outflows?

See Answer

Q: Wolf Pack Enterprises has total current assets of $346,002

Wolf Pack Enterprises has total current assets of $346,002 and fixed assets of $476,306. The company also has long-term debt of $276,400, $100,000 in its common stock account, and retained earnings of...

See Answer

Q: What is the difference between a book-value balance sheet and

What is the difference between a book-value balance sheet and a market-value balance sheet? Which provides better information to investors and management?

See Answer

Q: ACME Corporation had revenues of $867,030 in 2017.

ACME Corporation had revenues of $867,030 in 2017. It also had expenses (excluding depreciation) of $356,240, depreciation of $103,456, and interest expense of $52,423. What was the company’s net inco...

See Answer

Q: True Blue Company increased its investments in marketable securities by $323

True Blue Company increased its investments in marketable securities by $323,370 and paid $1,220,231 for new fixed assets during 2017. The company also repaid $779,200 of existing long-term debt while...

See Answer

Q: Your grandfather has agreed to deposit a certain amount of money each

Your grandfather has agreed to deposit a certain amount of money each year into an account paying 7.25 percent annually to help you go to graduate school. Starting next year, and for the following fou...

See Answer

Q: Sun Devil Corporation reported EBITDA of $7,300,125

Sun Devil Corporation reported EBITDA of $7,300,125 and net income of $3,328,950 for the fiscal year ended December 31, 2017. During the same period, the company had $1,155,378 in interest expense, $1...

See Answer

Q: Refer to the preceding balance sheet and income statement for Greenfern Corporation

Refer to the preceding balance sheet and income statement for Greenfern Corporation for the fiscal year ended July 31, 2017. What are the company’s current ratio and quick ratio? Wha...

See Answer

Q: Refer to the preceding balance sheet and income statement for Greenfern Corporation

Refer to the preceding balance sheet and income statement for Greenfern Corporation for the fiscal year ended July 31, 2017. Calculate the following ratios: Refer to the income statement of Greenfer...

See Answer

Q: Refer to the preceding balance sheet and income statement for Greenfern Corporation

Refer to the preceding balance sheet and income statement for Greenfern Corporation for the fiscal year ended July 31, 2017. Use the DuPont identity to calculate the return on equity (ROE). In the pro...

See Answer

Q: Last year Pontiff Enterprises reported net sales of $13,144

Last year Pontiff Enterprises reported net sales of $13,144,680, a gross profit $4,127,429, EBIT of $2,586,150, and net income of $867,555. Compute Pontiff’s cost of goods sold, gross profit margin, o...

See Answer

Q: National City Bank has 646,749,650 shares of common

National City Bank has 646,749,650 shares of common stock outstanding that are currently selling for $37.55 per share on the New York Stock Exchange. If National City’s net income was $2,780,955,000 i...

See Answer

Q: Juliette Bronson anticipates needing $500,000 to start a business

Juliette Bronson anticipates needing $500,000 to start a business. If she can earn 4.5 percent compounded annually on her investments, how much money would Juliette have to invest today to have $500,0...

See Answer

Q: Christopher Thompkins must decide how to invest $10,000 that

Christopher Thompkins must decide how to invest $10,000 that he just inherited. What would be the future value of his investment after five years under each of the following three investment opportuni...

See Answer

Q: Tina DeLeon deposited $2,500 today in an account paying

Tina DeLeon deposited $2,500 today in an account paying 6 percent interest annually. What would be the simple interest earned on this investment in five years? With annual compounding, how much intere...

See Answer

Q: The state of Texas had 42,725 active patient care physicians

The state of Texas had 42,725 active patient care physicians in 2013 and by 2017 this number had grown to 47,663. What was the compound annual growth rate (CAGR) in the number of active care physician...

See Answer

Q: Compound growth is exponential over time. Explain?

Compound growth is exponential over time. Explain?

See Answer

Q: Freisinger, Inc., is expecting a new project to start paying

Freisinger, Inc., is expecting a new project to start paying off, beginning at the end of next year. It expects cash flows to be as follows: If Freisinger can reinvest these cash flows to earn a retu...

See Answer

Q: You plan to set up an endowment at your alma mater that

You plan to set up an endowment at your alma mater that will fund $200,000 of scholarships each year indefinitely. If the principal (the amount you donate) can be invested at 5.5 percent, compounded a...

See Answer

Q: Annalise Genric wants to open a restaurant in a historic building.

Annalise Genric wants to open a restaurant in a historic building. The property can be leased for 20 years, but not purchased. She believes her restaurant can generate a net cash flow of $76,000 the f...

See Answer

Q: A credit card offers financing at an APR of 18 percent,

A credit card offers financing at an APR of 18 percent, with monthly compounding on outstanding charges. What is the effective annual rate (EAR)?

See Answer

Q: You are considering investing in a mutual fund. The fund is

You are considering investing in a mutual fund. The fund is expected to earn a return of 15 percent in the next year. If its annual return is normally distributed with a standard deviation of 6.5 perc...

See Answer

Q: What would you recommend to an investor who is considering making an

What would you recommend to an investor who is considering making an investment in a stock which plots below the security market line (SML)? Explain.

See Answer

Q: Why does an investor want a diversified portfolio? Can an investor

Why does an investor want a diversified portfolio? Can an investor eliminate all risk?

See Answer

Q: Seven years ago Eastern Corporation issued 20-year bonds that had

Seven years ago Eastern Corporation issued 20-year bonds that had a $1000 face value, paid interest annually, and that had a coupon rate of 7 percent. If the market rate of interest is 5.5 percent tod...

See Answer

Q: You are considering investing in a 10-year zero coupon bond

You are considering investing in a 10-year zero coupon bond that compounds interest semiannually. If the current market rate is 5.65 percent, what is the maximum price you should have pay for this bon...

See Answer

Q: What is the Rule of 72?

What is the Rule of 72?

See Answer

Q: Bigbox, Inc. has bonds outstanding that will mature in eight

Bigbox, Inc. has bonds outstanding that will mature in eight years. These bonds pay interest semiannually and have a coupon rate of 4.6 percent. If the bonds are currently selling at $888.92, what is...

See Answer

Q: Identify three fundamental types of decisions that financial managers make and identify

Identify three fundamental types of decisions that financial managers make and identify which part of the balance sheet each of these decisions affects?

See Answer

Q: What are the two basic mechanisms through which funds flow through the

What are the two basic mechanisms through which funds flow through the financial system, and how do they differ?

See Answer

Q: Why is stock value maximization superior to profit maximization as a goal

Why is stock value maximization superior to profit maximization as a goal for management?

See Answer

Q: What are agency costs? Explain.

What are agency costs? Explain.

See Answer

Q: In this chapter we showed that the formula for a perpetuity can

In this chapter we showed that the formula for a perpetuity can be obtained from the formula for the present value of an ordinary annuity if n is set equal to ∞. It is also possible to go the other wa...

See Answer

Q: In the chapter text, you saw that the formula for a

In the chapter text, you saw that the formula for a growing perpetuity can be obtained from the formula for the present value of a growing annuity if n is set equal to ∞. It is also possible to go the...

See Answer

Q: Why are capital budgeting decisions among the most important decisions in the

Why are capital budgeting decisions among the most important decisions in the life of a firm?

See Answer

Q: What are the three most basic types of financial decisions managers must

What are the three most basic types of financial decisions managers must make?

See Answer

Q: Explain why you would make an investment if the value of the

Explain why you would make an investment if the value of the expected cash flows exceeds the cost of the project?

See Answer

Q: Identify the steps involved in computing the future value when you have

Identify the steps involved in computing the future value when you have multiple cash flows?

See Answer

Q: What are some advantages and disadvantages of operating as a public corporation

What are some advantages and disadvantages of operating as a public corporation?

See Answer

Q: Explain why professional partnerships such as physicians’ groups organize as limited liability

Explain why professional partnerships such as physicians’ groups organize as limited liability partnerships?

See Answer

Q: Identify three financial officers who typically report to the CFO and describe

Identify three financial officers who typically report to the CFO and describe their duties?

See Answer

Q: Explain why maximizing the current market price of a firm’s stock is

Explain why maximizing the current market price of a firm’s stock is an appropriate goal for the firm’s management?

See Answer

Q: What is the fundamental determinant of an asset’s value?

What is the fundamental determinant of an asset’s value?

See Answer

Q: Why is profit maximization an unsatisfactory goal for managing a firm?

Why is profit maximization an unsatisfactory goal for managing a firm?

See Answer

Q: What are corporate raiders?

What are corporate raiders?

See Answer

Q: What is a conflict of interest in a business setting?

What is a conflict of interest in a business setting?

See Answer

Q: What critical economic role does the financial system play in the economy

What critical economic role does the financial system play in the economy?

See Answer

Q: Why might a firm prefer to have a security issue underwritten by

Why might a firm prefer to have a security issue underwritten by an investment banking firm?

See Answer

Q: Suppose three investments have equal lives and multiple cash flows. A

Suppose three investments have equal lives and multiple cash flows. A high discount rate tends to favor: a. The investment with large cash flows early. b. The investment with large cash flows late. c....

See Answer

Q: How and why do large business firms use money markets?

How and why do large business firms use money markets?

See Answer

Q: What is strong-form market efficiency? Semistrong-form market

What is strong-form market efficiency? Semistrong-form market efficiency? Weak-form market efficiency?

See Answer

Q: What are some services that commercial banks provide to businesses?

What are some services that commercial banks provide to businesses?

See Answer

Q: What is an IPO, and what role does an investment banker

What is an IPO, and what role does an investment banker play in the process?

See Answer

Q: Explain how the real rate of interest is determined?

Explain how the real rate of interest is determined?

See Answer

Q: Explain why interest rates follow the business cycle?

Explain why interest rates follow the business cycle?

See Answer

Q: What is the realization principle, and why may it lead to

What is the realization principle, and why may it lead to a difference in the timing of when revenues are recognized on the books and cash is collected?

See Answer

Q: What types of information does a firm’s annual report contain?

What types of information does a firm’s annual report contain?

See Answer

Q: What are some objections to the preparation of marked-to-

What are some objections to the preparation of marked-to-market balance sheets?

See Answer

Q: What is the difference between book value and market value?

What is the difference between book value and market value?

See Answer

Q: Explain whether or not each of the following statements is correct.

Explain whether or not each of the following statements is correct. a. A 15-year mortgage will have larger monthly payments than a 30-year mortgage of the same amount and same interest rate. b. If an...

See Answer

Q: What is EBITDA, and what does it measure?

What is EBITDA, and what does it measure?

See Answer

Q: Explain how the four financial statements are related?

Explain how the four financial statements are related?

See Answer

Q: What does it mean when a firm’s cash flow to investors is

What does it mean when a firm’s cash flow to investors is negative?

See Answer

Q: Which type of tax rate, marginal or average, should be

Which type of tax rate, marginal or average, should be used in analyzing the expansion of a product line, and why?

See Answer

Q: What is the primary concern of a firm’s creditors?

What is the primary concern of a firm’s creditors?

See Answer

Q: Why does it make sense to standardize financial statements?

Why does it make sense to standardize financial statements?

See Answer

Q: What are common-size, or standardized, financial statements,

What are common-size, or standardized, financial statements, and how are they prepared?

See Answer

Q: What are the efficiency ratios, and what do they measure?

What are the efficiency ratios, and what do they measure? Why, for some firms, is the total asset turnover more important than the fixed asset turnover?

See Answer

Q: List the leverage ratios discussed in this section, and explain how

List the leverage ratios discussed in this section, and explain how they are related?

See Answer

Q: List the profitability ratios discussed in this section, and explain how

List the profitability ratios discussed in this section, and explain how they differ from each other?

See Answer

Q: Why is the effective annual rate (EAR) superior to the

Why is the effective annual rate (EAR) superior to the annual percentage rate (APR) in measuring the true economic cost or return?

See Answer

Q: What are the three major shortcomings of ROE?

What are the three major shortcomings of ROE?

See Answer

Q: What is the purpose of the DuPont system of analysis?

What is the purpose of the DuPont system of analysis?

See Answer

Q: What is the equation for ROA in the DuPont system, and

What is the equation for ROA in the DuPont system, and how do the factors in that equation influence the ratio?

See Answer

Q: In what three ways can a financial manager choose a benchmark?

In what three ways can a financial manager choose a benchmark?

See Answer

Q: Explain what the SIC codes are, and discuss the pros and

Explain what the SIC codes are, and discuss the pros and cons of using them in financial analysis?

See Answer

Q: What are the limitations on traditional financial statement analysis?

What are the limitations on traditional financial statement analysis?

See Answer

Q: List some of the problems that financial analysts confront when analyzing financial

List some of the problems that financial analysts confront when analyzing financial statements?

See Answer

Q: Explain how the DuPont identity allows us to evaluate a firm’s performance

Explain how the DuPont identity allows us to evaluate a firm’s performance?

See Answer

Q: What is a time line, and why is it important in

What is a time line, and why is it important in financial analysis?

See Answer

Q: Why is a dollar today worth more than a dollar one year

Why is a dollar today worth more than a dollar one year from now?

See Answer

Q: Because the conversion feature in a convertible bond is valuable to bondholders

Because the conversion feature in a convertible bond is valuable to bondholders, convertible bond issues have lower coupon payments than otherwise similar bonds that are not convertible. Does this mea...

See Answer

Q: What is compounding, and how does it affect the future value

What is compounding, and how does it affect the future value of an investment?

See Answer

Q: How does changing the compounding period affect the amount of interest earned

How does changing the compounding period affect the amount of interest earned on an investment?

See Answer

Q: Explain why you would expect the discount factor to become smaller when

Explain why you would expect the discount factor to become smaller when based on the longer the time to payment?

See Answer

Q: Explain how to calculate the present value of a stream of cash

Explain how to calculate the present value of a stream of cash flows?

See Answer

Q: Why is it important to adjust all cash flows to a common

Why is it important to adjust all cash flows to a common date?

See Answer

Q: Explain how to calculate the future value of a stream of cash

Explain how to calculate the future value of a stream of cash flows?

See Answer

Q: How do an ordinary annuity, an annuity due, and a

How do an ordinary annuity, an annuity due, and a perpetuity differ?

See Answer

Q: Give two examples of perpetuities?

Give two examples of perpetuities?

See Answer

Q: What is the annuity transformation method?

What is the annuity transformation method?

See Answer

Q: What is the difference between a growing annuity and a growing perpetuity

What is the difference between a growing annuity and a growing perpetuity?

See Answer

Q: Refer above to the balance sheet and income statement for Abercrombie Supply

Refer above to the balance sheet and income statement for Abercrombie Supply Company for the year ended June 30, 2017. Calculate the following ratios: Refer data to problem 4.2: Prepare a common-size...

See Answer

Q: a. Investor A holds a 10-year bond, while

a. Investor A holds a 10-year bond, while investor B holds an 8-year bond. If interest rate increases by 1 percent, which investor has the higher interest rate risk? Explain. b. Investor A holds a 10-...

See Answer

Q: Distinguish between quoted interest rate, interest rate per period, and

Distinguish between quoted interest rate, interest rate per period, and effective annual interest rate?

See Answer

Q: What is the APR, and why are lending institutions required to

What is the APR, and why are lending institutions required to disclose this rate?

See Answer

Q: What is the correct way to annualize an interest rate in financial

What is the correct way to annualize an interest rate in financial decision making?

See Answer

Q: What does the coefficient of variation tell us, and how is

What does the coefficient of variation tell us, and how is it related to the Sharpe Ratio?

See Answer

Q: Why does the total risk of a portfolio not approach zero as

Why does the total risk of a portfolio not approach zero as the number of assets in a portfolio becomes very large?

See Answer

Q: Why are returns on the stock market used as a benchmark in

Why are returns on the stock market used as a benchmark in measuring systematic risk?

See Answer

Q: How is the expected return on an asset related to its systematic

How is the expected return on an asset related to its systematic risk?

See Answer

Q: If an asset’s expected return does not plot on the line in

If an asset’s expected return does not plot on the line in question 2 above, what does that imply about its price?

See Answer

Q: What are the main differences between the corporate bond markets and stock

What are the main differences between the corporate bond markets and stock markets?

See Answer

Q: A bond has a 7 percent coupon rate, a face value

A bond has a 7 percent coupon rate, a face value of $1,000, and a maturity of four years. On a time line, lay out the cash flows for the bond?

See Answer

Q: We know that a vanilla bond with a coupon rate below the

We know that a vanilla bond with a coupon rate below the market rate of interest will sell for a discount and that a vanilla bond with a coupon rate above the market rate of interest will sell for a p...

See Answer

Q: What are zero coupon bonds, and how are they priced?

What are zero coupon bonds, and how are they priced?

See Answer

Q: Explain conceptually how bonds are priced?

Explain conceptually how bonds are priced?

See Answer

Q: Explain how bond yields are calculated?

Explain how bond yields are calculated?

See Answer

Q: What is interest rate risk?

What is interest rate risk?

See Answer

Q: Explain why long-term bonds with zero coupons are riskier than

Explain why long-term bonds with zero coupons are riskier than short-term bonds that pay coupon interest?

See Answer

Q: Describe the three most prominent bond rating systems?

Describe the three most prominent bond rating systems?

See Answer

Q: What are the key factors that most affect the level and shape

What are the key factors that most affect the level and shape of the yield curve?

See Answer

Q: What are default risk premiums, and what do they measure?

What are default risk premiums, and what do they measure?

See Answer

Q: Economic units that need to borrow money are said to be:

Economic units that need to borrow money are said to be: a. Lender–savers b. Borrower–spenders c. Balanced budget keepers. d. None of the above.

See Answer

Q: What is the appropriate decision criterion for financial managers to use when

What is the appropriate decision criterion for financial managers to use when selecting a capital project?

See Answer

Q: Define yield to maturity. Why is it important?

Define yield to maturity. Why is it important?

See Answer

Q: What are some of the things that managers do to manage a

What are some of the things that managers do to manage a firm’s working capital?

See Answer

Q: Which one of the following characteristics does not pertain to corporations?

Which one of the following characteristics does not pertain to corporations? a. Can enter into contracts. b. Can borrow money. c. Are the easiest type of business to form. d. Can be sued. e. Can own...

See Answer

Q: What are typically the main components of an executive compensation package?

What are typically the main components of an executive compensation package?

See Answer

Q: Depreciation and amortization expenses are: a. Part of current

Depreciation and amortization expenses are: a. Part of current assets on the balance sheet. b. After-tax expenses that reduce a firm’s cash flows. c. Long-term liabilities that reduce a firm’s net wor...

See Answer

Q: Explain what the marketability of a security is and how it is

Explain what the marketability of a security is and how it is determined?

See Answer

Q: What are over-the-counter markets (OTCs), and

What are over-the-counter markets (OTCs), and how do they differ from organized exchanges?

See Answer

Q: The going concern assumption of GAAP implies that the firm:

The going concern assumption of GAAP implies that the firm: a. Is going under and needs to be liquidated at historical cost. b. Will continue to operate and its assets should be recorded at historical...

See Answer

Q: How does the business cycle affect the nominal interest rate and inflation

How does the business cycle affect the nominal interest rate and inflation rate?

See Answer

Q: CSB, Inc., has a beta of 1.35.

CSB, Inc., has a beta of 1.35. If the expected market return is 14.5 percent and the risk-free rate is 5.5 percent, what is the appropriate required return of CSB (using the CAPM)?

See Answer

Q: How would one explain a low receivables turnover ratio?

How would one explain a low receivables turnover ratio?

See Answer

Q: Define interest rate risk. How can the CFOs manage this risk

Define interest rate risk. How can the CFOs manage this risk?

See Answer

Q: Why is ROE generally much higher than ROA for banks relative to

Why is ROE generally much higher than ROA for banks relative to other industries?

See Answer

Q: What are the two factors to be considered in time value of

What are the two factors to be considered in time value of money?

See Answer

Q: Explain the difference between future value and present value?

Explain the difference between future value and present value?

See Answer

Q: If you were given a choice between investing in a savings account

If you were given a choice between investing in a savings account that paid quarterly interest and one that paid monthly interest, which one should you choose if they both offered the same stated inte...

See Answer

Q: What is the key economic principle involved in calculating the present value

What is the key economic principle involved in calculating the present value or future value of multiple cash flows?

See Answer

Q: What is the difference between a perpetuity and an annuity?

What is the difference between a perpetuity and an annuity?

See Answer

Q: Define annuity due. Would an investment be worth more if it

Define annuity due. Would an investment be worth more if it were an ordinary annuity or an annuity due? Explain?

See Answer

Q: Raymond Bartz is trying to choose between two equally risky annuities,

Raymond Bartz is trying to choose between two equally risky annuities, each paying $5,000 per year for five years. One is an ordinary annuity, the other is an annuity due. Which of the following state...

See Answer

Q: Which of the following investments will have the highest future value at

Which of the following investments will have the highest future value at the end of three years? Assume that the effective annual rate for all investments is the same. a. You earn $3,000 at the end of...

See Answer

Q: When will the annual percentage rate (APR) be the same

When will the annual percentage rate (APR) be the same as the effective annual rate (EAR)?

See Answer

Q: Explain why bond prices and interest rates are negatively related. What

Explain why bond prices and interest rates are negatively related. What are the roles of the coupon rate and the termtomaturity in this relation?

See Answer

Q: What economic conditions would prompt investors to take advantage of a bond’s

What economic conditions would prompt investors to take advantage of a bond’s convertibility feature?

See Answer

Q: If interest rates are expected to increase, should investors look to

If interest rates are expected to increase, should investors look to long-term bonds or short-term securities? Explain.

See Answer

Q: Explain why total financial assets in the economy must equal total financial

Explain why total financial assets in the economy must equal total financial liabilities?

See Answer

Q: Define book-value accounting and market-value accounting?

Define book-value accounting and market-value accounting?

See Answer

Q: What does it mean when a company’s return on assets (ROA

What does it mean when a company’s return on assets (ROA) is equal to its return on equity (ROE)?

See Answer

Q: Why don’t small businesses make greater use of the direct credit markets

Why don’t small businesses make greater use of the direct credit markets since these markets enable firms to finance their activities at a very low cost?

See Answer

Q: What is the difference between stockholders and stakeholders?

What is the difference between stockholders and stakeholders?

See Answer

Q: Suppose that a group of accountants wants to start an accounting business

Suppose that a group of accountants wants to start an accounting business. What organizational form would they most likely choose, and why?

See Answer

Q: Why would the owners of a business choose to form a corporation

Why would the owners of a business choose to form a corporation even though they will face double taxation?

See Answer

Q: What are two basic services that investment banks provide in the economy

What are two basic services that investment banks provide in the economy?

See Answer

Q: Explain what you would assume the yield curve would look like during

Explain what you would assume the yield curve would look like during economic expansion and why?

See Answer

Q: Why are taxes and the tax code important for managerial decision making

Why are taxes and the tax code important for managerial decision making?

See Answer

Q: Explain why firms prefer to use accelerated depreciation methods over the straight

Explain why firms prefer to use accelerated depreciation methods over the straight-line method for tax purposes?

See Answer

Q: What is the statement of cash flows, and what is its

What is the statement of cash flows, and what is its role?

See Answer

Q: You are concerned about one of the investments in your fully diversified

You are concerned about one of the investments in your fully diversified portfolio. You just have an uneasy feeling about the CFO, Iam Shifty, of that particular firm. You do believe, however, that th...

See Answer

Q: In recent years, investors have agreed that the market portfolio consists

In recent years, investors have agreed that the market portfolio consists of more than just a group of U.S. stocks and bonds. If you are an investor who invests in only U.S. stocks and bonds, describe...

See Answer

Q: What are the two basic sources of funds for all businesses?

What are the two basic sources of funds for all businesses?

See Answer

Q: Who are the owners in a corporation, and how is their

Who are the owners in a corporation, and how is their ownership represented?

See Answer

Q: Explain what is meant by stockholders’ limited liability?

Explain what is meant by stockholders’ limited liability?

See Answer

Q: What is double taxation?

What is double taxation?

See Answer

Q: What is the form of business organization taken by most large companies

What is the form of business organization taken by most large companies and why?

See Answer

Q: An investor holds a 10-year bond paying a coupon rate

An investor holds a 10-year bond paying a coupon rate of 9 percent. The yield to maturity of the bond is 7.8 percent. Would you expect the investor to be holding a par-value, premium, or discount bond...

See Answer

Q: What are some of the drawbacks to setting profit maximization as the

What are some of the drawbacks to setting profit maximization as the main goal of a company?

See Answer

Q: What are the major factors that affect a firm’s stock price?

What are the major factors that affect a firm’s stock price?

See Answer

Q: What is net working capital?

What is net working capital?

See Answer

Q: What can happen if a firm is poorly managed and its stock

What can happen if a firm is poorly managed and its stock price falls substantially below its maximum potential price?

See Answer

Q: What are some of the regulations that pertain to boards of directors

What are some of the regulations that pertain to boards of directors that were put in place to reduce agency conflicts?

See Answer

Q: Explain the difference between profitable and unprofitable firms?

Explain the difference between profitable and unprofitable firms?

See Answer

Q: What three major decisions are of most concern to financial managers?

What three major decisions are of most concern to financial managers?

See Answer

Q: What are some of the working capital decisions that a financial manager

What are some of the working capital decisions that a financial manager faces?

See Answer

Q: What are the common forms of business organization discussed in this chapter

What are the common forms of business organization discussed in this chapter?

See Answer

Q: What is the role of the financial system, and what are

What is the role of the financial system, and what are the two major components of the financial system?

See Answer

Q: Why were commercial banks prohibited from engaging in investment banking activities until

Why were commercial banks prohibited from engaging in investment banking activities until 1999?

See Answer

Q: List the ways a company’s financial manager can benchmark the company’s own

List the ways a company’s financial manager can benchmark the company’s own performance?

See Answer

Q: Cranjet Inc. is issuing 10,000 bonds, and its

Cranjet Inc. is issuing 10,000 bonds, and its investment banker has guaranteed a price of $985 per bond. If the investment banker sells the entire issue to investors for $10,150,000. a. What is the u...

See Answer

Q: What are some of the ways in which a financial institution or

What are some of the ways in which a financial institution or intermediary can raise money?

See Answer

Q: How do financial institutions act as intermediaries to provide services to small

How do financial institutions act as intermediaries to provide services to small businesses?

See Answer

Q: Which financial institution is usually the most important to businesses?

Which financial institution is usually the most important to businesses?

See Answer

Q: What is the main difference between money markets and capital markets?

What is the main difference between money markets and capital markets?

See Answer

Q: What is the relevant tax rate to use when making financial decisions

What is the relevant tax rate to use when making financial decisions? Explain why.

See Answer

Q: What are the main types of securities in the money markets?

What are the main types of securities in the money markets?

See Answer

Q: How do capital market instruments differ from money market instruments?

How do capital market instruments differ from money market instruments?

See Answer

Q: Given the data for Oakland Mills Company in problem 3.7

Given the data for Oakland Mills Company in problem 3.7 above, compute the cash flows to investors from operating activity? Refer to data in problem 3.7: The Oakland Mills Company has disclosed the f...

See Answer

Q: What is treasury stock? Why do firms have treasury stock?

What is treasury stock? Why do firms have treasury stock?

See Answer

Q: What does a competitive financial system imply about interest rates?

What does a competitive financial system imply about interest rates?

See Answer

Q: Hillman Corporation reported current assets of $3,495,055

Hillman Corporation reported current assets of $3,495,055 for the year ending December 31, 2017 and current assets of $3,103,839 on December 31, 2016. Current liabilities for the firm were $2,867,225...

See Answer

Q: Del Bridge Construction had long-term assets before depreciation of $

Del Bridge Construction had long-term assets before depreciation of $990,560 on December 31, 2016 and $1,211,105 on December 31, 2017. How much cash flow was invested in long-term assets for Del Brid...

See Answer

Q: Define average tax rate and marginal tax rate?

Define average tax rate and marginal tax rate?

See Answer

Q: What are the major differences between public and private markets?

What are the major differences between public and private markets?

See Answer

Q: What are the two risk-hedging instruments discussed in the chapter

What are the two risk-hedging instruments discussed in the chapter?

See Answer

Q: Zephyr Sales Company has sales of $1.125 million.

Zephyr Sales Company has sales of $1.125 million. If the company’s management expects sales to grow 6.5 percent annually, how long will it be before sales doubles? Use a financial calculator to solve...

See Answer

Q: Refer to the information given in Problem 3.21. What

Refer to the information given in Problem 3.21. What is the cash flow from operating activity for Nimitz Rental? Refer to information given in problem 3.21: Nimitz Rental Company provided the followi...

See Answer

Q: In February 2017 the risk free rate was 2.97 percent

In February 2017 the risk free rate was 2.97 percent, the market risk premium was 6 percent and the beta for Twitter stock was 0.99. What is the expected return that was consistent with the systematic...

See Answer

Q: The market risk premium is 6 percent, and the risk-

The market risk premium is 6 percent, and the risk-free rate is 5 percent. If the expected return on a bond is 6.5 percent, what is its beta?

See Answer

Q: Explain why profit maximization is not the best goal for a company

Explain why profit maximization is not the best goal for a company. What is a better goal?

See Answer

Q: David is going to purchase two stocks to form the initial holdings

David is going to purchase two stocks to form the initial holdings in his portfolio. Iron stock has an expected return of 15 percent, while Copper stock has an expected return of 20 percent. If David...

See Answer

Q: When are the nominal and real interest rates equal?

When are the nominal and real interest rates equal?

See Answer

Q: If the expected return on the market is 10 percent and the

If the expected return on the market is 10 percent and the risk-free rate is 4 percent, what is the expected return for a stock with a beta equal to 1.5? What is the market risk premium?

See Answer

Q: Tonalli is putting together a portfolio of 10 stocks in equal proportions

Tonalli is putting together a portfolio of 10 stocks in equal proportions. What is the relative importance of the variance for each stock versus the covariance for the pairs of stocks in her portfolio...

See Answer

Q: Jose is thinking about purchasing a soft drink machine and placing it

Jose is thinking about purchasing a soft drink machine and placing it in a business office. He knows that there is a 5 percent probability that someone who walks by the machine will make a purchase fr...

See Answer

Q: Kate recently invested in real estate with the intention of selling the

Kate recently invested in real estate with the intention of selling the property one year from today. She has modeled the returns on that investment based on three economic scenarios. She believes tha...

See Answer

Q: Given the returns and probabilities for the three possible states listed below

Given the returns and probabilities for the three possible states listed below, calculate the covariance between the returns of Stock A and Stock B. For convenience, assume that the expected returns o...

See Answer

Q: Damien knows that the beta of his portfolio is equal to 1

Damien knows that the beta of his portfolio is equal to 1, but he does not know the risk-free rate of return or the market risk premium. He also knows that the expected return on the market is 8 perce...

See Answer

Q: Define systematic risk?

Define systematic risk?

See Answer

Q: Common-size analysis is used in financial analysis to:

Common-size analysis is used in financial analysis to: a. Evaluate changes in a company’s operating cycle over time. b. Predict changes in a company’s capital structure using regression analysis. c. C...

See Answer

Q: Last year ABC companies had accounts receivable turnover of 15, total

Last year ABC companies had accounts receivable turnover of 15, total asset turnover of 4.5, and total assets of $1,000,000. What was the value of ABC’s accounts receivable? What was the value of its...

See Answer

Q: What are some of the major external and internal factors that affect

What are some of the major external and internal factors that affect a firm’s stock price? What is the difference between the two general types of factors?

See Answer

Q: List the two ways in which a transfer of funds takes place

List the two ways in which a transfer of funds takes place in an economy. What is the main difference between these two?

See Answer

Q: DuPont analysis involves breaking return-on-assets ratios into their

DuPont analysis involves breaking return-on-assets ratios into their a. Profit components. b. Marginal and average components. c. Operating and financing components. d. Profit margin and turnover comp...

See Answer

Q: Why are direct financial markets also called wholesale markets?

Why are direct financial markets also called wholesale markets?

See Answer

Q: Tanner invested $1,000 in large U.S.

Tanner invested $1,000 in large U.S. stocks at the beginning of 2012. This investment earned 15.98 percent in 2012, 32.41 percent in 2013, 13.69 percent in 2014, and 1.41 percent in 2015. What return...

See Answer

Q: What is a primary market? What does IPO stand for?

What is a primary market? What does IPO stand for?

See Answer

Q: Primary Markets: Identify whether the following transactions are primary market or

Primary Markets: Identify whether the following transactions are primary market or secondary market transactions. a. Jim Hendry bought 300 shares of IBM through his brokerage account. b. Peggy Jones b...

See Answer

Q: Holding all else constant, what will happen to the present value

Holding all else constant, what will happen to the present value of a future amount if you increase the discount rate? What if you increase the number of years?

See Answer

Q: Given a change in market interest rates, which will change more

Given a change in market interest rates, which will change more; the market price of a bond with 20 years until maturity or the market price of a bond with 5 years until maturity? Assume all the chara...

See Answer

Q: Which of the following classes of securities is likely to have the

Which of the following classes of securities is likely to have the lowest corporate borrowing cost? (LO 5) a. AAA rated bonds. b. A rated bonds. c. BB rated bonds. d. C rated bonds. e. All of the abov...

See Answer

Q: Which of the following is/are advantages of the corporate form

Which of the following is/are advantages of the corporate form of organization? a. Reduced start-up costs b. Greater access to capital markets c. Unlimited liability d. Single taxation

See Answer

Q: Identify the sources of agency costs. What are some ways these

Identify the sources of agency costs. What are some ways these costs can be controlled in a company?

See Answer

Q: You just purchased a share of IBM stock on the New York

You just purchased a share of IBM stock on the New York Stock Exchange. What kind of transaction was this? a. Primary market transaction. b. Secondary market transaction. c. Futures market transaction...

See Answer

Q: List the three forms of the efficient market hypothesis, and describe

List the three forms of the efficient market hypothesis, and describe what information is assumed to be reflected in security prices under each of these hypotheses.

See Answer

Q: If the nominal rate of interest is 4.25 percent and

If the nominal rate of interest is 4.25 percent and the expected rate of inflation is 1.75 percent, what is the real rate of interest?

See Answer

Q: What is the relation between business cycles and the general level of

What is the relation between business cycles and the general level of interest rates?

See Answer

Q: Compare an annuity due with an ordinary annuity. The payments for

Compare an annuity due with an ordinary annuity. The payments for both are made annually and are of the same dollar amounts. The two annuities also have the same duration in years and the same discoun...

See Answer

Q: Given the following information from Capstone Corporation, what price would CAPM

Given the following information from Capstone Corporation, what price would CAPM predict that the company’s stock will trade for 1 year from today. Assume that the risk free rate is 3 percent and that...

See Answer

Q: You have just invested in a portfolio of three stocks. The

You have just invested in a portfolio of three stocks. The amount of money that you invested in each stock and its beta are summarized below. Calculate the beta of the portfolio and use the capital as...

See Answer

Q: Why are many businesses operated as sole proprietorships or partnerships?

Why are many businesses operated as sole proprietorships or partnerships?

See Answer

Q: What are the major responsibilities of the CFO?

What are the major responsibilities of the CFO?

See Answer

Q: Why does the internal auditor report to both the CFO and the

Why does the internal auditor report to both the CFO and the board of directors?

See Answer

Q: What is the Sarbanes-Oxley Act, and what does it

What is the Sarbanes-Oxley Act, and what does it focus on? Why does it focus in these areas?

See Answer

Q: What are agency conflicts?

What are agency conflicts?

See Answer

Q: List the three main objectives of the Sarbanes-Oxley Act?

List the three main objectives of the Sarbanes-Oxley Act?

See Answer

Q: How would you define an ethical business culture?

How would you define an ethical business culture?

See Answer

Q: What are the two basic ways in which funds flow through the

What are the two basic ways in which funds flow through the financial system from lender–savers to borrower–spenders?

See Answer

Q: Why is it difficult for individuals to participate in the direct financial

Why is it difficult for individuals to participate in the direct financial markets?

See Answer

Q: What is the difference between primary and secondary markets?

What is the difference between primary and secondary markets?

See Answer

Q: What are capital markets, and why are they important to corporations

What are capital markets, and why are they important to corporations?

See Answer

Q: How does information about a firm’s prospects get reflected in its share

How does information about a firm’s prospects get reflected in its share price?

See Answer

Q: What is financial intermediation, and why is it important?

What is financial intermediation, and why is it important?

See Answer

Q: How are inflationary expectations accounted for in the nominal rate of interest

How are inflationary expectations accounted for in the nominal rate of interest?

See Answer

Q: Give an example of a conflict of interest in a business setting

Give an example of a conflict of interest in a business setting, other than the one involving the real estate agent discussed in the chapter text?

See Answer

Q: What is net working capital? Why might a low value for

What is net working capital? Why might a low value for this number be considered undesirable?

See Answer

Q: Explain the accounting concept behind depreciation?

Explain the accounting concept behind depreciation?

See Answer

Q: What is treasury stock?

What is treasury stock?

See Answer

Q: How is net income computed?

How is net income computed?

See Answer

Q: What accounting events trigger changes to the retained earnings account?

What accounting events trigger changes to the retained earnings account?

See Answer

Q: How do increases in fixed assets from one period to the next

How do increases in fixed assets from one period to the next affect cash holdings for the firm?

See Answer

Q: Name two working capital accounts that represent sources of cash for the

Name two working capital accounts that represent sources of cash for the firm?

See Answer

Q: Explain the difference between cash flows from financing and investing activities?

Explain the difference between cash flows from financing and investing activities?

See Answer

Q: How does the calculation of net income differ from the calculation of

How does the calculation of net income differ from the calculation of cash flow to investors from operating activity?

See Answer

Q: All else being equal, if a firm increases its accounts payable

All else being equal, if a firm increases its accounts payable, what effect will this have on cash flow to investors?

See Answer

Q: When determining the real interest rate, what happens to businesses that

When determining the real interest rate, what happens to businesses that find themselves with unfunded capital projects whose rate of return exceeds the cost of capital?

See Answer

Q: Why is it important to consider the consequences of taxes when financing

Why is it important to consider the consequences of taxes when financing a new project?

See Answer

Q: What are the tax implications of a decision to finance a project

What are the tax implications of a decision to finance a project using debt rather than new equity?

See Answer

Q: Why is it important to look at a firm’s historical financial statements

Why is it important to look at a firm’s historical financial statements?

See Answer

Q: What is the difference between simple interest and compound interest?

What is the difference between simple interest and compound interest?

See Answer

Q: What is the present value, and when is it used?

What is the present value, and when is it used?

See Answer

Q: What is the discount rate? How does the discount rate differ

What is the discount rate? How does the discount rate differ from the interest rate in the future value equation?

See Answer

Q: What is the relation between the present value factor and the future

What is the relation between the present value factor and the future value factor?

See Answer

Q: What is the difference between the interest rate (i) and

What is the difference between the interest rate (i) and the growth rate (g) in the future value equation?

See Answer

Q: What are the two components of a total holding period return?

What are the two components of a total holding period return?

See Answer

Q: How is the expected return on an investment calculated?

How is the expected return on an investment calculated?

See Answer

Q: How do large corporations adjust their liquidity in the money markets?

How do large corporations adjust their liquidity in the money markets?

See Answer

Q: What is the relation between the variance and the standard deviation?

What is the relation between the variance and the standard deviation?

See Answer

Q: What relation do we generally observe between risk and return when we

What relation do we generally observe between risk and return when we examine historical returns?

See Answer

Q: What type of return tells you the average compounded return earned by

What type of return tells you the average compounded return earned by an investor?

See Answer

Q: What are the two components of total risk?

What are the two components of total risk?

See Answer

Q: How is beta estimated?

How is beta estimated?

See Answer

Q: How would you interpret a beta of 1.5 for an

How would you interpret a beta of 1.5 for an asset? A beta of 0.75?

See Answer

Q: What name is given to the relation between risk and expected return

What name is given to the relation between risk and expected return implied by the CAPM?

See Answer

Q: Explain what a convertible bond is?

Explain what a convertible bond is?

See Answer

Q: What is the compounding period for most bonds sold in the United

What is the compounding period for most bonds sold in the United States?

See Answer

Q: The CFO of a certain company always wears his green suit on

The CFO of a certain company always wears his green suit on a day that the firm is about to release positive information about his company. You believe that you can profit from this information by buy...

See Answer

Q: Shouldn’t the nominal rate of interest (Equation 2.1)

Shouldn’t the nominal rate of interest (Equation 2.1) be determined by the actual rate of inflation (∆Pa), which can be easily measured, rather than by the expected rate of inflation (∆Pe)?

See Answer

Q: How does Exhibit 2.5 help explain why interest rates were

How does Exhibit 2.5 help explain why interest rates were so high during the early 1980s as compared to the relatively low interest rates in the early 1960s?

See Answer

Q: What is a major reason for the accounting scandals in the early

What is a major reason for the accounting scandals in the early 2000s? How do firms sometimes attempt to meet Wall Street analysts’ earnings projections?

See Answer

Q: The equity multiplier for Spiffy Corporation is 1.75, its

The equity multiplier for Spiffy Corporation is 1.75, its EBIT return on assets (EROA) is 0.07, and the value of its equity is $850,000. What is the value of Spiffy’s total assets? What is the value o...

See Answer

Q: How does a firm’s cash flow to investors from operating activity differ

How does a firm’s cash flow to investors from operating activity differ from net income, and why?

See Answer

Q: Identify the five fundamental principles of GAAP, and explain briefly their

Identify the five fundamental principles of GAAP, and explain briefly their importance?

See Answer

Q: Compare and contrast depreciation expense and amortization expense?

Compare and contrast depreciation expense and amortization expense?

See Answer

Q: Why are retained earnings not considered an asset of the firm?

Why are retained earnings not considered an asset of the firm?

See Answer

Q: Why is it not enough for an analyst to look at just

Why is it not enough for an analyst to look at just the short-term and long-term debt on a firm’s balance sheet when assessing the firm’s fixed obligations?

See Answer

Q: Why is too much liquidity not a good thing?

Why is too much liquidity not a good thing?

See Answer

Q: Inventory is excluded when the quick ratio or acid-test ratio

Inventory is excluded when the quick ratio or acid-test ratio is calculated because inventory is the most difficult current asset to convert to cash without loss of value. What types of inventory are...

See Answer

Q: What does a very high inventory turnover ratio signify?

What does a very high inventory turnover ratio signify?

See Answer

Q: What additional information does the fixed asset turnover ratio provide over the

What additional information does the fixed asset turnover ratio provide over the total asset turnover ratio? For which industries does it carry greater significance?

See Answer

Q: How does financial leverage help stockholders?

How does financial leverage help stockholders?

See Answer

Q: Give an example of a capital budgeting decision and a financing decision

Give an example of a capital budgeting decision and a financing decision?

See Answer

Q: Why is the ROE a more appropriate proxy (for stockholder value

Why is the ROE a more appropriate proxy (for stockholder value maximization) for some firms than for other firms?

See Answer

Q: Explain the phrase “a dollar today is worth more than a

Explain the phrase “a dollar today is worth more than a dollar tomorrow”?

See Answer

Q: You are planning to take a spring break trip to Cancun your

You are planning to take a spring break trip to Cancun your senior year. The trip is exactly two years away, but you want to be prepared and have enough money when the time comes. Explain how you woul...

See Answer

Q: Explain the importance of a time line?

Explain the importance of a time line?

See Answer

Q: Explain the difference between compounding and discounting?

Explain the difference between compounding and discounting?

See Answer

Q: Explain how compound interest differs from simple interest?

Explain how compound interest differs from simple interest?

See Answer

Q: Describe the cash flows between a firm and its stakeholders?

Describe the cash flows between a firm and its stakeholders?

See Answer

Q: What are the three fundamental decisions the financial manager is concerned with

What are the three fundamental decisions the financial manager is concerned with, and how do they affect the firm’s balance sheet?

See Answer

Q: Explain the economic role of brokers and dealers. How does each

Explain the economic role of brokers and dealers. How does each make a profit?

See Answer

Q: The correlation between stocks A and B is 0.50,

The correlation between stocks A and B is 0.50, while the correlation between stocks A and C is –0.5. You already own stock A and are thinking of buying either stock B or stock C. If you want your por...

See Answer

Q: Megan Gaumer expects to need $50,000 for a down

Megan Gaumer expects to need $50,000 for a down payment on a house in six years. How much does she need to invest today in an account paying 7.25 percent in order to have $50,000 in six years?

See Answer

Q: The idea that we can know the return on a security for

The idea that we can know the return on a security for each possible outcome is overly simplistic. However, even though we cannot possibly predict all possible outcomes, this fact has little bearing o...

See Answer

Q: Which investment category included in Exhibit 7.3 has shown the

Which investment category included in Exhibit 7.3 has shown the greatest degree of risk in the United States since 1926? Explain why that makes sense in a world where the value of an asset in this inv...

See Answer

Q: The CAPM is used to price the risk (estimate the expected

The CAPM is used to price the risk (estimate the expected return) for any asset. Our examples have focused on stocks, but we could also use CAPM to estimate the expected rate of return for bonds. Expl...

See Answer

Q: You may have heard the statement that you should not include your

You may have heard the statement that you should not include your home as an asset in your investment portfolio. Assume that your house will comprise up to 75 percent of your assets in the early part...

See Answer

Q: Suppose that you know the risk and the expected return for two

Suppose that you know the risk and the expected return for two stocks. Discuss the process you might utilize to determine which of the two stocks is a better buy. You may assume that the two stocks wi...

See Answer

Q: What is the difference between the expected rate of return and the

What is the difference between the expected rate of return and the required rate of return? What does it mean if they are different for a particular asset at a particular point in time?

See Answer

Q: Suppose that the standard deviation of the returns on the shares of

Suppose that the standard deviation of the returns on the shares of stock at two different companies is exactly the same. Does this mean that the required rate of return will be the same for these two...

See Answer

Q: What is a firm’s capital structure, and why is it important

What is a firm’s capital structure, and why is it important?

See Answer

Q: What are the advantages and disadvantages of a sole proprietorship?

What are the advantages and disadvantages of a sole proprietorship?

See Answer

Q: What is a partnership, and what is the biggest disadvantage of

What is a partnership, and what is the biggest disadvantage of this form of business organization? How can this disadvantage be avoided?

See Answer

Q: You have an opportunity to invest $2,500 today and

You have an opportunity to invest $2,500 today and receive $3,000 in three years. What would be the return on your investment if you accepted this opportunity?

See Answer

Q: What is the primary responsibility of the board of directors in a

What is the primary responsibility of the board of directors in a corporation?

See Answer

Q: All public companies must hire a certified public accounting firm to perform

All public companies must hire a certified public accounting firm to perform an independent audit of their financial statements. What exactly does the term audit mean?

See Answer

Q: What is the appropriate goal of financial managers? How do managers’

What is the appropriate goal of financial managers? How do managers’ decisions affect how successful the firm is in achieving this goal?

See Answer

Q: What is an agency relationship, and what is an agency conflict

What is an agency relationship, and what is an agency conflict? How can agency conflicts be reduced in a corporation?

See Answer

Q: How can a lack of business ethics negatively affect the performance of

How can a lack of business ethics negatively affect the performance of an economy? Give an example.

See Answer

Q: What are some ways to resolve a conflict of interest?

What are some ways to resolve a conflict of interest?

See Answer

Q: Describe what an information asymmetry is in a business transaction. Explain

Describe what an information asymmetry is in a business transaction. Explain how the inequity associated with an information asymmetry might be, at least partially, solved through the market for goods...

See Answer

Q: What ethical conflict does insider trading present?

What ethical conflict does insider trading present?

See Answer

Q: What is the appropriate decision rule for a firm considering undertaking a

What is the appropriate decision rule for a firm considering undertaking a capital project? Give a real-life example?

See Answer

Q: Infosys Technologies, Inc., an Indian technology company, reported net

Infosys Technologies, Inc., an Indian technology company, reported net income of $419 million this year. Analysts expect the company’s earnings to be $1.468 billion in five years. What is the expected...

See Answer

Q: What is the general formula used to calculate the price of a

What is the general formula used to calculate the price of a share of a stock? What does it mean?

See Answer

Q: What is the cost of capital?

What is the cost of capital?

See Answer

Q: The profitability index is a tool for measuring a project’s benefits relative

The profitability index is a tool for measuring a project’s benefits relative to its costs. How might this help to eliminate bias in project selection?

See Answer

Q: Explain why the cost of capital is referred to as the “

Explain why the cost of capital is referred to as the “hurdle” rate in capital budgeting?

See Answer

Q: Elkridge Construction Company has an overall (composite) cost of capital

Elkridge Construction Company has an overall (composite) cost of capital of 12 percent. This cost of capital reflects the cost of capital for an Elkridge Construction project with average risk. Howeve...

See Answer

Q: a. Sykes, Inc. management is considering two projects:

a. Sykes, Inc. management is considering two projects: a plant expansion and a new computer system for the firm’s production department. Classify these projects as independent, mutually exclusive, or...

See Answer

Q: In the context of capital budgeting, what is “capital rationing

In the context of capital budgeting, what is “capital rationing”?

See Answer

Q: a. A firm invests in a project that is expected to

a. A firm invests in a project that is expected to earn a return of 12 percent. If the appropriate cost of capital is also 12 percent, did the firm make the right decision. Explain. b. What is the im...

See Answer

Q: Identify the weaknesses of the payback period method?

Identify the weaknesses of the payback period method?

See Answer

Q: What are the strengths and weaknesses of the accounting rate of return

What are the strengths and weaknesses of the accounting rate of return approach?

See Answer

Q: Under what circumstances might the IRR and NPV approaches produce conflicting results

Under what circumstances might the IRR and NPV approaches produce conflicting results?

See Answer

Q: Do you agree or disagree with the following statement given the discussion

Do you agree or disagree with the following statement given the discussion in this chapter? We can calculate future cash flows precisely and obtain an exact value for the NPV of an investment?

See Answer

Q: Describe the process of capital rationing?

Describe the process of capital rationing?

See Answer

Q: When two mutually exclusive projects have different lives, how can an

When two mutually exclusive projects have different lives, how can an analyst determine which is better? What is the underlying assumption in this method?

See Answer

Q: What is the opportunity cost of using an existing asset? Give

What is the opportunity cost of using an existing asset? Give an example of the opportunity cost of using the excess capacity of a machine?

See Answer

Q: You are providing financial advice to a shrimp farmer who will be

You are providing financial advice to a shrimp farmer who will be harvesting his last crop of farm-raised shrimp. His current shrimp crop is very young and will, therefore, grow and become more valuab...

See Answer

Q: Suppose that FRA Corporation already has divisions in both Dallas and Houston

Suppose that FRA Corporation already has divisions in both Dallas and Houston. FRA is now considering setting up a third division in Austin. This expansion will require that one senior manager from Da...

See Answer

Q: MusicHeaven, Inc., is a producer of media players which currently

MusicHeaven, Inc., is a producer of media players which currently have either 20 gigabytes or 30 gigabytes of storage. Now the company is considering launching a new production line making mini media...

See Answer

Q: QualityLiving Trust is a real estate investment company that builds and remodels

QualityLiving Trust is a real estate investment company that builds and remodels apartment buildings in northern California. It is currently considering remodeling a few idle buildings that it owns in...

See Answer

Q: High-End Fashions, Inc., bought a production line of

High-End Fashions, Inc., bought a production line of ankle-length skirts last year at a cost of $500,000. This year, however, miniskirts are in and ankle-length skirts are completely out of fashion. H...

See Answer

Q: How is the MACRS depreciation method under IRS rules different from the

How is the MACRS depreciation method under IRS rules different from the straight-line depreciation allowed under GAAP rules? What is the implication on incremental after-tax free cash flows from firms...

See Answer

Q: Explain the difference between marginal and average tax rates, and identify

Explain the difference between marginal and average tax rates, and identify which of these rates is used in capital budgeting and why?

See Answer

Q: You are involved in the planning process for a firm that is

You are involved in the planning process for a firm that is expected to have a large increase in sales next year. Which type of firm would benefit the most from that sales increase: a firm with low fi...

See Answer

Q: Why are capital investments considered the most important decisions made by a

Why are capital investments considered the most important decisions made by a firm’s management?

See Answer

Q: What is the advantage of using a simulation analysis instead of a

What is the advantage of using a simulation analysis instead of a scenario analysis to assess the risk of a project?

See Answer

Q: You own a firm with a single new product that is about

You own a firm with a single new product that is about to be introduced to the public for the first time. Your marketing analysis suggests that the annual demand for this product could be anywhere bet...

See Answer

Q: Discuss the interpretation of the degree of accounting operating leverage and degree

Discuss the interpretation of the degree of accounting operating leverage and degree of pretax cash flow operating leverage?

See Answer

Q: Explain how EBITDA differs from incremental after-tax free cash flows

Explain how EBITDA differs from incremental after-tax free cash flows (FCF) and discuss the types of businesses for which this difference would be especially small or large?

See Answer

Q: Describe how the pretax operating cash flow break-even point is

Describe how the pretax operating cash flow break-even point is related to the economic break-even point?

See Answer

Q: Is it possible to have a crossover point where the accounting break

Is it possible to have a crossover point where the accounting break-even point is the same for two alternatives - that is, above the break-even point for a low-fixed-cost alternative but below the bre...

See Answer

Q: What is the fundamental difference between a sensitivity analysis and a scenario

What is the fundamental difference between a sensitivity analysis and a scenario analysis?

See Answer

Q: The economics break-even calculation assumes that the number of units

The economics break-even calculation assumes that the number of units sold is the same each year during the life of the project. It is possible for the NPV of a project to be negative if unit sakes ar...

See Answer

Q: How does the pretax operating cash flow for a project differ from

How does the pretax operating cash flow for a project differ from the economic profit for that project?

See Answer

Q: Explain why the required rate of return on a firm’s assets must

Explain why the required rate of return on a firm’s assets must be equal to the weighted average cost of capital associated with its liabilities and equity?

See Answer

Q: What are the differences between capital projects that are independent, mutually

What are the differences between capital projects that are independent, mutually exclusive, and contingent?

See Answer

Q: Your boss just finished computing your firm’s weighted average cost of capital

Your boss just finished computing your firm’s weighted average cost of capital. He is relieved because he says that he can now use that cost of capital to evaluate all projects that the firm is consid...

See Answer

Q: Which is easier to calculate directly, the expected rate of return

Which is easier to calculate directly, the expected rate of return on the assets of a firm or the expected rate of return on the firm’s debt and equity? Assume that you are an outsider to the firm?

See Answer

Q: With respect to the level of risk and the required return for

With respect to the level of risk and the required return for a firm’s portfolio of projects, discuss how the market and a firm’s management can have inconsistent information and expectations?

See Answer

Q: Your friend has recently told you that the federal government effectively subsidizes

Your friend has recently told you that the federal government effectively subsidizes the use of debt financing (vs. equity financing) for corporations. Do you agree with that statement? Explain?

See Answer

Q: Your firm will have a fixed interest expense for the next 10

Your firm will have a fixed interest expense for the next 10 years. You recently found out that the marginal income tax rate for the firm will change from 30 percent to 40 percent next year. Describe...

See Answer

Q: Describe why it is not usually appropriate to use the coupon rate

Describe why it is not usually appropriate to use the coupon rate on a firm’s bonds to estimate the pretax cost of debt for the firm?

See Answer

Q: Maltese Falcone, Inc., has not checked its weighted average cost

Maltese Falcone, Inc., has not checked its weighted average cost of capital for four years. Firm management claims that since Maltese has not had to raise capital for new projects in four years, they...

See Answer

Q: Ten years ago, the Edson Water Company issued preferred stock at

Ten years ago, the Edson Water Company issued preferred stock at a price equal to the par value of $100. If the dividend yield on that issue was 12 percent, explain why the firm’s current cost of pref...

See Answer

Q: Discuss under what circumstances you might be able to use a model

Discuss under what circumstances you might be able to use a model that assumes constant growth in dividends to calculate the current cost of equity capital for a firm?

See Answer

Q: Suppose that you live in a country where it takes 10 days

Suppose that you live in a country where it takes 10 days to settle a stock purchase. By how many days will the ex-dividend date precede the record date?

See Answer

Q: What is the NPV of a project?

What is the NPV of a project?

See Answer

Q: Lintner found that firms are reluctant to make dividend changes that might

Lintner found that firms are reluctant to make dividend changes that might have to be reversed. Discuss the rationale for that behavior?

See Answer

Q: You find that you are the only investor in a particular stock

You find that you are the only investor in a particular stock who is subject to a 15 percent tax rate on dividends (all other investors are subject to a 5 percent tax rate on dividends). Is there grea...

See Answer

Q: Discuss why the dividend payment process is so much simpler for private

Discuss why the dividend payment process is so much simpler for private companies than for public companies?

See Answer

Q: You are the CEO of a firm that appears to be the

You are the CEO of a firm that appears to be the target of a hostile takeover attempt. Thibeaux Piques has been accumulating the shares of your stock and now holds a substantial percentage of the outs...

See Answer

Q: You have accumulated stock in a firm that does not pay cash

You have accumulated stock in a firm that does not pay cash dividends. You have read that, according to Modigliani and Miller, you can create a “homemade” dividend should you require cash. Discuss why...

See Answer

Q: You have just read a press release in which a firm claims

You have just read a press release in which a firm claims that it will be able to generate a higher level of cash flows for its investors going forward. Justify the choice of a dividend payout that co...

See Answer

Q: Some people argue that a high tax rate on dividends creates incentives

Some people argue that a high tax rate on dividends creates incentives for managers to go about their business without credibly convincing investors that the firm is doing well, even when it is. Discu...

See Answer

Q: Fled Flightstone Mining’s management does not like to pay cash dividends due

Fled Flightstone Mining’s management does not like to pay cash dividends due to the volatility of the company’s cash flows. Fled management has found, however, that when it does not pay dividends, its...

See Answer

Q: Given that many new businesses fail in the first few years after

Given that many new businesses fail in the first few years after they are established, how should an entrepreneur think about the risk of failure associated with a new business? From what you have lea...

See Answer

Q: Explain why it is difficult to value a young, rapidly growing

Explain why it is difficult to value a young, rapidly growing company?

See Answer

Q: What are the five steps used in NPV analysis?

What are the five steps used in NPV analysis?

See Answer

Q: Explain how the taxation of a C-corporation differs from the

Explain how the taxation of a C-corporation differs from the taxation of the other forms of business organization discussed in this chapter?

See Answer

Q: What is a business plan? Explain how a business plan can

What is a business plan? Explain how a business plan can help an entrepreneur succeed in building a business?

See Answer

Q: You are entering negotiations to purchase a business and are trying to

You are entering negotiations to purchase a business and are trying to formulate a negotiating strategy. You want to determine the minimum price you should offer and the maximum you should be willing...

See Answer

Q: You have just received a business valuation report that is dated six

You have just received a business valuation report that is dated six months ago. Describe the factors that might have changed during the past six months and, therefore, caused the value of the busines...

See Answer

Q: Is the replacement cost of a business generally related to the value

Is the replacement cost of a business generally related to the value of the cash flows that the business is expected to produce in the future? Why or why not? Illustrate your answer with an example?

See Answer

Q: You want to estimate the value of a company that has three

You want to estimate the value of a company that has three very different lines of business. It manufactures aircraft, is in the data-processing business, and manufactures automobiles. How could you u...

See Answer

Q: Your boss has asked you to estimate the intrinsic value of the

Your boss has asked you to estimate the intrinsic value of the equity for Google, which does not currently pay any dividends. You are going to use an income approach and are trying to choose between t...

See Answer

Q: Explain how the financial statements of a private company might differ from

Explain how the financial statements of a private company might differ from those of a public company. What does this imply for valuing a private company?

See Answer

Q: Why can the market price of a stock differ from its true

Why can the market price of a stock differ from its true (intrinsic) value?

See Answer

Q: Why are investors and managers concerned about stock market efficiency?

Why are investors and managers concerned about stock market efficiency?

See Answer

Q: What are the major shortcomings of using the Accounting Rate of Return

What are the major shortcomings of using the Accounting Rate of Return (ARR) method as a capital budgeting method?

See Answer

Q: Why are common stockholders considered to be more at risk than the

Why are common stockholders considered to be more at risk than the holders of other types of securities?

See Answer

Q: Under what conditions does it make sense to use the constant-

Under what conditions does it make sense to use the constant-growth dividend model to value a stock?

See Answer

Q: What does it mean when a company has a very high P

What does it mean when a company has a very high P/E ratio? Give examples of industries in which you believe high P/E ratios are justified?

See Answer

Q: The Poseidon Shipping Company has paid a $0.25 dividend

The Poseidon Shipping Company has paid a $0.25 dividend per quarter for the past three years. Poseidon just lowered its declared dividend to $0.20 for the next dividend payment. Discuss what this new...

See Answer

Q: You have just encountered two identical firms with identical investment opportunities,

You have just encountered two identical firms with identical investment opportunities, as well as the ability to fund these opportunities. One of the firms has just announced that it will pay a divide...

See Answer

Q: Explain what the introduction of transaction costs does to the Modigliani and

Explain what the introduction of transaction costs does to the Modigliani and Miller assumption that dividends are irrelevant. Start with a firm that pays dividends to investors that do not want to re...

See Answer

Q: CashCo increased its cash dividend each quarter for the past eight quarters

CashCo increased its cash dividend each quarter for the past eight quarters. While this may signal that the firm is financially very healthy, what else could we conclude from these actions?

See Answer

Q: Undecided Corp. has excess cash on hand right now, although

Undecided Corp. has excess cash on hand right now, although management is not sure about the level of cash flows going forward. If management would like to put cash in stockholders’ hands, what kind o...

See Answer

Q: A firm can deliver a negative signal to stockholders by increasing the

A firm can deliver a negative signal to stockholders by increasing the level of dividends or by reducing the level of dividends. Explain why this is true?

See Answer

Q: A commentator on a financial talk show on TV says that “

A commentator on a financial talk show on TV says that “On average, firms pay out too little to stockholders. This is why stock prices go up with dividend increases and down with dividend decreases.”...

See Answer

Q: What is the Internal Rate of Return (IRR) method?

What is the Internal Rate of Return (IRR) method?

See Answer

Q: You own shares in a firm that has extra cash on hand

You own shares in a firm that has extra cash on hand to distribute to stockholders. You do not want the cash. What course of action would you prefer the firm take?

See Answer

Q: Stock repurchases, once announced, do not actually have to occur

Stock repurchases, once announced, do not actually have to occur in total or in part. From a signaling perspective, why would a special dividend be better than a stock repurchase?

See Answer

Q: Consider a firm that repurchases shares from its stockholders in the open

Consider a firm that repurchases shares from its stockholders in the open market, and explain why this action might be detrimental to the stockholders from whom the firm buys shares?

See Answer

Q: You read that a number of public companies have been financing their

You read that a number of public companies have been financing their dividend payments in recent years entirely through equity issues. A colleague of yours argues that this only increase taxes paid by...

See Answer

Q: Briefly discuss the methods available for a firm to repurchase its shares

Briefly discuss the methods available for a firm to repurchase its shares and explain why you might expect the stock price reaction to the announcement of each of these methods to differ?

See Answer

Q: What is the advantage of a Dutch auction over a fixed-

What is the advantage of a Dutch auction over a fixed-price tender offer?

See Answer

Q: In the early 1990s, the amount of time that elapsed between

In the early 1990s, the amount of time that elapsed between purchasing a stock and actually obtaining that stock was five business days. This period was known as the settlement period. The settlement...

See Answer

Q: Dividend reinvestment programs (DRIPs) sometimes sell shares at a discount

Dividend reinvestment programs (DRIPs) sometimes sell shares at a discount to stockholders who reinvest their dividends through such plans. Your boss tells you that such plans are just a scheme to tra...

See Answer

Q: WeAreProfits, Inc., has not issued any new debt securities in

WeAreProfits, Inc., has not issued any new debt securities in 10 years. It will begin paying cash dividends to its stockholders for the first time next year. Explain how a dividend might help the firm...

See Answer

Q: Shadows, Inc., had shares outstanding that were valued at $

Shadows, Inc., had shares outstanding that were valued at $120 per share before a two-for-one stock split. After the stock split, the shares were valued at $62 per share. If we accept that the firm’s...

See Answer

Q: Why should the NPV method be the primary decision tool used in

Why should the NPV method be the primary decision tool used in making capital investment decisions?

See Answer

Q: Saguaro Company currently has 30,000 shares outstanding. Each share

Saguaro Company currently has 30,000 shares outstanding. Each share has a market value of $20. If the firm pays $5 per share in dividends, what will each share be worth after the dividend payment? Ign...

See Answer

Q: Cholla Company currently has 30,000 shares outstanding. Each share

Cholla Company currently has 30,000 shares outstanding. Each share has a market value of $20. If the firm repurchases $150,000 worth of shares, then what will be the value of each share outstanding af...

See Answer

Q: Place the following in the proper chronological order, and describe the

Place the following in the proper chronological order, and describe the purpose of each: ex-dividend date, record date, payment date, and declaration date?

See Answer

Q: You purchased 1,000 shares of Zebulon Copper Co. five

You purchased 1,000 shares of Zebulon Copper Co. five years ago for $50 per share. Today Zebulon management is trying to decide whether to repurchase shares for $70 per share through a fixed-price ten...

See Answer

Q: You are the Chief Financial Officer (CFO) of a large

You are the Chief Financial Officer (CFO) of a large publicly traded company. You would like to convey positive information about the firm to the market. If you agree with the conclusions from the Lin...

See Answer

Q: You are the CFO of a public company that advises distressed companies

You are the CFO of a public company that advises distressed companies about how to manage their businesses. Your company has been performing extremely well. In fact, it has earned so much money that t...

See Answer

Q: Explain how the issuance of new securities by a firm can produce

Explain how the issuance of new securities by a firm can produce useful information about the issuing firm. How can this information make the shares of the firm more valuable, even if it only confirms...

See Answer

Q: Explain why holders of a firm’s debt should insist on a covenant

Explain why holders of a firm’s debt should insist on a covenant that restricts the amount of cash dividends the firm pays?

See Answer

Q: Explain why managers of firms might prefer that their firms’ shares trade

Explain why managers of firms might prefer that their firms’ shares trade in a moderate per-share price range rather than in a high per-share price range. How do managers of firms keep their shares tr...

See Answer

Q: Explain how a stock repurchase is different from a dividend payment?

Explain how a stock repurchase is different from a dividend payment?

See Answer

Q: What might cause a firm to face capital constraints?

What might cause a firm to face capital constraints?

See Answer

Q: List some common forms of business organization, and discuss how access

List some common forms of business organization, and discuss how access to capital differs across these forms of organization?

See Answer

Q: You want to estimate the total intrinsic value of a large gas

You want to estimate the total intrinsic value of a large gas and electric utility company. This company has publicly traded stock and has been paying a regular dividend for many years. You decide tha...

See Answer

Q: You are considering investing in a private company that is owned by

You are considering investing in a private company that is owned by a friend of yours. You have read through the company’s financial statements and believe that they are reliable. Multiples of similar...

See Answer

Q: Does the expected rate of return that is calculated using CAPM,

Does the expected rate of return that is calculated using CAPM, with a beta estimated from stock returns in the public market, reflect a minority or a controlling ownership position? How is it likely...

See Answer

Q: Discuss the pros and cons of an S-corporation compared with

Discuss the pros and cons of an S-corporation compared with a C-corporation?

See Answer

Q: You have started a business that sells a home gardening system that

You have started a business that sells a home gardening system that allows people to grow vegetables on the countertop in their kitchens. You are considering two options for marketing your product. Th...

See Answer

Q: Aggie Motors is a chain of used car dealerships that has publicly

Aggie Motors is a chain of used car dealerships that has publicly traded stock. Using the adjusted book value approach, you have estimated the value of Aggie Motors to be $45,646,000. The company has...

See Answer

Q: You are an analyst at a private equity firm that buys private

You are an analyst at a private equity firm that buys private companies, improves their operating performance, and sells them for a profit. Your boss has asked you to estimate the fair market value of...

See Answer

Q: Using the enterprise value/EBITDA multiple, what is the total

Using the enterprise value/EBITDA multiple, what is the total value of Johnson Machine Tool Company? What is the per share value of Johnson’s stock? Refer to the information Johnson Machine Tool Comp...

See Answer

Q: Which of the above multiples analyses do you believe is more appropriate

Which of the above multiples analyses do you believe is more appropriate? Refer to the information Johnson Machine Tool Company: Use the following information concerning Johnson Machine Tool Company....

See Answer

Q: How do dealers differ from brokers?

How do dealers differ from brokers?

See Answer

Q: How can the Profitability Index (PI) help in choosing projects

How can the Profitability Index (PI) help in choosing projects when a firm faces capital constraints? What are its limitations?

See Answer

Q: What are some of the things that the founder of a company

What are some of the things that the founder of a company must do to launch a new business?

See Answer

Q: You are using the FCFF approach to value a business. You

You are using the FCFF approach to value a business. You have estimated that the FCFF for next year will be $123.65 million and that it will increase at a rate of 8 percent for each of the following f...

See Answer

Q: You want to estimate the value of a privately owned restaurant that

You want to estimate the value of a privately owned restaurant that is financed entirely with equity. Its most recent income statement is as follows: Revenue ……………………………………………………. $3,000,000 Cost of g...

See Answer

Q: A few years ago, a friend of yours started a small

A few years ago, a friend of yours started a small business that develops gaming software. The company is doing well and is valued at $1.5 million based on multiples for comparable public companies af...

See Answer

Q: You plan to start a business that sells waterproof sun block with

You plan to start a business that sells waterproof sun block with a unique formula that reduces the damage of UVA radiation 30 percent more effectively than similar products on the market. You expect...

See Answer

Q: For the previous question, assume that you do not have sufficient

For the previous question, assume that you do not have sufficient savings to cover the entire amount required to start your sun-block business. You are going to have to get external financing. A local...

See Answer

Q: Your friend is starting a new company. He wants to write

Your friend is starting a new company. He wants to write a business plan to clarify the company’s business outlook and raise venture capital. Knowing that you have taken this course, he has asked you,...

See Answer

Q: A friend of yours is trying to value the equity of a

A friend of yours is trying to value the equity of a company and, knowing that you have read this book, has asked for your help. So far she has tried to use the FCFE approach. She estimated the cash f...

See Answer

Q: Forever Youth Technology is a biochemical company that is two years old

Forever Youth Technology is a biochemical company that is two years old. Its main product, an antioxidant drink that is supposed to energize the consumer and delay aging, is still under development. T...

See Answer

Q: Mad Rock Inc. is a company that sells mp3 music online

Mad Rock Inc. is a company that sells mp3 music online. It is expected to generate earnings of $1 per share this year after its Web site is upgraded and online marketing is stepped up. Given the popul...

See Answer

Q: What changes have taken place in the capital budgeting techniques used by

What changes have taken place in the capital budgeting techniques used by U.S. companies?

See Answer

Q: At the end of 2016 the value of the S&P

At the end of 2016 the value of the S&P 500 Index divided by the estimated 2016 earnings for S&P 500 firms (the S&P 500 P/E multiple) was 25.54. Assume that the long-term Treasury bond yield was 2.88...

See Answer

Q: Explain how financial liabilities differ among different forms of business organization?

Explain how financial liabilities differ among different forms of business organization?

See Answer

Q: The S&P 500 P/E multiple of 25.

The S&P 500 P/E multiple of 25.54 at the end of 2016 was higher than its historical average of approximately 15. Some financial commentators argued that this meant that the firms in the S&P 500 were,...

See Answer

Q: You own a company that produces and distributes course packets for classes

You own a company that produces and distributes course packets for classes at local universities via the Internet. You have asked a friend to invest $35,000 in the business. Your friend wants to know...

See Answer

Q: List two useful tools to help an entrepreneur to understand the cash

List two useful tools to help an entrepreneur to understand the cash requirements of a business and to estimate the financing needs of his or her business?

See Answer

Q: You believe you have a great business idea and want to start

You believe you have a great business idea and want to start your own company. However, you do not have enough savings to finance it. Where can you get the additional funds you need?

See Answer

Q: Why is it especially difficult for an entrepreneur with a new business

Why is it especially difficult for an entrepreneur with a new business to raise capital? What tool can help him or her to raise external capital?

See Answer

Q: It is April 4, 2018, and your company is considering

It is April 4, 2018, and your company is considering the possibility of purchasing the Chrysler automobile manufacturing business. Managers of Fiat Chrysler Automobiles N.V, the automobile manufacture...

See Answer

Q: Why is excess cash a nonoperating asset (NOA)? Why does

Why is excess cash a nonoperating asset (NOA)? Why does it make sense to add the value of excess cash to the value of the discounted cash flows when we use the WACC or FCFE approach to value a busines...

See Answer

Q: Fresno Corp. is a fast-growing company whose management expects

Fresno Corp. is a fast-growing company whose management expects it to grow at a rate of 30 percent over the next two years and then slow down to a growth rate of 18 percent for the following three yea...

See Answer

Q: Why do we care about incremental cash flows at the firm level

Why do we care about incremental cash flows at the firm level when we evaluate a project?

See Answer

Q: Riggs Corp. management is planning to spend $650,000

Riggs Corp. management is planning to spend $650,000 on a new- marketing campaign. They believe that this action will result in additional cash flows of $325,000 over the next three years. If the disc...

See Answer

Q: Quebec, Inc., is purchasing machinery at a cost of $

Quebec, Inc., is purchasing machinery at a cost of $3,768,966. The company’s management expects the machinery to produce cash flows of $979,225, $1,158,886, and $1,881,497 over the next three years, r...

See Answer

Q: Northern Specialties just purchased inventory-management computer software at a cost

Northern Specialties just purchased inventory-management computer software at a cost of $1,645,276. Cost savings from the investment over the next six years will produce the following cash flow stream...

See Answer

Q: Nakamichi Bancorp has made an investment in banking software at a cost

Nakamichi Bancorp has made an investment in banking software at a cost of $1,875,000. Management expects productivity gains and cost savings over the next several years. If, as a result of this invest...

See Answer

Q: Capitol Corp. management is expecting a project to generate after-

Capitol Corp. management is expecting a project to generate after-tax income of $63,435 in each of the next three years. The average book value of the project’s equipment over that period will be $212...

See Answer

Q: Refer to Problem 10.4. What is the IRR that

Refer to Problem 10.4. What is the IRR that Franklin Mints management can expect on this project? Refer to Problem 10.4. Management of Franklin Mints, a confectioner, is considering purchasing a new...

See Answer

Q: Reco Corp. is expected to pay a dividend of $2

Reco Corp. is expected to pay a dividend of $2.25 next year. The forecast for the stock price a year from now is $37.50. If the required rate of return is 14 percent, what is the current stock price?...

See Answer

Q: What is the profitability index, and why is it helpful in

What is the profitability index, and why is it helpful in the capital rationing process?

See Answer

Q: Champlain Corp. management is investigating two computer systems. The Alpha

Champlain Corp. management is investigating two computer systems. The Alpha 8300 costs $3,122,300 and will generate cost savings of $1,345,500 in each of the next five years. The Beta 2100 system cost...

See Answer

Q: Briarcrest Condiments is a spice-making firm. Recently, it

Briarcrest Condiments is a spice-making firm. Recently, it developed a new process for producing spices. The process requires new machinery that would cost $1,968,450, have a life of five years, and w...

See Answer

Q: Why is D&A first subtracted and then added back in

Why is D&A first subtracted and then added back in FCF calculations?

See Answer

Q: Cranjet Industries is expanding its product line and its production capacity.

Cranjet Industries is expanding its product line and its production capacity. The costs and expected cash flows of the two independent projects are given in the following table. The firm uses a discou...

See Answer

Q: Emporia Mills management is evaluating two alternative heating systems. Costs and

Emporia Mills management is evaluating two alternative heating systems. Costs and projected energy savings are given in the following table. The firm uses 11.5 percent to discount such project cash fl...

See Answer

Q: Creative Solutions, Inc., has just invested $4,615

Creative Solutions, Inc., has just invested $4,615,300 in new equipment. The firm uses a payback period criteria of not accepting any project that takes more than four years to recover its costs. Mana...

See Answer

Q: Timeline Manufacturing Co. management is evaluating two projects. The company

Timeline Manufacturing Co. management is evaluating two projects. The company uses payback criteria of three years or less. Project A has a cost of $912,855, and project B’s cost is...

See Answer

Q: Regent Corp. management is evaluating three competing types of equipment.

Regent Corp. management is evaluating three competing types of equipment. Costs and cash flow projections for all three are given in the following table. Which would be the best choice based on paybac...

See Answer

Q: Nugent Communication Corp. is investing $9,365,000

Nugent Communication Corp. is investing $9,365,000 in new technologies. The company’s management expects significant benefits in the first three years after installation (as can be s...

See Answer

Q: You own a company that competes with Old World DVD Company (

You own a company that competes with Old World DVD Company (in the previous problem). Instead of selling DVDs, however, your company sells music downloads from a web site. Things are going well now, b...

See Answer

Q: Tre-Bien, Inc., is a fast-growing technology

Tre-Bien, Inc., is a fast-growing technology company. Management projects rapid growth of 30 percent for the next two years, then a growth rate of 17 percent for the following two years. After that, a...

See Answer

Q: Management of ProCor, a biotech firm, forecasted the following growth

Management of ProCor, a biotech firm, forecasted the following growth rates for the next three years: 35 percent, 28 percent, and 22 percent. Management then expects the company to grow at a constant...

See Answer

Q: Revarop, Inc., is a fast-growth company that is

Revarop, Inc., is a fast-growth company that is expected to grow at a rate of 23 percent for the next four years. It is then expected to grow at a constant rate of 6 percent. Revarop’s first dividend,...

See Answer

Q: What is the difference between nominal and real dollars? Why is

What is the difference between nominal and real dollars? Why is it important not to mix them in an NPV analysis?

See Answer

Q: Quansi, Inc., management expects to pay no dividends for the

Quansi, Inc., management expects to pay no dividends for the next six years. It has projected a growth rate of 25 percent for the next seven years. After seven years, the firm will grow at a constant...

See Answer

Q: Staggert Corp. will pay dividends of $5.00,

Staggert Corp. will pay dividends of $5.00, $6.25, $4`.75, and $3.00 in the next four years. Thereafter, management expects the dividend growth rate to be constant at 6 percent. If the required rate o...

See Answer

Q: Diaz Corp. is expected to grow rapidly/ at a rate

Diaz Corp. is expected to grow rapidly/ at a rate of 35 percent for the next seven years. The company’s first dividend, to be paid three years from now, will be $5. After seven years, the company (and...

See Answer

Q: Tin-Tin Waste Management, Inc., is growing rapidly.

Tin-Tin Waste Management, Inc., is growing rapidly. Dividends are expected to grow at rates of 30 percent, 35 percent, 25 percent, and 18 percent over the next four years. Thereafter, management expec...

See Answer

Q: Equation 9.4 shows the relation between a stock’s value and

Equation 9.4 shows the relation between a stock’s value and the dividend that is expected next year if dividends grow at a constant rate forever. If a firm pays all of its earnings as dividends, show...

See Answer

Q: Riker Departmental Stores management has forecasted a growth rate of 40 percent

Riker Departmental Stores management has forecasted a growth rate of 40 percent for the next two years, followed by growth rates of 25 percent and 20 percent for the following two years. It then expec...

See Answer

Q: Refer to Problem 10.31. a. What are

Refer to Problem 10.31. a. What are the IRRs for the projects? b. Does the IRR criterion suggest a different decision than the NPV criterion? c. Explain how you would expect the management of Draconia...

See Answer

Q: Management of Dravid, Inc., is currently evaluating three projects that

Management of Dravid, Inc., is currently evaluating three projects that are independent. The cost of funds can be either 13.6 percent or 14.8 percent depending on their financing plan. All three proje...

See Answer

Q: Perry, Inc., paid a dividend of $2.50

Perry, Inc., paid a dividend of $2.50 yesterday. You are interested in investing in this company, which has forecasted a constant-growth rate of 7 percent for its dividends, forever. The required rate...

See Answer

Q: ZweitePharma is a fast-growing drug company. Management forecasts that

ZweitePharma is a fast-growing drug company. Management forecasts that in the next three years, the company’s dividend growth rates will be 30 percent, 28 percent, and 24 percent, respectively. Last w...

See Answer

Q: What is a progressive tax system? What is the difference between

What is a progressive tax system? What is the difference between a firm’s marginal and average tax rates?

See Answer

Q: Triton Inc., is expected to grow at a rate of 22

Triton Inc., is expected to grow at a rate of 22 percent for the next five years and then settle to a constant growth rate of 6 percent. The company recently paid a dividend of $2.35. The required rat...

See Answer

Q: Ceebros Builders is expanding very fast and is expected to grow at

Ceebros Builders is expanding very fast and is expected to grow at a rate of 25 percent for the next four years. The company recently paid a dividend of $3.60 but is not expected to pay any dividends...

See Answer

Q: Primus Corp. management is planning to convert an existing warehouse into

Primus Corp. management is planning to convert an existing warehouse into a new plant that will increase its production capacity by 45 percent. The cost of this project will be $7,125,000. It will res...

See Answer

Q: Quasar Tech Co. management is investing $6 million in new

Quasar Tech Co. management is investing $6 million in new machinery that will produce the next-generation routers. Sales to its customers will amount to $1,750,000 for the next three years and then in...

See Answer

Q: Management of Skywards, Inc., an airline caterer, is purchasing

Management of Skywards, Inc., an airline caterer, is purchasing refrigerated trucks at a total cost of $3.25 million. After-tax net income from this investment is expected to be $750,000 for the next...

See Answer

Q: Trident Corp. management is evaluating two independent projects. The costs

Trident Corp. management is evaluating two independent projects. The costs and expected cash flows are given in the following table. The cost of capital is 10 percent? a. Calculate the projects&acir...

See Answer

Q: Management of Tyler, Inc., is considering switching to a new

Management of Tyler, Inc., is considering switching to a new production technology. The cost of the required equipment will be $4 million. The discount rate is 12 percent. The cash flows that manageme...

See Answer

Q: You are analyzing two proposed capital investments with the following cash flows

You are analyzing two proposed capital investments with the following cash flows: The cost of capital for both projects is 10 percent. Calculate the profitability index (PI) for each project. Which p...

See Answer

Q: Given the following cash flows for a capital project, calculate the

Given the following cash flows for a capital project, calculate the NPV and IRR. The required rate of return is 8 percent. …. NPV ……â...

See Answer

Q: Given the following cash flows for a capital project, calculate its

Given the following cash flows for a capital project, calculate its payback period and discounted payback period. The required rate of return is 8 percent. The discounted payback period is a. 0.16 ye...

See Answer

Q: How can FCF in the terminal year of a project’s life differ

How can FCF in the terminal year of a project’s life differ from FCF in the other years?

See Answer

Q: An investment of $100 generates after-tax cash flows of

An investment of $100 generates after-tax cash flows of $40 in Year 1, $80 in Year 2, and $120 in Year 3. The required rate of return is 20 percent. The net present value is closest to a. $42.22 b. $5...

See Answer

Q: An investment of $150,000 is expected to generate an

An investment of $150,000 is expected to generate an after-tax cash flow of $100,000 in one year and another $120,000 in two years. The cost of capital is 10 percent. What is the internal rate of retu...

See Answer

Q: Hathaway, Inc., a resort management company, is refurbishing one

Hathaway, Inc., a resort management company, is refurbishing one of its hotels at a cost of $7.8 million. Management expects that this will lead to additional cash flows of $1.8 million for the next s...

See Answer

Q: Morningside Bakeries recently purchased equipment at a cost of $650,

Morningside Bakeries recently purchased equipment at a cost of $650,000. Management expects the equipment to generate cash flows of $275,000 in each of the next four years. The cost of capital is 14 p...

See Answer

Q: Management of Sycamore Home Furnishings is considering acquiring a new machine that

Management of Sycamore Home Furnishings is considering acquiring a new machine that can create customized window treatments. The equipment will cost $263,400 and will generate cash flows of $85,000 ov...

See Answer

Q: Management of Great Flights, Inc., an aviation firm, is

Management of Great Flights, Inc., an aviation firm, is considering purchasing three aircraft for a total cost of $161 million. The company would lease the aircraft to an airline. Cash flows from the...

See Answer

Q: Refer to problem 10.5. Compute the IRR for both

Refer to problem 10.5. Compute the IRR for both production System 1 and production System 2. Which has the higher IRR? Which production system has the higher NVP? Explain why the IRR and NPV ranki...

See Answer

Q: Ancala Corporation management is considering investments in two new golf apparel lines

Ancala Corporation management is considering investments in two new golf apparel lines for next season: golf hats and belts. Due to a funding constraint, these lines are mutually exclusive. A summary...

See Answer

Q: Compute the IRR on the following cash flow streams: a

Compute the IRR on the following cash flow streams: a. An initial investment of $25,000 followed by a single cash flow of $37,450 in year 6. b. An initial investment of $1 million followed by a single...

See Answer

Q: Compute the IRR for the following project cash flows: a

Compute the IRR for the following project cash flows: a. An initial outlay of $3,125,000 followed by annual cash flows of $565,325 for the next eight years. b. An initial investment of $33,750 followe...

See Answer

Q: What is the difference between variable and fixed costs, and what

What is the difference between variable and fixed costs, and what are examples of each?

See Answer

Q: Suppose that you could invest in the following projects but have only

Suppose that you could invest in the following projects but have only $30,000 to invest. How would you make your decision and in which projects would you invest?

See Answer

Q: Crescent Industries management is planning to replace some existing machinery in its

Crescent Industries management is planning to replace some existing machinery in its plant. The cost of the new equipment and the resulting cash flows are shown in the accompanying table. If the firm...

See Answer

Q: Suppose that you could invest in the same projects as in the

Suppose that you could invest in the same projects as in the previous problem, but have only $25,000 to invest. Which projects would you chose?

See Answer

Q: Management of Draconian Measures, Inc., is evaluating two independent projects

Management of Draconian Measures, Inc., is evaluating two independent projects. The company uses a 13.8 percent discount rate for such projects. The costs and cash flows for the projects are shown in...

See Answer

Q: Management of Intrepid, Inc., is considering investing in three independent

Management of Intrepid, Inc., is considering investing in three independent projects. The costs and the cash flows are given in the following table. The appropriate cost of capital is 14.5 percent. Co...

See Answer

Q: Jekyll & Hyde Corp. management is evaluating two mutually exclusive projects

Jekyll & Hyde Corp. management is evaluating two mutually exclusive projects. The cost of capital is 15 percent. Costs and cash flows for each project are given in the following table. Which proje...

See Answer

Q: Management of Larsen Automotive, a manufacturer of auto parts, is

Management of Larsen Automotive, a manufacturer of auto parts, is considering investing in two projects. The company typically compares project returns to a cost of funds of 17 percent. Compute the IR...

See Answer

Q: Compute the IRR for each of the following projects: /

Compute the IRR for each of the following projects:

See Answer

Q: Management of Franklin Mints, a confectioner, is considering purchasing a

Management of Franklin Mints, a confectioner, is considering purchasing a new jelly bean-making machine at a cost of $312,500. They project that the cash flows from this investment will be $121,450 fo...

See Answer

Q: Blanda Incorporated management is considering investing in two alternative production systems.

Blanda Incorporated management is considering investing in two alternative production systems. The systems are mutually exclusive, and the cost of the new equipment and the resulting cash flows are sh...

See Answer

Q: How are working capital items forecast? Why are accounts receivable typically

How are working capital items forecast? Why are accounts receivable typically forecast as a percentage of revenue and accounts payable and inventories as percentages of the cost of good sold?

See Answer

Q: Why do we use forecasted incremental after-tax free cash flows

Why do we use forecasted incremental after-tax free cash flows instead of forecasted accounting earnings in estimating the NPV of a project?

See Answer

Q: Keswick Supply Company wants to set up a division that provides copy

Keswick Supply Company wants to set up a division that provides copy and fax services to businesses. Customers will be given 20 days to pay for such services. The annual revenue of the division is est...

See Answer

Q: Define expected cash flows, and explain why this concept is important

Define expected cash flows, and explain why this concept is important in evaluating projects?

See Answer

Q: Explain the concept of equivalent annual cost and how it is used

Explain the concept of equivalent annual cost and how it is used to compare projects with different lives?

See Answer

Q: You are buying a sofa. You will pay $200 today

You are buying a sofa. You will pay $200 today and make three consecutive annual payments of $300 in the future. The real rate of return is 10 percent, and the expected inflation rate is 4 percent. Wh...

See Answer

Q: Healthy Potions, Inc., is considering investing in a new production

Healthy Potions, Inc., is considering investing in a new production line for eye drops. Other than investing in the equipment, the company needs to increase its cash and cash equivalents by $10,000, i...

See Answer

Q: Given the soaring price of gasoline, Ford is considering introducing a

Given the soaring price of gasoline, Ford is considering introducing a new production line of gas-electric hybrid sedans. The expected annual unit sales of the hybrid cars is 30,000; the price is $22,...

See Answer

Q: Archer Daniels Midland Company is considering buying a new farm that it

Archer Daniels Midland Company is considering buying a new farm that it plans to operate for 10 years. The farm will require an initial investment of $12 million. This investment will consist of $2 mi...

See Answer

Q: You are trying to choose between purchasing one of two machines for

You are trying to choose between purchasing one of two machines for a factory. Machine A costs $15,000 to purchase and has a three-year life. Machine B costs $17,700 to purchase but has a four year li...

See Answer

Q: You are starting a family pizza parlor and need to buy a

You are starting a family pizza parlor and need to buy a motorcycle for delivery orders. You have two models in mind. Model A costs $9,000 and is expected to run for 6 years; Model B is more expensive...

See Answer

Q: When can we not simply compare the NPVs of two mutually exclusive

When can we not simply compare the NPVs of two mutually exclusive projects?

See Answer

Q: Predator LLC, a leveraged-buyout specialist, recently bought a

Predator LLC, a leveraged-buyout specialist, recently bought a company and wants to determine the optimal time to sell it. The partner in charge of this investment has estimated the after-tax cash flo...

See Answer

Q: Bell Mountain Vineyards is considering updating its current manual accounting system with

Bell Mountain Vineyards is considering updating its current manual accounting system with a high-end electronic system. While the new accounting system would save the company money, the cost of the sy...

See Answer

Q: You have a 2000 Nissan that is expected to run for another

You have a 2000 Nissan that is expected to run for another three years, but you are considering buying a new Hyundai before the Nissan wears out. You will donate the Nissan to Goodwill when you buy th...

See Answer

Q: Assume that you are considering replacing your old Nissan with a new

Assume that you are considering replacing your old Nissan with a new Hyundai, as in the previous problem. However, the annual maintenance cost of the old Nissan increases as time goes by. It is $1,200...

See Answer

Q: Anaconda Manufacturing Company currently own a mine that is known to contain

Anaconda Manufacturing Company currently own a mine that is known to contain a certain amount of gold. Since Anaconda does not have any gold-mining expertise, the company plans to sell the entire mine...

See Answer

Q: You are thinking about delivering pizzas in your spare time. Since

You are thinking about delivering pizzas in your spare time. Since you must use your own car to deliver the pizzas, you will wear out your current car one year earlier, which is one year from today, t...

See Answer

Q: How do we adjust for depreciation when we calculate incremental after-

How do we adjust for depreciation when we calculate incremental after-tax free cash flow from EBITDA? What is the intuition for the adjustment?

See Answer

Q: You are the CFO of SlimBody, Inc., a retailer of

You are the CFO of SlimBody, Inc., a retailer of the exercise machine Slimbody6 and related accessories. Your firm is considering opening up a new store in Los Angeles. The store will have a life of 2...

See Answer

Q: Merton Shovel Corporation has decided to bid for a contract to supply

Merton Shovel Corporation has decided to bid for a contract to supply shovels to the Honduran Army. The Honduran Army intends to buy 1,000 shovels per year for the next three years. To supply these sh...

See Answer

Q: Rocky Mountain Lumber, Inc., is considering purchasing a new wood

Rocky Mountain Lumber, Inc., is considering purchasing a new wood saw that costs $50,000. The saw will generate revenues of $100,000 per year for five years. The cost of materials and labor needed to...

See Answer

Q: List the major stock market indexes, and explain what they tell

List the major stock market indexes, and explain what they tell us?

See Answer

Q: How do we decide when to harvest an asset?

How do we decide when to harvest an asset?

See Answer

Q: A beauty product company is developing a new fragrance named Happy Forever

A beauty product company is developing a new fragrance named Happy Forever. There is a probability of 0.5 that consumers will love Happy Forever, and in this case, annual sales will be 1 million bottl...

See Answer

Q: Great Fit, Inc., is a company that manufactures clothing.

Great Fit, Inc., is a company that manufactures clothing. The company has a production line that produces women’s tops of regular sizes. The same machine could be used to produce petite sizes as well....

See Answer

Q: Biotech Partners LLC has been farming a new strain of radioactive-

Biotech Partners LLC has been farming a new strain of radioactive-material-eating bacteria that the electrical utility industry can use to help dispose of its nuclear waste. Two opposing factors affec...

See Answer

Q: ACME Manufacturing is considering replacing an existing production line with a new

ACME Manufacturing is considering replacing an existing production line with a new line that has a greater output capacity and operates with less labor than the existing line. The new line would cost...

See Answer

Q: The alternative to investing in the new production line in Problem 11

The alternative to investing in the new production line in Problem 11.36 is to overhaul the existing line, which currently has both a book value and a salvage value of $0. It would cost $300,000 to ov...

See Answer

Q: FITCO is considering the purchase of new equipment. The equipment costs

FITCO is considering the purchase of new equipment. The equipment costs $350,000, and an additional $110,000 is needed to install it. The equipment will be depreciated straight-line to zero over a fiv...

See Answer

Q: After estimating a project’s NPV, the analyst is advised that the

After estimating a project’s NPV, the analyst is advised that the fixed capital outlay will be revised upward by $100,000. The fixed capital outlay is depreciated straight-line over an eight-year life...

See Answer

Q: What is the difference between nominal and real cash flows? Which

What is the difference between nominal and real cash flows? Which rate of return should we use to discount each type of cash flow?

See Answer

Q: What is the difference between average tax rate and the marginal tax

What is the difference between average tax rate and the marginal tax rate? Which one should we use in calculating incremental after-tax cash flows?

See Answer

Q: Healthy Potions, Inc., a pharmaceutical company, bought a machine

Healthy Potions, Inc., a pharmaceutical company, bought a machine at a cost of $2 million five years ago that produces pain-reliever medicine. The machine has been depreciated over the past five years...

See Answer

Q: Why do analysts care about how sensitive EBITDA and EBIT are to

Why do analysts care about how sensitive EBITDA and EBIT are to changes in revenue?

See Answer

Q: What are variable costs and fixed costs? What are some examples

What are variable costs and fixed costs? What are some examples of each? How are these costs estimated in forecasting operating expenses?

See Answer

Q: When forecasting operating expenses, explain the difference between a fixed cost

When forecasting operating expenses, explain the difference between a fixed cost and a variable cost?

See Answer

Q: Zippy Corporation just purchased computing equipment for $20,000.

Zippy Corporation just purchased computing equipment for $20,000. The equipment will be depreciated using a five-year MACRS depreciation schedule. If the equipment is sold at the end of its fourth yea...

See Answer

Q: Define variable costs and fixed costs, and give an example of

Define variable costs and fixed costs, and give an example of each?

See Answer

Q: Calculate the accounting operating profit break-even point and pretax operating

Calculate the accounting operating profit break-even point and pretax operating cash flow break-even point for each of the three production choices outlined below?

See Answer

Q: The accounting operating profit break-even point tells us the number

The accounting operating profit break-even point tells us the number of units that must be sold for a firm to break-even in a given year from an accounting operating profit perspective. What measure t...

See Answer

Q: What is simulation analysis, and how is it used?

What is simulation analysis, and how is it used?

See Answer

Q: Duplicate Footballs, Inc., expects to sell 15,000 balls

Duplicate Footballs, Inc., expects to sell 15,000 balls this year. The balls sell for $110 each and have a variable cost per unit of $80. Fixed costs, including depreciation and amortization, are curr...

See Answer

Q: Specialty Light Bulbs anticipates selling 3,000 light bulbs this year

Specialty Light Bulbs anticipates selling 3,000 light bulbs this year at a price of $15 per bulb. It costs Specialty $10 in variable costs to produce each light bulb, and the fixed costs for the firm...

See Answer

Q: The pretax operating cash flow of Memphis Motors declined so much during

The pretax operating cash flow of Memphis Motors declined so much during the recession of 2008 and 2009 that the company almost defaulted on its debt. The owner of the company wants to change the cost...

See Answer

Q: What do the degree of pretax cash flow operating leverage (Cash

What do the degree of pretax cash flow operating leverage (Cash Flow DOL) and the degree of accounting operating leverage (Accounting DOL) tell us?

See Answer

Q: For the Vinyl CD Co. in Self-Study Problem 12

For the Vinyl CD Co. in Self-Study Problem 12.3, what percentage increase in pretax operating cash flow will be driven by the additional revenue?

See Answer

Q: Calculate the accounting operating profit break-even point for both factory

Calculate the accounting operating profit break-even point for both factory choices for Dandle’s Candles? Use the following information below: Dandle’s Candles will be producing a new line of driples...

See Answer

Q: Calculate the number of candles for which the accounting operating profit at

Calculate the number of candles for which the accounting operating profit at Dandle's Candles is the same regardless of the factory choice? Use the following information below: Dandle’s Candles will...

See Answer

Q: Describe the role that the mix of variable versus fixed costs has

Describe the role that the mix of variable versus fixed costs has in the variation of earnings before interest and taxes (EBIT) for the firm?

See Answer

Q: Calculate the pretax operating cash flow break-even point for both

Calculate the pretax operating cash flow break-even point for both factory choices for Dandle’s Candles? Use the following information below: Dandle’s Candles will be producing a new line of dripless...

See Answer

Q: Your analysis tells you that at a projected level of sales,

Your analysis tells you that at a projected level of sales, a project your firm is considering will be below accounting break-even but above cash flow break-even. Explain why this might still be a via...

See Answer

Q: Management of March and Dine Inc. has estimated that the firm’s

Management of March and Dine Inc. has estimated that the firm’s new TV dinner project must generate $10,200 in FCF during each of the next six years to have an NPV of $0. Management anticipates that...

See Answer

Q: RoseWeiser Company Management is considering a project that will require an initial

RoseWeiser Company Management is considering a project that will require an initial investment of $50,000 and will last for 10 years. No other capital expenditures or increases in working capital are...

See Answer

Q: The BowGus Archery Company management estimates that its new Galactically Flexible Bow

The BowGus Archery Company management estimates that its new Galactically Flexible Bow project will have to generate EBIT of $20,000 each year to be viable. The project’s fixed cash expenses are expec...

See Answer

Q: Sensitivity analysis and scenario analysis are somewhat similar. Describe which is

Sensitivity analysis and scenario analysis are somewhat similar. Describe which is a more realistic method of analyzing the impact of different scenarios on a project?

See Answer

Q: What is the difference between the pretax operating cash flow break-

What is the difference between the pretax operating cash flow break-even point and the accounting operating profit break-even point?

See Answer

Q: Describe the circumstances under which sensitivity analysis might be a reasonable basis

Describe the circumstances under which sensitivity analysis might be a reasonable basis for determining changes to a firm’s EBIT or FCF?

See Answer

Q: Chip’s Home Brew Whiskey management forecasts that if the firm sells each

Chip’s Home Brew Whiskey management forecasts that if the firm sells each bottle of Snake-Bite for $20, then the demand for the product will be 15,000 bottles per year, whereas sales will be 90 percen...

See Answer

Q: If you were interested in calculating the probability that your project will

If you were interested in calculating the probability that your project will have a positive FCF, what type of risk analysis tool will you most likely use?

See Answer

Q: Mick’s Soft Lemonade is starting to develop a new product for which

Mick’s Soft Lemonade is starting to develop a new product for which the cash fixed costs are expected to be $80,000. The projected EBIT is $100,000, and the Accounting DOL is expected to be 2.0. What...

See Answer

Q: The Generic Publications Textbook Company sells all of its books for $

The Generic Publications Textbook Company sells all of its books for $100 per book, and it currently costs $50 in variable costs to produce each text. The fixed costs, which include depreciation and a...

See Answer

Q: If a firm has a fixed asset base, meaning that its

If a firm has a fixed asset base, meaning that its depreciation and amortization for any year is positive, discuss the relation between its Accounting DOL and its Cash flow DOL?

See Answer

Q: Silver Polygon, Inc., has determined that if its revenues were

Silver Polygon, Inc., has determined that if its revenues were to increase by 10 percent, then EBIT would increase by 25 percent to $100,000. The fixed costs (cash only) for the firm are $100,000. Giv...

See Answer

Q: If a firm’s costs (both variable as well as fixed)

If a firm’s costs (both variable as well as fixed) are known with certainty, then what are the only two sources of volatility for the firm’s operating profits or its operating cash flows?

See Answer

Q: Using the same logic as with the accounting break-even calculation

Using the same logic as with the accounting break-even calculation in Problem 12.19, adapt the formula for crossover level of unit sales to find the number of units sold where the pretax operating cas...

See Answer

Q: You are the project manager for Eagle Golf Corporation. You are

You are the project manager for Eagle Golf Corporation. You are considering manufacturing a new golf wedge with a unique groove design. You have put together the estimates in the following table abo...

See Answer

Q: How is the per-unit contribution related to the accounting operating

How is the per-unit contribution related to the accounting operating profit break-even point?

See Answer

Q: Commodore Motors management is considering a project to produce toy cars.

Commodore Motors management is considering a project to produce toy cars. The project would require an initial outlay of $100,000 and have an expected life of 10 years. Management estimates that eac...

See Answer

Q: The Fulcrum Company produces decorative swivel platforms for home televisions. If

The Fulcrum Company produces decorative swivel platforms for home televisions. If Fulcrum produces 40 million units, it estimates that it can sell them for $100 each. The variable production costs are...

See Answer

Q: WalkAbout Kangaroo Shoe Stores forecasts that it will sell 9,500

WalkAbout Kangaroo Shoe Stores forecasts that it will sell 9,500 pairs of shoes next year. The firm buys its shoes for $50 per pair from the wholesaler and sells them for $75 per pair. If the firm wil...

See Answer

Q: The degree of pretax cash flow operating leverage at Rackit Corporation is

The degree of pretax cash flow operating leverage at Rackit Corporation is 2.7 when it sells 100,000 units of its new tennis racket and its EBITDA is $95,000. Ignoring the effects of taxes, what are...

See Answer

Q: Caterpillar, Inc. is a manufacturer of large earth-moving

Caterpillar, Inc. is a manufacturer of large earth-moving and mining equipment. This firm, and other heavy equipment manufacturers, have accounting degrees of operating leverage that are relatively hi...

See Answer

Q: Why is the per-unit contribution important in a break-

Why is the per-unit contribution important in a break-even analysis?

See Answer

Q: KneeMan Markup Company has total debt obligations with book and market values

KneeMan Markup Company has total debt obligations with book and market values equal to $30 million and $28 million, respectively. It also has total equity with book and market values equal to $20 mill...

See Answer

Q: Describe the alternatives to using a firm’s WACC as a discount rate

Describe the alternatives to using a firm’s WACC as a discount rate when evaluating a project?

See Answer

Q: Explain why the total value of all of the securities used to

Explain why the total value of all of the securities used to finance a firm must be equal to the value of the firm?

See Answer

Q: Explain why the cost of capital for a firm is equal to

Explain why the cost of capital for a firm is equal to the expected rate of return to the investors in the firm?

See Answer

Q: What is a simulation analysis, and what can it tell us

What is a simulation analysis, and what can it tell us?

See Answer

Q: You know that the after-tax cost of debt capital for

You know that the after-tax cost of debt capital for Bubbles Champagne Company is 7 percent. If the firm has only one issue of five-year bonds outstanding, what is the current price of the bonds if th...

See Answer

Q: You are analyzing the cost of debt for a firm. You

You are analyzing the cost of debt for a firm. You know that the firm’s 14-year maturity, 8.5 percent coupon bonds are selling at a price of $823.48. The bonds pay interest semiannually. If these bond...

See Answer

Q: Holding all other things constant, does a decrease in the marginal

Holding all other things constant, does a decrease in the marginal tax rate for a firm provide incentive for the managers of a firm to increase or decrease its use of debt?

See Answer

Q: You are analyzing the after-tax cost of debt for a

You are analyzing the after-tax cost of debt for a firm. You know that the firm’s 12-year maturity, 9.5 percent semiannual coupon bonds are selling at a price of $1,200. If these bonds are the only de...

See Answer

Q: What is the weighted average cost of capital for a firm?

What is the weighted average cost of capital for a firm?

See Answer

Q: Underestimated Inc.’s common shares currently sell for $36 each

Underestimated Inc.’s common shares currently sell for $36 each. The firm’s management believes that its shares should really sell for $54 each. If the firm just paid an annual dividend of $2 per shar...

See Answer

Q: Write out the general equation for the price of the stock for

Write out the general equation for the price of the stock for a firm that will grow dividends very rapidly at a constant rate for the four years after the next dividend is paid and will grow dividends...

See Answer

Q: You have calculated the cost of common stock using all three methods

You have calculated the cost of common stock using all three methods described in this chapter. Unfortunately, all three methods have yielded different answers. Describe which answer (if any) is most...

See Answer

Q: The managers of a firm financed entirely with common stock are evaluating

The managers of a firm financed entirely with common stock are evaluating two distinct projects. The first project has a large amount of unsystematic risk and a small amount of systematic risk. The se...

See Answer

Q: The Imaginary Products Co. currently has debt with a market value

The Imaginary Products Co. currently has debt with a market value of $300 million outstanding. The debt consists of 9 percent coupon bonds (semiannual coupon payments) which have a maturity of 15 year...

See Answer

Q: How does a scenario analysis differ from a sensitivity analysis?

How does a scenario analysis differ from a sensitivity analysis?

See Answer

Q: For the Imaginary Products firm in Problem 13.24, calculate

For the Imaginary Products firm in Problem 13.24, calculate the appropriate cost of capital for a new project that is financed with the same proportion of debt, preferred shares, and common shares as...

See Answer

Q: If a firm’s management anticipates financing a project with a capital mix

If a firm’s management anticipates financing a project with a capital mix that is different from its current capital structure, describe how the firm is subjecting itself to a calculation error if its...

See Answer

Q: You are analyzing the cost of capital for MacroSwift Corporation, which

You are analyzing the cost of capital for MacroSwift Corporation, which develops software operating systems for computers. The firm’s dividend growth rate has been a very constant 3 percent per year f...

See Answer

Q: You are an external financial analyst evaluating the merits of a stock

You are an external financial analyst evaluating the merits of a stock. Since you are using a dividend discount model approach to evaluate a cost of equity capital, you need to estimate the dividend g...

See Answer

Q: How are taxes accounted for when we calculate the cost of debt

How are taxes accounted for when we calculate the cost of debt?

See Answer

Q: In your analysis of the cost of capital for a common stock

In your analysis of the cost of capital for a common stock, you calculate a cost of capital using a dividend discount model that is much lower than the calculation for the cost of capital using the CA...

See Answer

Q: RetRyder Hand Trucks has a preferred share issue outstanding that pays a

RetRyder Hand Trucks has a preferred share issue outstanding that pays a dividend of $1.30 per year. The current cost of preferred equity for RetRyder is 9 percent. If RetRyder issues additional prefe...

See Answer

Q: Enigma Corporation’s management believes that the firm’s cost of capital (WACC

Enigma Corporation’s management believes that the firm’s cost of capital (WACC) is too high because the firm has been too secretive with the market concerning its operations. Evaluate that statement?...

See Answer

Q: Discuss what valuable information would be lost if you decided to use

Discuss what valuable information would be lost if you decided to use book values in order to calculate the cost of each of the capital components within a firm’s capital structure?

See Answer

Q: Hurricane Corporation is financed with debt, preferred equity, and common

Hurricane Corporation is financed with debt, preferred equity, and common equity with market values of $20 million, $10 million, and $30 million, respectively. The betas for the debt, preferred stock,...

See Answer

Q: Why does the market value of the claims on the assets of

Why does the market value of the claims on the assets of a firm equal the market value of the assets?

See Answer

Q: You are working as an intern at Coral Gables Products, a

You are working as an intern at Coral Gables Products, a privately owned manufacturing company. Shortly after you read Chapter 13 in this book, you got into a discussion with the Chief Financial Offic...

See Answer

Q: The CFO described in Problem 13.35 asks you to estimate

The CFO described in Problem 13.35 asks you to estimate the beta for Coral Gables’s common stock. Since the common stock is not publicly traded, you do not have the data necessary to estimate the beta...

See Answer

Q: Estimate the weighted average cost of capital for Coral Gables using your

Estimate the weighted average cost of capital for Coral Gables using your estimated beta and the information in the problem statement in Problem 13.36? Assume that the average and marginal tax rates f...

See Answer

Q: List and describe each of the three methods used to calculate the

List and describe each of the three methods used to calculate the cost of common stock?

See Answer

Q: Whitewall Tire Co. just paid an annual dividend of $1

Whitewall Tire Co. just paid an annual dividend of $1.60 on its common shares. If Whitewall is expected to increase its annual dividend by 2 percent per year into the foreseeable future and the curren...

See Answer

Q: Two-Stage Rocket paid an annual dividend of $1.

Two-Stage Rocket paid an annual dividend of $1.25 yesterday, and it is commonly known that the firm’s management expects to increase its dividend by 8 percent for the next two years and by 2 percent t...

See Answer

Q: Kresler Autos has preferred shares outstanding that pay annual dividends of $

Kresler Autos has preferred shares outstanding that pay annual dividends of $12, and the current price of the shares is $80. What is the after-tax cost of new preferred shares for Kresler if the flota...

See Answer

Q: What is a stock market index?

What is a stock market index?

See Answer

Q: Kingston, Inc. management is considering purchasing a new machine at

Kingston, Inc. management is considering purchasing a new machine at a cost of $4,133,250. They expect this equipment to produce cash flows of $814,322, $863,275, $937,250, $1,017,112, $1,212,960, and...

See Answer

Q: What does NASDAQ stand for? What is NASDAQ?

What does NASDAQ stand for? What is NASDAQ?

See Answer

Q: How is the WACC for a firm calculated?

How is the WACC for a firm calculated?

See Answer

Q: What is a dividend yield? What does it tell us?

What is a dividend yield? What does it tell us?

See Answer

Q: The preferred stock of Wellcare Inc. is currently trading at $

The preferred stock of Wellcare Inc. is currently trading at $137.50 per share. If the required rate of return is 8 percent and this stock has no maturity date, what is the quarterly dividend paid by...

See Answer

Q: You are chairperson of the investment committee at your firm. Five

You are chairperson of the investment committee at your firm. Five projects have been submitted to your committee for approval this month. The investment required and the project profitability index f...

See Answer

Q: What do we know about that project’s IRR if we know that

What do we know about that project’s IRR if we know that it has a positive NPV?

See Answer

Q: West Street Automotive is considering adding state safety inspections to their service

West Street Automotive is considering adding state safety inspections to their service offerings. The equipment necessary to perform these inspections will cost $557,000 and will generate cash flows o...

See Answer

Q: You purchased 100 shares of stocks of an oil company, Texas

You purchased 100 shares of stocks of an oil company, Texas Energy, Inc., at $50 per share. The company has 1 million shares outstanding. Ten days later, Texas Energy announced an investment in an oil...

See Answer

Q: A division of Virginia City Highlands Manufacturing is considering purchasing for 1

A division of Virginia City Highlands Manufacturing is considering purchasing for 1,500,000 a machine that automates the process of inserting electronic components onto computer motherboards. The annu...

See Answer

Q: After examining the NPV analysis for a potential project that would increase

After examining the NPV analysis for a potential project that would increase the firm’s output by 5 percent, an analyst’s manager tells the analyst to increase the initial fixed capital outlay in the...

See Answer

Q: Managers of Central Embroidery have decided to purchase a new monogram machine

Managers of Central Embroidery have decided to purchase a new monogram machine and are considering two alternative machines. The first machine costs $100,000 and is expected to last five years. The se...

See Answer

Q: You have inherited an apple orchard and want to sell it in

You have inherited an apple orchard and want to sell it in the next four years. An expert in apple orchard valuation has estimated the after-tax cash flow you would receive if you sold at the end of e...

See Answer

Q: Why do analysts care about the current cost of long-term

Why do analysts care about the current cost of long-term debt when estimating a firm’s cost of capital?

See Answer

Q: Retro Inc. sells vintage football jerseys for $72 each.

Retro Inc. sells vintage football jerseys for $72 each. Variable costs are $58 per unit and total fixed costs (including depreciation and amortization expense) are $84,000 per year. If sales for next...

See Answer

Q: How would a capital intensive company fare during good and poor economic

How would a capital intensive company fare during good and poor economic times as compared with other companies? Explain?

See Answer

Q: The manager of Roy’s Restaurant has determined that if revenues were to

The manager of Roy’s Restaurant has determined that if revenues were to increase by 20 percent, then EBIT would increase by 45 percent to $87,000. What would be the corresponding change in EBITDA if r...

See Answer

Q: Luminosity Inc. produces modern light fixtures that sell for $150

Luminosity Inc. produces modern light fixtures that sell for $150 per unit. The firm’s management is considering purchasing a high-capacity manufacturing machine. If the high-capacity machine is purch...

See Answer

Q: Paper Christmas Trees Inc. is considering introducing a new line of

Paper Christmas Trees Inc. is considering introducing a new line of inexpensive Christmas trees. The initial outlay for the project is $175,000, and the company will have to invest $5,000 in working c...

See Answer

Q: Howard Power and Telecommunications Corporation has three divisions. The names of

Howard Power and Telecommunications Corporation has three divisions. The names of these divisions, along with the after-tax cost of capital for each division and the market value of the assets in each...

See Answer

Q: Quarri Industries has eight percent coupon bonds outstanding. These bonds have

Quarri Industries has eight percent coupon bonds outstanding. These bonds have a market price of $954.41, pay interest semiannually, and will mature in 6 years. If the tax rate is 35 percent, what are...

See Answer

Q: Use the information in questions 13.2 and 13.3

Use the information in questions 13.2 and 13.3 as well as the following information to compute the WACC for Quarri Industries. In addition to common stock, Quarri has 500,000 preferred shares outstand...

See Answer

Q: Staunton Energy Corporation managers are considering a capital budgeting project to replace

Staunton Energy Corporation managers are considering a capital budgeting project to replace some machinery used in one of the company’s oil refineries. Is the company’s WACC the appropriate discount r...

See Answer

Q: Why does an ongoing stock repurchase program offer management greater flexibility in

Why does an ongoing stock repurchase program offer management greater flexibility in distributing value to stockholders than a regular cash dividend?

See Answer

Q: What does the price-earnings ratio tell us?

What does the price-earnings ratio tell us?

See Answer

Q: How do you estimate the cost of debt for a firm with

How do you estimate the cost of debt for a firm with more than one type of debt?

See Answer

Q: MasterCard, Inc. completed a 10-for-1 stock

MasterCard, Inc. completed a 10-for-1 stock split on January 22, 2014. Immediately before the stock split there were 120.38 million shares outstanding at a price of $826.00 per share. After the split...

See Answer

Q: You plan to start a business to produce and sell custom kitchen

You plan to start a business to produce and sell custom kitchen cabinets. The targeted price for each order of cabinets is $10,000. You estimate that you will receive orders for cabinets for eight kit...

See Answer

Q: Winters Inc. management estimates that the company will generate after-

Winters Inc. management estimates that the company will generate after-tax free cash flows from the firm (FCFF) of $12.5 million, $16.8 million and $19.7 million, respectively, over the next three yea...

See Answer

Q: Testco Corporation is considering adding a new product line. The cost

Testco Corporation is considering adding a new product line. The cost of the factory and equipment to produce this product is $1,780,000. Company management expects net cash flows from the sale of thi...

See Answer

Q: Flowers Unlimited is considering purchasing an additional delivery truck which will have

Flowers Unlimited is considering purchasing an additional delivery truck which will have a seven year useful life. The new truck will cost $42,000. Cost savings with this truck are expected to be $12,...

See Answer

Q: What is the average accounting rate of return (ARR) on

What is the average accounting rate of return (ARR) on a piece of equipment that will cost $1.2 million and that will result in pretax cost savings of $380,000 for the first three years and then $280,...

See Answer

Q: Abacus Corporation will pay dividends of $2.25, $

Abacus Corporation will pay dividends of $2.25, $2.95, and $3.15 in the next three years. After three years, the dividends are expected to grow at a constant rate of 4 percent per year. If the require...

See Answer

Q: What is NASDAQ?

What is NASDAQ?

See Answer

Q: What three different models are used to value stocks based on different

What three different models are used to value stocks based on different dividend patterns?

See Answer

Q: If a firm accepts a project with a $10,000

If a firm accepts a project with a $10,000 NPV, what is the effect on the value of the firm?

See Answer

Q: Under what circumstances can you use the constant-growth dividend formula

Under what circumstances can you use the constant-growth dividend formula to estimate kcs?

See Answer

Q: What is the payback period?

What is the payback period?

See Answer

Q: Why does the payback period provide a measure of a project’s liquidity

Why does the payback period provide a measure of a project’s liquidity risk?

See Answer

Q: What are the main shortcomings of the payback method?

What are the main shortcomings of the payback method?

See Answer

Q: Under what circumstances do the NPV and IRR decision rules always yield

Under what circumstances do the NPV and IRR decision rules always yield the same decision?

See Answer

Q: In capital budgeting, what is a conventional cash flow pattern?

In capital budgeting, what is a conventional cash flow pattern?

See Answer

Q: What decision criteria should managers use in selecting projects when a firm

What decision criteria should managers use in selecting projects when a firm faces capital constraints?

See Answer

Q: What types of investments should be included in FCF calculations?

What types of investments should be included in FCF calculations?

See Answer

Q: What are the five general rules for calculating FCF?

What are the five general rules for calculating FCF?

See Answer

Q: Why is it important to understand that cash flow forecasts in an

Why is it important to understand that cash flow forecasts in an NPV analysis are expected values?

See Answer

Q: When choosing between mutually exclusive projects of unequal lives, how can

When choosing between mutually exclusive projects of unequal lives, how can we ensure that the best decision is made?

See Answer

Q: Under what conditions is the WACC the appropriate discount rate for a

Under what conditions is the WACC the appropriate discount rate for a project?

See Answer

Q: Under what circumstance would you replace an old machine that is still

Under what circumstance would you replace an old machine that is still operating with a new one?

See Answer

Q: How is the proportion of fixed costs in a project’s cost structure

How is the proportion of fixed costs in a project’s cost structure related to the sensitivity of EBITDA and EBIT to changes in revenue?

See Answer

Q: How does operating leverage change when there is an increase in the

How does operating leverage change when there is an increase in the proportion of a project’s costs that are fixed?

See Answer

Q: How is a sensitivity analysis used in project analysis?

How is a sensitivity analysis used in project analysis?

See Answer

Q: What does the WACC for a firm tell us?

What does the WACC for a firm tell us?

See Answer

Q: How do taxes affect the cost of debt?

How do taxes affect the cost of debt?

See Answer

Q: What information is needed to use the CAPM to estimate kcs or

What information is needed to use the CAPM to estimate kcs or kps?

See Answer

Q: What is the advantage of using a multistage-growth dividend model

What is the advantage of using a multistage-growth dividend model, rather than the constant-growth dividend model, to estimate kcs?

See Answer

Q: Do analysts use book values or market values to calculate the weights

Do analysts use book values or market values to calculate the weights when they use Equation13.7? Why?

See Answer

Q: What kinds of errors can be made when the WACC for a

What kinds of errors can be made when the WACC for a firm is used as the discount rate for evaluating all projects in the firm?

See Answer

Q: List and define four types of dividends?

List and define four types of dividends?

See Answer

Q: What is a stock repurchase?

What is a stock repurchase?

See Answer

Q: How do stock prices react to dividend announcements?

How do stock prices react to dividend announcements?

See Answer

Q: How does a stock dividend differ from other types of dividends?

How does a stock dividend differ from other types of dividends?

See Answer

Q: How are dividend payouts affected by expected earnings?

How are dividend payouts affected by expected earnings?

See Answer

Q: What else can a business plan be used for?

What else can a business plan be used for?

See Answer

Q: Why is it important to discuss the qualifications of the management team

Why is it important to discuss the qualifications of the management team in a business plan?

See Answer

Q: The market value of a firm’s assets is $3 billion.

The market value of a firm’s assets is $3 billion. If the market value of the firm’s liabilities is $2 billion, what is the market value of the stockholders’ investment and why?

See Answer

Q: Berron Comics, Inc., has borrowed $100 million and is

Berron Comics, Inc., has borrowed $100 million and is required to pay its lenders $8 million in interest this year. If Berron is in the 35 percent marginal tax bracket, then what is the after-tax cost...

See Answer

Q: You are considering opening another restaurant in the TexasBurgers chain. The

You are considering opening another restaurant in the TexasBurgers chain. The new restaurant will have annual revenue of $300,000 and operating expenses of $150,000. The annual depreciation and amorti...

See Answer

Q: Sunglass Heaven, Inc., is launching a new store in a

Sunglass Heaven, Inc., is launching a new store in a shopping mall in Houston. The annual revenue of the store depends on the weather conditions in the summer in Houston. The annual revenue will be $2...

See Answer

Q: What are the key events and dates in the dividend payment process

What are the key events and dates in the dividend payment process?

See Answer

Q: Explain why the after-tax cost of equity (common or

Explain why the after-tax cost of equity (common or preferred) does not have to be adjusted by the marginal income tax rate for the firm?

See Answer

Q: Management of the Veil Acts Company just announced that instead of a

Management of the Veil Acts Company just announced that instead of a regular dividend this quarter, it will be repurchasing shares using the same amount of cash that would have been paid in the suspen...

See Answer

Q: Management of the Bernie Rubbel Company has just declared a three-

Management of the Bernie Rubbel Company has just declared a three-for-one stock split. If you own 12,000 shares before the split, how many shares do you own after the split? What if it were a one-for-...

See Answer

Q: Under what circumstances will the sale of an asset result in a

Under what circumstances will the sale of an asset result in a taxable gain? How do you estimate the taxes or tax benefits associated with the sale of an asset?

See Answer

Q: What are the differences between cash flows used in capital budgeting calculations

What are the differences between cash flows used in capital budgeting calculations and past accounting earnings?

See Answer

Q: The price of a share of stock is $15.00

The price of a share of stock is $15.00 on Tuesday, November 14, 2017. The record date for a $0.50 dividend is Friday, November 17, 2017. If there are no taxes on dividends, what would you expect the...

See Answer

Q: Provide two conditions under which a set of projects might be characterized

Provide two conditions under which a set of projects might be characterized as mutually exclusive?

See Answer

Q: Stock A and Stock B are both priced at $50 per

Stock A and Stock B are both priced at $50 per share. Stock A has a P/E ratio of 17, while Stock B has a P/E ratio of 24. Which is the more attractive investment, considering everything else to be the...

See Answer

Q: Suppose the cost of capital of the Gadget Company is 10 percent

Suppose the cost of capital of the Gadget Company is 10 percent. If Gadget has a capital structure that is 50 percent debt and 50 percent equity, its before-tax cost of debt is 5 percent, and its marg...

See Answer

Q: Morgan Insurance Ltd. issued a fixed-rate perpetual preferred stock

Morgan Insurance Ltd. issued a fixed-rate perpetual preferred stock three years ago and placed it privately with institutional investors. The stock was issued at $25.00 per share with a $1.75 dividend...

See Answer

Q: How does a dividend affect the size of a stockholder’s investment in

How does a dividend affect the size of a stockholder’s investment in a firm?

See Answer

Q: Seerex Wok Co. is expected to pay a dividend of $

Seerex Wok Co. is expected to pay a dividend of $1.10 one year from today on its common shares. That dividend is expected to increase by 5 percent every year thereafter. If the price of Seerex common...

See Answer

Q: Fjord Luxury Liners has preferred shares outstanding that pay an annual dividend

Fjord Luxury Liners has preferred shares outstanding that pay an annual dividend equal to $15 per year. If the current price of Fjord preferred shares is $107.14, what is the after-tax cost of preferr...

See Answer

Q: At the end of 2012, the maximum tax rate on dividends

At the end of 2012, the maximum tax rate on dividends increased from 15 percent to 23.8 percent. How would you expect this increase to affect the prices of dividend-paying stocks versus those of nondi...

See Answer

Q: You purchased 1,000 shares of Koogal stock five years ago

You purchased 1,000 shares of Koogal stock five years ago for $30 per share. Today Koogal is repurchasing your shares through a fixed-price tender offer for $80 per share. What are the after-tax proce...

See Answer

Q: An investment requires an outlay of $100 and produces after-

An investment requires an outlay of $100 and produces after-tax cash flows of $40 annually for four years. A project enhancement increases the required outlay by $15 and the annual after-tax cash flow...

See Answer

Q: Llama Wool Company management is doing some financial planning for the coming

Llama Wool Company management is doing some financial planning for the coming year. Llama plans to raise $10,000 in new equity this year and wants to pay a dividend to stockholders of $30,000. The fir...

See Answer

Q: Scintilla, Inc., stock is trading for $10.00

Scintilla, Inc., stock is trading for $10.00 per share on the day before the ex-dividend date. If the dividend is $0.25 and there are no taxes, what should the price of the shares be on the ex-dividen...

See Answer

Q: A company’s management announces a $1.00 per share dividend

A company’s management announces a $1.00 per share dividend payment. Assuming all investors are subject to a 15 percent tax rate on dividends, how much should the company’s share price drop on the ex-...

See Answer

Q: Compare the characteristics of an LLC with those of a partnership and

Compare the characteristics of an LLC with those of a partnership and a C-corporation.

See Answer

Q: What is the replacement cost of a business?

What is the replacement cost of a business?

See Answer

Q: How do stock repurchases differ from dividends?

How do stock repurchases differ from dividends?

See Answer

Q: Moriband Corp. paid a dividend of $2.15 yesterday

Moriband Corp. paid a dividend of $2.15 yesterday. The company’s dividend is expected to grow at a steady rate of 5 percent for the foreseeable future. If investors in stocks of companies like Moriban...

See Answer

Q: Nyeil, Inc., is a consumer products firm that is growing

Nyeil, Inc., is a consumer products firm that is growing at a constant rate of 6.5 percent. The firm’s last dividend was $3.36. If the required rate of return is 18 percent, what is the market value o...

See Answer

Q: Refer to problem 10.5. What are the payback periods

Refer to problem 10.5. What are the payback periods for Production Systems 1 and 2? If the systems are mutually exclusive and the firm always chooses projects with the lowest payback period, in whic...

See Answer

Q: Proxicam, Inc., is expected to grow at a constant rate

Proxicam, Inc., is expected to grow at a constant rate of 7 percent. If the company’s next dividend, which will be paid in a year, is $1.15 and its current stock price is $22.35, what is the required...

See Answer

Q: X-Centric Energy Company has issued perpetual preferred stock with a

X-Centric Energy Company has issued perpetual preferred stock with a stated (par) value of $100 and a dividend of 4.5 percent. If the required rate of return is 8.25 percent, what is the stock’s curre...

See Answer

Q: The First Bank of Flagstaff has issued perpetual preferred stock with a

The First Bank of Flagstaff has issued perpetual preferred stock with a $100 par value. The bank pays a quarterly dividend of $1.65 on this stock. What is the current price of this preferred stock giv...

See Answer

Q: The preferred stock of Axim Corp. is currently selling at $

The preferred stock of Axim Corp. is currently selling at $47.13. If the required rate of return is 12.2 percent, what is the dividend paid by this stock?

See Answer

Q: Each quarter, Sirkota, Inc., pays a dividend on its

Each quarter, Sirkota, Inc., pays a dividend on its perpetual preferred stock. Todaythe stock is selling at $63.37. If the required rate of return for such stocks is 15.5 percent, what is the quarterl...

See Answer

Q: Kay Williams is interested in purchasing the common stock of Reckers,

Kay Williams is interested in purchasing the common stock of Reckers, Inc., which is currently priced at $37.45. The company is expected to pay a dividend of $2.58 next year and to increase its divide...

See Answer

Q: The required rate of return is 23 percent. Ninex Corp.

The required rate of return is 23 percent. Ninex Corp. has just paid a dividend of$3.12 and is expected to increase its dividend at a constant rate of 5 percent. What is the expected price of the stoc...

See Answer

Q: In what ways can a company repurchase its stock?

In what ways can a company repurchase its stock?

See Answer

Q: What is the Dow Jones Industrial Average?

What is the Dow Jones Industrial Average?

See Answer

Q: Jenny Banks is interested in buying the stock of Fervan, Inc

Jenny Banks is interested in buying the stock of Fervan, Inc., which is increasing its dividends at a constant rate of 6 percent. Last year the firm paid a dividend of $2.65. The required rate of retu...

See Answer

Q: You own shares of Old World DVD Company and are interested in

You own shares of Old World DVD Company and are interested in selling them. With so many people downloading music these days, sales, profits, and dividends at Old World have been declining 6 percent p...

See Answer

Q: Six Twelve, Inc., is considering opening up a new convenience

Six Twelve, Inc., is considering opening up a new convenience store in downtown New York City. The expected annual revenue at the new store is $800,000. To estimate the increase in working capital, an...

See Answer

Q: Explain how we determine the optimal time to replace an existing asset

Explain how we determine the optimal time to replace an existing asset with a new one?

See Answer

Q: If you had to choose between one project with an expected life

If you had to choose between one project with an expected life of five years and a second project with an expected life of six years, how could you do this without using the equivalent annual cost con...

See Answer

Q: You are graduating in two years. You want to invest your

You are graduating in two years. You want to invest your current savings of $5,000 in bonds and use the proceeds to purchase a new car when you graduate and start to work. You can invest the money in...

See Answer

Q: Given the U.S. Corporate Tax Rate Schedule in Exhibit

Given the U.S. Corporate Tax Rate Schedule in Exhibit 11.6, what was the marginal tax rate and average tax rate of a corporation that had a taxable income of $12 million in 2016?

See Answer

Q: If a manufacturing firm and a service firm have identical cash fixed

If a manufacturing firm and a service firm have identical cash fixed costs, but the manufacturing firm has much higher depreciation and amortization, then which firm is more likely to have a large dis...

See Answer

Q: Explain how the value of accounting operating leverage can be used?

Explain how the value of accounting operating leverage can be used?

See Answer

Q: What are the benefits and costs associated with dividends?

What are the benefits and costs associated with dividends?

See Answer

Q: Mick’s Soft Lemonade is starting to develop a new product for which

Mick’s Soft Lemonade is starting to develop a new product for which the cash fixed costs are expected to be $80,000. The projected EBIT is $100,000, and the Accounting DOL is expected to be 2.0. What...

See Answer

Q: How do we calculate incremental after-tax free cash flows from

How do we calculate incremental after-tax free cash flows from forecasted earnings of a project? What are the common adjustment items?

See Answer

Q: Capital Co. has a capital structure, based on current market

Capital Co. has a capital structure, based on current market values, that consists of 50 percent debt, 10 percent preferred stock, and 40 percent common stock. If the returns required by investors are...

See Answer

Q: What are direct out-of-pocket costs?

What are direct out-of-pocket costs?

See Answer

Q: Courtesy Bancorp issued perpetual preferred stock a few years ago. The

Courtesy Bancorp issued perpetual preferred stock a few years ago. The bank pays an annual dividend of $4.27 and your required rate of return is 12.2 percent. a. What is the value of the stock given y...

See Answer

Q: Rhea Kirby owns shares in Ryoko Corp. Currently, the market

Rhea Kirby owns shares in Ryoko Corp. Currently, the market price of the stock is $36.34. Management expects dividends to grow at a constant rate of 6 percent for the foreseeable future. Its last divi...

See Answer

Q: In most circumstances, given the choice between a higher fixed cost

In most circumstances, given the choice between a higher fixed cost structure and a lower fixed cost structure, which of the two would generate a larger contribution margin?

See Answer

Q: Perpetual Ltd. has issued bonds that never require the principal amount

Perpetual Ltd. has issued bonds that never require the principal amount to be repaid to investors. Correspondingly, Perpetual must make interest payments into the infinite future. If the bondholders r...

See Answer

Q: You know that the return of Momentum Cyclicals common shares is 1

You know that the return of Momentum Cyclicals common shares is 1.6 times as sensitive to macroeconomic information as the return of the market. If the risk-free rate of return is 2.88 percent and the...

See Answer

Q: Operating leverage is a measure of the: a. Sensitivity

Operating leverage is a measure of the: a. Sensitivity of net earnings to changes in operating earnings. b. Sensitivity of net earnings to changes in sales. c. Sensitivity of fixed operating costs to...

See Answer

Q: Why might stock repurchases be preferred to dividends?

Why might stock repurchases be preferred to dividends?

See Answer

Q: The cost of equity is equal to the: a.

The cost of equity is equal to the: a. Expected market return. b. Rate of return required by stockholders. c. Cost of retained earnings plus dividends. d. Risk the company incurs when financing.

See Answer

Q: Dot.Com has determined that it could issue $1,

Dot.Com has determined that it could issue $1,000 face value bonds with an 8 percent coupon paid semiannually and a five-year maturity at $900 per bond. If Dot.Com’s marginal tax rate is 38 percent, i...

See Answer

Q: Marx Political Consultants has decided to discontinue all of its business operations

Marx Political Consultants has decided to discontinue all of its business operations. The firm has total debt of $7 million, and the liquidation value of its assets is $10 million. If the book value o...

See Answer

Q: When assembling the cash flows to calculate an NPV or IRR,

When assembling the cash flows to calculate an NPV or IRR, the project’s after-tax interest expenses should be subtracted from the cash flows for: a. The NPV calculation, but not the IRR calculation....

See Answer

Q: The Gearing Company has an after-tax cost of debt capital

The Gearing Company has an after-tax cost of debt capital of 4 percent, a cost of preferred stock of 8 percent, a cost of equity capital of 10 percent, and a weighted average cost of capital of 7 perc...

See Answer

Q: The law firm of Dewey, Cheatem, and Howe has monthly

The law firm of Dewey, Cheatem, and Howe has monthly fixed costs of $100,000, EBIT of $250,000, and depreciation charges on its office furniture and computers of $5,000. Calculate the Cash Flow DOL fo...

See Answer

Q: Nynet, Inc., paid a dividend of $4.18

Nynet, Inc., paid a dividend of $4.18 last year. The company’s management does not expect to increase its dividend in the foreseeable future. If the required rate of return is 18.5 percent, what is th...

See Answer

Q: Knight Supply Corp. has not grown for the past several years

Knight Supply Corp. has not grown for the past several years and management expects this lack of growth to continue. The firm last paid a dividend of $3.56. If you require a rate of return of 13 perce...

See Answer

Q: Ron Santana is interested in buying the stock of First National Bank

Ron Santana is interested in buying the stock of First National Bank. While the bank’s management expects no growth in the near future, Ron is attracted by the dividend income. Last year the bank paid...

See Answer

Q: The current stock price of Largent, Inc., is $44

The current stock price of Largent, Inc., is $44.72. If the required rate of return is 19 percent, what is the dividend paid by this firm if the dividend is not expected to grow in the future?

See Answer

Q: Why do some people view preferred stock as a special type of

Why do some people view preferred stock as a special type of a bond rather than a stock?

See Answer

Q: How does a stock dividend differ from a stock split?

How does a stock dividend differ from a stock split?

See Answer

Q: Which type of secondary market provides the most efficient market for financial

Which type of secondary market provides the most efficient market for financial securities?

See Answer

Q: Which of the following is/are usually included in an entrepreneur’s

Which of the following is/are usually included in an entrepreneur’s business plan? a. Detailed description of the company’s products and services. b. Discussion of the management team, including organ...

See Answer

Q: Sessler Corporation is a private company that had EBIT of $186

Sessler Corporation is a private company that had EBIT of $186 million and depreciation and amortization of $22 million in the most recent fiscal year. At the end of that year, a similar, public firm...

See Answer

Q: Do private companies have audited financial statements prepared in accordance with GAAP

Do private companies have audited financial statements prepared in accordance with GAAP?

See Answer

Q: Is preferred stock a debt or an equity security?

Is preferred stock a debt or an equity security?

See Answer

Q: Burnes, Inc. is a mature firm that is growing at

Burnes, Inc. is a mature firm that is growing at a constant rate of 5.5 percent per year. The last dividend that the firm paid was $1.50 per share. If dividends are expected to grow at the same rate a...

See Answer

Q: Which of the following are relevant cash flows in the evaluation of

Which of the following are relevant cash flows in the evaluation of a proposal to produce a new product? a. Decrease in the cash flows of a substitute product. b. Alternative of leasing an existing bu...

See Answer

Q: Quarri Industries has a beta of 1.6. If the

Quarri Industries has a beta of 1.6. If the market risk free rate is 4 percent and the expected return on the market 9 percent, what is Babbitt’s cost of common stock?

See Answer

Q: Miron’s Copper Corp. management expects its common stock dividends to grow

Miron’s Copper Corp. management expects its common stock dividends to grow 1.5 percent per year for the indefinite future. The firm’s shares are currently selling for $18.45, and the firm just paid a...

See Answer

Q: Shares of Convoy West, Inc. are trading for $55

Shares of Convoy West, Inc. are trading for $55.45 on the day before the ex-dividend date. If the quarterly dividend is $0.16 per share and there are no taxes, how will the share price change on the e...

See Answer

Q: What did the 2005 study conclude about how managers view stock repurchases

What did the 2005 study conclude about how managers view stock repurchases?

See Answer

Q: Three years ago, you purchased 4,000 shares of Metwa

Three years ago, you purchased 4,000 shares of Metwa Inc. for $17 per share. Today Metwa is repurchasing its shares through a fixed-price tender offer at a price of $45 per share. What are the after-t...

See Answer

Q: Why would management of a company undertake a reverse stock split?

Why would management of a company undertake a reverse stock split?

See Answer

Q: List three practical considerations that managers should take into account when setting

List three practical considerations that managers should take into account when setting a dividend policy?

See Answer

Q: What are three general reasons that new businesses fail?

What are three general reasons that new businesses fail?

See Answer

Q: How do financing considerations affect the choice of organizational form?

How do financing considerations affect the choice of organizational form?

See Answer

Q: How does a cash budget help an entrepreneur?

How does a cash budget help an entrepreneur?

See Answer

Q: Why is a business plan important in raising capital for a young

Why is a business plan important in raising capital for a young company?

See Answer

Q: What are the two market approaches that can be used to value

What are the two market approaches that can be used to value a business, and how do they differ?

See Answer

Q: What is a nonoperating asset, and how are such assets accounted

What is a nonoperating asset, and how are such assets accounted for in business valuation?

See Answer

Q: What are three income approaches used to value a business?

What are three income approaches used to value a business?

See Answer

Q: What are growth stocks, and why do they typically pay little

What are growth stocks, and why do they typically pay little or no dividends?

See Answer

Q: What is the difference between FCFE and dividends?

What is the difference between FCFE and dividends?

See Answer

Q: Why is it important to specify a valuation date when you value

Why is it important to specify a valuation date when you value a business?

See Answer

Q: What is the difference between investment value and fair market value?

What is the difference between investment value and fair market value?

See Answer

Q: Why is marketability an important issue in business valuation?

Why is marketability an important issue in business valuation?

See Answer

Q: What is a key person?

What is a key person?

See Answer

Q: How might financial statements for private companies differ from those for public

How might financial statements for private companies differ from those for public companies?

See Answer

Q: Merriweather Manufacturing Company has been growing at a rate of 6 percent

Merriweather Manufacturing Company has been growing at a rate of 6 percent for the past two years, and the CEO expects the company to continue to grow at this rate for the next several years. The comp...

See Answer

Q: Clarion Corp. has been selling electrical supplies for the past 20

Clarion Corp. has been selling electrical supplies for the past 20 years. The company’s product line has changed very little in the past five years, and the company’s management does not expect to add...

See Answer

Q: Barrymore Infotech is a fast-growing communications company. The company

Barrymore Infotech is a fast-growing communications company. The company did not pay a dividend last year and is not expected to do so for the next two years. Last year the company’s growth accelerate...

See Answer

Q: You are interested in buying the preferred stock of a bank that

You are interested in buying the preferred stock of a bank that pays a dividend of $1.80 every quarter. If you discount such cash flows at 8 percent, what is the value of this stock?

See Answer

Q: Explain why the growth rate g must always be less than the

Explain why the growth rate g must always be less than the rate of return R for the constant-growth rate model?

See Answer

Q: The Management of Premium Manufacturing Company is evaluating two forklift systems to

The Management of Premium Manufacturing Company is evaluating two forklift systems to use in its plant that produces the towers for a windmill power farm. The costs and the cash flows from these syste...

See Answer

Q: Perryman Crafts Corp. management is evaluating two independent capital projects that

Perryman Crafts Corp. management is evaluating two independent capital projects that will each cost the company $250,000. The two projects will provide the following cash flows: Which project will be...

See Answer

Q: Terrell Corp. management is considering purchasing a machine that will cost

Terrell Corp. management is considering purchasing a machine that will cost $117,250 and will be depreciated on a straight-line basis over a five-year period. The sales and expenses (excluding depreci...

See Answer

Q: Refer to Problem 10.1. Compute the IRR for each

Refer to Problem 10.1. Compute the IRR for each of the two systems. Is the investment decision different from the one determined by NPV? Refer to Problem 10.1: The Management of Premium Manufacturing...

See Answer

Q: You are considering a project that has an initial outlay of $

You are considering a project that has an initial outlay of $1 million. The profitability index of the project is 2.24. What is the NPV of the project?

See Answer

Q: Explain why the announcement of a new investment is usually accompanied by

Explain why the announcement of a new investment is usually accompanied by a change in the firm’s stock price?

See Answer

Q: In calculating the NPV of a project, should we use all

In calculating the NPV of a project, should we use all of the after-tax cash flows associated with the project, or incremental after-tax cash flows from the project? Why?

See Answer

Q: Sprigg Lane Manufacturing, Inc., needs to purchase a new central

Sprigg Lane Manufacturing, Inc., needs to purchase a new central air-conditioning system for a plant. There are two choices. The first system costs $50,000 and is expected to last 10 years, and the se...

See Answer

Q: The Yellow Shelf Company sells all of its shelves for $100

The Yellow Shelf Company sells all of its shelves for $100 per shelf, and incurs $50 in variable costs to produce each. If the fixed costs for the firm are $2,000,000 per year, what will the EBIT for...

See Answer

Q: Hydrogen Batteries sells its specialty automobile batteries for $85 each,

Hydrogen Batteries sells its specialty automobile batteries for $85 each, while its current variable cost per unit is $65. Total fixed costs (including depreciation and amortization expense) are $150,...

See Answer

Q: Why can skipping payment of a preferred dividend be a bad signal

Why can skipping payment of a preferred dividend be a bad signal?

See Answer

Q: The Vinyl CD Co. is going to take on a project

The Vinyl CD Co. is going to take on a project that is expected to increase its EBIT by $90,000, its fixed cost cash expenditures by $100,000, and its depreciation and amortization by $80,000 next yea...

See Answer

Q: You are considering investing in a business that has monthly fixed costs

You are considering investing in a business that has monthly fixed costs of $5,500 and sells a single product that costs $35 per unit to make. This product sells for $90 per unit. What is the annual p...

See Answer

Q: Belt Bottoms, Inc. is considering a five-year project

Belt Bottoms, Inc. is considering a five-year project with an initial investment of $20,000. What annual free cash flow (FCF) would be required for this project to have an NPV of $0 if the opportunit...

See Answer

Q: Mike’s T-Shirts, Inc., has debt claims of $

Mike’s T-Shirts, Inc., has debt claims of $400 (market value) and equity claims of $600 (market value). If the after-tax cost of debt financing is 11 percent and the cost of equity is 17 percent, then...

See Answer

Q: You are analyzing a firm that is financed with 60 percent debt

You are analyzing a firm that is financed with 60 percent debt and 40 percent equity. The current cost of debt financing is 10 percent, but due to a recent downgrade by the rating agencies, the firm’s...

See Answer

Q: You would like to own a common stock that has a record

You would like to own a common stock that has a record date of Friday, September 8, 2017. What is the last date that you can purchase the stock and still receive the dividend?

See Answer

Q: You believe that the average investor is subject to a 20 percent

You believe that the average investor is subject to a 20 percent tax rate on dividend payments. If a firm is going to pay a $0.30 dividend, by what amount would you expect the stock price to drop on t...

See Answer

Q: Two publicly traded companies in the same industry are similar in all

Two publicly traded companies in the same industry are similar in all respects except one. Whereas Publicks has issued debt in the public markets, Privicks has never borrowed from any public source. I...

See Answer

Q: Your sister wants to open a store that sells antique-style

Your sister wants to open a store that sells antique-style jewelry and accessories. She has $15,000 of savings to invest, but opening the store will require an initial investment of $20,000. Net cash...

See Answer

Q: You have the following information for a company you are valuing and

You have the following information for a company you are valuing and for a comparable company: Comparable company: ………….……………………………… Company you are valuing: Stock price = $23.45 ………………… …………………………………...

See Answer

Q: How is a preferred stock with a fixed maturity valued?

How is a preferred stock with a fixed maturity valued?

See Answer

Q: How do the cash flows that are discounted when the WACC approach

How do the cash flows that are discounted when the WACC approach (FCFF approach) is used to value a business differ from those that are discounted when the free cash flow to equity (FCFE) approach is...

See Answer

Q: You are valuing a company using the WACC approach and have estimated

You are valuing a company using the WACC approach and have estimated that the free cash flows from the firm (FCFF) in the next five years will be $36.7, $42.6, $45.1, $46.3, and $46.6 million, respect...

See Answer

Q: You want to estimate the value of a local advertising firm.

You want to estimate the value of a local advertising firm. The earnings of the firm are expected to be $2 million next year. Based on expected earnings next year, the average price-to-earnings ratio...

See Answer

Q: Ted McKay has just bought the common stock of Ryland Corp.

Ted McKay has just bought the common stock of Ryland Corp. The company expects to grow at the following rates for the next three years: 30 percent, 25 percent, and 15 percent. Last year the company pa...

See Answer

Q: Explain why preferred stock is considered to be a hybrid of equity

Explain why preferred stock is considered to be a hybrid of equity and debt securities?

See Answer

Q: Why is stock valuation more difficult than bond valuation?

Why is stock valuation more difficult than bond valuation?

See Answer

Q: You are currently thinking about investing in a stock valued at $

You are currently thinking about investing in a stock valued at $25.00 per share. The stock recently paid a dividend of $2.25 and its dividend is expected to grow at a rate of 5 percent for the forese...

See Answer

Q: The modified IRR (MIRR) alleviates two concerns with using the

The modified IRR (MIRR) alleviates two concerns with using the IRR method for evaluating capital investments. What are they?

See Answer

Q: Facebook does not pay dividends. How can it have a positive

Facebook does not pay dividends. How can it have a positive stock price?

See Answer

Q: High Tech Monopoly Co. has plenty of cash to fund any

High Tech Monopoly Co. has plenty of cash to fund any conceivable positive NPV project. Can you describe a situation in which capital rationing could still occur?

See Answer

Q: What are some benefits of using debt financing?

What are some benefits of using debt financing?

See Answer

Q: What is a lease? What are the two types of leases

What is a lease? What are the two types of leases?

See Answer

Q: A bond covenant is a part of a bond contract that restricts

A bond covenant is a part of a bond contract that restricts the behavior of the firm, barring it from taking certain actions. Using the terminology of options, explain why a bond contract might includ...

See Answer

Q: An American option will never be worth less than a European option

An American option will never be worth less than a European option. Evaluate this statement?

See Answer

Q: Explain why, in the binomial pricing theory, the probabilities of

Explain why, in the binomial pricing theory, the probabilities of an upward move versus a downward move are not important?

See Answer

Q: Future Enterprises is considering building a factory that will include an option

Future Enterprises is considering building a factory that will include an option to expand operations in three years. If things go well, the anticipated expansion will have a value of $10 million and...

See Answer

Q: Corporations frequently include employee stock options as a part of the compensation

Corporations frequently include employee stock options as a part of the compensation for their managers and sometimes for all of their employees. These options allow the holder to buy the stock of the...

See Answer

Q: You own ABC Corp. bonds. Using option pricing theory,

You own ABC Corp. bonds. Using option pricing theory, explain what agency concerns you would have if ABC were in danger of bankruptcy?

See Answer

Q: Royal Dutch Shell, an oil company, has headquarters in both

Royal Dutch Shell, an oil company, has headquarters in both the Netherlands and the United Kingdom. What type of firm is it?

See Answer

Q: Give examples of U.S. banks facing different risks in

Give examples of U.S. banks facing different risks in international lending?

See Answer

Q: International economic integration and technological changes in the last couple of decades

International economic integration and technological changes in the last couple of decades have dramatically increased globalization across many industries. Explain how a biotech firm or a medical fir...

See Answer

Q: In the United States, managers are asked to focus on maximizing

In the United States, managers are asked to focus on maximizing stockholder value. Is this consistent with the goals of managers in Germany and Japan?

See Answer

Q: What types of conflicts arise with leases and why?

What types of conflicts arise with leases and why?

See Answer

Q: A Canadian cooperative of wheat farmers sold wheat to a grain company

A Canadian cooperative of wheat farmers sold wheat to a grain company in Russia. Under what circumstances will the Canadian farmers be exposed to foreign exchange risk? When will the Russian importer...

See Answer

Q: Stardust, Inc., is an exporter of plumbing fixtures. About

Stardust, Inc., is an exporter of plumbing fixtures. About 30 percent of its sales are made in Canada. The sales department just found out that the Canadian dollar is at a premium against the U.S. dol...

See Answer

Q: What factors must a financial manager consider when making decisions about accounts

What factors must a financial manager consider when making decisions about accounts receivable?

See Answer

Q: List some of the working capital management practices you would expect to

List some of the working capital management practices you would expect to see in a manufacturing company following just-in-time inventory practices?

See Answer

Q: What costs would a firm following a flexible current asset management strategy

What costs would a firm following a flexible current asset management strategy consider, and why?

See Answer

Q: SpinTheWheel Co. has assets currently worth $10 million in the

SpinTheWheel Co. has assets currently worth $10 million in the form of one-year risk-free bonds that will return 10 percent. The company has debt with a face value of $5.5 million due in one year. (No...

See Answer

Q: What is the payoff for a call option with a strike price

What is the payoff for a call option with a strike price of $50 if the stock price at expiration is $40? What if the stock price is $65?

See Answer

Q: The price of a stock that does not pay dividends is currently

The price of a stock that does not pay dividends is currently $35, and the risk-free rate is 4 percent. A European call option on the stock, with a strike price of $35 and which expires in six months,...

See Answer

Q: Two call options have been written on the same underlying stock.

Two call options have been written on the same underlying stock. Call #1 has a strike price of $42, and call #2 has a strike price of $52. Call #1 is selling for $5.00, and call #2 is selling for $6.0...

See Answer

Q: Husky Motors has two debt issues outstanding, both of which mature

Husky Motors has two debt issues outstanding, both of which mature in five years. The senior debt issue, which has a face value of $10 million, must be paid in full before any of the principal for the...

See Answer

Q: What are some of the trade-offs required in the management

What are some of the trade-offs required in the management of working capital accounts?

See Answer

Q: What does the seller of a put option hope will happen?

What does the seller of a put option hope will happen?

See Answer

Q: What is the value of a call option if the stock price

What is the value of a call option if the stock price is zero? What if the stock price is extremely high (relative to the strike price)?

See Answer

Q: Like owners of stock, owners of options can lose no more

Like owners of stock, owners of options can lose no more than the amount they invested. They are far more likely to lose that full amount, but they cannot lose more. Do sellers of options have the sam...

See Answer

Q: Use the data in Exhibit 21.5 to answer these questions

Use the data in Exhibit 21.5 to answer these questions: a. What is the six-month forward rate (in U.S. dollars) for Swiss francs? Is the Swiss franc selling for a premium or a discount? b. What is th...

See Answer

Q: Ford Motor Company maintains production facilities in many different countries including Brazil

Ford Motor Company maintains production facilities in many different countries including Brazil, Taiwan, and the United States. Given the data in Exhibit 21.6, which production plant is likely to face...

See Answer

Q: How is transaction exposure different from operating exposure?

How is transaction exposure different from operating exposure?

See Answer

Q: The foreign exchange department at Tokyo’s Daiwa Bank quoted the spot rate

The foreign exchange department at Tokyo’s Daiwa Bank quoted the spot rate on the euro at €0.007269/¥. The 90-day forward rate is quoted at a premium of 5.42 percent on the euro. What is the 90-day fo...

See Answer

Q: The spot rate of the Australian dollar (A$) is A

The spot rate of the Australian dollar (A$) is A$1.1667/$. The Australian dollar is quoted at a 30-day forward premium of 4.90 percent against the U.S. dollar. What is the 30-day forward quote?

See Answer

Q: The foreign exchange department of Bank of America has a bid quote

The foreign exchange department of Bank of America has a bid quote on Canadian dollars (C$) of C$1.0800/$. If the bank typically tries to make a bid-ask spread of 0.5 percent on these foreign exchan...

See Answer

Q: Banco Santiago wants to make a bid-ask spread of 0

Banco Santiago wants to make a bid-ask spread of 0.65 percent on its foreign exchange transactions. If the ask rate on the Mexican peso (MP) is MP18.3092/$, what does the bid rate have to be?

See Answer

Q: What is the operating cycle, and how is it related to

What is the operating cycle, and how is it related to the cash conversion cycle?

See Answer

Q: Alcor Pharma just received revenues of $3,165,300

Alcor Pharma just received revenues of $3,165,300 in Australian dollars (A$). Management has the following exchange rates: A$1.08010/£ and $1.5906/£. What is the U.S. dollar value of the company’s rev...

See Answer

Q: Tricolor Industries has purchased equipment from a Brazilian firm for a total

Tricolor Industries has purchased equipment from a Brazilian firm for a total cost of 272,500 Brazilian reals. The firm has to pay in 30 days. Citibank has given the firm a 30-day forward quote of $0....

See Answer

Q: Covington Industries just sold equipment to a Mexican firm. Payment of

Covington Industries just sold equipment to a Mexican firm. Payment of 11,315,000 pesos will be due to Covington in 30 days. Covington has the option of selling the pesos today at a 30-day forward rat...

See Answer

Q: Barrington Fertilizers, Inc., exports its specialized lawn care products to

Barrington Fertilizers, Inc., exports its specialized lawn care products to Canada. It made a sale worth C$1,150,000, with the payment due in 90 days. Barrington’s banker gave it a forward quote of $0...

See Answer

Q: Moon Rhee Auto Supply, a Korean supplier of parts to Kia

Moon Rhee Auto Supply, a Korean supplier of parts to Kia Motors, is evaluating an opportunity to set up a plant in Alabama, where Kia Motors has an auto assembly plant for its SUVs. The cost of this p...

See Answer

Q: The Boeing Company has two different debt issues, both maturing four

The Boeing Company has two different debt issues, both maturing four years from now. The domestic bond issue pays semiannual coupons and has a coupon rate of 4.80 percent. The current price on the bon...

See Answer

Q: Caterpillar, Inc. management is trying to decide between selling a

Caterpillar, Inc. management is trying to decide between selling a new bond issue in the U.S. or the Eurodollar bond market. In either market the bonds will be denominated in dollars and will have a t...

See Answer

Q: IBM’s German unit is looking to borrow €7.5 million

IBM’s German unit is looking to borrow €7.5 million from Deutsche Bank. Deutsche Bank quotes a rate of three-month LIBOR plus 0.25 percent for the 90-day loan. Currently, the three-month LIBOR is 3.87...

See Answer

Q: Toyota is interested in borrowing $5 million for 90 days.

Toyota is interested in borrowing $5 million for 90 days. Bank of America has quoted a rate that is 1.125 percent under the prime rate of 6.25 percent. Daiwa Bank is offering Toyota a rate that is 0.7...

See Answer

Q: Explain the relation between each pair of currencies. /

Explain the relation between each pair of currencies.

See Answer

Q: What are the two general current asset management strategies discussed in this

What are the two general current asset management strategies discussed in this section, and how do they differ?

See Answer

Q: If the spot rate was $1.0413/C$

If the spot rate was $1.0413/C$ and the 90-day forward rate was $1.0507/C$, how much more (in U.S. dollars) would you receive by selling C$ 1,000,000 at the forward rate than at the spot rate?

See Answer

Q: Cerberus Security Company produces a cash flow of $200 per year

Cerberus Security Company produces a cash flow of $200 per year and is expected to continue doing so in the infinite future. The cost of equity capital for Cerberus is 20 percent, and the firm is fina...

See Answer

Q: Based on your knowledge from this and previous chapters, what are

Based on your knowledge from this and previous chapters, what are some methods an investment banker uses to determine an IPO price? What factors will play a significant role in the calculation?

See Answer

Q: A majority of firms choose a firm-commitment underwriting arrangement rather

A majority of firms choose a firm-commitment underwriting arrangement rather than a best-effort arrangement for their IPO? Explain why.

See Answer

Q: Describe the risks that are associated with a restrictive current asset management

Describe the risks that are associated with a restrictive current asset management strategy?

See Answer

Q: Sybex Corp. sells its goods with terms of 2/10

Sybex Corp. sells its goods with terms of 2/10 EOM, net 30. What is the implicit cost of the trade credit?

See Answer

Q: Mill Street Corporation sells its goods with terms of 4/10

Mill Street Corporation sells its goods with terms of 4/10 EOM, net 60. What is the implicit cost of the trade credit?

See Answer

Q: Why might a negotiated sale be the lowest cost means of issuing

Why might a negotiated sale be the lowest cost means of issuing a complex debt security?

See Answer

Q: Pacific Traders has annual sales of $1,895,000

Pacific Traders has annual sales of $1,895,000. The firm’s financial manager has determined that using a lockbox will reduce collection time by 2.3 days. If the firm’s opportunity cost on savings is 5...

See Answer

Q: The Kellogg Bank requires borrowers to keep an 8 percent compensating balance

The Kellogg Bank requires borrowers to keep an 8 percent compensating balance. Gorman Jewels borrows $340,000 at a 7 percent stated APR. What is the effective interest rate on the loan?

See Answer

Q: What does “4/15, net 30” mean?

What does “4/15, net 30” mean?

See Answer

Q: Morgan Contractors borrowed $1.75 million at an APR of

Morgan Contractors borrowed $1.75 million at an APR of 10.2 percent. The loan called for a compensating balance of 12 percent. What is the effective interest rate on the loan?

See Answer

Q: Maltz Landscaping has an average collection period of 38 days for its

Maltz Landscaping has an average collection period of 38 days for its accounts receivable. Currently, Maltz factors all of its receivables at a 2 percent discount. What is the effective annual interes...

See Answer

Q: Winegartner Cosmetics management is setting up a line of credit at the

Winegartner Cosmetics management is setting up a line of credit at the company’s bank for $5 million for up to two years. The interest rate is 5.875 percent and the loan agreement calls for an annual...

See Answer

Q: Your boss asks you to compute your company’s cash conversion cycle.

Your boss asks you to compute your company’s cash conversion cycle. Looking at the financial statements, you see that the average inventory for the year was $26,300, accounts receivable averaged $17,9...

See Answer

Q: Blackwell Automotive, Inc., reported the following financial information for the

Blackwell Automotive, Inc., reported the following financial information for the last fiscal year. Calculate the firm’s cash conversion cycle and operating cycle?

See Answer

Q: Elsee, Inc., has net sales of $13 million,

Elsee, Inc., has net sales of $13 million, and 75 percent of these are credit sales. Its cost of goods sold is 65 percent of annual net sales. The firm’s cash conversion cycle is 41.3 days. The invent...

See Answer

Q: Joanna Handicrafts, Inc., has net sales of $4.

Joanna Handicrafts, Inc., has net sales of $4.23 million with 50 percent being credit sales. Its cost of goods sold is $2.54 million. The firm’s cash conversion cycle is 47.9 days and its operating cy...

See Answer

Q: Aviva Technology’s operating cycle is 81 days. Its inventory was $

Aviva Technology’s operating cycle is 81 days. Its inventory was $134,000 at the end of last year, and the company had cost of goods sold of $1.1 million. How long does it take Aviva to collect its re...

See Answer

Q: Premier Corp. has net sales of $812,344,

Premier Corp. has net sales of $812,344, and cost of goods sold equal to 70 percent of net sales. Assume all sales are credit sales. If the firm’s accounts receivable total $113,902 and its operating...

See Answer

Q: Telecraft Enterprises carries 45 days of inventory in its stores. Last

Telecraft Enterprises carries 45 days of inventory in its stores. Last year Telecraft reported net sales of $1,400,000 and the company had receivables of $325,000 at the end of the year. What is the...

See Answer

Q: Why can cash investment in inventory be costly?

Why can cash investment in inventory be costly?

See Answer

Q: Given the data for Telecraft Enterprises in problem 14.21,

Given the data for Telecraft Enterprises in problem 14.21, re-estimate the firm’s operating cycle if days’ sales outstanding decreased to 75 days. For the same level of net sales, what is the implied...

See Answer

Q: Suppose a biotech company in Boston, Massachusetts, completes an $

Suppose a biotech company in Boston, Massachusetts, completes an $85 million IPO priced to the public at $75 per share. The firm receives $72 per share, and the out-of-pocket expenses are $340,000. Th...

See Answer

Q: Longhorn Traders is one of the largest RV dealers in Austin,

Longhorn Traders is one of the largest RV dealers in Austin, Texas, and sells about 2,800 recreational vehicles a year. The cost of placing an order with Longhorn’s supplier is $800, and the inventory...

See Answer

Q: The Clarkson Designer Company Management wants to borrow $750,000

The Clarkson Designer Company Management wants to borrow $750,000.The bank will provide the loan at an APR of 6.875 percent. Since the loan calls for a compensating balance, the effective interest rat...

See Answer

Q: The Colonial Window Treatments Company is borrowing $1.5 million

The Colonial Window Treatments Company is borrowing $1.5 million. The loan requires a 10 percent compensating balance, and the effective interest rate on the loan is 9.75 percent. What is the stated A...

See Answer

Q: Gruppa, Inc., has just set up a formal line of

Gruppa, Inc., has just set up a formal line of credit of $10 million with First Community Commercial Bank. The line of credit is good for up to five years. The bank will charge Gruppa an interest rate...

See Answer

Q: Lansdowne Electronics has a formal line of credit of $1 million

Lansdowne Electronics has a formal line of credit of $1 million for up to three years with HND Bank. The interest rate on the loan is 5.3 percent, and under the agreement, Lansdowne has to pay an annu...

See Answer

Q: Jennifer Electrical is evaluating whether a lockbox it is currently using is

Jennifer Electrical is evaluating whether a lockbox it is currently using is worth keeping. Management estimates that the lockbox reduces the mail float by 1.8 days and the processing by half a day. T...

See Answer

Q: Hazel Corp. has just signed up for a lockbox. Management

Hazel Corp. has just signed up for a lockbox. Management expects the lockbox to reduce the mail float by 2.1 days. Hazel Corp.’s remittances average $37,000 a day, and the average check is $125. The b...

See Answer

Q: Ginseng Company collects 50 percent of its receivables in 10 days or

Ginseng Company collects 50 percent of its receivables in 10 days or fewer, 31 percent in 11 to 30 days, 7 percent in 31 to 45 days, 7 percent in 46 to 60 days, and 5 percent in more than 60 days. The...

See Answer

Q: Explain how lockboxes are used?

Explain how lockboxes are used?

See Answer

Q: A partial aging of accounts receivable for Lincoln Cleaning Services is given

A partial aging of accounts receivable for Lincoln Cleaning Services is given in the accompanying table. What percent of receivables are in the 45-day range? Determine the firm’s eff...

See Answer

Q: Keswick Fencing Company collects 45 percent of its receivables in 10 days

Keswick Fencing Company collects 45 percent of its receivables in 10 days or fewer, 34 percent in 11 to 30 days, 12 percent in 31 to 45 days, 5 percent in 46 to 60 days, and 4 percent in more than 60...

See Answer

Q: Zenex, Inc., sells $250,000 of its accounts

Zenex, Inc., sells $250,000 of its accounts receivable to factors at a 3 percent discount. The firm’s average collection period is 90 days. What is the dollar cost of the factoring service? What is th...

See Answer

Q: Explain the difference between a competitive and negotiated cash sale. Which

Explain the difference between a competitive and negotiated cash sale. Which method of sale is likely to yield the lowest funding cost for firms selling plain vanilla bonds in stable markets?

See Answer

Q: What impact would the following actions have on the operating and cash

What impact would the following actions have on the operating and cash conversion cycles? Would the cycles increase, decrease, or remain unchanged? a. More raw material than usual is purchased. b. The...

See Answer

Q: What impact would the following actions have on the operating and cash

What impact would the following actions have on the operating and cash conversion cycles? Would the cycles increase, decrease, or remain unchanged? a. Less raw material than usual is purchased. b. The...

See Answer

Q: Morgan Sports Company just reported the following financial information. /

Morgan Sports Company just reported the following financial information. a. Calculate the firm’s days’ sales outstanding. b. What is the firm’s...

See Answer

Q: Jackson Electrical, one of the largest generator dealers in Phoenix,

Jackson Electrical, one of the largest generator dealers in Phoenix, sells about 2,000 generators a year. The cost of placing an order with its supplier is $750, and the inventory carrying costs are $...

See Answer

Q: Tanzaniqe, Inc., sells $200,000 of its accounts

Tanzaniqe, Inc., sells $200,000 of its accounts receivable to factors at a 5 percent discount. The firm’s average collection period is 90 days. a. What is the dollar cost of the factoring service? b....

See Answer

Q: Explain how term to maturity affects the price of a bank loan

Explain how term to maturity affects the price of a bank loan?

See Answer

Q: List and briefly describe the three main strategies a firm may use

List and briefly describe the three main strategies a firm may use to finance its working capital and fixed assets.

See Answer

Q: Suppose a company uses trade credit with the terms of 2/

Suppose a company uses trade credit with the terms of 2/10, net 50. If the company pays its account on the 50th day, the effective borrowing cost of skipping the discount on Day 10 is closest to a. 14...

See Answer

Q: Nalco Holding is an international company that operates in 130 countries,

Nalco Holding is an international company that operates in 130 countries, has a market capitalization (market value of equity) of $2.3 billion, and reported net income of $45 million on $3.3 billion i...

See Answer

Q: You work for a venture capital firm and are approached to finance

You work for a venture capital firm and are approached to finance a new high-tech start-up. While you believe in the business idea, you also believe it is very risky. What strategies can help to mitig...

See Answer

Q: Which of the companies reduced the average time it took to collect

Which of the companies reduced the average time it took to collect on accounts receivable from 2016 to 2017? a. Company A. b. Company B. c. Company C. d. Company D.

See Answer

Q: Deere and Bros. is a broker that brings new issues of

Deere and Bros. is a broker that brings new issues of small firms to the public market. Its most recent deal for Dextra, Inc., had the following characteristics: Number of shares: ………………………… 1,000,000...

See Answer

Q: Suppose that a biotech firm in Pittsburgh raised $120 million in

Suppose that a biotech firm in Pittsburgh raised $120 million in an IPO. The firm received $23 per share, and the stock sold to the public for $25 per share. The firm’s legal fees, SEC registration fe...

See Answer

Q: The 20-year Treasury rate is 4.67 percent,

The 20-year Treasury rate is 4.67 percent, and a firm’s credit rating is BB. Suppose management of the firm decides to raise $20 million by selling 20-year bonds. Management determines that since it h...

See Answer

Q: Trajax, Inc., a high-technology firm in Portland,

Trajax, Inc., a high-technology firm in Portland, raised a total of $90 million in an IPO. The company received $27 of the $30 per share offering price. The firm’s legal fees, SEC registration fees, a...

See Answer

Q: Myriad Biotech management plans a $114 million IPO in which the

Myriad Biotech management plans a $114 million IPO in which the offering price to the public will be $51 per share. The company will receive $47.50 per share. The firm’s legal fees, SEC registration f...

See Answer

Q: The Modigliani and Miller theory suggests that the value of the firm’s

The Modigliani and Miller theory suggests that the value of the firm’s assets is equal to the value of the claims on those assets and is not dependent on how the asset claims are divided. The common a...

See Answer

Q: Give some examples of sources of short-term financing?

Give some examples of sources of short-term financing?

See Answer

Q: List and describe three practical considerations that concern managers when they make

List and describe three practical considerations that concern managers when they make capital structure decisions?

See Answer

Q: Keyboard Chiropractic Clinics produces $300,000 of cash flow each

Keyboard Chiropractic Clinics produces $300,000 of cash flow each year. The firm has no debt outstanding, and its cost of equity capital is 25 percent. The firm’s management would like to repurchase $...

See Answer

Q: What is the effect on Modigliani and Miller’s Proposition 1 of relaxing

What is the effect on Modigliani and Miller’s Proposition 1 of relaxing the assumption that there are no information or transaction costs?

See Answer

Q: Backwards Resources Company has a WACC of 12.6 percent,

Backwards Resources Company has a WACC of 12.6 percent, and it is subject to a 40 percent marginal tax rate. Backwards has $250 million of debt outstanding at an interest rate of 9 percent and $750 mi...

See Answer

Q: Briefly discuss costs of financial distress to a firm that may arise

Briefly discuss costs of financial distress to a firm that may arise when employees believe it is highly likely that the firm will declare bankruptcy?

See Answer

Q: Describe what exactly is meant when someone is describing the value of

Describe what exactly is meant when someone is describing the value of the firm versus the value of the equity of the firm?

See Answer

Q: Santa’s Shoes is a retailer that has just begun having financial difficulty

Santa’s Shoes is a retailer that has just begun having financial difficulty. Santa’s suppliers are aware of the increased possibility of bankruptcy. What might Santa’s suppliers do based on this infor...

See Answer

Q: Deficit Corp. management has determined that they will be $50

Deficit Corp. management has determined that they will be $50 million short of being able to pay the firm’s debt obligations at the end of this year. Management has identified a positive NPV project t...

See Answer

Q: Use the information in the following table to make a suggestion concerning

Use the information in the following table to make a suggestion concerning the proportion of debt that the firm should utilize in its capital structure?

See Answer

Q: Problem 16.22 introduces taxes and information and transaction costs to

Problem 16.22 introduces taxes and information and transaction costs to the simplified Modigliani and Miller model. If the marginal tax rate for the firm were to suddenly increase by a material amount...

See Answer

Q: What is M&M Proposition 1? M&M Proposition

What is M&M Proposition 1? M&M Proposition 2?

See Answer

Q: Explain bootstrapping, and list the most common sources of seed money

Explain bootstrapping, and list the most common sources of seed money?

See Answer

Q: Describe the order of financial sources for managers who subscribe to the

Describe the order of financial sources for managers who subscribe to the pecking order theory of financing. Evaluate that order by observing the costs of each source relative to the costs of other so...

See Answer

Q: The pecking order theory suggests that managers prefer to first use internally

The pecking order theory suggests that managers prefer to first use internally generated equity to finance new projects. Does this preference mean that these funds represent an even cheaper source of...

See Answer

Q: Discuss how the legal costs of financial distress may increase with the

Discuss how the legal costs of financial distress may increase with the probability that a firm will formally declare bankruptcy, even if the firm has not reached that point yet?

See Answer

Q: Operating a firm without debt is generally considered to be a conservative

Operating a firm without debt is generally considered to be a conservative practice. Discuss how such a conservative approach to a firm’s capital structure is good or bad for the value of the firm in...

See Answer

Q: Finite Corp. has $250 million of debt outstanding at an

Finite Corp. has $250 million of debt outstanding at an interest rate of 11 percent. What is the present value of the debt tax shield if the debt will mature in five years (and no new debt will replac...

See Answer

Q: The Boring Corporation is currently valued at $900 million, but

The Boring Corporation is currently valued at $900 million, but management wants to completely pay off its perpetual debt of $300 million. Boring is subject to a 30 percent marginal tax rate. If Borin...

See Answer

Q: If we drop the assumption that there are no information or transaction

If we drop the assumption that there are no information or transaction costs, in addition to dropping the no-tax assumption, then will the Modigliani and Miller model still suggest that the firm shoul...

See Answer

Q: PolyAna Corporation has an abundant cash flow. It is so high

PolyAna Corporation has an abundant cash flow. It is so high that the managers take Fridays off for a weekly luncheon in Cancun using the corporate jet. Describe how altering the capital structure of...

See Answer

Q: Forwards Resources Company is currently an all-equity firm with a

Forwards Resources Company is currently an all-equity firm with a WACC of 14% and a 40 percent marginal tax rate. Forwards wants to move to a capital structure with $250 million of debt outstanding at...

See Answer

Q: You own all of the equity in a debt-free App

You own all of the equity in a debt-free App development business that generates cash flows of $400,000 each year in perpetuity. The cost of assets, kAssets is 10 percent and the tax rate is 25 perce...

See Answer

Q: Who are venture capitalists, and what do they do?

Who are venture capitalists, and what do they do?

See Answer

Q: A firm that is financed completely with equity currently has a cost

A firm that is financed completely with equity currently has a cost of capital equal to 15 percent. If Modigliani and Miller’s Proposition 1 holds and the firm’s management is thinking about changing...

See Answer

Q: Swan Specialty Cycles is currently financed with 50 percent debt and 50

Swan Specialty Cycles is currently financed with 50 percent debt and 50 percent equity. The firm pays $125 each year to its debt investors (at a 10 percent cost of debt), and the debt has no maturity...

See Answer

Q: Legitron Corporation has $350 million of debt outstanding at an interest

Legitron Corporation has $350 million of debt outstanding at an interest rate of 9 percent. What is the dollar value of the tax shield on that debt, just for this year, if Legitron is subject to a 35...

See Answer

Q: Given the data for Cattail Corporation in Problem 19.9,

Given the data for Cattail Corporation in Problem 19.9, if you assume that all balance sheet items also vary with the change in sales, develop a pro forma balance sheet for Cattail for the next fiscal...

See Answer

Q: For McDonald Metal Works in Problem 19.13, how much

For McDonald Metal Works in Problem 19.13, how much must net sales grow if the capital intensity ratio has to drop to 60 percent? State your answer as both a percent of sales and a dollar sales increa...

See Answer

Q: Swan Supply Company has net income of $1,212,

Swan Supply Company has net income of $1,212,335, assets of $12,522,788 and retains 70 percent of its income every year. What is the company’s internal growth rate?

See Answer

Q: Refer to Problem 19.7. Northwood expects to increase its

Refer to Problem 19.7. Northwood expects to increase its sales by 15 percent next year. All costs vary directly with sales. If Northwood wants to retain $65,000 of earnings next year, will it have to...

See Answer

Q: Tomey Supply Company’s financial statements for the most recent fiscal year are

Tomey Supply Company’s financial statements for the most recent fiscal year are shown below. The company management projects that sales will increase by 20 percent next year. Assume...

See Answer

Q: Use the following pro forma information for Tomey Supply Company for next

Use the following pro forma information for Tomey Supply Company for next year: net income = $508,275; addition to retained earnings = $340,544; common equity = $848,171; net sales = $2,121,745. Assum...

See Answer

Q: Rowan Company has a net profit margin of 8.3 percent

Rowan Company has a net profit margin of 8.3 percent, debt ratio of 45 percent, total assets of $4,157,550, and sales of $6,852,654. If the company has a dividend payout ratio of 67 percent, what is i...

See Answer

Q: How do venture capitalists reduce the risk of their investments?

How do venture capitalists reduce the risk of their investments?

See Answer

Q: Refer to the information for Rowan Company in Problem 19.23

Refer to the information for Rowan Company in Problem 19.23. The firm’s management desires a sustainable growth rate (SGR) of 10 percent but does not wish to change the company’s level of debt or its...

See Answer

Q: Rocky Sales, Inc., has current sales of $1,

Rocky Sales, Inc., has current sales of $1,215,326 and net income of $211,253. It also has a debt ratio of 25 percent and a dividend payout ratio of 75 percent. The company’s total assets are $712,455...

See Answer

Q: Ellicott Textile Mills management has reported the following financial information for the

Ellicott Textile Mills management has reported the following financial information for the year ended September 30, 2017. The company generated a net income of $915, 366 on a net profit margin of 6.4...

See Answer

Q: Maryland Micro Brewers generated revenues of $12,125,800

Maryland Micro Brewers generated revenues of $12,125,800 with a 72 percent capital intensity ratio during the year ended September 30, 2017. Its net income was $873,058. With the introduction of a hal...

See Answer

Q: What are the elements of a financing plan?

What are the elements of a financing plan?

See Answer

Q: Ritchie Marble Company has total assets of $12,899,

Ritchie Marble Company has total assets of $12,899,450, sales of $18,174,652, and net income of $4,589,774. Management expects sales to grow by 25 percent next year. All assets and costs (including ta...

See Answer

Q: Norton Group, Inc., expects to add $1,213

Norton Group, Inc., expects to add $1,213,777 to retained earnings and currently has total assets of $23,159,852. If the company has the ability to borrow up to $1 million, how much growth can the fir...

See Answer

Q: Capstone Marketing Group has total assets of $5,568,

Capstone Marketing Group has total assets of $5,568,000, sales of $3,008,725, and net income of $822,000. The company expects its sales to grow by 12 percent next year. All assets and costs (includin...

See Answer

Q: Given the data for Capstone Marketing Group in Problem 19.32

Given the data for Capstone Marketing Group in Problem 19.32, what would Capstone’s payout ratio have to be for the firm’s EFN to be zero? Refer to the given data for Capstone Marketing Group in Prob...

See Answer

Q: Rockville Consulting Group expects to add $271,898 to retained

Rockville Consulting Group expects to add $271,898 to retained earnings this year. The company has total assets of $3,425,693 and wishes to add no new external funds for the coming year. If assets and...

See Answer

Q: Explain the venture capital funding cycle?

Explain the venture capital funding cycle?

See Answer

Q: The financial statements for the year ended June 30, 2017,

The financial statements for the year ended June 30, 2017, are given below for Morgan Construction Company. The firm’s sales are projected to grow at a rate of 25 percent next year,...

See Answer

Q: Use the financial information for Morgan Construction Company from Problem 19.

Use the financial information for Morgan Construction Company from Problem 19.35. Assume now that equity accounts do not vary directly with sales but change when retained earnings change or new equity...

See Answer

Q: Using the information for Morgan Construction Company in the preceding problem,

Using the information for Morgan Construction Company in the preceding problem, calculate the firm’s internal growth rate and sustainable growth rate?

See Answer

Q: Use the information for Morgan Construction Company from Problems 19.35

Use the information for Morgan Construction Company from Problems 19.35 and 19.36. Assume that equity accounts do not vary directly with sales, but change when retained earnings change or new equity i...

See Answer

Q: Munson Communications Company has just reported earnings for the year ended June

Munson Communications Company has just reported earnings for the year ended June 30, 2017. Below are the firm’s income statement and balance sheet. The company had a 55 percent divid...

See Answer

Q: Identify the steps in the financial planning process?

Identify the steps in the financial planning process?

See Answer

Q: Define the retention (plowback) ratio and the dividend payout ratio

Define the retention (plowback) ratio and the dividend payout ratio?

See Answer

Q: Goodwin Corp. has revenues of $12,112,659

Goodwin Corp. has revenues of $12,112,659, costs of $9,080,545, interest payments of $412,375, and a tax rate of 34 percent. It paid dividends of $1,025,000 to its stockholders. What are the firm’s di...

See Answer

Q: What is the value at expiration of a call option with a

What is the value at expiration of a call option with a strike price of $65 if the stock price is $1? $50? $65? $100? $1,000?

See Answer

Q: List and describe four different types of real options that are associated

List and describe four different types of real options that are associated with investment projects?

See Answer

Q: What is a seasoned offering, and why are seasoned securities valued

What is a seasoned offering, and why are seasoned securities valued more highly than securities sold in an IPO?

See Answer

Q: How are options related to the agency costs of debt and equity

How are options related to the agency costs of debt and equity?

See Answer

Q: Suppose that you own a call option and a put option on

Suppose that you own a call option and a put option on the same stock and that these options have the same exercise price. Explain how the relative values of these two options will change as the stock...

See Answer

Q: A convertible bond is a bond that can be exchanged for stock

A convertible bond is a bond that can be exchanged for stock at the discretion of the bondholder. How would you go about finding the value of such a bond? Would the bond be worth more or less than an...

See Answer

Q: Explain how the payoff functions differ for the owner (buyer)

Explain how the payoff functions differ for the owner (buyer) and the seller:(1) of a call option; (2) of a put option?

See Answer

Q: The stock of Socrates Motors is currently trading for $40 and

The stock of Socrates Motors is currently trading for $40 and will either rise to $50 or fall to $35 in one month. The risk-free rate for one month is 1.5 percent. What is the value of a one-month cal...

See Answer

Q: Assume that the stock of Socrates Motors is currently trading for $

Assume that the stock of Socrates Motors is currently trading for $40 and will either rise to $50 or fall to $35 in one month. The risk-free rate for one month is 1.5 percent. What is the value of a o...

See Answer

Q: You are considering buying a three-month put option on Wing

You are considering buying a three-month put option on Wing and a Prayer Construction stock. The company’s stock currently trades for $10 per share and its price will either rise to $15 or fall to $7...

See Answer

Q: You hold a European put option on Tubes, Inc., stock

You hold a European put option on Tubes, Inc., stock, with a strike price of $100. Things haven’t been going too well for Tubes. The current stock price is $2, and you think that it will either rise t...

See Answer

Q: A golden parachute is a part of a manager’s compensation package that

A golden parachute is a part of a manager’s compensation package that makes a large lump-sum payment in the event that the manager is fired (or loses his or her job in a merger, for example). Providin...

See Answer

Q: Consider the following payoff diagram. / Find a

Consider the following payoff diagram. Find a combination of calls, puts, risk-free bonds, and stock that has this payoff. (You need not use all of these instruments, and there are many possible solu...

See Answer

Q: Explain the two ways a security issue can be underwritten?

Explain the two ways a security issue can be underwritten?

See Answer

Q: Consider the payoff structures of the following two portfolios: a

Consider the payoff structures of the following two portfolios: a. Buying a one month call option on one share of stock at a strike price of $50 and saving the present value of $50 (so that at expirat...

See Answer

Q: One way to extend the binomial pricing model is by including multiple

One way to extend the binomial pricing model is by including multiple time periods. Suppose Splittime, Inc., is currently trading for $100 per share. In one month, the price will either increase by $1...

See Answer

Q: Wolfgang’s Masonry management estimates that it takes the company 27 days on

Wolfgang’s Masonry management estimates that it takes the company 27 days on average to pay its suppliers. Management also knows that the company has days’ sales in inventory of 64 days and days’ sale...

See Answer

Q: What items in a business plan does a venture capitalist look for

What items in a business plan does a venture capitalist look for in deciding whether to provide initial financing?

See Answer

Q: Devon Automotive management estimates that it takes the company 62 days to

Devon Automotive management estimates that it takes the company 62 days to collect cash from customers on finished goods from the day it receives raw materials, and it takes 65 days to pay its supplie...

See Answer

Q: Montrose, Inc. sells its products with terms of 3/

Montrose, Inc. sells its products with terms of 3/15 EOM, net 30. What is the cost of the trade credit it provides its customers?

See Answer

Q: FRA Manufacturing Company purchases 9,000 units of Part 3BX each

FRA Manufacturing Company purchases 9,000 units of Part 3BX each year. The cost of placing an order is $5 and the cost of carrying one part in inventory for a year is $1. What is the Economic Order Qu...

See Answer

Q: Management of Southern Parts Company has decided to sell 10-year

Management of Southern Parts Company has decided to sell 10-year bonds to finance expansion into the Pacific Northwest. The loan rate on these bonds is 8 percent and the 3-month Treasury bill rate is...

See Answer

Q: Sunny Way Landscaping has a formal line of credit of $500

Sunny Way Landscaping has a formal line of credit of $500,000 with First Commerce Bank. The interest rate on the loan is 6 percent, and under the agreement, Sunny Skies must pay an annual fee of 75 ba...

See Answer

Q: Why are traditional sources of funding not usually available for new or

Why are traditional sources of funding not usually available for new or emerging businesses?

See Answer

Q: What is underpricing, and why is it a cost to the

What is underpricing, and why is it a cost to the stockholders?

See Answer

Q: Hilton Worldwide Holdings Inc. completed an initial public offering on December

Hilton Worldwide Holdings Inc. completed an initial public offering on December 12, 2013. The offer price was $20.00 per share and the closing price at the end of the first day was $21.50. The firm is...

See Answer

Q: SMA Inc. is considering issuing the following securities. For which

SMA Inc. is considering issuing the following securities. For which security would a competitive sale be less costly than a negotiated sale under stable market conditions? Why? (LO 5) a) Plain vanilla...

See Answer

Q: Central Grocers Inc. produces annual cash flows of $175,

Central Grocers Inc. produces annual cash flows of $175,000, which are expected to continue indefinitely. The company is financed entirely with equity capital at an annual cost of 12 percent. Manageme...

See Answer

Q: Southwest Airlines has substantial cash reserves and an investment-grade bond

Southwest Airlines has substantial cash reserves and an investment-grade bond rating. How would the trade-off theory predict that managers of Southwest would raise capital and choose the company’s cap...

See Answer

Q: What control implications do a firm’s capital structure decisions have?

What control implications do a firm’s capital structure decisions have?

See Answer

Q: Mars Company had net sales of $18 million in the year

Mars Company had net sales of $18 million in the year that just ended. Next year, the company’s management expects a 15 percent increase in sales. If cost of goods sold is 60 percent of sales and inve...

See Answer

Q: Lavaca Inc. management expects net sales to be $855,

Lavaca Inc. management expects net sales to be $855,000, total costs to be $647,000, and to pay taxes at an average rate of 32 percent this year. If the Lavaca pays out 38 percent of its earnings as d...

See Answer

Q: Spurlock Inc. had net income of $266,778 in

Spurlock Inc. had net income of $266,778 in its most recent fiscal year and total assets of $1,833,400 at the end of the year. The company’s total debt ratio (total debt to total assets) is 35 percent...

See Answer

Q: Using the information in Sample Test Problem 19.3, what

Using the information in Sample Test Problem 19.3, what is Spurlock’s capital intensity ratio if the company has net sales of $3,557,100? What does this ratio tell us? Refer to the information in pro...

See Answer

Q: Edgefield Excavation Company has total assets of $4,976,

Edgefield Excavation Company has total assets of $4,976,456, sales of $1,225,700, and net income of $587,000. The company’s management expects sales to grow by 9 percent next year. All costs (includin...

See Answer

Q: Explain why firms generally sell their equity and complicated debt issues through

Explain why firms generally sell their equity and complicated debt issues through negotiated sales?

See Answer

Q: You own a call option on Pepsico stock with a strike price

You own a call option on Pepsico stock with a strike price of $60 per share that expires in 60 days. The current market price of Pepsico stock is $63.50 per share. What are the limits on the value of...

See Answer

Q: Assume that the current market price of Montrose Industrials stock is $

Assume that the current market price of Montrose Industrials stock is $28 per share and will either rise to $38 per share or fall to $21 per share in one month. The risk-free rate for one month is 1 p...

See Answer

Q: Why is it hard to account for real options in an NPV

Why is it hard to account for real options in an NPV analysis?

See Answer

Q: Fuel costs are a significant fraction of total costs in the airline

Fuel costs are a significant fraction of total costs in the airline industry. How might airline managers use options to manage fuel costs? What is the downside of doing this?

See Answer

Q: A bank in India has offered a spot rate quote on Indian

A bank in India has offered a spot rate quote on Indian rupees (Rs) of Rs62.2905/$. The Indian rupee is quoted at a 30-day forward premium of 5.22 percent against the dollar. What is the 30-day forwar...

See Answer

Q: Technocorp has purchased industrial parts from a German company for a total

Technocorp has purchased industrial parts from a German company for a total cost of €1,225,000. The firm has 30 days to pay. A bank has given Technocorp a 30-day forward quote of $1.1278/€. Assume tha...

See Answer

Q: Tass Co., Ltd, a Japanese electrical parts producer, is

Tass Co., Ltd, a Japanese electrical parts producer, is considering building a plant in the U.S. The cost of this plant will be $20 million and the current spot exchange rate between the yen and the U...

See Answer

Q: The Whole Foods Market, Inc. balance sheet for the fiscal

The Whole Foods Market, Inc. balance sheet for the fiscal year ending September 25, 2016 included the following: total current assets of $1,975 million, total assets of $6,341 million, total current l...

See Answer

Q: Last year Perpetual Plastics Company took an average of 46 days to

Last year Perpetual Plastics Company took an average of 46 days to pay suppliers and 38 days to collect its receivables. The company’s average days’ sales in inventory was 52 days. What was Perpetual’...

See Answer

Q: What is a real option?

What is a real option?

See Answer

Q: Explain the importance of shelf registration?

Explain the importance of shelf registration?

See Answer

Q: What is globalization?

What is globalization?

See Answer

Q: What is the optimal capital structure for a firm?

What is the optimal capital structure for a firm?

See Answer

Q: What is the trade-off theory of capital structure?

What is the trade-off theory of capital structure?

See Answer

Q: How can capital structure decisions affect the risk associated with net income

How can capital structure decisions affect the risk associated with net income?

See Answer

Q: How can the three conditions specified by M&M help us

How can the three conditions specified by M&M help us understand how the capital structure of a firm affects its value?

See Answer

Q: What is the primary motivation for leasing?

What is the primary motivation for leasing?

See Answer

Q: How do you calculate net working capital, and why is it

How do you calculate net working capital, and why is it important?

See Answer

Q: What are the types of costs associated with each of these strategies

What are the types of costs associated with each of these strategies?

See Answer

Q: What is an aging schedule, and what is its purpose?

What is an aging schedule, and what is its purpose?

See Answer

Q: What is the economic order quantity model?

What is the economic order quantity model?

See Answer

Q: What are the disadvantages of a private placement sale compared with a

What are the disadvantages of a private placement sale compared with a public sale?

See Answer

Q: What is float?

What is float?

See Answer

Q: What are the advantages and disadvantages of short-term financing?

What are the advantages and disadvantages of short-term financing?

See Answer

Q: List the steps in the IPO process?

List the steps in the IPO process?

See Answer

Q: What are the components of the cost associated with an IPO?

What are the components of the cost associated with an IPO?

See Answer

Q: What is the strategic plan?

What is the strategic plan?

See Answer

Q: How are the investment decision and financing decision related?

How are the investment decision and financing decision related?

See Answer

Q: Why is the sales forecast the key component of a financial model

Why is the sales forecast the key component of a financial model?

See Answer

Q: What are pro forma financial statements, and why are they an

What are pro forma financial statements, and why are they an important part of the financial planning process?

See Answer

Q: What is the plug factor in a financial model?

What is the plug factor in a financial model?

See Answer

Q: Why might you expect accounts receivable to vary with sales?

Why might you expect accounts receivable to vary with sales?

See Answer

Q: What is the difference between business risk and financial risk?

What is the difference between business risk and financial risk?

See Answer

Q: Why do companies engage in PIPE transactions?

Why do companies engage in PIPE transactions?

See Answer

Q: Why is it that some working capital accounts may not vary proportionately

Why is it that some working capital accounts may not vary proportionately with sales?

See Answer

Q: What are lumpy assets, and how do these assets vary with

What are lumpy assets, and how do these assets vary with sales?

See Answer

Q: What two factors determine the amount of EFN?

What two factors determine the amount of EFN?

See Answer

Q: What variables affect the value of a call option?

What variables affect the value of a call option?

See Answer

Q: What are four different types of real options commonly found in business

What are four different types of real options commonly found in business?

See Answer

Q: What types of risks can options be used to manage?

What types of risks can options be used to manage?

See Answer

Q: What are multinational corporations?

What are multinational corporations?

See Answer

Q: Explain the difference between American and European business executives’ views on wealth

Explain the difference between American and European business executives’ views on wealth maximization?

See Answer

Q: What is foreign exchange rate risk?

What is foreign exchange rate risk?

See Answer

Q: How is the equilibrium exchange rate determined?

How is the equilibrium exchange rate determined?

See Answer

Q: What are the four planning documents on which the financial plan is

What are the four planning documents on which the financial plan is based?

See Answer

Q: What does it mean to hedge a financial transaction?

What does it mean to hedge a financial transaction?

See Answer

Q: When do companies have to consider country or political risk?

When do companies have to consider country or political risk?

See Answer

Q: Management of Oakley, Inc., is planning to raise $1

Management of Oakley, Inc., is planning to raise $1 million in new equity through a private placement. If the sale price is $18 per share, how many shares does the company have to issue?

See Answer

Q: Of the two parties to an option contract, the buyer and

Of the two parties to an option contract, the buyer and the seller, who has a right and who has an obligation?

See Answer

Q: If a Volkswagen Passat costs $26,350 in Baltimore and

If a Volkswagen Passat costs $26,350 in Baltimore and €21,675 in Frankfurt, what is the exchange rate between the U.S. dollar and the euro?

See Answer

Q: Calculate the following exchange rates given the following information? /

Calculate the following exchange rates given the following information?

See Answer

Q: Columbia Corp. has just made a sale to a British customer

Columbia Corp. has just made a sale to a British customer. The sale was for a total value of £135,000 and is to be paid 60 days from now. Columbia is concerned that the British pound will depreciate a...

See Answer

Q: American Bancorp is planning to make a $3.5 million

American Bancorp is planning to make a $3.5 million loan to a French firm. Currently, LIBOR is at 1.5 percent. American considers a default risk premium of 1.15 percent, a foreign exchange risk premiu...

See Answer

Q: Suppose a firm is doing an IPO and the investment bank offers

Suppose a firm is doing an IPO and the investment bank offers to buy the securities for $39 per share with an offering price of $42. What is the underwriter’s spread? Assume that the underwriter’s cos...

See Answer

Q: Management of The Stride Rite Corporation, designer and marketer of athletic

Management of The Stride Rite Corporation, designer and marketer of athletic apparel, is planning an expansion into foreign markets and needs to raise $10 million to finance this move. Management anti...

See Answer

Q: How are historical financial data used to determine the forecasted values of

How are historical financial data used to determine the forecasted values of balance sheet accounts?

See Answer

Q: Dean Foods Co. needs to borrow $23 million for a

Dean Foods Co. needs to borrow $23 million for a factory equipment upgrade. Management decides to sell 10-year bonds. They determine that the 3-month Treasury bill yields 4.32 percent, the firm’s cred...

See Answer

Q: You are considering starting a new online dating service, but you

You are considering starting a new online dating service, but you lack the initial capital. What are your options for financing?

See Answer

Q: If any of the three assumptions in Modigliani and Miller Proposition 1

If any of the three assumptions in Modigliani and Miller Proposition 1 are relaxed, which has the most predictably quantifiable impact on the value of the firm?

See Answer

Q: If we assume that the cash flows for a firm with financial

If we assume that the cash flows for a firm with financial leverage are equal to the cash flows for the same firm without financial leverage, what can we say about the value of this firm if its cost o...

See Answer

Q: Northwood Corp. has a dividend payout ratio of 60 percent,

Northwood Corp. has a dividend payout ratio of 60 percent, return on equity of 14.5 percent, total assets of $11,500,450, and equity of $4,652,125. Calculate the firm’s internal rate of growth (IGR)?...

See Answer

Q: Renewal Company has net income of $1.25 million and

Renewal Company has net income of $1.25 million and a dividend payout ratio of 35 percent. It currently has equity of $2,875,223. What is the firm’s sustainable growth rate?

See Answer

Q: Mello Wines, a California winery, grows its grapes locally,

Mello Wines, a California winery, grows its grapes locally, uses local labor, and sells its wines only in the United States. Can this firm be exposed to foreign exchange risk?

See Answer

Q: How can the insurance policy on a car be viewed as an

How can the insurance policy on a car be viewed as an option?

See Answer

Q: A U.S. firm owns a subsidiary in Belgium.

A U.S. firm owns a subsidiary in Belgium. What kind of foreign exchange risk does the U.S. firm face?

See Answer

Q: Suppose GE issues bearer bonds in France denominated in British pounds.

Suppose GE issues bearer bonds in France denominated in British pounds. What type of bonds are these?

See Answer

Q: What is IGR, and why is it of interest to management

What is IGR, and why is it of interest to management?

See Answer

Q: Why is the capital budget an important part of a firm’s financial

Why is the capital budget an important part of a firm’s financial planning?

See Answer

Q: Why do financing and investment decisions have to be made concurrently?

Why do financing and investment decisions have to be made concurrently?

See Answer

Q: A writer (seller) of a call option may or may

A writer (seller) of a call option may or may not actually own the underlying asset. If he or she owns the asset, and therefore will have the asset available to deliver should the option be exercised,...

See Answer

Q: Like all other models, the binomial pricing model is a simplification

Like all other models, the binomial pricing model is a simplification of reality. In this model, how do we represent high volatility or low volatility of the value of the underlying asset?

See Answer

Q: List all the accounts that can be affected by the “plug

List all the accounts that can be affected by the “plug” value. How does this value help managers?

See Answer

Q: What kinds of real options should be considered in the following situations

What kinds of real options should be considered in the following situations? a. Wingnuts R Us is considering two sites for a new factory. One is just large enough for the planned facility, while the o...

See Answer

Q: Ray Corp is a U.S. electronics manufacturer with a

Ray Corp is a U.S. electronics manufacturer with a production plant in Turkey. This morning, the Turkish government introduced a new law prohibiting the repatriation of any funds from the country for...

See Answer

Q: Explain why time might play a significant role during low-interest

Explain why time might play a significant role during low-interest periods in a decision of whether to choose a private placement or public sale?

See Answer

Q: Assuming nothing else changes, what happens to the value of an

Assuming nothing else changes, what happens to the value of an option as time passes and the expiration date gets closer?

See Answer

Q: Ryan wants to buy a pair of leather shoes at Harrods in

Ryan wants to buy a pair of leather shoes at Harrods in London that cost £113.60. If the exchange rate is $1.6177/£, what is Ryan’s cost in U.S. dollars?

See Answer

Q: If a firm continually exceeds its SGR, what problems may it

If a firm continually exceeds its SGR, what problems may it face in the future?

See Answer

Q: Triumph Autos has contracted with an Indian software firm for design software

Triumph Autos has contracted with an Indian software firm for design software. The payment of 22,779,750 rupees (₹) is due in 30 days. What is the cost in dollars if the 30-day forward rate is ₹64.39/...

See Answer

Q: Nova Scotia Bank offers quotes on the Canadian dollar as shown below

Nova Scotia Bank offers quotes on the Canadian dollar as shown below. What is the bid-ask spread based on these quotes? Bid …………………………. Ask C$ 1.3081/$ …………………. C$ 1.3087/$

See Answer

Q: A local community bank has requested foreign exchange quotes for the Swiss

A local community bank has requested foreign exchange quotes for the Swiss Franc from Citibank. Citibank quotes a bid rate of $1.0934/SF and an ask rate of $1.0997/SF. What is the bid-ask spread?

See Answer

Q: A foreign exchange dealer is willing to buy the Danish krone (

A foreign exchange dealer is willing to buy the Danish krone (DKr) at $0.1556/DKr and will sell it at a rate of $0.1563/DKr. What is the bid-ask spread on the Danish krone?

See Answer

Q: Given the following quotes, calculate the €/£ cross rate.

Given the following quotes, calculate the €/£ cross rate. Bank of America ……………….. $1.663/£ JP Morgan Chase ……………… $1.3914/€

See Answer

Q: Barclays Bank of London has offered the following exchange rate quotes:

Barclays Bank of London has offered the following exchange rate quotes: ¥134.64/£ and Korean won 13.8374/¥. What is the cross rate between the Korean won and the British pound?

See Answer

Q: Bremer Corporation observes that the Swiss franc (SF) is being

Bremer Corporation observes that the Swiss franc (SF) is being quoted at €0.7660/SF, while the Swedish krona (SK) is quoted at €0.1114/SK. What is the SK/SF cross rate?

See Answer

Q: Crescent Corporation’s recent sale to a firm in Mexico produced revenues of

Crescent Corporation’s recent sale to a firm in Mexico produced revenues of 13,144,800 Mexican pesos (MPs). If the firm sold the pesos to its bank and was credited with $1,077,873.60, what was the spo...

See Answer

Q: What are Yankee bonds?

What are Yankee bonds?

See Answer

Q: The spot rate on the London market was £0.7531

The spot rate on the London market was £0.7531/$, while the 90-day forward rate is £0.7602/$. What is the annualized forward premium or discount on the British pound?

See Answer

Q: What is a put option, and what do the payoff functions

What is a put option, and what do the payoff functions for the owner and seller of a put option look like?

See Answer

Q: Bank of America quoted the 180-day forward rate on the

Bank of America quoted the 180-day forward rate on the Swiss franc at $1.0407/SF. The spot rate was quoted at $1.0268/SF. What is the forward premium or discount on the Swiss franc?

See Answer

Q: Flint Corp. recently purchased auto parts worth 17.5 million

Flint Corp. recently purchased auto parts worth 17.5 million Mexican pesos (MP) on credit. Management needs to find out the U.S. dollar cost of the purchase. It has access to two quotes for Canadian d...

See Answer

Q: Given the following direct quotes, calculate the equivalent indirect quotes.

Given the following direct quotes, calculate the equivalent indirect quotes. a. $0.0844/Mexican pesos b. £0.8513/€ c. Rs 54.64/ C$

See Answer

Q: A Swiss sporting goods company borrows in yen in the Eurocredit market

A Swiss sporting goods company borrows in yen in the Eurocredit market at a rate of 4.35 percent from Bank of America using a three-month rollover loan. Bank of America assigns a default risk premium...

See Answer

Q: Convert the following indirect quotes to the appropriate American quotes.

Convert the following indirect quotes to the appropriate American quotes. a. £0.6917/$ b. ¥104.28/$ c. SF 1.0769/$

See Answer

Q: Suppose a BMW 528i is priced at $68,750 in

Suppose a BMW 528i is priced at $68,750 in New York and € 50,267 in Berlin. In which place is the car more expensive if the spot rate is $1.1077/€?

See Answer

Q: Crane, Inc., sold equipment to an Irish firm and will

Crane, Inc., sold equipment to an Irish firm and will receive €1,319,405 in 30 days. If the company entered a forward contract to sell at the 30-day forward rate of $1.0812/€, what is the dollar reven...

See Answer

Q: Brilliant Equipment purchased machinery from a Japanese firm and must make a

Brilliant Equipment purchased machinery from a Japanese firm and must make a payment of ¥313.25 million in 45 days. The bank quotes a forward rate of ¥103.01/$ to buy the required yen. What is the cos...

See Answer

Q: Explain the importance of the strategic plan.

Explain the importance of the strategic plan.

See Answer

Q: Rosenthal Design has daily sales of $59,000. The

Rosenthal Design has daily sales of $59,000. The financial management team has determined that a lockbox would reduce the collection time by 1.6 days. Assuming the company can earn 5.2 percent interes...

See Answer

Q: Why does the payoff function for an option have a kink in

Why does the payoff function for an option have a kink in it?

See Answer

Q: What are the steps in a general cash offering? Explain each

What are the steps in a general cash offering? Explain each of them.

See Answer

Q: Are the following statements true or false. a. Shelf

Are the following statements true or false. a. Shelf registration allows firms to register an inventory of securities for an unlimited time. b. The securities can be taken off the shelf at any time wi...

See Answer

Q: Explain what is meant by economies of scale in issuing securities?

Explain what is meant by economies of scale in issuing securities?

See Answer

Q: Marx and Spender Corp. currently has a WACC of 21 percent

Marx and Spender Corp. currently has a WACC of 21 percent. If the cost of debt capital for the firm is 12 percent and the firm is currently financed with 25 percent debt, then what is the current cost...

See Answer

Q: Springer Corp. has $250 million of debt outstanding at an

Springer Corp. has $250 million of debt outstanding at an interest rate of 11 percent. What is the present value of the debt tax shield if the debt has no maturity and if Springer is subject to a 40 p...

See Answer

Q: Suppose two firms want to borrow money from a bank for a

Suppose two firms want to borrow money from a bank for a period of one year. Firm A has excellent credit, whereas Firm B’s credit standing is such that it would pay prime + 2 percent. The current prim...

See Answer

Q: Now suppose that Firm B from Problem 15.16 decides to

Now suppose that Firm B from Problem 15.16 decides to get a term loan for 10 years. How does this affect the company’s borrowing cost?

See Answer

Q: Cartco needs to borrow $5 million for an upgrade to its

Cartco needs to borrow $5 million for an upgrade to its headquarters and manufacturing facility. Management has decided to borrow using a five-year term loan from its existing commercial bank. The p...

See Answer

Q: Structural Corp. currently has an equity cost of capital equal to

Structural Corp. currently has an equity cost of capital equal to 15 percent. If the Modigliani and Miller Proposition 1 assumptions hold, with the exception of the assumption that there are no taxes,...

See Answer

Q: Northern Manufacturing Company management found that during the last year it took

Northern Manufacturing Company management found that during the last year it took an average of 47 days to pay its suppliers, whereas it took 63 days to collect its receivables. The company’s days’ sa...

See Answer

Q: What is a call option, and what do the payoff functions

What is a call option, and what do the payoff functions for the owner and seller of a call option look like?

See Answer

Q: On August 19, 2004, Google completed its IPO of 19

On August 19, 2004, Google completed its IPO of 19.6 million shares to the initial investors at $85.00 per share. The closing price of the stock that same day was $100.34. What was the dollar value of...

See Answer

Q: The weighted average cost of capital for a firm (assuming all

The weighted average cost of capital for a firm (assuming all three Modigliani and Miller assumptions apply) is 15 percent. What is the current cost of equity capital for the firm if its cost of debt...

See Answer

Q: When Global Partners went public in September 2016, the offer price

When Global Partners went public in September 2016, the offer price was $22.00 per share and the closing price at the end of the first day was $23.90. The firm issued 4.9 million shares. What was the...

See Answer

Q: Bellex Technologies agreed to complete its IPO on a best-effort

Bellex Technologies agreed to complete its IPO on a best-effort basis. The company’s investment bank demanded a spread of 17 percent of the offer price, which was set at $30 per share. Three million s...

See Answer

Q: Sliver Computing Inc., reported the following balance sheet information for the

Sliver Computing Inc., reported the following balance sheet information for the last fiscal year. Sliver Computing also reported cost of goods sold of $291,240 for the same period. Internal auditors s...

See Answer

Q: An online medical advice company just completed an IPO with an investment

An online medical advice company just completed an IPO with an investment bank on a firm-commitment basis. The firm issued five million shares of common stock, and the underwriting fees were $1.90 per...

See Answer

Q: Mikos Processed Foods is currently valued at $500 million. Mikos

Mikos Processed Foods is currently valued at $500 million. Mikos will be repurchasing $100 million of its equity by issuing a nonmaturing debt issue at a 10 percent annual interest rate. Mikos is subj...

See Answer

Q: Under Modigliani and Miller’s Proposition 1, where all three of the

Under Modigliani and Miller’s Proposition 1, where all three of the assumptions remain in effect, explain how the value of the firm changes due to changes in the proportion of debt and equity utilized...

See Answer

Q: The weighted average cost of capital for a firm, assuming all

The weighted average cost of capital for a firm, assuming all three Modigliani and Miller assumptions hold, is 10 percent. What is the current cost of equity capital for the firm if the cost of debt f...

See Answer

Q: Define capital intensity ratio, and explain its significance?

Define capital intensity ratio, and explain its significance?

See Answer

Q: What are the limits on the value of a call option prior

What are the limits on the value of a call option prior to its expiration date?

See Answer

Q: What are some viable exit strategies for investors in a start-

What are some viable exit strategies for investors in a start-up company?

See Answer

Q: Tantrix Confectioners has total assets of $3,257,845

Tantrix Confectioners has total assets of $3,257,845 and net sales of $5,123,951. What is the firm’s capital intensity ratio?

See Answer

Q: McDonald Metal Works has been able to generate net sales of $

McDonald Metal Works has been able to generate net sales of $13,445,196 on assets of $9,145,633. What is the firm’s capital intensity ratio?

See Answer

Q: If Newell Corp. has a ROE of 13.7 percent

If Newell Corp. has a ROE of 13.7 percent and a dividend payout ratio of 32 percent, what is its sustainable growth rate?

See Answer

Q: Identify two industries (other than airlines) that are capital intensive

Identify two industries (other than airlines) that are capital intensive. Using online or other data sources, compute the capital intensity ratio for the largest firm in each of the chosen industries?...

See Answer

Q: Consider two companies that operate in the same line of business and

Consider two companies that operate in the same line of business and have the same degree of operating leverage: the Basic Company and the Grundlegend Company. The Basic Company has no debt in its cap...

See Answer

Q: According to the pecking order theory: a. New debt

According to the pecking order theory: a. New debt is preferable to new equity. b. New equity is preferable to internally generated funds. c. New debt is preferable to internally generated funds. d. N...

See Answer

Q: A firm sells $100,000 of its accounts receivable to

A firm sells $100,000 of its accounts receivable to factors at a 2 percent discount. The firm’s average collection period is one month. What is the dollar cost of the factoring service?

See Answer

Q: According to the trade-off theory: a. The

According to the trade-off theory: a. The amount of debt a company has is irrelevant. b. Debt should be used only as a last resort. c. Debt will not be used if a company’s tax rate is high. d. Compani...

See Answer

Q: What are the various steps in preparing a capital budget?

What are the various steps in preparing a capital budget?

See Answer

Q: Why are the variables that affect the value of a put option

Why are the variables that affect the value of a put option the same as those that affect the value of a call option?

See Answer

Q: Using the pro forma financial statements for Tomey Supply Company developed in

Using the pro forma financial statements for Tomey Supply Company developed in Problem 19.20, find the internal growth rate for Tomey?

See Answer

Q: Lilly Bakery distributes its products to more than 75 restaurants and delis

Lilly Bakery distributes its products to more than 75 restaurants and delis. The company’s average collection period is 27 days, and it keeps its inventory for an average of four days. What is Lilly’s...

See Answer

Q: Given the information in Problem 19.26, what is the

Given the information in Problem 19.26, what is the internal growth rate of Ellicott Textile Mills?

See Answer

Q: Fantasy Travel Company has a return on equity of 17.5

Fantasy Travel Company has a return on equity of 17.5 percent, a total equity/total assets ratio of 65 percent, and a dividend payout ratio of 75 percent. What is the company’s internal growth rate?

See Answer

Q: A company increasing its credit terms for customers from 1/10

A company increasing its credit terms for customers from 1/10, net 30 to 1/10, net 60 will likely experience: a. An increase in cash on hand. b. An increase in the average collection period. c. Higher...

See Answer

Q: List the various elements of financial modeling?

List the various elements of financial modeling?

See Answer

Q: Which of the companies has the lowest accounts receivable turnover in 2017

Which of the companies has the lowest accounts receivable turnover in 2017? a. Company A. b. Company B. c. Company C. d. Company D.

See Answer

Q: The industry average receivables collection period: a. Increased from

The industry average receivables collection period: a. Increased from 2016 to 2017. b. Decreased from 2016 to 2017. c. Did not change from 2016 to 2017. d. Increased along with the increase in the ind...

See Answer

Q: Northwood, Inc., has revenue of $455,316,

Northwood, Inc., has revenue of $455,316, costs of $316,487, and a tax rate of 31 percent. If the firm pays out 45 percent of its earnings as dividends every year, how much earnings are retained and w...

See Answer

Q: Cattail Corporation’s financial statements for the fiscal year just ended are shown

Cattail Corporation’s financial statements for the fiscal year just ended are shown below: Cattail management expects sales to increase by 14 percent next year. Assume that the fin...

See Answer

Q: What are bankruptcy costs, and what are the two types of

What are bankruptcy costs, and what are the two types of bankruptcy costs?

See Answer

Q: Is it always possible to estimate the value of a real option

Is it always possible to estimate the value of a real option? Why or why not?

See Answer

Q: What is an option?

What is an option?

See Answer

Q: Suppose you have an option to buy a share of ABC Corp

Suppose you have an option to buy a share of ABC Corp. stock for $100. The option expires tomorrow, and the current price of ABC Corp. is $95. How much is your option worth?

See Answer

Q: NetSpeed Technologies is a telecom component manufacturer. The firm typically has

NetSpeed Technologies is a telecom component manufacturer. The firm typically has a collection period of 44 days and days’ sales in inventory of 29 days. What is the operating cycle for NetSpeed?

See Answer

Q: You hold an American option to sell one share of Zyther Co

You hold an American option to sell one share of Zyther Co. stock. The option expires tomorrow. The strike price of the option is $50, and the current stock price is $49. What is the value of exercisi...

See Answer

Q: What is the difference between a financial option and a real option

What is the difference between a financial option and a real option?

See Answer

Q: A callable bond is a bond that can be bought back by

A callable bond is a bond that can be bought back by the bond issuer before maturity for some pre-specified price (normally a small amount above face value) at the discretion of the bond issuer. How w...

See Answer

Q: The seller of an option can never make any money from a

The seller of an option can never make any money from a change in the value of the underlying asset; he or she can only hope that the option will not be exercised and that he or she will not lose any...

See Answer

Q: Again assume that the price of Socrates Motors stock will either rise

Again assume that the price of Socrates Motors stock will either rise to $50 or fall to $35 in one month and that the risk-free rate for one month is 1.5 percent. How much is an option with a strike p...

See Answer

Q: The payoff function for the holder of straight debt looks like that

The payoff function for the holder of straight debt looks like that for the seller of a put option. Convertible debt is straight debt plus a call option on a firm’s stock. How does the addition of a c...

See Answer

Q: What is the payoff for a put option with a strike price

What is the payoff for a put option with a strike price of $50 if the stock price at expiration is $40? What if the stock price is $65?

See Answer

Q: What do the payoff functions for stockholders and lenders look like?

What do the payoff functions for stockholders and lenders look like?

See Answer

Q: What are the five variables that affect the value of an option

What are the five variables that affect the value of an option, and how do changes in each of these variables affect the value of a call option?

See Answer

Q: Devlin Construction Inc., reported the following balance sheet information for the

Devlin Construction Inc., reported the following balance sheet information for the last fiscal year. Devlin also reported net sales of $980,770 and days sales in inventory of 48.90 days. Devlin Co...

See Answer

Q: You finally decide to act on your brilliant idea and start an

You finally decide to act on your brilliant idea and start an online textbook rental company. You develop a detailed business plan and calculate that you will need about $350,000 of initial funding to...

See Answer

Q: Briefly describe the IPO process?

Briefly describe the IPO process?

See Answer

Q: FAJ, Inc. has $500 million of debt outstanding at

FAJ, Inc. has $500 million of debt outstanding at an interest rate of 9 percent. What is the present value of the tax shield on that debt if it has no maturity and if FAJ is subject to a 30 percent ma...

See Answer

Q: What are six factors that cause international transactions to differ from domestic

What are six factors that cause international transactions to differ from domestic transactions?

See Answer

Q: A firm is making an initial public offering. The investment bankers

A firm is making an initial public offering. The investment bankers agree to a firm underwriting commitment for 500,000 shares that would be priced to the public at $36 a share. The underwriter’s spre...

See Answer

Q: Your boss at Box and Freight Company asks you how much additional

Your boss at Box and Freight Company asks you how much additional debt the company would have to add through a capital restructuring in order to create $9 million in present value from the resulting i...

See Answer

Q: The required rate of return on the assets of a firm is

The required rate of return on the assets of a firm is 12 percent, the firm has a debt-to-common-stock ratio of 40 percent, and the cost of debt is 6 percent. If the firm has no preferred stock and th...

See Answer

Q: Rosemary Corporation has daily sales of $139,000. The

Rosemary Corporation has daily sales of $139,000. The financial manager at the firm has determined that a lockbox would reduce collection time by 2.2 days. Assuming the company can earn 5.5 percent in...

See Answer

Q: What does the payoff function for a typical manager look like?

What does the payoff function for a typical manager look like?

See Answer

Q: The market value of Whole Foods stock is currently $53.

The market value of Whole Foods stock is currently $53.73 per share, and the annual risk-free rate is 3 percent. A three-month call option on the stock with a strike price of $55 sells for $2.15. What...

See Answer

Q: If a Dell Studio laptop sells for $999 in Austin,

If a Dell Studio laptop sells for $999 in Austin, Texas and £689 in London, what is the implied exchange rate between the U.S. dollar and the euro?

See Answer

Q: Refer to Exhibits 19.10 and 19.11 in the

Refer to Exhibits 19.10 and 19.11 in the text. The EFN for several growth rates for Empire Enterprises are as follows: Growth Rate (%) ………….. EFN ($ millions) 0% ………………...

See Answer

Q: What is hedging?

What is hedging?

See Answer

Q: Why is the repatriation of cash flows from an overseas project considered

Why is the repatriation of cash flows from an overseas project considered critical to the project’s value?

See Answer

Q: What difficulties do firms face in estimating cash flows from an overseas

What difficulties do firms face in estimating cash flows from an overseas project?

See Answer

Q: What is the difference between foreign bonds and Eurobonds?

What is the difference between foreign bonds and Eurobonds?

See Answer

Q: Which currency is the preferred currency of exchange in global financial markets

Which currency is the preferred currency of exchange in global financial markets? Why?

See Answer

Q: Why is credit risk higher in international markets?

Why is credit risk higher in international markets?

See Answer

Q: List the inputs that are used in calculating a Eurocredit price?

List the inputs that are used in calculating a Eurocredit price?

See Answer

Q: What are agency costs, and how are they related to the

What are agency costs, and how are they related to the use of debt financing?

See Answer

Q: Use the financial information for Starlight from Problem 19.1.

Use the financial information for Starlight from Problem 19.1. Assume now that equity accounts do not vary directly with sales but change when retained earning change or new equity is issued. The comp...

See Answer

Q: Use the financial statements from Problem 16.1 and the information

Use the financial statements from Problem 16.1 and the information from Problem 16.2 to calculate the company’s retention(plowback) ratio, external funds needed (EFN), internal growt...

See Answer

Q: The stock of Augusta Light and Power is currently selling at $

The stock of Augusta Light and Power is currently selling at $12 per share. Over the next year the company is undertaking a new electricity production project. If the project is successful, the compan...

See Answer

Q: ADCAP International is a U.S.-based company which sells

ADCAP International is a U.S.-based company which sells its products primarily in overseas markets. The company’s stock is currently trading at $50 per share. Depending on the outcome of U.S. trade ne...

See Answer

Q: Your company is considering opening a new factory in Europe to serve

Your company is considering opening a new factory in Europe to serve the growing demand for your product there. What real options might you want to consider in your capital budgeting analysis of the f...

See Answer

Q: Your firm, which uses oil as an input to its production

Your firm, which uses oil as an input to its production processes, hedges its exposure to changes in the price of oil by buying call options on oil at today’s price. If the price of oil goes down by t...

See Answer

Q: Digital, Inc., an electronic games manufacturer, is planning to

Digital, Inc., an electronic games manufacturer, is planning to purchase flash memory from one of two sources. Kyoto, Inc., quotes a price of ¥6,800 per gigabyte. The current exchange rate is ¥102.30/...

See Answer

Q: You are provided the following working capital information for the Blue Ridge

You are provided the following working capital information for the Blue Ridge Company: If all sales are made on credit, what are the firm’s operating and cash conversion cycles?

See Answer

Q: Merrifield Cosmetics calculates that its operating cycle for last year was 76

Merrifield Cosmetics calculates that its operating cycle for last year was 76 days. The company had $230,000 in its accounts receivable account and had sales of $1.92 million. Approximately how many...

See Answer

Q: Below is a partial aging of accounts receivable for Bitar Roofing Services

Below is a partial aging of accounts receivable for Bitar Roofing Services. Fill in the rest of the information and determine Bitar’s days’ sales outstanding. How d...

See Answer

Q: What is the pecking order theory of capital structure?

What is the pecking order theory of capital structure?

See Answer

Q: By obtaining a lockbox, Nizam’s Manufacturing was able to reduce its

By obtaining a lockbox, Nizam’s Manufacturing was able to reduce its total cash collection time by two days. The firm has annual sales of $570,000 and can earn 4.75 percent annual interest. Assuming t...

See Answer

Q: Rockville Corporation is going to borrow $250,000 from its

Rockville Corporation is going to borrow $250,000 from its bank at an APR of 8.5 percent. The bank requires its customers to maintain a 10 percent compensating balance. What is the effective interest...

See Answer

Q: Are taxes necessary for the cost of debt financing to be less

Are taxes necessary for the cost of debt financing to be less than the cost of equity financing?

See Answer

Q: You are offered jobs with identical responsibilities by two different firms in

You are offered jobs with identical responsibilities by two different firms in the same industry. One has no debt in its capital structure, and the other has 99 percent debt in its capital structure....

See Answer

Q: You are valuing two otherwise identical firms in the same industry.

You are valuing two otherwise identical firms in the same industry. One firm has a corporate jet for every executive at the vice president level and above, while the other does not have a single corpo...

See Answer

Q: The Starlight, Inc. financial statements for the fiscal year ended

The Starlight, Inc. financial statements for the fiscal year ended June 30, 2017, are presented below. The firm’s sales are projected to grow at a rate of 20 percent next year, and a...

See Answer

Q: Consider the WACC for a firm that pays taxes. Explain what

Consider the WACC for a firm that pays taxes. Explain what a firm’s best course of action would be to minimize its WACC and thereby maximize the firm value. Use the WACC formula for your explanation?...

See Answer

Q: The Modigliani and Miller propositions, when the no-tax assumption

The Modigliani and Miller propositions, when the no-tax assumption is relaxed, suggest that the firm should finance itself with as much debt as possible. Taking this suggestion to the extreme, is it e...

See Answer

Q: Crossler Automobiles sells autos in a market where the standard auto comes

Crossler Automobiles sells autos in a market where the standard auto comes with a 10-year/100,000-mile warranty on all parts and labor. Describe how an increased probability of bankruptcy could affect...

See Answer

Q: Agency problems occur because the nonowner managers and stockholders of a firm

Agency problems occur because the nonowner managers and stockholders of a firm have different interests. Propose a capital structure change that might help better align these different interests?

See Answer

Q: What does the empirical evidence tell us about the two theories?

What does the empirical evidence tell us about the two theories?

See Answer

Q: If a firm increases its debt to a very high level,

If a firm increases its debt to a very high level, then the positive effect of debt in aligning the interests of management with those of stockholders tends to become negative? Explain why this occurs...

See Answer

Q: Using the Modigliani and Miller framework but excluding the assumptions that there

Using the Modigliani and Miller framework but excluding the assumptions that there are no taxes and no information or transaction costs, describe the value of the firm as a function of the proportion...

See Answer

Q: When we observe the capital structure of many firms, we find

When we observe the capital structure of many firms, we find that they tend to utilize lower levels of debt than that predicted by the trade-off theory. Offer an explanation for this?

See Answer

Q: List and briefly describe the three key assumptions in Modigliani and Miller’s

List and briefly describe the three key assumptions in Modigliani and Miller’s Proposition 1 that are required for total firm value to be independent of capital structure?

See Answer

Q: Evaluate the statement that the weighted average cost of capital (WACC

Evaluate the statement that the weighted average cost of capital (WACC) for a firm (assuming that all three assumptions of Modigliani and Miller’s propositions hold) is always less than or equal to th...

See Answer

Q: If the value of the firm remains constant as a function of

If the value of the firm remains constant as a function of its capital structure and the three Modigliani and Miller assumptions apply, why might the overall cost of capital change or not change as ca...

See Answer

Q: How are customers and suppliers affected by a firm’s working capital management

How are customers and suppliers affected by a firm’s working capital management decisions?

See Answer

Q: A beverage bottling company in Vermont has days’ sales outstanding of 23

A beverage bottling company in Vermont has days’ sales outstanding of 23.7 days. Is this good? Explain?

See Answer

Q: How do the following circumstances affect the cash conversion cycle:

How do the following circumstances affect the cash conversion cycle: (a) favorable credit terms allow the firm to pay its accounts payable slower, (b) inventory turnover increases, (c) accounts receiv...

See Answer

Q: What are some industries in which the use of lockboxes would especially

What are some industries in which the use of lockboxes would especially benefit companies? Explain.

See Answer

Q: Why is financial flexibility important in the choice of a capital structure

Why is financial flexibility important in the choice of a capital structure?

See Answer

Q: Suppose you are a financial manager at a big firm and you

Suppose you are a financial manager at a big firm and you expect interest rates to decline in the near future. What current asset investment strategy would you recommend that the company pursue?

See Answer

Q: Why is the commercial paper market available only to the most creditworthy

Why is the commercial paper market available only to the most creditworthy companies?

See Answer

Q: Explain what a negative cash conversion cycle means?

Explain what a negative cash conversion cycle means?

See Answer

Q: Assume you work for a venture capital firm and have been approached

Assume you work for a venture capital firm and have been approached by a couple of recent college graduates with a request to fund their new business. If you are interested in the idea, what process w...

See Answer

Q: Managers at a large firm are looking for a medium-size

Managers at a large firm are looking for a medium-size loan with a long term to maturity and low liquidity. Which of the following types of debt would be the most appropriate? a. Public bond. b. Priva...

See Answer

Q: Identify the three basic services investment bankers provide to help firms bring

Identify the three basic services investment bankers provide to help firms bring new security issues to the market. During which stage of the typical IPO does the investment banker take on the risk of...

See Answer

Q: Define underpricing, and explain why the majority of IPOs are underpriced

Define underpricing, and explain why the majority of IPOs are underpriced. What role do investment banks play in the price-setting process?

See Answer

Q: Explain why the owners of a company might choose to keep it

Explain why the owners of a company might choose to keep it private?

See Answer

Q: Identify the three cost components that make up the total cost of

Identify the three cost components that make up the total cost of issuing securities for a company. Briefly describe each?

See Answer

Q: What are the characteristics of a public bond? (Think in

What are the characteristics of a public bond? (Think in terms of comparing it to private placement and bank term loans.)

See Answer

Q: How can capital structure decisions affect the control of a firm?

How can capital structure decisions affect the control of a firm?

See Answer

Q: Discuss the advantages of shelf registration. What kinds of securities are

Discuss the advantages of shelf registration. What kinds of securities are most likely to be registered this way?

See Answer

Q: Identify whether each of the following factors implies a lower or higher

Identify whether each of the following factors implies a lower or higher price for a bond? a. Low marketability of the security. b. Short term to maturity. c. Low credit rating of the issuer. d. No ca...

See Answer

Q: What is financial planning? What four types of plans/budgets

What is financial planning? What four types of plans/budgets are involved in financial planning?

See Answer

Q: What is the sustainable growth rate? Why is it important?

What is the sustainable growth rate? Why is it important?

See Answer

Q: Explain how sales can be used to develop pro forma financial statements

Explain how sales can be used to develop pro forma financial statements?

See Answer

Q: Why is sales not always a good measure to use in forecasting

Why is sales not always a good measure to use in forecasting fixed assets?

See Answer

Q: Explain why the fixed asset account may or may not vary with

Explain why the fixed asset account may or may not vary with sales?

See Answer

Q: How does the dividend payout ratio affect the amount of funds needed

How does the dividend payout ratio affect the amount of funds needed to finance growth?

See Answer

Q: Define internal growth rate (IGR). Identify the characteristics of a

Define internal growth rate (IGR). Identify the characteristics of a high-growth firm that has no external funds needed?

See Answer

Q: Options can be combined to create more complicated payoff structures. Consider

Options can be combined to create more complicated payoff structures. Consider the combination of one put option and one call option with the same expiration date and the same strike price. Draw the p...

See Answer

Q: In 2014, Japanese liquor company Suntory Holdings Ltd. acquired U

In 2014, Japanese liquor company Suntory Holdings Ltd. acquired U.S. bourbon maker Beam Inc., for $14 billion. Is this a horizontal or vertical acquisition? How do you suppose Beam’s nationality affec...

See Answer

Q: For each of the following scenarios, discuss whether profit opportunities exist

For each of the following scenarios, discuss whether profit opportunities exist from trading in the stock of the firm under the conditions that (1) the market is not weak form efficient, (2) the mark...

See Answer

Q: In 2016, automobile manufacturer BMW completed its $1 billion investment

In 2016, automobile manufacturer BMW completed its $1 billion investment to increase production at its South Carolina plant by 50 percent. BMW apparently felt that it would be better able to compete a...

See Answer

Q: In the chapter, we gave several examples of so-called

In the chapter, we gave several examples of so-called payday loans. As you saw, the interest rates on these loans can be extremely high and are even called predatory by some. Do you think such high in...

See Answer

Q: In the chapter, we mentioned that many companies have been under

In the chapter, we mentioned that many companies have been under pressure to declassify their boards of directors. Why would investors want a board to be declassified? What are the advantages of a cla...

See Answer

Q: What are the difficulties in using the PE ratio to value stock

What are the difficulties in using the PE ratio to value stock?

See Answer

Q: You are at work when a co-worker excitedly comes to

You are at work when a co-worker excitedly comes to your desk and shows you the scenario analysis that he has just completed for a potential new project. All three scenarios show a positive NPV. He st...

See Answer

Q: With some insurance policies, the value of the asset to be

With some insurance policies, the value of the asset to be replaced is the current market value. For example, if you bought a couch five years ago for $1,000, and the current value of the couch is $30...

See Answer

Q: In 2009 and 2010, investment banks were under fire for interest

In 2009 and 2010, investment banks were under fire for interest rate swaps sold to municipalities and nonprofits. For example, California’s water resource authority paid about $305 million, North Caro...

See Answer

Q: You have been at your job with S&S Air for

You have been at your job with S&S Air for a week now and have decided you need to sign up for the company’s 401(k) plan. Even after your discussion with Audrey Sanborn, the Bledsoe Financial Services...

See Answer

Q: For initial public offerings of common stock, 2017 was a slow

For initial public offerings of common stock, 2017 was a slow year, with about $24.53 billion raised by the process. Relatively few of the 108 firms involved paid cash dividends. Why do you think that...

See Answer

Q: Define each of the following terms: a. Greenmail

Define each of the following terms: a. Greenmail b. White knight c. Golden parachute d. Crown jewels e. Shark repellent f. Corporate raider g. Poison pill h. Tender offer i. Leveraged buyout (LBO)

See Answer

Q: How could overconfidence affect the financial manager of the firm and the

How could overconfidence affect the financial manager of the firm and the firm’s shareholders?

See Answer

Q: In February 2017, Air Lease Corporation (ALC) announced a

In February 2017, Air Lease Corporation (ALC) announced a deal to lease five new Boeing 787-9 passenger aircraft to China Southern Airlines. ALC had the aircraft on purchase order from Boeing and plan...

See Answer

Q: The following material represents the cover page and summary of the prospectus

The following material represents the cover page and summary of the prospectus for the initial public offering of the Pest Investigation Control Corporation (PICC), which is going public tomorrow with...

See Answer

Q: In the chapter, we presented an example where you have lost

In the chapter, we presented an example where you have lost $78 and are given the opportunity to make a wager in which you have an 80 percent chance that your loss will increase to $100 and a 20 perce...

See Answer

Q: Acquiring firm stockholders seem to benefit very little from takeovers. Why

Acquiring firm stockholders seem to benefit very little from takeovers. Why is this finding a puzzle? What are some of the reasons offered in explanation?

See Answer

Q: Natural resource extraction facilities (such as oil wells or gold mines

Natural resource extraction facilities (such as oil wells or gold mines) provide a good example of the value of the option to suspend operations. Why?

See Answer

Q: You own stock in the Hendrix Guitar Company. The company has

You own stock in the Hendrix Guitar Company. The company has implemented a plan to award employee stock options. As a shareholder, does the plan benefit you? If so, what are the benefits?

See Answer

Q: Given that AK Steel was up by about 359 percent for 2016

Given that AK Steel was up by about 359 percent for 2016, why didn’t all investors hold this stock?

See Answer

Q: What impact does an increase in the volatility of the underlying stock’s

What impact does an increase in the volatility of the underlying stock’s return have on an option’s value? Explain.

See Answer

Q: The protective put strategy we discussed in the chapter is sometimes referred

The protective put strategy we discussed in the chapter is sometimes referred to as stock price insurance. Why?

See Answer

Q: Suppose the exchange rate for the Swiss franc is quoted as SF

Suppose the exchange rate for the Swiss franc is quoted as SF 1.50 in the spot market and SF 1.53 in the 90-day forward market. a. Is the dollar selling at a premium or a discount relative to the fran...

See Answer

Q: Shank Manufacturing, Inc., has recently installed a just-in

Shank Manufacturing, Inc., has recently installed a just-in-time (JIT) inventory system. Describe the effect this is likely to have on the company’s carrying costs, shortage costs, and operating cycle...

See Answer

Q: Suppose you are looking at a company with no change in capital

Suppose you are looking at a company with no change in capital spending, no change in net working capital, and no depreciation. Since the company is not increasing its assets, EBIT is constant. What i...

See Answer

Q: Take a look back at the times interest earned (TIE)

Take a look back at the times interest earned (TIE) ratio we discussed in Chapter 3. For interest paid to be fully deductible by a company after the Tax Cuts and Jobs Act of 2017, what must be true ab...

See Answer

Q: What is the value of a put option at maturity? Based

What is the value of a put option at maturity? Based on your answer, what is the intrinsic value of a put option?

See Answer

Q: In evaluating the Cayenne, would you use the term erosion to

In evaluating the Cayenne, would you use the term erosion to describe the possible damage to Porsche’s reputation?

See Answer

Q: Porsche was one of the last manufacturers to enter the sports utility

Porsche was one of the last manufacturers to enter the sports utility vehicle market. Why would one company decide to proceed with a product when other companies, at least initially, decide not to ent...

See Answer

Q: In evaluating the Cayenne, what do you think Porsche needs to

In evaluating the Cayenne, what do you think Porsche needs to assume regarding the substantial profit margins that exist in this market? Is it likely they will be maintained as the market becomes more...

See Answer

Q: In February 2017, Air Lease Corporation (ALC) announced a

In February 2017, Air Lease Corporation (ALC) announced a deal to lease five new Boeing 787-9 passenger aircraft to China Southern Airlines. ALC had the aircraft on purchase order from Boeing and plan...

See Answer

Q: In February 2017, Air Lease Corporation (ALC) announced a

In February 2017, Air Lease Corporation (ALC) announced a deal to lease five new Boeing 787-9 passenger aircraft to China Southern Airlines. ALC had the aircraft on purchase order from Boeing and plan...

See Answer

Q: In Chapter 10, we discussed Porsche’s launch of its new Cayenne

In Chapter 10, we discussed Porsche’s launch of its new Cayenne. Suppose sales of the Cayenne go extremely well and Porsche is forced to expand output to meet demand. Porsche’s action in this case wou...

See Answer

Q: In 2016, activist investor Elliott Management was pressuring Marathon Petroleum to

In 2016, activist investor Elliott Management was pressuring Marathon Petroleum to split up the company. Elliott felt that Marathon should sell off its Speedway gas and retail stores. Why might invest...

See Answer

Q: Historically, the U.S. tax code treated dividend payments

Historically, the U.S. tax code treated dividend payments made to shareholders as ordinary income. Thus, dividends were taxed at the investor’s marginal tax rate, which was as high as 38.6 percent in...

See Answer

Q: Suppose you own a home that costs $200,000 and

Suppose you own a home that costs $200,000 and you buy homeowner’s insurance to cover your house against fire, wind, tornados, and other disasters. One way to view your insurance is that you purchased...

See Answer

Q: Suppose a firm has a book balance of $2 million.

Suppose a firm has a book balance of $2 million. On the bank’s website, the cash manager finds out that the bank balance is $2.5 million. What is the situation here? If this is an ongoing situation, w...

See Answer

Q: Historically, the U.S. tax code treated dividend payments

Historically, the U.S. tax code treated dividend payments made to shareholders as ordinary income. Thus, dividends were taxed at the investor’s marginal tax rate, which was as high as 38.6 percent in...

See Answer

Q: What is the option to abandon? Explain why we underestimate NPV

What is the option to abandon? Explain why we underestimate NPV if we ignore this option.

See Answer

Q: What is the option to expand? Explain why we underestimate NPV

What is the option to expand? Explain why we underestimate NPV if we ignore this option.

See Answer

Q: We discussed five international capital market relationships: relative PPP, IRP

We discussed five international capital market relationships: relative PPP, IRP, UFR, UIP, and the international Fisher effect. Which of these would you expect to hold most closely?

See Answer

Q: Some people argue that the efficient markets hypothesis cannot explain the 1987

Some people argue that the efficient markets hypothesis cannot explain the 1987 market crash or the high price-to-earnings ratio of Internet stocks during the late 1990s. What alternative hypothesis i...

See Answer

Q: Proponents of behavioral finance use three concepts to argue that markets are

Proponents of behavioral finance use three concepts to argue that markets are not efficient. What are these arguments?

See Answer

Q: What does the lack of transparency in the bond market imply for

What does the lack of transparency in the bond market imply for bond investors?

See Answer

Q: Discuss the accounting criteria for determining whether or not a lease must

Discuss the accounting criteria for determining whether or not a lease must be reported on the balance sheet using the accounting rules in place before 2019. In each case, give a rationale for the cri...

See Answer

Q: Often, junk bonds are not rated. Why?

Often, junk bonds are not rated. Why?

See Answer

Q: Common advice on Wall Street is “Keep your alpha high and

Common advice on Wall Street is “Keep your alpha high and your beta low.” Why?

See Answer

Q: Some companies, such as Alphabet, have created classes of stock

Some companies, such as Alphabet, have created classes of stock with no voting rights at all. Why would investors buy such stock?

See Answer

Q: Referring back to the Boeing example used at the beginning of the

Referring back to the Boeing example used at the beginning of the chapter, note that we suggested that Boeing’s stockholders probably didn’t suffer as a result of the reported loss. What do you think...

See Answer

Q: Discuss the IRS criteria for determining whether or not a lease is

Discuss the IRS criteria for determining whether or not a lease is tax deductible. In each case, give a rationale for the criterion.

See Answer

Q: You are CEO of Titan Industries and have just been awarded a

You are CEO of Titan Industries and have just been awarded a large number of employee stock options. The company has two mutually exclusive projects available. The first project has a large NPV and wi...

See Answer

Q: Specialized ratios are sometimes used in specific industries. For example,

Specialized ratios are sometimes used in specific industries. For example, the so-called book-to-bill ratio is closely watched for semiconductor manufacturers. A ratio of .93 indicates that for every...

See Answer

Q: In our discussion of the SML, we defined alpha. What

In our discussion of the SML, we defined alpha. What does alpha measure? What alpha would you like to see on your investments?

See Answer

Q: Bismark Co. is in the process of considering a change

Bismark Co. is in the process of considering a change in its terms of sale. The current policy is cash only; the new policy will involve one period’s credit. Sales are 25,000 units per period at a pri...

See Answer

Q: The Wildcat Oil Company is trying to decide whether to

The Wildcat Oil Company is trying to decide whether to lease or buy a new computer-assisted drilling system for its oil exploration business. Management has decided that it must use the system to stay...

See Answer

Q: The most recent financial statements for Crosby, Inc., follow. Sales

The most recent financial statements for Crosby, Inc., follow. Sales for 2018 are projected to grow by 20 percent. Interest expense will remain constant; the tax rate and the dividend payout rate will...

See Answer

Q: In the previous problem, suppose the firm was operating at

In the previous problem, suppose the firm was operating at only 80 percent capacity in 2017. What is EFN now?Data from Problem 24The most recent financial statements for Crosby, Inc., follow. Sales fo...

See Answer

Q: In Problem 24, suppose the firm wishes to keep its

In Problem 24, suppose the firm wishes to keep its debt-equity ratio constant. What is EFN now?Data from Problem 24:The most recent financial statements for Crosby, Inc., follow. Sales for 2018 are pr...

See Answer

Q: Redo Problem 24 using sales growth rates of 15 and

Redo Problem 24 using sales growth rates of 15 and 25 percent in addition to 20 percent. Illustrate graphically the relationship between EFN and the growth rate, and use this graph to determine the re...

See Answer

Q: Redo Problem 26 using sales growth rates of 30 and

Redo Problem 26 using sales growth rates of 30 and 35 percent in addition to 20 percent. Assume the firm wishes to maintain its debt-equity ratio. Illustrate graphically the relationship between EFN a...

See Answer

Q: Sig, Inc., wishes to maintain a growth rate of 12

Sig, Inc., wishes to maintain a growth rate of 12 percent per year and a debt-equity ratio of .43. The profit margin is 5.9 percent, and the ratio of total assets to sales is constant at 1.80. Is this...

See Answer

Q: The most recent financial statements for Kerch, Inc., are shown

The most recent financial statements for Kerch, Inc., are shown here (assuming no income taxes):

See Answer

Q: What is Tobin’s Q for Smolira Golf? What assumptions are

What is Tobin’s Q for Smolira Golf? What assumptions are you making about the book value of debt and the market value of debt? What about the book value of assets and the market valu...

See Answer

Q: Based on the result in Problem 30, show that the internal and sustainable

Based on the result in Problem 30, show that the internal and sustainable growth rates are as given in the chapter. Hint: For the internal growth rate, set EFN equal to zero and solve for g. Data...

See Answer

Q: In the chapter, we discussed the two versions of the

In the chapter, we discussed the two versions of the sustainable growth rate formula. Derive the formula ROE × b from the formula given in the chapter, where ROE is based on beginning of period equity...

See Answer

Q: An asset costs $630,000 and will be depreciated in a

An asset costs $630,000 and will be depreciated in a straight-line manner over its three-year life. It will have no salvage value. The lessor can borrow at 6 percent and the lessee can borrow at 9 per...

See Answer

Q: Klingon Widgets, Inc., purchased new cloaking machinery three years ago

Klingon Widgets, Inc., purchased new cloaking machinery three years ago for $6 million. The machinery can be sold to the Romulans today for $5.1 million. Klingon’s current balance sheet shows net fixe...

See Answer

Q: Square Hammer Corp. shows the following information on its 2018

Square Hammer Corp. shows the following information on its 2018 income statement: Sales = $305,000; Costs = $176,000; Other expenses = $8,900; Depreciation expense = $18,700; Interest expense = $12,90...

See Answer

Q: Prepare a 2018 balance sheet for Rogers Corp. based on

Prepare a 2018 balance sheet for Rogers Corp. based on the following information: Cash = $127,000; Patents and copyrights = $660,000; Accounts payable = $210,000; Accounts receivable = $115,000; Tangi...

See Answer

Q: Bishop, Inc., is obligated to pay its creditors $7,800 during

Bishop, Inc., is obligated to pay its creditors $7,800 during the year. a. What is the market value of the shareholders’ equity if assets have a market value of $9,400? b. What if assets equal $6,700?

See Answer

Q: During 2018, Raines Umbrella Corp. had sales of $705,000. Cost

During 2018, Raines Umbrella Corp. had sales of $705,000. Cost of goods sold, administrative and selling expenses, and depreciation expenses were $445,000, $95,000, and $140,000, respectively. In addi...

See Answer

Q: In Problem 16, suppose Raines Umbrella Corp. paid out $102,000

In Problem 16, suppose Raines Umbrella Corp. paid out $102,000 in cash dividends. Is this possible? If net capital spending and net working capital were both zero, and if no new stock was issued durin...

See Answer

Q: The most recent financial statements for Cardinal, Inc., are shown

The most recent financial statements for Cardinal, Inc., are shown here:

See Answer

Q: Cardinal Industries had the following operating results for 2018: Sales

Cardinal Industries had the following operating results for 2018: Sales = $33,106; Cost of goods sold = $23,624; Depreciation expense = $5,877; Interest expense = $2,650; Dividends paid = $1,888. At t...

See Answer

Q: Consider the following abbreviated financial statements for Parrothead Enterprises:

Consider the following abbreviated financial statements for Parrothead Enterprises:a. What is owners’ equity for 2017 and 2018?b. What is the change in net working capital for 2018?c...

See Answer

Q: Griffin’s Goat Farm, Inc., has sales of $796,000, costs of

Griffin’s Goat Farm, Inc., has sales of $796,000, costs of $327,000, depreciation expense of $42,000, interest expense of $34,000, and a tax rate of 21 percent. What is the net income for this firm?

See Answer

Q: Automobiles are often leased, and several terms are unique to

Automobiles are often leased, and several terms are unique to auto leases. Suppose you are considering leasing a car. The price you and the dealer agree on for the car is $41,900. This is the base cap...

See Answer

Q: On the balance sheet, the net fixed assets (NFA) account

On the balance sheet, the net fixed assets (NFA) account is equal to the gross fixed assets (FA) account (which records the acquisition cost of fixed assets) minus the accumulated depreciation (AD) ac...

See Answer

Q: Draw up an income statement and balance sheet for this

Draw up an income statement and balance sheet for this company for 2017 and 2018

See Answer

Q: For 2018, calculate the cash flow from assets, cash flow

For 2018, calculate the cash flow from assets, cash flow to creditors, and cash flow to stockholders

See Answer

Q: Suppose the firm in Problem 2 paid out $95,000 in

Suppose the firm in Problem 2 paid out $95,000 in cash dividends. What is the addition to retained earnings?Data from Problem 2:Griffin’s Goat Farm, Inc., has sales of $796,000, costs of $327,000, dep...

See Answer

Q: Suppose the firm in Problem 3 had 80,000 shares of

Suppose the firm in Problem 3 had 80,000 shares of common stock outstanding. What is the earnings per share, or EPS, figure? What is the dividends per share figure?Problem 3:Suppose the firm in Proble...

See Answer

Q: Pompeii, Inc., has sales of $46,200, costs of $23,100, depreciation

Pompeii, Inc., has sales of $46,200, costs of $23,100, depreciation expense of $2,200, and interest expense of $1,700. If the tax rate is 22 percent, what is the operating cash flow, or OCF?

See Answer

Q: Logano Driving School’s 2017 balance sheet showed net fixed assets

Logano Driving School’s 2017 balance sheet showed net fixed assets of $2.4 million, and the 2018 balance sheet showed net fixed assets of $3.3 million. The company’s 2018 income statement showed a dep...

See Answer

Q: The most recent financial statements for Assouad, Inc., are shown

The most recent financial statements for Assouad, Inc., are shown here:

See Answer

Q: The 2017 balance sheet of Dream, Inc., showed current assets

The 2017 balance sheet of Dream, Inc., showed current assets of $4,810 and current liabilities of $2,230. The 2018 balance sheet showed current assets of $5,360 and current liabilities of $2,970. What...

See Answer

Q: The 2017 balance sheet of Kerber’s Tennis Shop, Inc., showed

The 2017 balance sheet of Kerber’s Tennis Shop, Inc., showed long-term debt of $1.87 million, and the 2018 balance sheet showed long-term debt of $2.21 million. The 2018 income statement showed an int...

See Answer

Q: Return to the case of the diagnostic scanner used in

Return to the case of the diagnostic scanner used in Problems 1 through 6. Suppose the entire $4,800,000 purchase price of the scanner is borrowed. The rate on the loan is 8 percent, and the loan will...

See Answer

Q: The 2017 balance sheet of Kerber’s Tennis Shop, Inc., showed

The 2017 balance sheet of Kerber’s Tennis Shop, Inc., showed $650,000 in the common stock account and $3.98 million in the additional paid-in surplus account. The 2018 balance sheet showed $805,000 an...

See Answer

Q: SDJ, Inc., has net working capital of $2,170, current liabilities

SDJ, Inc., has net working capital of $2,170, current liabilities of $4,590, and inventory of $3,860. What is the current ratio? What is the quick ratio?

See Answer

Q: Heritage, Inc., had a cost of goods sold of $68,314.

Heritage, Inc., had a cost of goods sold of $68,314. At the end of the year, the accounts payable balance was $15,486. How long on average did it take the company to pay off its suppliers during the y...

See Answer

Q: The market value of the equity of Hudgins, Inc., is

The market value of the equity of Hudgins, Inc., is $645,000. The balance sheet shows $53,000 in cash and $215,000 in debt, while the income statement has EBIT of $91,000 and a total of $157,000 in de...

See Answer

Q: SME Company has a debt-equity ratio of .57. Return on

SME Company has a debt-equity ratio of .57. Return on assets is 7.9 percent, and total equity is $620,000. What is the equity multiplier? Return on equity? Net income?

See Answer

Q: Prepare the 2017 and 2018 common-size balance sheets for Just

Prepare the 2017 and 2018 common-size balance sheets for Just Dew It.

See Answer

Q: Prepare the 2018 common-base year balance sheet for Just Dew

Prepare the 2018 common-base year balance sheet for Just Dew It.

See Answer

Q: Prepare the 2018 combined common-size, common-base year balance sheet for

Prepare the 2018 combined common-size, common-base year balance sheet for Just Dew It.

See Answer

Q: The most recent financial statements for Bello Co. are shown

The most recent financial statements for Bello Co. are shown here:

See Answer

Q: For each account on this company’s balance sheet, show the

For each account on this company’s balance sheet, show the change in the account during 2018 and note whether this change was a source or use of cash. Do your numbers add up and make...

See Answer

Q: Harrods PLC has a market value of £95 million and

Harrods PLC has a market value of £95 million and 4.5 million shares outstanding. Selfridge Department Store has a market value of £32 million and 1.8 million shares outstanding. Harrods is contemplat...

See Answer

Q: Based on the balance sheets given for Just Dew It,

Based on the balance sheets given for Just Dew It, calculate the following financial ratios for each year: b. Quick ratio.c. Cash ratio.d. NWC to total assets ratioe. Debt-equity ratio and equity mult...

See Answer

Q: Y3K, Inc., has sales of $6,183, total assets of $2,974,

Y3K, Inc., has sales of $6,183, total assets of $2,974, and a debt-equity ratio of .57. If its return on equity is 11 percent, what is its net income?

See Answer

Q: A company has net income of $196,500, a profit margin

A company has net income of $196,500, a profit margin of 6.8 percent, and an accounts receivable balance of $119,630. Assuming 65 percent of sales are on credit, what is the company’s days’ sales in r...

See Answer

Q: DTO, Inc., has sales of $16.7 million, total assets of

DTO, Inc., has sales of $16.7 million, total assets of $12.9 million, and total debt of $5.7 million. If the profit margin is 5 percent, what is net income? What is ROA? What is ROE?

See Answer

Q: The Maurer Company has a long-term debt ratio of .35

The Maurer Company has a long-term debt ratio of .35 and a current ratio of 1.30. Current liabilities are $955, sales are $7,210, profit margin is 8.3 percent, and ROE is 17.5 percent. What is the amo...

See Answer

Q: In response to complaints about high prices, a grocery chain

In response to complaints about high prices, a grocery chain runs the following advertising campaign: “If you pay your child $1.50 to go buy $50 worth of groceries, then your child makes twice as much...

See Answer

Q: Firm A and Firm B have debt-total asset ratios of

Firm A and Firm B have debt-total asset ratios of 65 percent and 45 percent, respectively, and returns on total assets of 5 percent and 9 percent, respectively. Which firm has a greater return on equi...

See Answer

Q: Pop Evil Inc.’s net income for the most recent year

Pop Evil Inc.’s net income for the most recent year was $16,481. The tax rate was 34 percent. The firm paid $3,681 in total interest expense and deducted $4,385 in depreciation expense. What was the c...

See Answer

Q: Highly Suspect Corp. has current liabilities of $415,000, a quick

Highly Suspect Corp. has current liabilities of $415,000, a quick ratio of .79, inventory turnover of 9.5, and a current ratio of 1.25. What is the cost of goods sold for the company?

See Answer

Q: For the company in Problem 6, what is the sustainable

For the company in Problem 6, what is the sustainable growth rate?Data from Problem 6:The most recent financial statements for Bello Co. are shown here:

See Answer

Q: BQ, Inc., is considering making an offer to purchase iReport

BQ, Inc., is considering making an offer to purchase iReport Publications. The vice president of finance has collected the following information:BQ also knows that securities analysts expect the earni...

See Answer

Q: Prince Albert Canning PLC had a net loss of £29,157

Prince Albert Canning PLC had a net loss of £29,157 on sales of £315,650. What was the company’s profit margin? Does the fact that these figures are quoted in a foreign currency make any difference? W...

See Answer

Q: Find the following financial ratios for Smolira Golf Corp. (use

Find the following financial ratios for Smolira Golf Corp. (use year-end figures rather than average values where appropriate):Short-term solvency ratios:Current ratio. Quick ratio. Cash ratio. Asset...

See Answer

Q: Construct the DuPont identity for Smolira Golf Corp.

Construct the DuPont identity for Smolira Golf Corp.

See Answer

Q: Prepare the 2018 statement of cash flows for Smolira Golf

Prepare the 2018 statement of cash flows for Smolira Golf Corp.

See Answer

Q: Smolira Golf Corp. has 20,000 shares of common stock outstanding,

Smolira Golf Corp. has 20,000 shares of common stock outstanding, and the market price for a share of stock at the end of 2018 was $58. What is the price-earnings ratio? What are the dividends per sha...

See Answer

Q: Wims, Inc., has current assets of $4,900, net fixed assets

Wims, Inc., has current assets of $4,900, net fixed assets of $27,300, current liabilities of $4,100, and long-term debt of $10,200. What is the value of the shareholders’ equity account for this firm...

See Answer

Q: Given the information for Kerber’s Tennis Shop, Inc., in Problems

Given the information for Kerber’s Tennis Shop, Inc., in Problems 8 and 9, suppose you also know that the firm’s net capital spending for 2018 was $1,250,000 and that the firm reduced its net working...

See Answer

Q: The most recent financial statements for Alexander Co. are shown

The most recent financial statements for Alexander Co. are shown here:Assets and costs are proportional to sales. The company maintains a constant 40 percent dividend payout ratio and a constant debt-...

See Answer

Q: What is the future value of $3,100 in 17 years

What is the future value of $3,100 in 17 years assuming an interest rate of 8.4 percent compounded semiannually?

See Answer

Q: For the company in the previous problem, what is the

For the company in the previous problem, what is the dividend yield? What is the expected capital gains yield?Previous problem:The next dividend payment by Savitz, Inc., will be $2.34 per share. The d...

See Answer

Q: Hudson Corporation will pay a dividend of $3.28 per share

Hudson Corporation will pay a dividend of $3.28 per share next year. The company pledges to increase its dividend by 3.75 percent per year indefinitely. If you require a return of 10 percent on your i...

See Answer

Q: Grateful Eight Co. is expected to maintain a constant 3.7

Grateful Eight Co. is expected to maintain a constant 3.7 percent growth rate in its dividends indefinitely. If the company has a dividend yield of 5.6 percent, what is the required return on the comp...

See Answer

Q: Suppose you know that a company’s stock currently sells for

Suppose you know that a company’s stock currently sells for $74 per share and the required return on the stock is 10.6 percent. You also know that the total return on the stock is evenly divided betwe...

See Answer

Q: The put-call parity condition is altered when dividends are paid.

The put-call parity condition is altered when dividends are paid. The dividend adjusted put-call parity formula is:S × e dt + P =E × e Rt + Cwhere d is again the continuously compounded dividend yield...

See Answer

Q: Burnett Corp. pays a constant $8.25 dividend on its stock.

Burnett Corp. pays a constant $8.25 dividend on its stock. The company will maintain this dividend for the next 13 years and will then cease paying dividends forever. If the required return on this st...

See Answer

Q: Bedekar, Inc., has an issue of preferred stock outstanding that

Bedekar, Inc., has an issue of preferred stock outstanding that pays a $3.40 dividend every year in perpetuity. If this issue currently sells for $91 per share, what is the required return?

See Answer

Q: Red, Inc., Yellow Corp., and Blue Company each will pay

Red, Inc., Yellow Corp., and Blue Company each will pay a dividend of $3.65 next year. The growth rate in dividends for all three companies is 4 percent. The required return for each company’s stock i...

See Answer

Q: What is the payback period for the following set of

What is the payback period for the following set of cash flows?Year …………………………………………………………………………………….. Cash Flow0 ……………………………………………………………………………………………. −$8,3001 ………………………………………………………………………………………….……. 2...

See Answer

Q: What is the IRR of the following set of cash

What is the IRR of the following set of cash flows? Year ……………………………………………………………………………………… Cash Flow0 …………………………………………………………………………………………… −$15,4001 ………………………………………………………………………………………………… 7,3002 …………………...

See Answer

Q: Spartan Credit Bank is offering 8.1 percent compounded daily on

Spartan Credit Bank is offering 8.1 percent compounded daily on its savings accounts. If you deposit $6,500 today, how much will you have in the account in 5 years? In 10 years? In 20 years?

See Answer

Q: For the cash flows in the previous problem, what is

For the cash flows in the previous problem, what is the NPV at a discount rate of zero percent? What if the discount rate is 10 percent? If it is 20 percent? If it is 30 percent?

See Answer

Q: Bruin, Inc., has identified the following two mutually exclusive projects:

Bruin, Inc., has identified the following two mutually exclusive projects:b. If the required return is 11 percent, what is the NPV for each of these projects? Which project will the company choose if...

See Answer

Q: Consider the following two mutually exclusive projects:

Consider the following two mutually exclusive projects:

See Answer

Q: Light Sweet Petroleum, Inc., is trying to evaluate a generation

Light Sweet Petroleum, Inc., is trying to evaluate a generation project with the followingcash flows:Year …………………………………………………………………………………………… Cash Flow0 ………………………………………………………………………………………… −$48,000,000...

See Answer

Q: What is the profitability index for the following set of

What is the profitability index for the following set of cash flows if the relevant discount rate is 10 percent? What if the discount rate is 15 percent? If it is 22 percent? Year ……………………………………………………...

See Answer

Q: In the previous problem, instead of a perpetual growth rate

In the previous problem, instead of a perpetual growth rate in adjusted cash flow from assets, you decide to calculate the terminal value of the company with the price-sales ratio. You believe that Ye...

See Answer

Q: The Sloan Corporation is trying to choose between the following

The Sloan Corporation is trying to choose between the following two mutually exclusive design projects: a. If the required return is 10 percent and the company applies the profitability index decision...

See Answer

Q: Consider the following two mutually exclusive projects:

Consider the following two mutually exclusive projects: Whichever project you choose, if any, you require a return of 11 percent on your investment.a. If you apply the payback criterion, which invest...

See Answer

Q: An investment has an installed cost of $527,630. The cash

An investment has an installed cost of $527,630. The cash flows over the four-year life of the investment are projected to be $212,200, $243,800, $203,500, and $167,410, respectively. If the discount...

See Answer

Q: Solo Corp. is evaluating a project with the following cash

Solo Corp. is evaluating a project with the following cash flows: Year ………………………………………………………………………………………………. Cash Flow0 …………………………………………………………………………………………………… −$47,0001 …………………………………………………………………………………...

See Answer

Q: An investment will pay you $80,000 in 10 years. If

An investment will pay you $80,000 in 10 years. If the appropriate discount rate is 9 percent compounded daily, what is the present value?

See Answer

Q: An investment project provides cash inflows of $745 per year

An investment project provides cash inflows of $745 per year for eight years. What is the project payback period if the initial cost is $1,700? What if the initial cost is $3,300? What if it is $6,100...

See Answer

Q: Suppose the company in the previous problem uses a discount

Suppose the company in the previous problem uses a discount rate of 11 percent and a reinvestment rate of 8 percent on all of its projects. Calculate the MIRR of the project using all three methods us...

See Answer

Q: Mannix Corporation stock currently sells for $57 per share. The

Mannix Corporation stock currently sells for $57 per share. The market requires a return of 11 percent on the firm’s stock. If the company maintains a constant 3.75 percent growth rate in dividends, w...

See Answer

Q: E-Eyes.com just issued some new preferred stock. The issue will

E-Eyes.com just issued some new preferred stock. The issue will pay an annual dividend of $20 in perpetuity, beginning 20 years from now. If the market requires a return of 5.65 percent on this invest...

See Answer

Q: An investment under consideration has a payback of seven years

An investment under consideration has a payback of seven years and a cost of $685,000. If the required return is 11 percent, what is the worst-case NPV? The best-case NPV? Explain. Assume the cash flo...

See Answer

Q: This problem is useful for testing the ability of financial

This problem is useful for testing the ability of financial calculators and spreadsheets. Consider the following cash flows. How many different IRRs are there? (Hint: Search between 20 percent and 70...

See Answer

Q: Use the Black-Scholes model for pricing a call, put-call parity,

Use the Black-Scholes model for pricing a call, put-call parity, and Problem 25 to show that the Black-Scholes model for directly pricing a put can be written as:P = E × e Rt × N (d2) − S × N (d1)

See Answer

Q: The Yurdone Corporation wants to set up a private cemetery

The Yurdone Corporation wants to set up a private cemetery business. According to the CFO, Barry M. Deep, business is “looking up.” As a result, the cemetery project will provide a net cash inflow of...

See Answer

Q: A project has the following cash flows:

A project has the following cash flows: Year …………………………………………………………………………………………. Cash Flow0 ………………………………………………………………………………………………... $59,0001 ……………………………………………………………………………………………….. − 34,0002 …………………………...

See Answer

Q: McKeekin Corp. has a project with the following cash flows:

McKeekin Corp. has a project with the following cash flows: Year ………………………………………………………………….. Cash Flow0 ………………………………………………………………………… $25,0001 ………………………………………………………………………… − 11,0002 …………………………………………………...

See Answer

Q: Big Dom’s Pawn Shop charges an interest rate of 27

Big Dom’s Pawn Shop charges an interest rate of 27 percent per month on loans to its customers. Like all lenders, Big Dom must report an APR to consumers. What rate should the shop report? What is the...

See Answer

Q: Anderson International Limited is evaluating a project in Erewhon. The

Anderson International Limited is evaluating a project in Erewhon. The project will create the following cash flows:Year …………………………………………………………………………………………….. Cash Flow0 ………………………………………………………………………………...

See Answer

Q: RAK, Inc., currently has an EPS of $2.45 and an

RAK, Inc., currently has an EPS of $2.45 and an earnings growth rate of 8 percent. If the benchmark PE ratio is 23, what is the target share price five years from now?

See Answer

Q: Bronco, Inc., imposes a payback cutoff of three years for

Bronco, Inc., imposes a payback cutoff of three years for its international investment projects. If the company has the following two projects available, should it accept either of them? ,,,

See Answer

Q: In practice, a common way to value a share of

In practice, a common way to value a share of stock when a company pays dividends is to value the dividends over the next five years or so, then find the “terminal” stock price using a benchmark PE ra...

See Answer

Q: Storico Co. just paid a dividend of $3.15 per share.

Storico Co. just paid a dividend of $3.15 per share. The company will increase its dividend by 20 percent next year and then reduce its dividend growth rate by 5 percentage points per year until it re...

See Answer

Q: This one’s a little harder. Suppose the current share price

This one’s a little harder. Suppose the current share price for the firm in the previous problem is $54.50 and all the dividend information remains the same. What required return must investors be dem...

See Answer

Q: You’ve just joined the investment banking firm of Dewey, Cheatum,

You’ve just joined the investment banking firm of Dewey, Cheatum, and Howe. They’ve offered you two different salary arrangements. You can have $85,000 per year for the next two years, or you can have...

See Answer

Q: In the previous problem, suppose you believe that sales in

In the previous problem, suppose you believe that sales in five years will be $29.2 million and the price-sales ratio will be 2.45. What is the share price now?Previous problem:You have looked at the...

See Answer

Q: When Marilyn Monroe died, ex-husband Joe DiMaggio vowed to place

When Marilyn Monroe died, ex-husband Joe DiMaggio vowed to place fresh flowers on her grave every Sunday as long as he lived. The week after she died in 1962, a bunch of fresh flowers that the former...

See Answer

Q: You are planning to save for retirement over the next

You are planning to save for retirement over the next 30 years. To save for retirement, you will invest $800 per month in a stock account in real dollars and $400 per month in a bond account in real d...

See Answer

Q: You want to buy a new sports coupe for $84,500,

You want to buy a new sports coupe for $84,500, and the finance office at the dealership has quoted you an APR of 5.2 percent for a 60-month loan to buy the car. What will your monthly payments be? Wh...

See Answer

Q: What is the relationship between the value of an annuity

What is the relationship between the value of an annuity and the level of interest rates? Suppose you just bought an annuity with 11 annual payments of $8,500 per year at the current interestof 10 per...

See Answer

Q: An investment project has annual cash inflows of $2,800, $3,700,

An investment project has annual cash inflows of $2,800, $3,700, $5,100, and $4,300, for the next four years, respectively. The discount rate is 11 percent. What is the discounted payback period for t...

See Answer

Q: You’re prepared to make monthly payments of $175, beginning at

You’re prepared to make monthly payments of $175, beginning at the end of this month, into an account that pays 7 percent interest compounded monthly. How many payments will you have made when your ac...

See Answer

Q: An investment project costs $17,000 and has annual cash flows

An investment project costs $17,000 and has annual cash flows of $4,700 for six years. What is the discounted payback period if the discount rate is zero percent? What if the discount rate is 5 percen...

See Answer

Q: You’re trying to determine whether to expand your business by

You’re trying to determine whether to expand your business by building a new manufacturing plant. The plant has an installation cost of $13.5 million, which will be depreciated straight-line to zero o...

See Answer

Q: A firm evaluates all of its projects by applying the

A firm evaluates all of its projects by applying the IRR rule. If the required return is 14 percent, should the firm accept the following project? Year …………………………………………………………………………………….. Cash Flow0 ……...

See Answer

Q: For the cash flows in the previous problem, suppose the

For the cash flows in the previous problem, suppose the firm uses the NPV decision rule. At a required return of 11 percent, should the firm accept this project? What if the required return is 24 perc...

See Answer

Q: A project that provides annual cash flows of $11,700 for

A project that provides annual cash flows of $11,700 for nine years costs $63,000 today. Is this a good project if the required return is 8 percent? What if it’s 20 percent? At what discount rate woul...

See Answer

Q: Photochronograph Corporation (PC) manufactures time series photographic equipment. It is

Photochronograph Corporation (PC) manufactures time series photographic equipment. It is currently at its target debt-equity ratio of .60. It’s considering building a new $65 million manufacturing fac...

See Answer

Q: Parker & Stone, Inc., is looking at setting up a

Parker & Stone, Inc., is looking at setting up a new manufacturing plant in South Park to produce garden tools. The company bought some land six years ago for $3.6 million in anticipation of using it...

See Answer

Q: Sunburn Sunscreen has a zero coupon bond issue outstanding with

Sunburn Sunscreen has a zero coupon bond issue outstanding with a $15,000 face value that matures in one year. The current market value of the firm’s assets is $16,200. The standard deviation of the r...

See Answer

Q: In the previous problem, suppose the required return on

In the previous problem, suppose the required return on the project is 12 percent. What is the project’s NPV?Data from Problem 9:Quad Enterprises is considering a new three-year expansion project that...

See Answer

Q: In the previous problem, suppose the project requires an initial

In the previous problem, suppose the project requires an initial investment in net working capital of $250,000, and the fixed asset will have a market value of $180,000 at the end of the project. What...

See Answer

Q: In the previous problem, suppose the fixed asset actually falls

In the previous problem, suppose the fixed asset actually falls into the three-year MACRS class. All the other facts are the same. What is the project’s Year 1 net cash flow now? Year 2? Year 3? What...

See Answer

Q: In the previous problem, suppose the fixed asset actually

In the previous problem, suppose the fixed asset actually qualifies for 100 percent bonus depreciation in the first year. All the other facts are the same. What is the project’s Year 1 net cash flow n...

See Answer

Q: Dog Up! Franks is looking at a new sausage system

Dog Up! Franks is looking at a new sausage system with an installed cost of $460,000. This cost will be depreciated straight-line to zero over the project’s five-year life, at the end of which the sau...

See Answer

Q: In the previous problem, suppose the fixed asset actually qualifies

In the previous problem, suppose the fixed asset actually qualifies for 100 percent bonus depreciation in the first year. What is the new NPV?Data from Problem 14:Dog Up! Franks is looking at a new sa...

See Answer

Q: In the previous problem, suppose your required return on the

In the previous problem, suppose your required return on the project is 11 percent and your pretax cost savings are $150,000 per year. Will you accept the project? What if the pretax cost savings are...

See Answer

Q: A five-year project has an initial fixed asset investment of

A five-year project has an initial fixed asset investment of $315,000, an initial NWC investment of $25,000, and an annual OCF of −$35,000. The fixed asset is fully depreciated over the life of the pr...

See Answer

Q: You are evaluating two different silicon wafer milling machines. The

You are evaluating two different silicon wafer milling machines. The Techron I costs $245,000, has a three-year life, and has pretax operating costs of $63,000 per year. The Techron II costs $420,000,...

See Answer

Q: What is the NAL of the lease from the lessor’s

What is the NAL of the lease from the lessor’s viewpoint? Assume a 21 percent tax rate

See Answer

Q: One of your customers is delinquent on his accounts payable

One of your customers is delinquent on his accounts payable balance. You’ve mutually agreed to a repayment schedule of $450 per month. You will charge 1.3 percent per month interest on the overdue bal...

See Answer

Q: Lucas Corp. has a debt-equity ratio of .65. The company

Lucas Corp. has a debt-equity ratio of .65. The company is considering a new plant that will cost $51 million to build. When the company issues new equity, it incurs a flotation cost of 7 percent. The...

See Answer

Q: Winnebagel Corp. currently sells 20,000 motor homes per year at

Winnebagel Corp. currently sells 20,000 motor homes per year at $97,000 each and 14,000 luxury motor coaches per year at $145,000 each. The company wants to introduce a new portable camper to fill out...

See Answer

Q: Martin Enterprises needs someone to supply it with 125,000 cartons

Martin Enterprises needs someone to supply it with 125,000 cartons of machine screws per year to support its manufacturing needs over the next five years, and you’ve decided to bid on the contract. It...

See Answer

Q: Masters Machine Shop is considering a four-year project to improve

Masters Machine Shop is considering a four-year project to improve its production efficiency. Buying a new machine press for $385,000 is estimated to result in $145,000 in annual pretax cost savings....

See Answer

Q: Eggz, Inc., is considering the purchase of new equipment that

Eggz, Inc., is considering the purchase of new equipment that will allow the company to collect loose hen feathers for sale. The equipment will cost $425,000 and will be eligible for 100 percent bonus...

See Answer

Q: Letang Industrial Systems Company (LISC) is trying to decide between

Letang Industrial Systems Company (LISC) is trying to decide between two different conveyor belt systems. System A costs $265,000, has a four-year life, and requires $73,000 in pretax annual operating...

See Answer

Q: Consider a project to supply 100 million postage stamps per

Consider a project to supply 100 million postage stamps per year to the U.S. Postal Service for the next five years. You have an idle parcel of land available that cost $750,000 five years ago; if the...

See Answer

Q: In the previous problem, suppose you were going to use

In the previous problem, suppose you were going to use a three-year MACRS depreciation schedule for your manufacturing equipment and you could keep working capital investments down to only $25,000 per...

See Answer

Q: Vandelay Industries is considering the purchase of a new machine

Vandelay Industries is considering the purchase of a new machine for the production of latex. Machine A costs $2,900,000 and will last for six years. Variable costs are 35 percent of sales, and fixed...

See Answer

Q: Light-emitting diode (LED) light bulbs have become required in recent

Light-emitting diode (LED) light bulbs have become required in recent years, but do they make financial sense? Suppose a typical 60-watt incandescent light bulb costs $.45 and lasts for 1,000 hours. A...

See Answer

Q: Gilmore, Inc., had equity of $145,000 at the beginning of

Gilmore, Inc., had equity of $145,000 at the beginning of the year. At the end of the year, the company had total assets of $210,000. During the year, the company sold no new equity. Net income for th...

See Answer

Q: The previous problem suggests that using LEDs instead of incandescent

The previous problem suggests that using LEDs instead of incandescent bulbs is a no-brainer. However, electricity costs actually vary quite a bit depending on location and user type (you can get infor...

See Answer

Q: This is a comprehensive project evaluation problem bringing together much

This is a comprehensive project evaluation problem bringing together much of what you have learned in this and previous chapters. Suppose you have been hired as a financial consultant to Defense Elect...

See Answer

Q: A proposed new investment has projected sales of $585,000. Variable

A proposed new investment has projected sales of $585,000. Variable costs are 44 percent of sales, and fixed costs are $187,000; depreciation is $51,000. Prepare a pro forma income statement assuming...

See Answer

Q: The previous two problems suggest that using LEDs is a

The previous two problems suggest that using LEDs is a good idea from a purely financial perspective unless you live in an area where power is relatively inexpensive, but there is another wrinkle. Sup...

See Answer

Q: Consider the following income statement:

Consider the following income statement:Sales ……………………………………………………………………………………… $747,300Costs ……………………………………………………………………………………….. 582,600Depreciation ………………………………………………………………………………… 89,300EBIT …………………...

See Answer

Q: A proposed new project has projected sales of $175,000, costs

A proposed new project has projected sales of $175,000, costs of $93,000, and depreciation of $24,800. The tax rate is 23 percent. Calculate operating cash flow using the four different approaches des...

See Answer

Q: A piece of newly purchased industrial equipment costs $1,375,000 and

A piece of newly purchased industrial equipment costs $1,375,000 and is classified as seven-year property under MACRS. Calculate the annual depreciation allowances and end-of-the-year book values for...

See Answer

Q: Consider an asset that costs $680,000 and is depreciated straight-line

Consider an asset that costs $680,000 and is depreciated straight-line to zero over its eight-year tax life. The asset is to be used in a five-year project; at the end of the project, the asset can be...

See Answer

Q: An asset used in a four-year project falls in the

An asset used in a four-year project falls in the five-year MACRS class for tax purposes. The asset has an acquisition cost of $5,100,000 and will be sold for $1,600,000 at the end of the project. If...

See Answer

Q: Quad Enterprises is considering a new three-year expansion project that

Quad Enterprises is considering a new three-year expansion project that requires an initial fixed asset investment of $2.32 million. The fixed asset will be depreciated straight-line to zero over its...

See Answer

Q: Live Forever Life Insurance Co. is selling a perpetuity contract

Live Forever Life Insurance Co. is selling a perpetuity contract that pays $1,250 monthly. The contract currently sells for $245,000. What is the monthly return on this investment vehicle? What is the...

See Answer

Q: Night Shades, Inc. (NSI), manufactures biotech sunglasses. The variable materials

Night Shades, Inc. (NSI), manufactures biotech sunglasses. The variable materials cost is $11.13 per unit, and the variable labor cost is $7.29 per unit.a. What is the variable cost per unit?b. Suppos...

See Answer

Q: Consider a project with the following data: Accounting break-even quantity

Consider a project with the following data: Accounting break-even quantity = 13,700 units; cash break-even quantity = 9,600 units; life = five years; fixed costs = $185,000; variable costs = $23 per u...

See Answer

Q: Perine, Inc., has balance sheet equity of $6.8 million. At

Perine, Inc., has balance sheet equity of $6.8 million. At the same time, the income statement shows net income of $815,000. The company paid dividends of $285,000 and has 245,000 shares of stock outs...

See Answer

Q: At an output level of 45,000 units, you calculate that

At an output level of 45,000 units, you calculate that the degree of operating leverage is 2.79. If output rises to 48,000 units, what will the percentage change in operating cash flow be? Will the ne...

See Answer

Q: In the previous problem, suppose fixed costs are $175,000. What

In the previous problem, suppose fixed costs are $175,000. What is the operating cash flow at 43,000 units? The degree of operating leverage?Data from Problem 11:At an output level of 45,000 units, yo...

See Answer

Q: Input area:

Input area:Fixed costs89000Units sold10400OCF127400New units sold11100Output area:Original DOL1.69858712715856%DQ0.0673076923076923%DOCF0.114327979712595New OCF141965.3846New DOL1.62691338625342

See Answer

Q: At an output level of 17,500 units, you have calculated

At an output level of 17,500 units, you have calculated that the degree of operating leverage is 3.26. The operating cash flow is $78,000 in this case. Ignoring the effect of taxes, what are fixed cos...

See Answer

Q: In the previous problem, what will be the new degree

In the previous problem, what will be the new degree of operating leverage in each case?Data from Problem 14:At an output level of 17,500 units, you have calculated that the degree of operating levera...

See Answer

Q: Your firm is contemplating the purchase of a new $485,000

Your firm is contemplating the purchase of a new $485,000 computer-based order entry system. The system will be depreciated straight-line to zero over its five-year life. It will be worth $35,000 at t...

See Answer

Q: Consider a four-year project with the following information: Initial fixed

Consider a four-year project with the following information: Initial fixed asset investment = $575,000; straight-line depreciation to zero over the four-year life; zero salvage value; price = $29; var...

See Answer

Q: You are planning to make monthly deposits of $475 into

You are planning to make monthly deposits of $475 into a retirement account that pays 10 percent interest compounded monthly. If your first deposit will be made one month from now, how large will your...

See Answer

Q: K-Too Everwear Corporation can manufacture mountain climbing shoes for $33.18

K-Too Everwear Corporation can manufacture mountain climbing shoes for $33.18 per pair in variable raw material costs and $24.36 per pair in variable labor expense. The shoes sell for $170 per pair. L...

See Answer

Q: McGilla Golf has decided to sell a new line of

McGilla Golf has decided to sell a new line of golf clubs. The clubs will sell for $845 per set and have a variable cost of $405 per set. The company has spent $150,000 for a marketing study that dete...

See Answer

Q: In the previous problem, you feel that the values are

In the previous problem, you feel that the values are accurate to within only ±10 percent. What are the best-case and worst-case NPVs? Hint: The price and variable costs for the two existing sets of c...

See Answer

Q: Consider four different stocks, all of which have a required

Consider four different stocks, all of which have a required return of 13 percent and a most recent dividend of $3.75 per share. Stocks W, X, and Y are expected to maintain constant growth rates in di...

See Answer

Q: Suppose in the previous problem that the company always needs

Suppose in the previous problem that the company always needs a conveyor belt system; when one wears out, it must be replaced. Which project should the firm choose now?Previous problem:Letang Industri...

See Answer

Q: Sloan Transmissions, Inc., has the following estimates for its new

Sloan Transmissions, Inc., has the following estimates for its new gear assembly project: Price = $1,440 per unit; variable costs = $460 per unit; fixed costs = $3.9 million; quantity = 85,000 units....

See Answer

Q: For the company in the previous problem, suppose management is

For the company in the previous problem, suppose management is most concerned about the impact of its price estimate on the project’s profitability. How could you address this concern? Describe how yo...

See Answer

Q: We are evaluating a project that costs $786,000, has an

We are evaluating a project that costs $786,000, has an eight-year life, and has no salvage value. Assume that depreciation is straight-line to zero over the life of the project. Sales are projected a...

See Answer

Q: In each of the following cases, calculate the accounting break-even

In each of the following cases, calculate the accounting break-even and the cash breakeven points. Ignore any tax effects in calculating the cash break-even.,,,

See Answer

Q: In each of the following cases, find the unknown variable:

In each of the following cases, find the unknown variable:,,,

See Answer

Q: Consider the following income statement for the Heir Jordan Corporation:

Consider the following income statement for the Heir Jordan Corporation:A 20 percent growth rate in sales is projected. Prepare a pro forma income statement assuming costs vary with sales and the divi...

See Answer

Q: In the previous problem, suppose you make $5,700 annual deposits

In the previous problem, suppose you make $5,700 annual deposits into the same retirement account. How large will your account balance be in 30 years?Previous problem:You are planning to make monthly...

See Answer

Q: A project has the following estimated data: Price = $62

A project has the following estimated data: Price = $62 per unit; variable costs = $28 per unit; fixed costs = $27,300; required return = 12 percent; initial investment = $34,800; life = four years. I...

See Answer

Q: Suppose a stock had an initial price of $65 per

Suppose a stock had an initial price of $65 per share, paid a dividend of $1.45 per share during the year, and had an ending share price of $71. Compute the percentage total return.

See Answer

Q: For Problem 9, suppose the average inflation rate over this

For Problem 9, suppose the average inflation rate over this period was 3.1 percent and the average T-bill rate over the period was 3.9 percent.Data from Problem 9:You’ve observed the following returns...

See Answer

Q: Given the information in Problem 10, what was the average

Given the information in Problem 10, what was the average real risk-free rate over this time period? What was the average real risk premium?Data from Problem 10:For Problem 9, suppose the average infl...

See Answer

Q: Most corporations pay quarterly dividends on their common stock rather

Most corporations pay quarterly dividends on their common stock rather than annual dividends. Barring any unusual circumstances during the year, the board raises, lowers, or maintains the current divi...

See Answer

Q: Look at Table 12.1 and Figure 12.7 in the text.

Look at Table 12.1 and Figure 12.7 in the text. When were T-bill rates at their highest over the period from 1926 through 2016? Why do you think they were so high during this period? What relationship...

See Answer

Q: You bought one of Great White Shark Repellant Co.’s 5.8

You bought one of Great White Shark Repellant Co.’s 5.8 percent coupon bonds one year ago for $1,030. These bonds make annual payments and mature 14 years from now. Suppose you decide to sell your bon...

See Answer

Q: You find a certain stock that had returns of 9

You find a certain stock that had returns of 9 percent, −16 percent, 18 percent, and 14 percent for four of the last five years. If the average return of the stock over this period was 10.3 percent, w...

See Answer

Q: A stock has had returns of 8 percent, 26 percent,

A stock has had returns of 8 percent, 26 percent, 14 percent, −17 percent, 31 percent, and −1 percent over the last six years. What are the arithmetic and geometric average returns for the stock?

See Answer

Q: A stock has had the following year-end prices and dividends:

A stock has had the following year-end prices and dividends:What are the arithmetic and geometric average returns for the stock?

See Answer

Q: Beginning three months from now, you want to be able

Beginning three months from now, you want to be able to withdraw $2,500 each quarter from your bank account to cover college expenses over the next four years. If the account pays .57 percent interest...

See Answer

Q: Suppose the returns on long-term corporate bonds are normally distributed.

Suppose the returns on long-term corporate bonds are normally distributed. Based on the historical record, what is the approximate probability that your return on these bonds will be less than −2.1 pe...

See Answer

Q: In the previous problem, what is the degree of operating

In the previous problem, what is the degree of operating leverage at the given level of output? What is the degree of operating leverage at the accounting break-even level of output?Data from Problem...

See Answer

Q: You are considering a new product launch. The project will

You are considering a new product launch. The project will cost $1,950,000, have a four-year life, and have no salvage value; depreciation is straight-line to zero. Sales are projected at 210 units pe...

See Answer

Q: In Problem 1, what was the dividend yield? The capital

In Problem 1, what was the dividend yield? The capital gains yield?Data from Problem 1:Suppose a stock had an initial price of $65 per share, paid a dividend of $1.45 per share during the year, and ha...

See Answer

Q: Over a 40-year period, an asset had an arithmetic return

Over a 40-year period, an asset had an arithmetic return of 11.2 percent and a geometric return of 9.4 percent. Using Blume’s formula, what is your best estimate of the future annual returns over 5 ye...

See Answer

Q: You have been hired as a consultant for Pristine Urban-Tech

You have been hired as a consultant for Pristine Urban-Tech Zither, Inc. (PUTZ), manufacturers of fine zithers. The market for zithers is growing quickly. The company bought some land three years ago...

See Answer

Q: Hybrid cars are touted as a “green” alternative; however, the

Hybrid cars are touted as a “green” alternative; however, the financial aspects of hybrid ownership are not as clear. Consider the 2016 Toyota Camry Hybrid LE, which had a list price of $5,500 (includ...

See Answer

Q: Suppose the returns on long-term corporate bonds and T-bills are

Suppose the returns on long-term corporate bonds and T-bills are normally distributed. Based on the historical record, use the NORMDIST function in Excel® to answer the following questions:a. What is...

See Answer

Q: This problem concerns the effect of taxes on the various

This problem concerns the effect of taxes on the various break-even measures. a. Show that, when we consider taxes, the general relationship between operating cash flow, OCF, and sales volume, Q, can...

See Answer

Q: If the appropriate discount rate for the following cash flows

If the appropriate discount rate for the following cash flows is 7.17 percent per year, what is the present value of the cash flows?Year …………………………………………………………………………… Cash Flow1 …………………………………………………………...

See Answer

Q: In Problem 27, suppose you’re confident about your own projections,

In Problem 27, suppose you’re confident about your own projections, but you’re a little unsure about Detroit’s actual machine screw requirement. What is the sensitivity of the project OCF to changes i...

See Answer

Q: Rework Problems 1 and 2 assuming the ending share price

Rework Problems 1 and 2 assuming the ending share price is $58.Data from Problem 2:In Problem 1, what was the dividend yield? The capital gains yield?Data from Problem 1:Suppose a stock had an initial...

See Answer

Q: Suppose you bought a bond with an annual coupon of

Suppose you bought a bond with an annual coupon of 7 percent one year ago for $1,010. The bond sells for $985 today.a. Assuming a $1,000 face value, what was your total dollar return on this investmen...

See Answer

Q: What was the average annual return on large-company stocks from

What was the average annual return on large-company stocks from 1926 through 2016: a. In nominal terms?b. In real terms?

See Answer

Q: In the previous problem, suppose the projections given for price,

In the previous problem, suppose the projections given for price, quantity, variable costs, and fixed costs are all accurate to within ±10 percent. Calculate the best-case and worst-case NPV figures.P...

See Answer

Q: In the previous problem, suppose the fixed asset actually qualifies

In the previous problem, suppose the fixed asset actually qualifies for 100 percent bonus depreciation in the first year. What is the new NPV?Previous problem:You have been hired as a consultant for P...

See Answer

Q: Using the following returns, calculate the arithmetic average returns, the

Using the following returns, calculate the arithmetic average returns, the variances, and the standard deviations for X and Y.

See Answer

Q: Refer to Table 12.1 in the text and look at

Refer to Table 12.1 in the text and look at the period from 1970 through 1975. Data from Table 12.1:a. Calculate the arithmetic average returns for large-company stocks and T-bills over this period.b....

See Answer

Q: You’ve observed the following returns on Crash-n-Burn Computer’s stock over

You’ve observed the following returns on Crash-n-Burn Computer’s stock over the past five years: 8 percent, −15 percent, 19 percent, 31 percent, and 21 percent.a. What was the arithmetic average retur...

See Answer

Q: First Simple Bank pays 6.4 percent simple interest on its

First Simple Bank pays 6.4 percent simple interest on its investment accounts. If First Complex Bank pays interest on its accounts compounded annually, what rate should the bank set if it wants to mat...

See Answer

Q: What are the portfolio weights for a portfolio that has

What are the portfolio weights for a portfolio that has 115 shares of Stock A that sell for $43 per share and 180 shares of Stock B that sell for $19 per share?

See Answer

Q: Consider the following information:

Consider the following information:b. What is the variance of this portfolio? The standard deviation?

See Answer

Q: You own a stock portfolio invested 20 percent in Stock

You own a stock portfolio invested 20 percent in Stock Q, 30 percent in Stock R, 35 percent in Stock S, and 15 percent in Stock T. The betas for these four stocks are .79, 1.23, 1.13, and 1.36, respec...

See Answer

Q: You own a portfolio equally invested in a risk-free asset

You own a portfolio equally invested in a risk-free asset and two stocks. If one of the stocks has a beta of 1.17 and the total portfolio is equally as risky as the market, what must the beta be for t...

See Answer

Q: A stock has a beta of 1.15, the expected return

A stock has a beta of 1.15, the expected return on the market is 10.3 percent, and the risk-free rate is 3.1 percent. What must the expected return on this stock be?

See Answer

Q: A stock has an expected return of 10.2 percent, the

A stock has an expected return of 10.2 percent, the risk-free rate is 3.9 percent, and the market risk premium is 7.2 percent. What must the beta of this stock be?

See Answer

Q: A stock has an expected return of 10.45 percent,

A stock has an expected return of 10.45 percent, its beta is .93, and the risk-free rate is 3.6 percent. What must the expected return on the market be?

See Answer

Q: Aria Acoustics, Inc. (AAI), projects unit sales for a new

Aria Acoustics, Inc. (AAI), projects unit sales for a new seven-octave voice emulation implant as follows:Year ………………………………………………………………………………………….. Unit Sales1 …………………………………………………………………………………………………….....

See Answer

Q: A stock has an expected return of 11.85 percent, its

A stock has an expected return of 11.85 percent, its beta is 1.24, and the expected return on the market is 10.2 percent. What must the risk-free rate be?

See Answer

Q: Asset W has an expected return of 11.8 percent and

Asset W has an expected return of 11.8 percent and a beta of 1.10. If the risk-free rate is 3.3 percent, complete the following table for portfolios of Asset W and a risk-free asset. Illustrate the re...

See Answer

Q: Fuente, Inc., has identified an investment project with the following

Fuente, Inc., has identified an investment project with the following cash flows. If the discount rate is 8 percent, what is the future value of these cash flows in Year 4? What is the future value at...

See Answer

Q: Stock Y has a beta of 1.2 and an expected

Stock Y has a beta of 1.2 and an expected return of 11.1 percent. Stock Z has a beta of .80 and an expected return of 7.85 percent. If the risk-free rate is 2.4 percent and the market risk premium is...

See Answer

Q: In the previous problem, what would the risk-free rate have

In the previous problem, what would the risk-free rate have to be for the two stocks to be correctly priced?Previous problem:Stock Y has a beta of 1.2 and an expected return of 11.1 percent. Stock Z h...

See Answer

Q: You own a portfolio that has $3,480 invested in Stock

You own a portfolio that has $3,480 invested in Stock A and $7,430 invested in Stock B. If the expected returns on these stocks are 8 percent and 11 percent, respectively, what is the expected return...

See Answer

Q: A stock has a beta of 1.12 and an expected

A stock has a beta of 1.12 and an expected return of 10.8 percent. A risk-free asset currently earns 2.7 percent.

See Answer

Q: Assume that the historical return on large-company stocks is a

Assume that the historical return on large-company stocks is a predictor of the future returns. What return would you estimate for large-company stocks over the next year? The next 10 years? 20 years?...

See Answer

Q: In Problem 20, McGilla Golf would like to know the

In Problem 20, McGilla Golf would like to know the sensitivity of NPV to changes in the price of the new clubs and the quantity of new clubs sold. What is the sensitivity of the NPV to each of these v...

See Answer

Q: Consider the following information about three stocks:

Consider the following information about three stocks:b. If the expected T-bill rate is 3.80 percent, what is the expected risk premium on the portfolio?c. If the expected inflation rate is 3.30 perce...

See Answer

Q: You want to create a portfolio equally as risky as

You want to create a portfolio equally as risky as the market, and you have $1,000,000 to invest. Given this information, fill in the rest of the following table:,,,

See Answer

Q: A proposed cost-saving device has an installed cost of $735,000.

A proposed cost-saving device has an installed cost of $735,000. The device will be used in a five-year project but is classified as three-year MACRS property for tax purposes. The required initial ne...

See Answer

Q: You have $100,000 to invest in a portfolio containing Stock

You have $100,000 to invest in a portfolio containing Stock X and Stock Y. Your goal is to create a portfolio that has an expected return of 12.7 percent. If Stock X has an expected return of 11.4 per...

See Answer

Q: You receive a credit card application from Shady Banks Savings

You receive a credit card application from Shady Banks Savings and Loan offering an introductory rate of 1.25 percent per year, compounded monthly for the first six months, increasing thereafter to 17...

See Answer

Q: Consider the following information about Stocks I and II:

Consider the following information about Stocks I and II:The market risk premium is 7 percent, and the risk-free rate is 3.5 percent. Which stock has the most systematic risk? Which one has the most u...

See Answer

Q: Suppose you observe the following situation:

Suppose you observe the following situation:

See Answer

Q: Suppose you observe the following situation:

Suppose you observe the following situation:a. Calculate the expected return on each stock.b. Assuming the capital asset pricing model holds and Stock A’s beta is greater than Stock...

See Answer

Q: Use the results of Problem 25 to find the accounting,

Use the results of Problem 25 to find the accounting, cash, and financial break-even quantities for the company in Problem 27.Data from Problem 27:Consider a project to supply Detroit with 30,000 tons...

See Answer

Q: You own a portfolio that is invested 35 percent in

You own a portfolio that is invested 35 percent in Stock X, 20 percent in Stock Y, and 45 percent in Stock Z. The expected returns on these three stocks are 9 percent, 15 percent, and 12 percent, resp...

See Answer

Q: Use the results of Problem 26 to find the degree

Use the results of Problem 26 to find the degree of operating leverage for the company in Problem 27 at the base-case output level of 30,000 tons. How does this number compare to the sensitivity figur...

See Answer

Q: You have $10,000 to invest in a stock portfolio. Your

You have $10,000 to invest in a stock portfolio. Your choices are Stock X with an expected return of 12.1 percent and Stock Y with an expected return of 9.8 percent. If your goal is to create a portfo...

See Answer

Q: Based on the following information, calculate the expected return:

Based on the following information, calculate the expected return:,,,

See Answer

Q: Based on the following information, calculate the expected return:

Based on the following information, calculate the expected return:,,,

See Answer

Q: To solve the bid price problem presented in the text,

To solve the bid price problem presented in the text, we set the project NPV equal to zero and found the required price using the definition of OCF. Thus the bid price represents a financial breakeven...

See Answer

Q: Suppose the firm in Problem 16 is considering two mutually

Suppose the firm in Problem 16 is considering two mutually exclusive investments. Project A has an NPV of $1,900, and Project B has an NPV of $2,800. As the result of taking Project A, the standard de...

See Answer

Q: Based on the following information, calculate the expected return:

Based on the following information, calculate the expected return:

See Answer

Q: A portfolio is invested 25 percent in Stock G, 55

A portfolio is invested 25 percent in Stock G, 55 percent in Stock J, and 20 percent in Stock K. The expected returns on these stocks are 11 percent, 9 percent, and 15 percent, respectively. What is t...

See Answer

Q: Consider the following information:

Consider the following information:

See Answer

Q: The Drogon Co. just issued a dividend of $2.80 per

The Drogon Co. just issued a dividend of $2.80 per share on its common stock. The company is expected to maintain a constant 4.5 percent growth rate in its dividends indefinitely. If the stock sells f...

See Answer

Q: Lannister Manufacturing has a target debt-equity ratio of .55. Its

Lannister Manufacturing has a target debt-equity ratio of .55. Its cost of equity is 11 percent, and its cost of debt is 6 percent. If the tax rate is 21 percent, what is the company’s WACC?

See Answer

Q: Fama’s Llamas has a weighted average cost of capital of

Fama’s Llamas has a weighted average cost of capital of 7.9 percent. The company’s cost of equity is 11 percent, and its pretax cost of debt is 5.8 percent. The tax rate is 25 percent. What is the com...

See Answer

Q: Dinklage Corp. has 7 million shares of common stock outstanding.

Dinklage Corp. has 7 million shares of common stock outstanding. The current share price is $68, and the book value per share is $8. The company also has two bond issues outstanding. The first bond is...

See Answer

Q: In Problem 12, suppose the most recent dividend was $3.25

In Problem 12, suppose the most recent dividend was $3.25 and the dividend growth rate is 5 percent. Assume that the overall cost of debt is the weighted average of that implied by the two outstanding...

See Answer

Q: Starset, Inc., has a target debt-equity ratio of .85. Its

Starset, Inc., has a target debt-equity ratio of .85. Its WACC is 9.1 percent, and the tax rate is 23 percent.a. If the company’s cost of equity is 14 percent, what is its pretax cost of debt?b. If in...

See Answer

Q: Given the following information for Watson Power Co., find the

Given the following information for Watson Power Co., find the WACC. Assume the company’s tax rate is 21 percent.Debt: …………………….. 15,000 bonds with a 5.8 percent coupon outstanding, $1,000 par value,...

See Answer

Q: You are planning to save for retirement over the next

You are planning to save for retirement over the next 30 years. To do this, you will invest $750 per month in a stock account and $250 per month in a bond account. The return of the stock account is e...

See Answer

Q: What would the lease payment have to be for both

What would the lease payment have to be for both lessor and lessee to be indifferent about the lease?

See Answer

Q: Your company has been approached to bid on a contract

Your company has been approached to bid on a contract to sell 4,800 voice recognition (VR) computer keyboards per year for four years. Due to technological improvements, beyond that time they will be...

See Answer

Q: The Rhaegel Corporation’s common stock has a beta of 1.07.

The Rhaegel Corporation’s common stock has a beta of 1.07. If the risk-free rate is 3.5 percent and the expected return on the market is 10 percent, what is the company’s cost of equity capital?

See Answer

Q: Stock in Daenerys Industries has a beta of 1.05. The

Stock in Daenerys Industries has a beta of 1.05. The market risk premium is 7 percent, and T-bills are currently yielding 3.4 percent. The company’s most recent dividend was $2.35 per share, and divid...

See Answer

Q: Suppose Stark, Ltd., just issued a dividend of $2.51 per

Suppose Stark, Ltd., just issued a dividend of $2.51 per share on its common stock. The company paid dividends of $2.01, $2.17, $2.25, and $2.36 per share in the last four years. If the stock currentl...

See Answer

Q: Holdup Bank has an issue of preferred stock with a

Holdup Bank has an issue of preferred stock with a stated dividend of $4.25 that just sold for $93 per share. What is the bank’s cost of preferred stock?

See Answer

Q: Viserion, Inc., is trying to determine its cost of debt.

Viserion, Inc., is trying to determine its cost of debt. The firm has a debt issue outstanding with 23 years to maturity that is quoted at 103 percent of face value. The issue makes semiannual payment...

See Answer

Q: Jiminy’s Cricket Farm issued a 30-year, 6 percent semiannual bond

Jiminy’s Cricket Farm issued a 30-year, 6 percent semiannual bond three years ago. The bond currently sells for 93 percent of its face value. The company’s tax rate is 22 percent.a. What is the pretax...

See Answer

Q: For the firm in Problem 7, suppose the book value

For the firm in Problem 7, suppose the book value of the debt issue is $95 million. In addition, the company has a second debt issue on the market, a zero coupon bond with eight years left to maturity...

See Answer

Q: Targaryen Corporation has a target capital structure of 70 percent

Targaryen Corporation has a target capital structure of 70 percent common stock, 5 percent preferred stock, and 25 percent debt. Its cost of equity is 10 percent, the cost of preferred stock is 5 perc...

See Answer

Q: You have an investment that will pay you 0.67 percent

You have an investment that will pay you 0.67 percent per month. How much will you have per dollar invested in one year? In two years?

See Answer

Q: Leah, Inc., is proposing a rights offering. Presently there are

Leah, Inc., is proposing a rights offering. Presently there are 375,000 shares outstanding at $67 each. There will be 50,000 new shares offered at $58 each.a. What is the new market value of the compa...

See Answer

Q: The Metallica Heavy Metal Mining (MHMM) Corporation wants to diversify

The Metallica Heavy Metal Mining (MHMM) Corporation wants to diversify its operations. Some recent financial information for the company is shown here:Stock price …………………………………………………………………………………………… $...

See Answer

Q: Suppose we are thinking about replacing an old computer with

Suppose we are thinking about replacing an old computer with a new one. The old one cost us $1,560,000; the new one will cost $1,872,000. The new machine will be depreciated straight-line to zero over...

See Answer

Q: In Problem 10, what would the ROE on the investment

In Problem 10, what would the ROE on the investment have to be if we wanted the price after the offering to be $75 per share? (Assume the PE ratio remains constant.) What is the NPV of this investment...

See Answer

Q: Bell Hill Mfg. is considering a rights offer. The company

Bell Hill Mfg. is considering a rights offer. The company has determined that the ex-rights price would be $63. The current price is $68 per share, and there are 26 million shares outstanding. The rig...

See Answer

Q: Show that the value of a right just prior to

Show that the value of a right just prior to expiration can be written as:Value of a right=PRO PX = (PRO Ps) / (N + 1)where PRO, PS, and PX stand for the rights-on price, the subscription price, and t...

See Answer

Q: Prahm Corp. wants to raise $4.7 million via a rights

Prahm Corp. wants to raise $4.7 million via a rights offering. The company currently has 530,000 shares of common stock outstanding that sell for $55 per share. Its underwriter has set a subscription...

See Answer

Q: Knight Inventory Systems, Inc., has announced a rights offer. The

Knight Inventory Systems, Inc., has announced a rights offer. The company has announced that it will take four rights to buy a new share in the offering at a subscription price of $35. At the close of...

See Answer

Q: Titan Mining Corporation has 7.5 million shares of common stock

Titan Mining Corporation has 7.5 million shares of common stock outstanding, 250,000 shares of 4.2 percent preferred stock outstanding, and 140,000 bonds with a semiannual coupon of 5.1 percent outsta...

See Answer

Q: An all-equity firm is considering the following projects:

An all-equity firm is considering the following projects:The T-bill rate is 4 percent, and the expected return on the market is 11 percent.a. Which projects have a higher expected return than the firm...

See Answer

Q: You want to be a millionaire when you retire in

You want to be a millionaire when you retire in 40 years. How much do you have to save each month if you can earn an annual return of 9.7 percent? How much do you have to save each month if you wait 1...

See Answer

Q: Suppose your company needs $24 million to build a new

Suppose your company needs $24 million to build a new assembly line. Your target debt-equity ratio is .75. The flotation cost for new equity is 7 percent, but the flotation cost for debt is only 3 per...

See Answer

Q: Cully Company needs to raise $80 million to start a

Cully Company needs to raise $80 million to start a new project and will raise the money by selling new bonds. The company will generate no internal equity for the foreseeable future. The company has...

See Answer

Q: The Clifford Corporation has announced a rights offer to raise

The Clifford Corporation has announced a rights offer to raise $35 million for a new journal, the Journal of Financial Excess. This journal will review potential articles after the author pays a nonre...

See Answer

Q: You want to borrow $115,000 from your local bank to

You want to borrow $115,000 from your local bank to buy a new sailboat. You can afford to make monthly payments of $2,250, but no more. Assuming monthly compounding, what is the highest rate you can a...

See Answer

Q: Sommer, Inc., is considering a project that will result in

Sommer, Inc., is considering a project that will result in initial aftertax cash savings of $2.3 million at the end of the first year, and these savings will grow at a rate of 2 percent per year indef...

See Answer

Q: Being Human, Inc., recently issued new securities to finance a

Being Human, Inc., recently issued new securities to finance a new TV show. The project cost $35 million, and the company paid $2.2 million in flotation costs. In addition, the equity issued had a flo...

See Answer

Q: Ying Import has several bond issues outstanding, each making semiannual

Ying Import has several bond issues outstanding, each making semiannual interest payments. The bonds are listed in the following table. If the corporate tax rate is 22 percent, what is the aftertax co...

See Answer

Q: Red Shoe Co. has concluded that additional equity financing will

Red Shoe Co. has concluded that additional equity financing will be needed to expand operations and that the needed funds will be best obtained through a rights offering. It has correctly determined t...

See Answer

Q: The Woods Co. and the Spieth Co. have both announced

The Woods Co. and the Spieth Co. have both announced IPOs at $40 per share. One of these is undervalued by $9, and the other is overvalued by $4, but you have no way of knowing which is which. You pla...

See Answer

Q: The Whistling Straits Corporation needs to raise $60 million to

The Whistling Straits Corporation needs to raise $60 million to finance its expansion into new markets. The company will sell new shares of equity via a general cash offering to raise the needed funds...

See Answer

Q: The Maybe Pay Life Insurance Co. is trying to sell

The Maybe Pay Life Insurance Co. is trying to sell you an investment policy that will pay you and your heirs $35,000 per year forever. If the required return on this investment is 4.7 percent, how muc...

See Answer

Q: Given the following information for Bowie Pizza Co., calculate the

Given the following information for Bowie Pizza Co., calculate the depreciation expense: Sales = $64,000; Costs = $30,700; Addition to retained earnings = $5,700; Dividends paid = $1,980; Interest exp...

See Answer

Q: In Problem 5, if the SEC filing fee and associated

In Problem 5, if the SEC filing fee and associated administrative expenses of the offering are $1.2 million, how many shares need to be sold?Problem 5:The Whistling Straits Corporation needs to raise...

See Answer

Q: The Raven Co. has just gone public. Under a firm

The Raven Co. has just gone public. Under a firm commitment agreement, Raven received $21.39 for each of the 20 million shares sold. The initial offering price was $23 per share, and the stock rose to...

See Answer

Q: Nemesis, Inc., has 165,000 shares of stock outstanding. Each share

Nemesis, Inc., has 165,000 shares of stock outstanding. Each share is worth $77, so the company’s market value of equity is $12,705,000. Suppose the firm issues 30,000 new shares at the following pric...

See Answer

Q: Wayne, Inc., wishes to expand its facilities. The company currently

Wayne, Inc., wishes to expand its facilities. The company currently has 6 million shares outstanding and no debt. The stock sells for $64 per share, but the book value per share is $19. Net income is...

See Answer

Q: You need a 30-year, fixed-rate mortgage to buy a new

You need a 30-year, fixed-rate mortgage to buy a new home for $235,000. Your mortgage bank will lend you the money at an APR of 5.35 percent for this 360-month loan. However, you can afford monthly pa...

See Answer

Q: Ghost, Inc., has no debt outstanding and a total market

Ghost, Inc., has no debt outstanding and a total market value of $185,000. Earnings before interest and taxes, EBIT, are projected to be $29,000 if economic conditions are normal. If there is strong e...

See Answer

Q: Thrice Corp. uses no debt. The weighted average cost of

Thrice Corp. uses no debt. The weighted average cost of capital is 8.4 percent. If the current market value of the equity is $16.3 million and there are no taxes, what is EBIT?

See Answer

Q: In Problem 10, suppose the corporate tax rate is 22

In Problem 10, suppose the corporate tax rate is 22 percent. What is EBIT in this case? What is the WACC? Explain.Problem 10:Thrice Corp. uses no debt. The weighted average cost of capital is 8.4 perc...

See Answer

Q: Blitz Industries has a debt-equity ratio of 1.25. Its WACC

Blitz Industries has a debt-equity ratio of 1.25. Its WACC is 8.3 percent, and its cost of debt is 5.1 percent. The corporate tax rate is 21 percent.a. What is the company’s cost of equity capital?b....

See Answer

Q: Citee Corp. has no debt but can borrow at 6.1

Citee Corp. has no debt but can borrow at 6.1 percent. The firm’s WACC is currently 9.4 percent, and the tax rate is 21 percent.a. What is the company’s cost of equity?b. If the firm converts to 25 pe...

See Answer

Q: You have just won the lottery and will receive $1,500,000

You have just won the lottery and will receive $1,500,000 in one year. You will receive payments for 30 years, and the payments will increase by 2.7 percent per year. If the appropriate discount rate...

See Answer

Q: Meyer & Co. expects its EBIT to be $97,000 every

Meyer & Co. expects its EBIT to be $97,000 every year forever. The firm can borrow at 8 percent. The company currently has no debt, and its cost of equity is 13 percent. If the tax rate is 24 percent,...

See Answer

Q: In Problem 14, what is the cost of equity after

In Problem 14, what is the cost of equity after recapitalization? What is the WACC? What are the implications for the firm’s capital structure decision?Problem 14:Meyer & Co. expects its EBIT to be $9...

See Answer

Q: Tool Manufacturing has an expected EBIT of $51,000 in perpetuity

Tool Manufacturing has an expected EBIT of $51,000 in perpetuity and a tax rate of 21 percent. The firm has $126,000 in outstanding debt at an interest rate of 5.35 percent, and its unlevered cost of...

See Answer

Q: Repeat parts (a) and (b) in Problem 1 assuming the

Repeat parts (a) and (b) in Problem 1 assuming the company has a tax rate of 21 percent, a market-to-book ratio of 1.0, and the stock price remains constant.(a) and (b) in Problem 1:a. Calculate earni...

See Answer

Q: Suppose the company in Problem 1 has a market-to-book ratio

Suppose the company in Problem 1 has a market-to-book ratio of 1.0 and the stock price remains constant.Problem 1:Ghost, Inc., has no debt outstanding and a total market value of $185,000. Earnings be...

See Answer

Q: The present value of the following cash flow stream is

The present value of the following cash flow stream is $7,500 when discounted at 9 percent annually. What is the value of the missing cash flow? Year …………………………………………………………………….. Cash Flow1 …………………………...

See Answer

Q: Round Hammer is comparing two different capital structures: An all-equity

Round Hammer is comparing two different capital structures: An all-equity plan (Plan I) and a levered plan (Plan II). Under Plan I, the company would have 180,000 shares of stock outstanding. Under Pl...

See Answer

Q: In Problem 4, use M&M Proposition I to find the

In Problem 4, use M&M Proposition I to find the price per share of equity under each of the two proposed plans. What is the value of the firm?Problem 4:Round Hammer is comparing two different capital...

See Answer

Q: Bellwood Corp. is comparing two different capital structures. Plan I

Bellwood Corp. is comparing two different capital structures. Plan I would result in 12,700 shares of stock and $109,250 in debt. Plan II would result in 9,800 shares of stock and $247,000 in debt.The...

See Answer

Q: Ignoring taxes in Problem 6, what is the price per

Ignoring taxes in Problem 6, what is the price per share of equity under Plan I? Plan II? What principle is illustrated by your answers?Problem 6:Bellwood Corp. is comparing two different capital stru...

See Answer

Q: Your job pays you only once a year for all

Your job pays you only once a year for all the work you did over the previous 12 months. Today, December 31, you just received your salary of $55,000 and you plan to spend all of it. However, you want...

See Answer

Q: FCOJ, Inc., a prominent consumer products firm, is debating whether

FCOJ, Inc., a prominent consumer products firm, is debating whether to convert its all-equity capital structure to one that is 30 percent debt. Currently, there are 5,800 shares outstanding, and the p...

See Answer

Q: ABC Co. and XYZ Co. are identical firms in all

ABC Co. and XYZ Co. are identical firms in all respects except for their capital structure. ABC is all-equity financed with $720,000 in stock. XYZ uses both stock and perpetual debt; its stock is wort...

See Answer

Q: You own 1,000 shares of stock in Avondale Corporation. You

You own 1,000 shares of stock in Avondale Corporation. You will receive a $3.15 per share dividend in one year. In two years, the company will pay a liquidating dividend of $57 per share. The required...

See Answer

Q: In Problem 10, suppose you want only $1,500 total in

In Problem 10, suppose you want only $1,500 total in dividends the first year. What will your homemade dividend be in two years?Problem 10:You own 1,000 shares of stock in Avondale Corporation. You wi...

See Answer

Q: Awake Corporation is evaluating an extra dividend versus a share

Awake Corporation is evaluating an extra dividend versus a share repurchase. In either case, $17,500 would be spent. Current earnings are $1.89 per share, and the stock currently sells for $64 per sha...

See Answer

Q: The Gecko Company and the Gordon Company are two firms

The Gecko Company and the Gordon Company are two firms that have the same business risk but different dividend policies. Gecko pays no dividend, whereas Gordon has an expected dividend yield of 2.9 pe...

See Answer

Q: You just won the TVM Lottery. You will receive $1

You just won the TVM Lottery. You will receive $1 million today plus another 10 annual payments that increase by $375,000 per year. Thus, in one year, you receive $1.375 million. In two years, you get...

See Answer

Q: As discussed in the text, in the absence of market

As discussed in the text, in the absence of market imperfections and tax effects, we would expect the share price to decline by the amount of the dividend payment when the stock goes ex dividend. Once...

See Answer

Q: National Business Machine Co. (NBM) has $4 million of extra

National Business Machine Co. (NBM) has $4 million of extra cash after taxes have been paid. NBM has two choices to make use of this cash. One alternative is to invest the cash in financial assets. Th...

See Answer

Q: After completing its capital spending for the year, Carlson Manufacturing

After completing its capital spending for the year, Carlson Manufacturing has $1,000 extra cash. Carlson’s managers must choose between investing the cash in Treasury bonds that yield 3 percent or pay...

See Answer

Q: An investment offers $4,350 per year for 15 years, with

An investment offers $4,350 per year for 15 years, with the first payment occurring one year from now. If the required return is 6 percent, what is the value of the investment? What would the value be...

See Answer

Q: Change Corporation expects an EBIT of $31,200 every year forever.

Change Corporation expects an EBIT of $31,200 every year forever. The company currently has no debt, and its cost of equity is 11 percent.a. What is the current value of the company?b. Suppose the com...

See Answer

Q: The Day Company and the Knight Company are identical in

The Day Company and the Knight Company are identical in every respect except that Day is not levered. Financial information for the two firms appears in the following table. All earnings streams are p...

See Answer

Q: The owners’ equity accounts for Vidi International are shown here:

The owners’ equity accounts for Vidi International are shown here:Common stock ($.50 par value) ……………………………………………………… $ 25,000Capital surplus ………………………………………………………………………………. 215,000Retained earnings …...

See Answer

Q: The balance sheet for Sinking Ship Corp. is shown here

The balance sheet for Sinking Ship Corp. is shown here in market value terms. There are 14,000 shares of stock outstanding.The company has declared a dividend of $1.30 per share. The stock goes ex div...

See Answer

Q: In Problem 5, suppose the company has announced it is

In Problem 5, suppose the company has announced it is going to repurchase $18,200 worth of stock. What effect will this transaction have on the equity of the firm? How many shares will be outstanding?...

See Answer

Q: The market value balance sheet for Bobaflex Manufacturing is shown

The market value balance sheet for Bobaflex Manufacturing is shown here. The company has declared a 25 percent stock dividend. The stock goes ex dividend tomorrow (the chronology for a stock dividend...

See Answer

Q: The company with the common equity accounts shown here has

The company with the common equity accounts shown here has declared a 15 percent stock dividend when the market value of its stock is $53 per share. What effects will the distribution of the stock div...

See Answer

Q: You have just purchased a new warehouse. To finance the

You have just purchased a new warehouse. To finance the purchase, you’ve arranged for a 30- year mortgage loan for 80 percent of the $3,400,000 purchase price. The monthly payment on this loan will be...

See Answer

Q: In Problem 8, suppose the company instead decides on a

In Problem 8, suppose the company instead decides on a four-for-one stock split. The firm’s 65 cent per-share cash dividend on the new (postsplit) shares represents an increase of 10...

See Answer

Q: Ginger, Inc., has declared a $5.35 per share dividend. Suppose

Ginger, Inc., has declared a $5.35 per share dividend. Suppose capital gains are not taxed, but dividends are taxed at 15 percent. New IRS regulations require that taxes be withheld at the time the di...

See Answer

Q: If you put up $41,000 today in exchange for a

If you put up $41,000 today in exchange for a 5.1 percent, 15-year annuity, what will the annual cash flow be?

See Answer

Q: The following is the sales budget for Profit, Inc., for

The following is the sales budget for Profit, Inc., for the first quarter of 2018:Credit sales are collected as follows:65 percent in the month of the sale20 percent in the month after the sale15 perc...

See Answer

Q: Here are some important figures from the budget of Nashville

Here are some important figures from the budget of Nashville Nougats, Inc., for the second quarter of 2018:The company predicts that 5 percent of its credit sales will never be collected, 35 percent o...

See Answer

Q: Below are the most recent balance sheets for Country Kettles,

Below are the most recent balance sheets for Country Kettles, Inc. Excluding accumulated depreciation, determine whether each item is a source or a use of cash and the amount:,,,

See Answer

Q: Cori’s Corp. has an equity value of $13,315. Long-term debt

Cori’s Corp. has an equity value of $13,315. Long-term debt is $8,200. Net working capital, other than cash, is $2,750. Fixed assets are $17,380. How much cash does the company have? If current liabil...

See Answer

Q: For the company in Problem 2, show how the equity

For the company in Problem 2, show how the equity accounts will change if:Problem 2:The owners’ equity accounts for Vidi International are shown here:Common stock ($.50 par value) ……………………………………………………...

See Answer

Q: Simmons Mineral Operations, Inc. (SMO), currently has 530,000 shares of

Simmons Mineral Operations, Inc. (SMO), currently has 530,000 shares of stock outstanding that sell for $68 per share. Assuming no market imperfections or tax effects exist, what will the share price...

See Answer

Q: Morning Jolt Coffee Company has projected the following quarterly sales

Morning Jolt Coffee Company has projected the following quarterly sales amounts for the coming year:a. Accounts receivable at the beginning of the year are $365. The company has a 45-day collection pe...

See Answer

Q: Consider the following financial statement information for the Newk Corporation:

Consider the following financial statement information for the Newk Corporation:Calculate the operating and cash cycles. How do you interpret your answer?

See Answer

Q: Consider a firm with a contract to sell an asset

Consider a firm with a contract to sell an asset for $145,000 four years from now. The asset costs $91,700 to produce today. Given a relevant discount rate of 11 percent per year, will the firm make a...

See Answer

Q: Your firm has an average collection period of 31 days.

Your firm has an average collection period of 31 days. Current practice is to factor all receivables immediately at a discount of 1.25 percent. What is the effective cost of borrowing in this case? As...

See Answer

Q: Your company will generate $47,000 in annual revenue each year

Your company will generate $47,000 in annual revenue each year for the next seven years from a new information database. If the appropriate interest rate is 7.1 percent, what is the present value of t...

See Answer

Q: Sexton Corp. has projected the following sales for the coming

Sexton Corp. has projected the following sales for the coming year:a. Calculate payments to suppliers assuming that the company places orders during each quarter equal to 30 percent of projected sales...

See Answer

Q: The Torrey Pine Corporation’s purchases from suppliers in a quarter

The Torrey Pine Corporation’s purchases from suppliers in a quarter are equal to 75 percent of the next quarter’s forecast sales. The payables period is 60 days. Wa...

See Answer

Q: In a typical month, the Monk Corporation receives 80 checks

In a typical month, the Monk Corporation receives 80 checks totaling $113,000. These are delayed four days on average. What is the average daily float? Assume 30 days in a month

See Answer

Q: No More Books Corporation has an agreement with Floyd Bank

No More Books Corporation has an agreement with Floyd Bank whereby the bank handles $4.5 million in collections per day and requires a $340,000 compensating balance. No More Books is contemplating can...

See Answer

Q: Bird’s Eye Treehouses, Inc., a Kentucky company, has determined that

Bird’s Eye Treehouses, Inc., a Kentucky company, has determined that a majority of its customers are located in the Pennsylvania area. It therefore is considering using a lockbox system offered by a b...

See Answer

Q: Cow Chips, Inc., a large fertilizer distributor based in California,

Cow Chips, Inc., a large fertilizer distributor based in California, is planning to use a lockbox system to speed up collections from its customers located on the East Coast. A Philadelphia-area bank...

See Answer

Q: A bank offers your firm a revolving credit arrangement for

A bank offers your firm a revolving credit arrangement for up to $50 million at an interest rate of 1.65 percent per quarter. The bank also requires you to maintain a compensating balance of 5 percent...

See Answer

Q: Wildcat, Inc., has estimated sales (in millions) for the next

Wildcat, Inc., has estimated sales (in millions) for the next four quarters as follows:Sales for the first quarter of the following year are projected at $180 million. Accounts receivable at the begin...

See Answer

Q: Rework Problem 15 assuming:

Rework Problem 15 assuming:Problem 15:Wildcat, Inc., has estimated sales (in millions) for the next four quarters as follows:Sales for the first quarter of the following year are projected at $180 mil...

See Answer

Q: What is the value today of $4,400 per year, at

What is the value today of $4,400 per year, at a discount rate of 8.3 percent, if the first payment is received 6 years from today and the last payment is received 20 years from today?

See Answer

Q: If you deposit $4,500 at the end of each of

If you deposit $4,500 at the end of each of the next 20 years into an account paying 9.7 percent interest, how much money will you have in the account in 20 years? How much will you have if you make d...

See Answer

Q: In exchange for a $300 million fixed commitment line of

In exchange for a $300 million fixed commitment line of credit, your firm has agreed to do the following:1. Pay 1.85 percent per quarter on any funds actually borrowed.2. Maintain a 4.5 percent compen...

See Answer

Q: Cheap Money Bank offers your firm a discount interest loan

Cheap Money Bank offers your firm a discount interest loan at 8.25 percent for up to $25 million and, in addition, requires you to maintain a 5 percent compensating balance against the amount borrowed...

See Answer

Q: In a world of corporate taxes only, show that the

In a world of corporate taxes only, show that the WACC can be written as WACC = RU × [1 − TC(D/V)]

See Answer

Q: Each business day, on average, a company writes checks totaling

Each business day, on average, a company writes checks totaling $17,000 to pay its suppliers. The usual clearing time for the checks is four days. Meanwhile, the company is receiving payments from its...

See Answer

Q: Assume a firm’s debt is risk-free, so that the cost

Assume a firm’s debt is risk-free, so that the cost of debt equals the risk-free rate, Rf. Define βA as the firm’s asset beta—that is, the systematic risk of the firm’s assets. Define βE to be the bet...

See Answer

Q: Suppose a firm’s business operations are such that they mirror

Suppose a firm’s business operations are such that they mirror movements in the economy as a whole very closely; that is, the firm’s asset beta is 1.0. Use the result of Problem 21 to find the equity...

See Answer

Q: Purple Feet Wine, Inc., receives an average of $17,500 in

Purple Feet Wine, Inc., receives an average of $17,500 in checks per day. The delay in clearing is typically three days. The current interest rate is .017 percent per day.a. What is the company’s floa...

See Answer

Q: Your neighbor goes to the post office once a month

Your neighbor goes to the post office once a month and picks up two checks, one for $10,700 and one for $4,600. The larger check takes four days to clear after it is deposited; the smaller one takes t...

See Answer

Q: Your firm has an average receipt size of $125. A

Your firm has an average receipt size of $125. A bank has approached you concerning a lockbox service that will decrease your total collection time by two days. You typically receive 5,100 checks per...

See Answer

Q: A mail-order firm processes 5,300 checks per month. Of these,

A mail-order firm processes 5,300 checks per month. Of these, 60 percent are for $47 and 40 percent are for $79. The $47 checks are delayed two days on average; the $79 checks are delayed three days o...

See Answer

Q: Frostbite Thermal wear has a zero coupon bond issue outstanding

Frostbite Thermal wear has a zero coupon bond issue outstanding with a face value of $23,000 that matures in one year. The current market value of the firm’s assets is $26,200. The standard deviation...

See Answer

Q: A 15-year annuity pays $1,475 per month, and payments are

A 15-year annuity pays $1,475 per month, and payments are made at the end of each month. If the interest rate is 9 percent compounded monthly for the first seven years, and 6 percent compounded monthl...

See Answer

Q: Paper Submarine Manufacturing is investigating a lockbox system to reduce

Paper Submarine Manufacturing is investigating a lockbox system to reduce its collection time. It has determined the following:Average number of payments per day ……………………………………………………. 485Average value...

See Answer

Q: It takes Cookie Cutter Modular Homes, Inc., about six days

It takes Cookie Cutter Modular Homes, Inc., about six days to receive and deposit checks from customers. The company’s management is considering a lockbox system to reduce the firm’s collection times....

See Answer

Q: Every two weeks, No More Pencils, Inc., disburses checks that

Every two weeks, No More Pencils, Inc., disburses checks that average $107,000 and take seven days to clear. How much interest can the company earn annually if it delays transfer of funds from an inte...

See Answer

Q: Sanchez, Inc., is considering a change in its cash-only sales

Sanchez, Inc., is considering a change in its cash-only sales policy. The new terms of sale would be net one month. Based on the following information, determine if the company should proceed or not....

See Answer

Q: Provenza Manufacturing uses 3,400 switch assemblies per week and then

Provenza Manufacturing uses 3,400 switch assemblies per week and then reorders another 3,400. If the relevant carrying cost per switch assembly is $7.45, and the fixed order cost is $1,100, is the com...

See Answer

Q: The Trektronics store begins each week with 450 phasers in

The Trektronics store begins each week with 450 phasers in stock. This stock is depleted each week and reordered. If the carrying cost per phaser is $34 per year and the fixed order cost is $130, what...

See Answer

Q: You’ve worked out a line of credit arrangement that allows

You’ve worked out a line of credit arrangement that allows you to borrow up to $40 million at any time. The interest rate is .36 percent per month. In addition, 4 percent of the amount that you borrow...

See Answer

Q: The Snedecker Corporation is considering a change in its cash-only

The Snedecker Corporation is considering a change in its cash-only policy. The new terms would be net one period. Based on the following information, determine if the company should proceed or not. Th...

See Answer

Q: Veni, Inc., currently has an all-cash credit policy. It is

Veni, Inc., currently has an all-cash credit policy. It is considering making a change in the credit policy by going to terms of net 30 days. Based on the following information, what do you recommend?...

See Answer

Q: You want to have $60,000 in your savings account 12

You want to have $60,000 in your savings account 12 years from now, and you’re prepared to make equal annual deposits into the account at the end of each year. If the account pays 6.4 percent interest...

See Answer

Q: The Silver Spokes Bicycle Shop has decided to offer credit

The Silver Spokes Bicycle Shop has decided to offer credit to its customers during the spring selling season. Sales are expected to be 125 bicycles. The average cost to the shop of a bicycle is $750....

See Answer

Q: Assume that your company does not anticipate paying taxes for

Assume that your company does not anticipate paying taxes for the next several years. What are the cash flows from leasing in this case?

See Answer

Q: You have your choice of two investment accounts. Investment A

You have your choice of two investment accounts. Investment A is a 13-year annuity that features end-of-month $1,250 payments and has an interest rate of 7.5 percent compounded monthly. Investment B i...

See Answer

Q: In Problem 14, what is the break-even quantity for the

In Problem 14, what is the break-even quantity for the new credit policy?Problem 14:The Snedecker Corporation is considering a change in its cash-only policy. The new terms would be net one period. Ba...

See Answer

Q: Skye Flyer, Inc., has weekly credit sales of $21,900, and

Skye Flyer, Inc., has weekly credit sales of $21,900, and the average collection period is 33 days. What is the average accounts receivable figure?

See Answer

Q: A firm offers terms of 1/10, net 30. What effective

A firm offers terms of 1/10, net 30. What effective annual interest rate does the firm earn when a customer does not take the discount? Without doing any calculations, explain what will happen to this...

See Answer

Q: Starset, Inc., has an average collection period of 27 days.

Starset, Inc., has an average collection period of 27 days. Its average daily investment in receivables is $46,300. What are annual credit sales? What is the receivables turnover? Assume 365 days per...

See Answer

Q: Essence of Skunk Fragrances, Ltd., sells 7,900 units of its

Essence of Skunk Fragrances, Ltd., sells 7,900 units of its perfume collection each year at a price per unit of $385. All sales are on credit with terms of 1/10, net 40. The discount is taken by 65 pe...

See Answer

Q: The Arizona Bay Corporation sells on credit terms of net

The Arizona Bay Corporation sells on credit terms of net 30. Its accounts are, on average, five days past due. If annual credit sales are $8.35 million, what is the company’s balance sheet amount in a...

See Answer

Q: Air Spares is a wholesaler that stocks engine components and

Air Spares is a wholesaler that stocks engine components and test equipment for the commercial aircraft industry. A new customer has placed an order for eight high-bypass turbine engines, which increa...

See Answer

Q: Prescott Bank offers you a five-year loan for $75,000 at

Prescott Bank offers you a five-year loan for $75,000 at an annual interest rate of 6.8 percent. What will your annual loan payment be?

See Answer

Q: You place an order for 300 units of inventory at

You place an order for 300 units of inventory at a unit price of $140. The supplier offers terms of 1/10, net 30. a. How long do you have to pay before the account is overdue? If you take the full per...

See Answer

Q: Suppose the spot and six-month forward rates on the Norwegian

Suppose the spot and six-month forward rates on the Norwegian krone are Kr 8.39 and Kr 8.48, respectively. The annual risk-free rate in the United States is 3.8 percent, and the annual risk-free rate...

See Answer

Q: The Red Zeppelin Corporation has annual sales of $34 million.

The Red Zeppelin Corporation has annual sales of $34 million. The average collection period is 28 days. What is the average investment in accounts receivable as shown on the balance sheet? Assume 365...

See Answer

Q: Given an interest rate of 5.3 percent per year, what

Given an interest rate of 5.3 percent per year, what is the value at date t = 7 of a perpetual stream of $6,400 payments that begins at date t = 15?

See Answer

Q: Kyoto Joe, Inc., sells earnings forecasts for Japanese securities. Its

Kyoto Joe, Inc., sells earnings forecasts for Japanese securities. Its credit terms are 2/10, net 30. Based on experience, 70 percent of all customers will take the discount.a. What is the average col...

See Answer

Q: Suppose the spot exchange rate for the Canadian dollar is

Suppose the spot exchange rate for the Canadian dollar is Can$1.29 and the six-month forward rate is Can$1.31.a. Which is worth more, a U.S. dollar or a Canadian dollar?b. Assuming absolute PPP holds,...

See Answer

Q: Suppose the Japanese yen exchange rate is ¥114 = $1,

Suppose the Japanese yen exchange rate is ¥114 = $1, and the British pound exchange rate is £1 = $1.26.a. What is the cross-rate in terms of yen per pound?b. Suppose the cross-rate is ¥147 = £1. Is th...

See Answer

Q: Use Figure 21.1 to answer the following questions: Suppose interest

Use Figure 21.1 to answer the following questions: Suppose interest rate parity holds, and the current six-month risk-free rate in the United States is 1.3 percent. What must the six-month risk free r...

See Answer

Q: The treasurer of a major U.S. firm has $30 million

The treasurer of a major U.S. firm has $30 million to invest for three months. The interest rate in the United States is .28 percent per month. The interest rate in Great Britain is .31 percent per mo...

See Answer

Q: Refer to Table 23.1 in the text to answer this

Refer to Table 23.1 in the text to answer this question. Suppose you purchase a March 2017 cocoa futures contract this day at the last price of the day. What will your profit or loss be if cocoa price...

See Answer

Q: A stock is currently priced at $47. A call option

A stock is currently priced at $47. A call option with an expiration of one year has an exercise price of $50. The risk-free rate is 12 percent per year, compounded continuously, and the standard devi...

See Answer

Q: McCann Co. has identified an investment project with the following

McCann Co. has identified an investment project with the following cash flows. If the discount rate is 10 percent, what is the present value of these cash flows? What is the present value at 18 percen...

See Answer

Q: In calculating insurance premiums, the actuarially fair insurance premium is

In calculating insurance premiums, the actuarially fair insurance premium is the premium that results in a zero NPV for both the insured and the insurer. As such, the present value of the expected los...

See Answer

Q: You observe that the inflation rate in the United States

You observe that the inflation rate in the United States is 3.5 percent per year and that T-bills currently yield 4.1 percent annually. Using the approximate international Fisher effect, what do you e...

See Answer

Q: Suppose the spot and three-month forward rates for the yen

Suppose the spot and three-month forward rates for the yen are ¥113.65 and ¥113.18, respectively. a. Is the yen expected to get stronger or weaker?b. What would you estimate is the difference between...

See Answer

Q: Suppose the spot exchange rate for the Hungarian forint is

Suppose the spot exchange rate for the Hungarian forint is HUF 289.97. The inflation rate in the United States will be 2.9 percent per year. It will be 4.5 percent in Hungary. What do you predict the...

See Answer

Q: A local finance company quotes an interest rate of 17.1

A local finance company quotes an interest rate of 17.1 percent on one-year loans. So, if you borrow $20,000, the interest for the year will be $3,420. Because you must repay a total of $23,420 in one...

See Answer

Q: Lakonishok Equipment has an investment opportunity in Europe. The

Lakonishok Equipment has an investment opportunity in Europe. The project costs €10.5 million and is expected to produce cash flows of €1.7 million in Year 1, €2.4 million in Year 2, and €3.3 million...

See Answer

Q: You are evaluating a proposed expansion of an existing subsidiary

You are evaluating a proposed expansion of an existing subsidiary located in Switzerland. The cost of the expansion would be SF 13.8 million. The cash flows from the project would be SF 3.9 million pe...

See Answer

Q: Atreides International has operations in Arrakis. The balance sheet for

Atreides International has operations in Arrakis. The balance sheet for this division in Arrakeen solaris shows assets of 38,000 solaris, debt in the amount of 12,000 solaris, and equity of 26,000 sol...

See Answer

Q: In Problem 16, assume the equity increases by 1,250 solaris

In Problem 16, assume the equity increases by 1,250 solaris due to retained earnings. If the exchange rate at the end of the year is 1.54 solaris per dollar, what does the balance sheet look like?Prob...

See Answer

Q: In Problem 14, what is the break-even price per unit

In Problem 14, what is the break-even price per unit that should be charged under the new credit policy? Assume that the sales figure under the new policy is 3,310 units and all other values remain th...

See Answer

Q: Yan Yan Corp. has a $2,000 par value bond outstanding

Yan Yan Corp. has a $2,000 par value bond outstanding with a coupon rate of 4.4 percent paid semiannually and 13 years to maturity. The yield to maturity of the bond is 4.8 percent. What is the price...

See Answer

Q: In Problem 15, what is the break-even price per unit

In Problem 15, what is the break-even price per unit under the new credit policy? Assume all other values remain the same.Problem 15:Veni, Inc., currently has an all-cash credit policy. It is consider...

See Answer

Q: Refer to Table 23.1 in the text to answer this

Refer to Table 23.1 in the text to answer this question. Suppose you sell five March 2017 silver futures contracts this day at the last price of the day. What will your profit or loss be if silver pri...

See Answer

Q: Assuming a world of corporate taxes only, show that the

Assuming a world of corporate taxes only, show that the cost of equity, RE, is as given in the chapter by M&M Proposition II with corporate taxes

See Answer

Q: Solar Engines manufactures solar engines for tractor-trailers. Given the fuel

Solar Engines manufactures solar engines for tractor-trailers. Given the fuel savings available, new orders for 125 units have been made by customers requesting credit. The variable cost is $6,900 per...

See Answer

Q: In Problem 21, assume that the probability of default is

In Problem 21, assume that the probability of default is 15 percent. Should the orders be filled now? Assume the number of repeat customers is affected by the defaults. In other words, 30 percent of t...

See Answer

Q: A five-year annuity of 10 $5,900 semiannual payments will begin

A five-year annuity of 10 $5,900 semiannual payments will begin 9 years from now, with the first payment coming 9.5 years from now. If the discount rate is 8 percent compounded monthly, what is the va...

See Answer

Q: Refer to Table 23.2 in the text to answer this

Refer to Table 23.2 in the text to answer this question. Suppose you purchase the May 2017 call option on corn futures with a strike price of $3.85. Assume you purchased the option at the last price....

See Answer

Q: Suppose a financial manager buys call options on 50,000 barrels

Suppose a financial manager buys call options on 50,000 barrels of oil with an exercise price of $57 per barrel. She simultaneously sells a put option on 50,000 barrels of oil with the same exercise p...

See Answer

Q: Refer to Table 23.2 in the text to answer this

Refer to Table 23.2 in the text to answer this question. Suppose you purchase the May 2017 put option on corn futures with a strike price of $3.80. Assume your purchase was at the last price. What is...

See Answer

Q: Suppose your company has a building worth $165 million. Because

Suppose your company has a building worth $165 million. Because it is located in a high-risk area for natural disasters, the probability of a total loss in any particular year is 1.15 percent. What is...

See Answer

Q: Union Local School District has a bond outstanding with a

Union Local School District has a bond outstanding with a coupon rate of 2.8 percent paid semiannually and 16 years to maturity. The yield to maturity on this bond is 3.4 percent, and the bond has a p...

See Answer

Q: Refer to Table 23.1 in the text to answer this

Refer to Table 23.1 in the text to answer this question. Suppose today is February 10, 2017, and your firm produces breakfast cereal and needs 145,000 bushels of corn in May 2017 for an upcoming promo...

See Answer

Q: Suppose the current exchange rate for the Polish zloty is

Suppose the current exchange rate for the Polish zloty is Z 4.04. The expected exchange rate in three years is Z 4.13. What is the difference in the annual inflation rates for the United States and Po...

See Answer

Q: Suppose your company imports computer motherboards from Singapore. The exchange

Suppose your company imports computer motherboards from Singapore. The exchange rate is given in Figure 21.1. You have just placed an order for 30,000 motherboards at a cost to you of 218.50 Singapore...

See Answer

Q: T-bills currently yield 3.4 percent. Stock in Deadwood Manufacturing is

T-bills currently yield 3.4 percent. Stock in Deadwood Manufacturing is currently selling for $58 per share. There is no possibility that the stock will be worth less than $50 per share in one year.a....

See Answer

Q: The following facts apply to a convertible bond making semiannual

The following facts apply to a convertible bond making semiannual payments:Conversion price …………………………………………………………………… $37/shareCoupon rate …………………………………………………………………………………. 2.6%Par value …………………………………...

See Answer

Q: You have been hired to value a new 30-year callable,

You have been hired to value a new 30-year callable, convertible bond. The bond has a coupon rate of 2.7 percent, payable semiannually, and its face value is $1,000. The conversion price is $54, and t...

See Answer

Q: Suppose you are going to receive $13,500 per year for

Suppose you are going to receive $13,500 per year for five years. The appropriate interest rate is 6.8 percent. a. What is the present value of the payments if they are in the form of an ordinary annu...

See Answer

Q: A bond with 20 detachable warrants has just been offered

A bond with 20 detachable warrants has just been offered for sale at $1,000. The bond matures in 20 years and has an annual coupon of $24. Each warrant gives the owner the right to purchase two shares...

See Answer

Q: Your company is deciding whether to invest in a new

Your company is deciding whether to invest in a new machine. The new machine will increase cash flow by $275,000 per year. You believe the technology used in the machine has a 10-year life; in other w...

See Answer

Q: We are examining a new project. We expect to sell

We are examining a new project. We expect to sell 7,100 units per year at $56 net cash flow apiece for the next 10 years. In other words, the annual cash flow is projected to be $56 × 7,100 = $397,600...

See Answer

Q: If Treasury bills are currently paying 4.7 percent and the

If Treasury bills are currently paying 4.7 percent and the inflation rate is 2.2 percent, what is the approximate real rate of interest? The exact real rate?

See Answer

Q: In Problem 14, suppose you think it is likely that

In Problem 14, suppose you think it is likely that expected sales will be revised upward to 10,800 units if the first year is a success and revised downward to 3,900 units if the first year is not a s...

See Answer

Q: In Problem 15, suppose the scale of the project can

In Problem 15, suppose the scale of the project can be doubled in one year in the sense that twice as many units can be produced and sold. Naturally, expansion would be desirable only if the project i...

See Answer

Q: Suppose a share of stock sells for $63. The risk-free

Suppose a share of stock sells for $63. The risk-free rate is 5 percent, and the stock price in one year will be either $70 or $80.a. What is the value of a call option with an exercise price of $70?b...

See Answer

Q: Which of the following two sets of relationships, at time

Which of the following two sets of relationships, at time of issuance for convertible bonds, is more typical? Why?,,,

See Answer

Q: Campbell, Inc., has a $1,000 face value convertible bond issue

Campbell, Inc., has a $1,000 face value convertible bond issue that is currently selling in the market for $960. Each bond is exchangeable at any time for 18 shares of the company’s stock. The convert...

See Answer

Q: Use the option quote information shown here to answer the

Use the option quote information shown here to answer the questions that follow. The stock is currently selling for $85.a. Are the call options in the money? What is the intrinsic value of an RWJ Corp...

See Answer

Q: Liberty Products, Inc., is considering a new product launch. The

Liberty Products, Inc., is considering a new product launch. The firm expects to have annual operating cash flow of $5.3 million for the next eight years. The company uses a discount rate of 11 percen...

See Answer

Q: You want to buy a new sports car from Muscle

You want to buy a new sports car from Muscle Motors for $57,500. The contract is in the form of a 60-month annuity due at an APR of 5.9 percent. What will your monthly payment be?

See Answer

Q: You have been hired to value a new 25-year callable,

You have been hired to value a new 25-year callable, convertible bond. The bond has a coupon rate of 2.3 percent, payable annually. The conversion price is $68, and the stock currently sells for $27.8...

See Answer

Q: Consider the following project of Hand Clapper, Inc. The company

Consider the following project of Hand Clapper, Inc. The company is considering a four-year project to manufacture clap-command garage door openers. This project requires an initial investment of $14...

See Answer

Q: Suppose the real rate is 2.1 percent and the inflation

Suppose the real rate is 2.1 percent and the inflation rate is 3.4 percent. What rate would you expect to see on a Treasury bill?

See Answer

Q: Minder Industries stock has a beta of 1.08. The company

Minder Industries stock has a beta of 1.08. The company just paid a dividend of $.65, and the dividends are expected to grow at 4 percent. The expected return on the market is 10.5 percent, and Treasu...

See Answer

Q: Pearl Corp. is expected to have an EBIT of $1.8

Pearl Corp. is expected to have an EBIT of $1.8 million next year. Depreciation, the increase in net working capital, and capital spending are expected to be $155,000, $75,000, and $115,000, respectiv...

See Answer

Q: You have looked at the current financial statements for Reigle

You have looked at the current financial statements for Reigle Homes, Co. The company has an EBIT of $3.15 million this year. Depreciation, the increase in net working capital, and capital spending we...

See Answer

Q: Use the option quote information shown here to answer the

Use the option quote information shown here to answer the questions that follow. The stock is currently selling for $40.a. Suppose you buy 10 contracts of the February 38 call option. How much will yo...

See Answer

Q: The price of Chive Corp. stock will be either $67

The price of Chive Corp. stock will be either $67 or $91 at the end of the year. Call options are available with one year to expiration. T-bills currently yield 4 percent.a. Suppose the current price...

See Answer

Q: The price of Cilantro, Inc., stock will be either $70

The price of Cilantro, Inc., stock will be either $70 or $90 at the end of the year. Call options are available with one year to expiration. T-bills currently yield 6 percent.a. Suppose the current pr...

See Answer

Q: A one-year call option contract on Cheesy Poofs Co. stock

A one-year call option contract on Cheesy Poofs Co. stock sells for $845. In one year, the stock will be worth $64 or $81 per share. The exercise price on the call option is $70. What is the current v...

See Answer

Q: Rackin Pinion Corporation’s assets are currently worth $1,065. In one

Rackin Pinion Corporation’s assets are currently worth $1,065. In one year, they will be worth either $1,000 or $1,340. The risk-free interest rate is 3.9 percent. Suppose the company has an outstandi...

See Answer

Q: Prepare an amortization schedule for a five-year loan of $71,500.

Prepare an amortization schedule for a five-year loan of $71,500. The interest rate is 7 percent per year, and the loan calls for equal annual payments. How much interest is paid in the third year? Ho...

See Answer

Q: Buckeye Industries has a bond issue with a face value

Buckeye Industries has a bond issue with a face value of $1,000 that is coming due in one year. The value of the company’s assets is currently $1,040. Urban Meyer, the CEO, believes that the assets in...

See Answer

Q: An investment offers a total return of 12.3 percent over

An investment offers a total return of 12.3 percent over the coming year. Janice Yellen thinks the total real return on this investment will be only 8 percent. What does Janice believe the inflation r...

See Answer

Q: A $1,000 par convertible debenture has a conversion price for

A $1,000 par convertible debenture has a conversion price for common stock of $27 per share. With the common stock selling at $31, what is the conversion value of the bond?

See Answer

Q: If you have $1,275 today, how much will it be

If you have $1,275 today, how much will it be worth in six years at 8 percent per year compounded continuously?

See Answer

Q: In Problem 9, suppose you wanted the option to sell

In Problem 9, suppose you wanted the option to sell the land to the buyer in one year. Assuming all the facts are the same, describe the transaction that would occur today. What is the price of the tr...

See Answer

Q: A call option with an exercise price of $25 and

A call option with an exercise price of $25 and four months to expiration has a price of $2.75. The stock is currently priced at $23.80, and the risk-free rate is 2.5 percent per year, compounded cont...

See Answer

Q: A call option matures in six months. The underlying stock

A call option matures in six months. The underlying stock price is $75, and the stock’s return has a standard deviation of 20 percent per year. The risk-free rate is 4 percent per year, compounded con...

See Answer

Q: A call option has an exercise price of $60 and

A call option has an exercise price of $60 and matures in six months. The current stock price is $64, and the risk-free rate is 5 percent per year, compounded continuously. What is the price of the ca...

See Answer

Q: If you need $20,000 in 12 years, how much will

If you need $20,000 in 12 years, how much will you need to deposit today if you can earn 9 percent per year compounded continuously?

See Answer

Q: A stock is currently selling for $67 per share. A

A stock is currently selling for $67 per share. A call option with an exercise price of $70 sells for $3.21 and expires in three months. If the risk-free rate of interest is 2.6 percent per year, comp...

See Answer

Q: A put option that expires in six months with an

A put option that expires in six months with an exercise price of $45 sells for $2.34. The stock is currently priced at $48, and the risk-free rate is 3.5 percent per year, compounded continuously. Wh...

See Answer

Q: Rework Problem 55 assuming that the loan agreement calls for

Rework Problem 55 assuming that the loan agreement calls for a principal reduction of $14,300 every year instead of equal annual payments.Data from Problem 55:Prepare an amortization schedule for a fi...

See Answer

Q: Say you own an asset that had a total return

Say you own an asset that had a total return last year of 11.65 percent. If the inflation rate last year was 2.75 percent, what was your real return?

See Answer

Q: A put option and a call option with an exercise

A put option and a call option with an exercise price of $70 and three months to expiration sell for $1.30 and $6.25, respectively. If the risk-free rate is 3.1 percent per year, compounded continuous...

See Answer

Q: A put option and call option with an exercise price

A put option and call option with an exercise price of $50 expire in four months and sell for $5.99 and $8.64, respectively. If the stock is currently priced at $52.27, what is the annual continuously...

See Answer

Q: What are the prices of a call option and a

What are the prices of a call option and a put option with the following characteristics?,,,

See Answer

Q: What are the deltas of a call option and a

What are the deltas of a call option and a put option with the following characteristics? What does the delta of the option tell you?,,,

See Answer

Q: You own a lot in Key West, Florida, that is

You own a lot in Key West, Florida, that is currently unused. Similar lots have recently sold for $1,250,000. Over the past five years, the price of land in the area has increased 7 percent per year,...

See Answer

Q: Pearl, Inc., has offered $228 million cash for all of

Pearl, Inc., has offered $228 million cash for all of the common stock in Jam Corporation. Based on recent market information, Jam is worth $214 million as an independent operation. If the merger make...

See Answer

Q: Consider the following premerger information about Firm A and Firm

Consider the following premerger information about Firm A and Firm B:Assume that Firm A acquires Firm B via an exchange of stock at a price of $49 for each share of B’s stock. Both F...

See Answer

Q: You are given the following information concerning options on a

You are given the following information concerning options on a particular stock:a. What is the intrinsic value of the call option? Of the put option?b. What is the time value of the call option? Of t...

See Answer

Q: Fly-By-Night Couriers is analyzing the possible acquisition of Flash-in-the-Pan Restaurants.

Fly-By-Night Couriers is analyzing the possible acquisition of Flash-in-the-Pan Restaurants.Neither firm has debt. The forecasts of Fly-By-Night show that the purchase would increase its annual aftert...

See Answer

Q: Consider the following premerger information about Firm X and Firm

Consider the following premerger information about Firm X and Firm Y:Assume that Firm X acquires Firm Y by issuing new long-term debt for all the shares outstanding at a merger premium of $6 per share...

See Answer

Q: Locate the Treasury issue in Figure 7.4 maturing in May

Locate the Treasury issue in Figure 7.4 maturing in May 2038. What is its coupon rate? What is its bid price? What was the previous day’s asked price? Assume a par value of $10,000....

See Answer

Q: Bilbo Baggins wants to save money to meet three objectives.

Bilbo Baggins wants to save money to meet three objectives. First, he would like to be able to retire 30 years from now with retirement income of $17,500 per month for 25 years, with the first payment...

See Answer

Q: Assume that the following balance sheets are stated at book

Assume that the following balance sheets are stated at book value. Suppose that Meat Co. purchases Loaf, Inc.The fair market value of Loaf’s fixed assets is $9,800 versus the $6,900...

See Answer

Q: Silver Enterprises has acquired All Gold Mining in a merger

Silver Enterprises has acquired All Gold Mining in a merger transaction. Construct the balance sheet for the new corporation if the merger is treated as a purchase of interests for accounting purposes...

See Answer

Q: Penn Corp. is analyzing the possible acquisition of Teller Company.

Penn Corp. is analyzing the possible acquisition of Teller Company. Both firms have no debt. Penn believes the acquisition will increase its total aftertax annual cash flows by $1.6 million indefinite...

See Answer

Q: Three Guys Burgers, Inc., has offered $16.5 million for all

Three Guys Burgers, Inc., has offered $16.5 million for all of the common stock in Two Guys Fries, Corp. The current market capitalization of Two Guys as an independent company is $13.4 million. Assum...

See Answer

Q: The shareholders of Bread Company have voted in favor of

The shareholders of Bread Company have voted in favor of a buyout offer from Butter Corporation. Information about each firm is given here:Bread’s shareholders will receive one share...

See Answer

Q: Consider the following premerger information about a bidding firm (Firm

Consider the following premerger information about a bidding firm (Firm B) and a target firm (Firm T). Assume that both firms have no debt outstanding.Firm B has estimated that the value of the synerg...

See Answer

Q: In Problem 8, are the shareholders of Firm T better

In Problem 8, are the shareholders of Firm T better off with the cash offer or the stock offer? At what exchange ratio of B shares to T shares would the shareholders in T be indifferent between the tw...

See Answer

Q: Sunset Boards is a small company that manufactures and sells

Sunset Boards is a small company that manufactures and sells surfboards in Malibu. Tad Marks, the founder of the company, is in charge of the design and sale of the surfboards, but his background is i...

See Answer

Q: You have recently been hired by Swan Motors, Inc. (SMI),

You have recently been hired by Swan Motors, Inc. (SMI), in its relatively new treasury management department. SMI was founded eight years ago by Joe Swan. Joe found a method to manufacture a cheaper...

See Answer

Q: Assume that the tax rate is 21 percent. You can

Assume that the tax rate is 21 percent. You can borrow at 8 percent before taxes. Should you lease or buy?

See Answer

Q: Mark Sexton and Todd Story have been discussing the future

Mark Sexton and Todd Story have been discussing the future of S&S Air. The company has been experiencing fast growth, and the two see only clear skies in the company’s future. However, the fast growth...

See Answer

Q: After deciding to buy a new car, you can either

After deciding to buy a new car, you can either lease the car or purchase it on a three-year loan. The car you wish to buy costs $43,000. The dealer has a special leasing arrangement where you pay $4,...

See Answer

Q: Stephenson Real Estate Company was founded 25 years ago by

Stephenson Real Estate Company was founded 25 years ago by the current CEO, Robert Stephenson. The company purchases real estate, including land and buildings, and rents the property to tenants. The c...

See Answer

Q: You have recently been hired by Piepkorn Manufacturing to work

You have recently been hired by Piepkorn Manufacturing to work in the newly established treasury department. Piepkorn Manufacturing is a small company that produces cardboard boxes in a variety of siz...

See Answer

Q: Webb Corporation was founded 20 years ago by its president,

Webb Corporation was founded 20 years ago by its president, Bryan Webb. The company originally began as a mail-order company, but it has grown rapidly in recent years, in large part due to its website...

See Answer

Q: Sterling Wyatt, the president of Howlett Industries, has been exploring

Sterling Wyatt, the president of Howlett Industries, has been exploring ways of improving the company’s financial performance. Howlett manufactures and sells office equipment to reta...

See Answer

Q: Joi Chatman recently received her finance degree and has decided

Joi Chatman recently received her finance degree and has decided to enter the mortgage broker business. Rather than working for someone else, she will open her own shop. Her cousin Mike has approached...

See Answer

Q: S&S Air is preparing its first public securities offering. In

S&S Air is preparing its first public securities offering. In consultation with Renata Harper of underwriter Raines and Warren, Chris Guthrie decided that a convertible bond with a 20-year maturity wa...

See Answer

Q: As a new graduate, you’ve taken a management position with

As a new graduate, you’ve taken a management position with Exotic Cuisines, Inc., a restaurant chain that just went public last year. The company’s restaurants specialize in exotic main dishes, using...

See Answer

Q: Birdie Golf, Inc., has been in merger talks with Hybrid

Birdie Golf, Inc., has been in merger talks with Hybrid Golf Company for the past six months. After several rounds of negotiations, the offer under discussion is a cash offer of $185 million for Hybri...

See Answer

Q: Suppose Sunburn Sunscreen and Frostbite Thermal wear in the previous

Suppose Sunburn Sunscreen and Frostbite Thermal wear in the previous problems have decided to merge. Because the two companies have seasonal sales, the combined firm’s return on assets will have a sta...

See Answer

Q: Warf Computers has decided to proceed with the manufacture and

Warf Computers has decided to proceed with the manufacture and distribution of the virtual keyboard (VK) the company has developed. To undertake this venture, the company needs to obtain equipment for...

See Answer

Q: First City Bank pays 9 percent simple interest on its

First City Bank pays 9 percent simple interest on its savings account balances, whereas Second City Bank pays 9 percent interest compounded annually. If you made a deposit of $7,500 in each bank, how...

See Answer

Q: In Problem 4, over what range of lease payments will

In Problem 4, over what range of lease payments will the lease be profitable for both parties?Problem 4:Assume that your company does not anticipate paying taxes for the next several years. What are t...

See Answer

Q: An All-Pro defensive lineman is in contract negotiations. The team

An All-Pro defensive lineman is in contract negotiations. The team has offered the following salary structure:Time ……………………………………………………………………………..…….. Salary0 ……………………………………………………………………………….. $8,400,0...

See Answer

Q: For each of the following, compute the future value:

For each of the following, compute the future value:

See Answer

Q: For each of the following, compute the present value:

For each of the following, compute the present value:

See Answer

Q: Solve for the unknown interest rate in each of the

Solve for the unknown interest rate in each of the following:

See Answer

Q: Solve for the unknown number of years in each of

Solve for the unknown number of years in each of the following:

See Answer

Q: Assume the total cost of a college education will be

Assume the total cost of a college education will be $345,000 when your child enters college in 18 years. You presently have $73,000 to invest. What annual rate of interest must you earn on your inves...

See Answer

Q: At 6.1 percent interest, how long does it take to

At 6.1 percent interest, how long does it take to double your money? To quadruple it?

See Answer

Q: In the previous problem, suppose a sales associate told you

In the previous problem, suppose a sales associate told you the policy costs $800,000. At what interest rate would this be a fair deal?Data from Problem 10:The Maybe Pay Life Insurance Co. is trying t...

See Answer

Q: Locate the Treasury bond in Figure 7.4 maturing in February

Locate the Treasury bond in Figure 7.4 maturing in February 2040. Is this a premium or a discount bond? What is its current yield? What is its yield to maturity? What is the bid-ask spread in dollars?...

See Answer

Q: According to the Census Bureau, in October 2016, the average

According to the Census Bureau, in October 2016, the average house price in the United States was $354,900. In October 2000, the average price was $215,100. What was the annual increase in the price o...

See Answer

Q: You’re trying to save to buy a new $245,000 Ferrari.

You’re trying to save to buy a new $245,000 Ferrari. You have $50,000 today that can be invested at your bank. The bank pays 4.3 percent annual interest on its accounts. How long will it be before you...

See Answer

Q: Imprudential, Inc., has an unfunded pension liability of $415 million

Imprudential, Inc., has an unfunded pension liability of $415 million that must be paid in 20 years. To assess the value of the firm’s stock, financial analysts want to discount this liability back to...

See Answer

Q: You have just received notification that you have won the

You have just received notification that you have won the $2 million first prize in the Centennial Lottery. However, the prize will be awarded on your 100th birthday (assuming you’re around to collect...

See Answer

Q: This question illustrates what is known as discount interest. Imagine

This question illustrates what is known as discount interest. Imagine you are discussing a loan with a somewhat unscrupulous lender. You want to borrow $25,000 for one year. The interest rate is 14.9...

See Answer

Q: Your coin collection contains fifty 1952 silver dollars. If your

Your coin collection contains fifty 1952 silver dollars. If your grandparents purchased them for their face value when they were new, how much will your collection be worth when you retire in 2067, as...

See Answer

Q: In 1895, the first U.S. Open Golf Championship was held.

In 1895, the first U.S. Open Golf Championship was held. The winner’s prize money was $150. In 2016, the winner’s check was $1,800,000. What was the percentage increase per year in the winner’s check...

See Answer

Q: The “Brasher doubloon,” which was featured in the plot of

The “Brasher doubloon,” which was featured in the plot of the Raymond Chandler novel, The High Window, was sold at auction in 2014 for $4,582,500. The coin had a face value of $15 when it was first is...

See Answer

Q: Refer back to the Series EE savings bonds we discussed

Refer back to the Series EE savings bonds we discussed at the very beginning of the chapter.a. Assuming you purchased a $50 face value bond, what is the exact rate of return you would earn if you held...

See Answer

Q: Suppose you are still committed to owning a $245,000 Ferrari

Suppose you are still committed to owning a $245,000 Ferrari (see Problem 9). If you believe your mutual fund can achieve an annual rate of return of 11.2 percent and you want to buy the car in 9 year...

See Answer

Q: Bond X is a premium bond making semiannual payments. The

Bond X is a premium bond making semiannual payments. The bond pays a coupon rate of 7.4 percent, has a YTM of 6.8 percent, and has 13 years to maturity. Bond Y is a discount bond making semiannual pay...

See Answer

Q: You have just made your first $5,500 contribution to your

You have just made your first $5,500 contribution to your retirement account. Assuming you earn a return of 10 percent per year and make no additional contributions, what will your account be worth wh...

See Answer

Q: You are scheduled to receive $20,000 in two years. When

You are scheduled to receive $20,000 in two years. When you receive it, you will invest it for six more years at 6.8 percent per year. How much will you have in eight years?

See Answer

Q: You expect to receive $10,000 at graduation in two years.

You expect to receive $10,000 at graduation in two years. You plan on investing it at 9 percent until you have $60,000. How long will you wait from now?

See Answer

Q: The Johnson Company sells 2,400 pairs of running shoes per

The Johnson Company sells 2,400 pairs of running shoes per month at a cash price of $99 per pair. The firm is considering a new policy that involves 30 days’ credit and an increase in price to $100 pe...

See Answer

Q: Silicon Wafers, Inc. (SWI), is debating whether or not to

Silicon Wafers, Inc. (SWI), is debating whether or not to extend credit to a particular customer. SWI’s products, primarily used in the manufacture of semiconductors, currently sell for $975 per unit....

See Answer

Q: You are serving on a jury. A plaintiff is suing

You are serving on a jury. A plaintiff is suing the city for injuries sustained after a freak street sweeper accident. In the trial, doctors testified that it will be five years before the plaintiff i...

See Answer

Q: Consider the following information about two alternative credit strategies:

Consider the following information about two alternative credit strategies:The higher cost per unit reflects the expense associated with credit orders, and the higher price per unit reflects the exist...

See Answer

Q: Suppose a corporation currently sells Q units per month for

Suppose a corporation currently sells Q units per month for a cash-only price of P. Under a new credit policy that allows one month’s credit, the quantity sold will be Q′ and the price per unit will b...

See Answer

Q: All Night, Inc., uses a Miller-Orr cash management approach with

All Night, Inc., uses a Miller-Orr cash management approach with a lower limit of $43,000, an upper limit of $125,000, and a target balance of $80,000. Explain what each of these points represents; th...

See Answer

Q: Slap Shot Corporation has a fixed cost of $40 associated

Slap Shot Corporation has a fixed cost of $40 associated with buying and selling marketable securities. The interest rate is currently .013 percent per day, and the firm has estimated that the standar...

See Answer

Q: Both Bond Sam and Bond Dave have 7.3 percent coupons,

Both Bond Sam and Bond Dave have 7.3 percent coupons, make semiannual payments, and are priced at par value. Bond Sam has three years to maturity, whereas Bond Dave has 20 years to maturity. If intere...

See Answer

Q: Based on the Miller-Orr model, describe what will happen to

Based on the Miller-Orr model, describe what will happen to the lower limit, the upper limit, and the spread (the distance between the two) if the variation in net cash flow grows. Give an intuitive e...

See Answer

Q: The variance of the daily cash flows for the Pele

The variance of the daily cash flows for the Pele Bicycle Shop is $890,000. The opportunity cost to the firm of holding cash is 4.1 percent per year. What should the target cash level and the upper li...

See Answer

Q: Rise Against Corporation has determined that its target cash balance

Rise Against Corporation has determined that its target cash balance if it uses the BAT model is $5,100. The total cash needed for the year is $31,000, and the order cost is $10. What interest rate mu...

See Answer

Q: Given the following information, calculate the target cash balance using

Given the following information, calculate the target cash balance using the BAT model:Annual interest rate ………………………………………………………………………. 4.5%Fixed order cost ………………………………………………………………………………. $25Total c...

See Answer

Q: White Whale Corporation has an average daily cash balance of

White Whale Corporation has an average daily cash balance of $1,700. Total cash needed for the year is $64,000. The interest rate is 5 percent, and replenishing the cash costs $8 each time. What are t...

See Answer

Q: Debit and Credit Bookkeepers needs a total of $21,000 in

Debit and Credit Bookkeepers needs a total of $21,000 in cash during the year for transactions and other purposes. Whenever cash runs low, it sells $1,500 in securities and transfers in the cash. The...

See Answer

Q: You are looking at a one-year loan of $10,000. The

You are looking at a one-year loan of $10,000. The interest rate is quoted as 9.8 percent plus 2 points. A point on a loan is 1 percent (one percentage point) of the loan amount. Quotes similar to thi...

See Answer

Q: The All Day Company is currently holding $690,000 in cash.

The All Day Company is currently holding $690,000 in cash. It projects that over the next year its cash outflows will exceed cash inflows by $140,000 per month. How much of the current cash holdings s...

See Answer

Q: Chris Guthrie was recently hired by S&S Air, Inc., to

Chris Guthrie was recently hired by S&S Air, Inc., to assist the company with its financial planning and to evaluate the company’s performance. Chris graduated from college five...

See Answer

Q: After Chris completed the ratio analysis for S&S Air (see

After Chris completed the ratio analysis for S&S Air (see Chapter 3), Mark and Todd approached him about planning for next year’s sales. The company had historically used little...

See Answer

Q: Investment X offers to pay you $4,200 per year for

Investment X offers to pay you $4,200 per year for eight years, whereas Investment Y offers to pay you $6,100 per year for five years. Which of these cash flow streams has the higher present value if...

See Answer

Q: Ben Bates graduated from college six years ago with a

Ben Bates graduated from college six years ago with a finance undergraduate degree. Although he is satisfied with his current job, his goal is to become an investment banker. He feels that an MBA degr...

See Answer

Q: Ragan, Inc., was founded nine years ago by brother and

Ragan, Inc., was founded nine years ago by brother and sister Carrington and Genevieve Ragan. The company manufactures and installs commercial heating, ventilation, and cooling (HVAC) units. Ragan, In...

See Answer

Q: Seth Bullock, the owner of Bullock Gold Mining, is evaluating

Seth Bullock, the owner of Bullock Gold Mining, is evaluating a new gold mine in South Dakota. Dan Dority, the company’s geologist, has just finished his analysis of the mine site. He has estimated th...

See Answer

Q: Conch Republic Electronics is a midsized electronics manufacturer located in

Conch Republic Electronics is a midsized electronics manufacturer located in Key West, Florida. The company president is Shelley Couts, who inherited the company. When it was founded over 70 years ago...

See Answer

Q: Shelley Couts, the owner of Conch Republic Electronics, has received

Shelley Couts, the owner of Conch Republic Electronics, has received the capital budgeting analysis from Jay McCanless for the new smartphone the company is considering. Shelley is pleased with the re...

See Answer

Q: "You recently graduated from college, and your job search led

"You recently graduated from college, and your job search led you to S&S Air. Because you felt the company’s business was taking off, you accepted a job offer. The first day on t...

See Answer

Q: Joey Moss, a recent finance graduate, has just begun his

Joey Moss, a recent finance graduate, has just begun his job with the investment firm of Covili and Wyatt. Paul Covili, one of the firm’s founders, has been talking to Joey about the firm’s investment...

See Answer

Q: The interest rate on a one-year loan is quoted as

The interest rate on a one-year loan is quoted as 12 percent plus 3 points (see the previous problem). What is the EAR? Is your answer affected by the loan amount?

See Answer

Q: You are buying a house and will borrow $225,000 on

You are buying a house and will borrow $225,000 on a 30-year fixed rate mortgage with monthly payments to finance the purchase. Your loan officer has offered you a mortgage with an APR of 4.3 percent....

See Answer

Q: In the previous problem, suppose that you believe that you

In the previous problem, suppose that you believe that you will only live in the house for eight years before selling the house and buying another house. This means that in eight years, you will pay o...

See Answer

Q: Bond J has a coupon rate of 3 percent. Bond

Bond J has a coupon rate of 3 percent. Bond K has a coupon rate of 9 percent. Both bonds have 14 years to maturity, make semiannual payments, and have a YTM of 6 percent. If interest rates suddenly ri...

See Answer

Q: Two banks in the area offer 30-year, $275,000 mortgages at

Two banks in the area offer 30-year, $275,000 mortgages at 5.1 percent and charge a $4,300 loan application fee. However, the application fee charged by Insecurity Bank and Trust is refundable if the...

See Answer

Q: This problem illustrates a deceptive way of quoting interest rates

This problem illustrates a deceptive way of quoting interest rates called add-on interest. Imagine that you see an advertisement for Crazy Judy’s Stereo City that reads something like this: “$1,000 In...

See Answer

Q: You have successfully started and operated a company for the

You have successfully started and operated a company for the past 10 years. You have decided that it is time to sell your company and spend time on the beaches of Hawaii. A potential buyer is interest...

See Answer

Q: Rework Problem 1 assuming that the scanner will be depreciated

Rework Problem 1 assuming that the scanner will be depreciated as three-year property under MACRS (see Chapter 10 for the depreciation allowances).Problem 1:Assume that the tax rate is 21 percent. You...

See Answer

Q: Your Christmas ski vacation was great, but it unfortunately ran

Your Christmas ski vacation was great, but it unfortunately ran a bit over budget. All is not lost: You just received an offer in the mail to transfer your $15,000 balance from your current credit car...

See Answer

Q: An insurance company is offering a new policy to its

An insurance company is offering a new policy to its customers. Typically, the policy is bought by a parent or grandparent for a child at the child’s birth. The purchaser (say, the parent) makes the f...

See Answer

Q: You have just arranged for a $2,350,000 mortgage to finance

You have just arranged for a $2,350,000 mortgage to finance the purchase of a large tract of land. The mortgage has an APR of 5.2 percent, and it calls for monthly payments over the next 30 years. How...

See Answer

Q: A financial planning service offers a college savings program.

A financial planning service offers a college savings program. The plan calls for you to make six annual payments of $15,000 each, with the first payment occurring today, your child’s 12th birthday. B...

See Answer

Q: Your financial planner offers you two different investment plans. Plan

Your financial planner offers you two different investment plans. Plan X is an annual perpetuity of $35,000 per year. Plan Y is an annuity for 15 years and an annual payment of $47,000. Both plans wil...

See Answer

Q: What is the value of an investment that pays $25,000

What is the value of an investment that pays $25,000 every other year forever, if the first payment occurs one year from today and the discount rate is 7 percent compounded daily? What is the value to...

See Answer

Q: Workman Software has 6.4 percent coupon bonds on the market

Workman Software has 6.4 percent coupon bonds on the market with 18 years to maturity. The bonds make semiannual payments and currently sell for 94.31 percent of par. What is the current yield on the...

See Answer

Q: Twist Corp. has a current accounts receivable balance of $537,810.

Twist Corp. has a current accounts receivable balance of $537,810. Credit sales for the year just ended were $5,473,640. What is the receivables turnover? The days’ sales in receivables? How long did...

See Answer

Q: You have 45 years left until retirement and want to

You have 45 years left until retirement and want to retire with $4 million. Your salary is paid annually, and you will receive $50,000 at the end of the current year. Your salary will increase at 3 pe...

See Answer

Q: A check-cashing store is in the business of making personal

A check-cashing store is in the business of making personal loans to walk-up customers. The store makes only one-week loans at 6.8 percent interest per week. a. What APR must the store report to its c...

See Answer

Q: The King Corporation has ending inventory of $386,735, and cost

The King Corporation has ending inventory of $386,735, and cost of goods sold for the year just ended was $4,981,315. What is the inventory turnover? The days’ sales in inventory? How long on average...

See Answer

Q: The Wildcat Oil Company is trying to decide whether to

The Wildcat Oil Company is trying to decide whether to lease or buy a new computer-assisted drilling system for its oil exploration business. Management has decided that it must use the system to stay...

See Answer

Q: Queen, Inc., has a total debt ratio of .46. What

Queen, Inc., has a total debt ratio of .46. What is its debt-equity ratio? What is its equity multiplier?

See Answer

Q: Makers Corp. had additions to retained earnings for the year

Makers Corp. had additions to retained earnings for the year just ended of $415,000. The firm paid out $220,000 in cash dividends, and it has ending total equity of $5.6 million. If the company curren...

See Answer

Q: If Roten Rooters, Inc., has an equity multiplier of 1.27,

If Roten Rooters, Inc., has an equity multiplier of 1.27, total asset turnover of 2.10, and a profit margin of 6.1 percent, what is its ROE?

See Answer

Q: Jack Corp. has a profit margin of 6.4 percent, total

Jack Corp. has a profit margin of 6.4 percent, total asset turnover of 1.77, and ROE of 15.84 percent. What is this firm’s debt-equity ratio?

See Answer

Q: Based only on the following information for Thrice Corp., did

Based only on the following information for Thrice Corp., did cash go up or down? By how much? Classify each event as a source or use of cash.Decrease in inventory ………………………………………………………………………….. $375D...

See Answer

Q: Chamberlain Co. wants to issue new 20-year bonds for some

Chamberlain Co. wants to issue new 20-year bonds for some much-needed expansion projects. The company currently has 6 percent coupon bonds on the market that sell for $1,083, make semiannual payments,...

See Answer

Q: Consider the following simplified financial statements for the Wims Corporation

Consider the following simplified financial statements for the Wims Corporation (assuming no income taxes):

See Answer

Q: The balance sheet for the Heir Jordan Corporation follows. Based

The balance sheet for the Heir Jordan Corporation follows. Based on this information and the income statement in the previous problem, supply the missing information using the percentage of sales appr...

See Answer

Q: From the previous two questions, prepare a pro forma balance

From the previous two questions, prepare a pro forma balance sheet showing EFN, assuming an increase in sales of 15 percent, no new external debt or equity financing, and a constant payout ratio.Data...

See Answer

Q: If A7X Co. has an ROA of 7.6 percent and

If A7X Co. has an ROA of 7.6 percent and a payout ratio of 25 percent, what is its internal growth rate?

See Answer

Q: If Synyster Corp. has an ROE of 14.7 percent and

If Synyster Corp. has an ROE of 14.7 percent and a payout ratio of 30 percent, what is itss ustainable growth rate?

See Answer

Q: The Wildcat Oil Company is trying to decide whether to

The Wildcat Oil Company is trying to decide whether to lease or buy a new computer-assisted drilling system for its oil exploration business. Management has decided that it must use the system to stay...

See Answer

Q: Based on the following information, calculate the sustainable growth rate

Based on the following information, calculate the sustainable growth rate for Kaleb’s Heavy Equipment:,,,

See Answer

Q: Assuming the following ratios are constant, what is the sustainable

Assuming the following ratios are constant, what is the sustainable growth rate?,,,

See Answer

Q: Hodgkiss Mfg., Inc., is currently operating at only 91 percent

Hodgkiss Mfg., Inc., is currently operating at only 91 percent of fixed asset capacity. Current sales are $715,000. How fast can sales grow before any new fixed assets are needed?

See Answer

Q: For the company in Problem 16, suppose fixed assets are

For the company in Problem 16, suppose fixed assets are $520,000 and sales are projected to grow to $790,000. How much in new fixed assets are required to support this growth in sales? Assume the comp...

See Answer

Q: You purchase a bond with an invoice price of $948.

You purchase a bond with an invoice price of $948. The bond has a coupon rate of 5.9 percent, and there are four months to the next semiannual coupon date. What is the clean price of the bond?

See Answer

Q: Ramble On Co. wishes to maintain a growth rate of

Ramble On Co. wishes to maintain a growth rate of 12 percent per year, a debt-equity ratio of .90, and a dividend payout ratio of 25 percent. The ratio of total assets to sales is constant at 0.85. Wh...

See Answer

Q: A firm wishes to maintain an internal growth rate of

A firm wishes to maintain an internal growth rate of 7.1 percent and a dividend payout ratio of 25 percent. The current profit margin is 6.5 percent, and the firm uses no external financing sources. W...

See Answer

Q: In Question 1, assume the company pays out half of

In Question 1, assume the company pays out half of net income in the form of a cash dividend. Costs and assets vary with sales, but debt and equity do not. Prepare the pro forma statements and determi...

See Answer

Q: Based on the following information, calculate the sustainable growth rate

Based on the following information, calculate the sustainable growth rate for Hendrix Guitars, Inc.:,,,

See Answer

Q: You’ve collected the following information about Molino, Inc.:

You’ve collected the following information about Molino, Inc.:What is the sustainable growth rate for the company? If it does grow at this rate, how much new borrowing will take plac...

See Answer

Q: Calculate the internal growth rate for the company in Problem

Calculate the internal growth rate for the company in Problem 22. Now calculate the internal growth rate using ROA × b for both beginning of period and end of period total assets. What do you observe?...

See Answer

Q: You purchase a bond with a coupon rate of 5.3

You purchase a bond with a coupon rate of 5.3 percent and a clean price of $951. If the next semiannual coupon payment is due in two months, what is the invoice price?

See Answer

Q: Excey Corp. has 8 percent coupon bonds making annual payments

Excey Corp. has 8 percent coupon bonds making annual payments with a YTM of 7.2 percent. The current yield on these bonds is 7.55 percent. How many years do these bonds have left until they mature?

See Answer

Q: Find the EAR in each of the following cases:

Find the EAR in each of the following cases:

See Answer

Q: A company has a single zero coupon bond outstanding that

A company has a single zero coupon bond outstanding that matures in five years with a face value of $17.5 million. The current value of the company’s assets is $15.9 million, and the standard deviatio...

See Answer

Q: Suppose the following bond quotes for IOU Corporation appear in

Suppose the following bond quotes for IOU Corporation appear in the financial page of today’s newspaper. Assume the bond has a face value of $2,000 and the current date is April 19,...

See Answer

Q: If the appropriate discount rate for the following cash flows

If the appropriate discount rate for the following cash flows is 9 percent compounded quarterly, what is the present value of the cash flows? Year …………………………….……………………. Cash Flow1 …….……………………………………………...

See Answer

Q: Imagination Dragons Corporation needs to raise funds to finance a

Imagination Dragons Corporation needs to raise funds to finance a plant expansion, and it has decided to issue 25-year zero coupon bonds with a par value of $1,000 each to raise the money. The require...

See Answer

Q: Suppose your company needs to raise $53 million and you

Suppose your company needs to raise $53 million and you want to issue 20-year bonds for this purpose. Assume the required return on your bond issue will be 5.3 percent, and you’re evaluating two issue...

See Answer

Q: Even though most corporate bonds in the United States make

Even though most corporate bonds in the United States make coupon payments semiannually, bonds issued elsewhere often have annual coupon payments. Suppose a German company issues a bond with a par val...

See Answer

Q: You are looking at an investment that has an effective

You are looking at an investment that has an effective annual rate of 11.6 percent. What is the effective semiannual return? The effective quarterly return? The effective monthly return?

See Answer

Q: You want to have $2.5 million in real dollars in

You want to have $2.5 million in real dollars in an account when you retire in 40 years. The nominal return on your investment is 10.3 percent and the inflation rate is 3.7 percent. What real amount m...

See Answer

Q: Bond P is a premium bond with a coupon rate

Bond P is a premium bond with a coupon rate of 9 percent. Bond D has a coupon rate of 5 percent and is currently selling at a discount. Both bonds make annual payments, have a YTM of 7 percent, and ha...

See Answer

Q: The YTM on a bond is the interest rate you

The YTM on a bond is the interest rate you earn on your investment if interest rates don’t change. If you actually sell the bond before it matures, your realized return is known as the holding period...

See Answer

Q: Find the APR, or stated rate, in each of the

Find the APR, or stated rate, in each of the following cases:

See Answer

Q: Jallouk Corporation has two different bonds currently outstanding. Bond M

Jallouk Corporation has two different bonds currently outstanding. Bond M has a face value of $20,000 and matures in 20 years. The bond makes no payments for the first six years, then pays $900 every...

See Answer

Q: Zevon Industries has a zero coupon bond issue that matures

Zevon Industries has a zero coupon bond issue that matures in two years with a face value of $40,000. The current value of the company’s assets is $26,700, and the standard deviation of the return on...

See Answer

Q: At one point, certain U.S. Treasury bonds were callable. Consider

At one point, certain U.S. Treasury bonds were callable. Consider the prices in the following three Treasury issues as of May 15, 2017:

See Answer

Q: A Japanese company has a bond outstanding that sells for

A Japanese company has a bond outstanding that sells for 105.43 percent of its ¥100,000 par value. The bond has a coupon rate of 3.4 percent paid annually and matures in 16 years. What is the yield to...

See Answer

Q: Gabriele Enterprises has bonds on the market making annual payments,

Gabriele Enterprises has bonds on the market making annual payments, with eight years to maturity, a par value of $1,000, and selling for $948. At this price, the bonds yield 5.1 percent. What must th...

See Answer

Q: Weismann Co. issued 15-year bonds a year ago at a

Weismann Co. issued 15-year bonds a year ago at a coupon rate of 4.9 percent. The bonds make semiannual payments and have a par value of $1,000. If the YTM on these bonds is 4.5 percent, what is the c...

See Answer

Q: West Corp. issued 25-year bonds two years ago at a

West Corp. issued 25-year bonds two years ago at a coupon rate of 5.3 percent. The bonds make semiannual payments. If these bonds currently sell for 105 percent of par value, what is the YTM?

See Answer

Q: McConnell Corporation has bonds on the market with 14.5 years

McConnell Corporation has bonds on the market with 14.5 years to maturity, a YTM of 5.3 percent, a par value of $1,000, and a current price of $1,045. The bonds make semiannual payments. What must the...

See Answer

Q: You find a zero coupon bond with a par value

You find a zero coupon bond with a par value of $10,000 and 17 years to maturity. If the yield to maturity on this bond is 4.2 percent, what is the price of the bond? Assume semiannual compounding per...

See Answer

Q: The Jackson-Timberlake Wardrobe Co. just paid a dividend of $2.15

The Jackson-Timberlake Wardrobe Co. just paid a dividend of $2.15 per share on its stock. The dividends are expected to grow at a constant rate of 4 percent per year indefinitely. If investors require...

See Answer

Q: First National Bank charges 13.1 percent compounded monthly on its

First National Bank charges 13.1 percent compounded monthly on its business loans. First United Bank charges 13.4 percent compounded semiannually. As a potential borrower, which bank would you go to f...

See Answer

Q: After successfully completing your corporate finance class, you feel the

After successfully completing your corporate finance class, you feel the next challenge ahead is to serve on the board of directors of Schenkel Enterprises. Unfortunately, you will be the only person...

See Answer

Q: In the previous problem, assume that the company uses cumulative

In the previous problem, assume that the company uses cumulative voting, and there are four seats in the current election. How much will it cost you to buy a seat now?Previous problem:After successful...

See Answer

Q: Colosseum Corp. has a zero coupon bond that matures in

Colosseum Corp. has a zero coupon bond that matures in five years with a face value of $65,000. The current value of the company’s assets is $62,000, and the standard deviation of its return on assets...

See Answer

Q: The Perfect Rose Co. has earnings of $3.18 per share.

The Perfect Rose Co. has earnings of $3.18 per share. The benchmark PE for the company is 18. What stock price would you consider appropriate? What if the benchmark PE were 21?

See Answer

Q: TwitterMe, Inc., is a new company and currently has negative

TwitterMe, Inc., is a new company and currently has negative earnings. The company’s sales are $2.1 million and there are 130,000 shares outstanding. If the benchmark price-sales ratio is 4.3, what is...

See Answer

Q: Moody Farms just paid a dividend of $2.65 on its

Moody Farms just paid a dividend of $2.65 on its stock. The growth rate in dividends is expected to be a constant 3.8 percent per year indefinitely. Investors require a return of 15 percent for the fi...

See Answer

Q: Metallica Bearings, Inc., is a young start-up company. No dividends

Metallica Bearings, Inc., is a young start-up company. No dividends will be paid on the stock over the next nine years because the firm needs to plow back its earnings to fuel growth. The company will...

See Answer

Q: Maurer, Inc., has an odd dividend policy. The company has

Maurer, Inc., has an odd dividend policy. The company has just paid a dividend of $2.75 per share and has announced that it will increase the dividend by $4.50 per share for each of the next five year...

See Answer

Q: Lohn Corporation is expected to pay the following dividends over

Lohn Corporation is expected to pay the following dividends over the next four years: $13, $9, $6, and $2.75. Afterward, the company pledges to maintain a constant 5 percent growth rate in dividends f...

See Answer

Q: Synovec Co. is growing quickly. Dividends are expected to grow

Synovec Co. is growing quickly. Dividends are expected to grow at a rate of 30 percent for the next three years, with the growth rate falling off to a constant 4 percent thereafter. If the required re...

See Answer

Q: Elliott Credit Corp. wants to earn an effective annual return

Elliott Credit Corp. wants to earn an effective annual return on its consumer loans of 17.1 percent per year. The bank uses daily compounding on its loans. What interest rate is the bank required by l...

See Answer

Q: Mobray Corp. is experiencing rapid growth. Dividends are expected to

Mobray Corp. is experiencing rapid growth. Dividends are expected to grow at 25 percent per year during the next three years, 15 percent over the following year, and then 6 percent per year indefinite...

See Answer

Q: The next dividend payment by Savitz, Inc., will be $2.34

The next dividend payment by Savitz, Inc., will be $2.34 per share. The dividends are anticipated to maintain a growth rate of 4.5 percent forever. If the stock currently sells for $37 per share, what...

See Answer

Q: Antiques R Us is a mature manufacturing firm. The company

Antiques R Us is a mature manufacturing firm. The company just paid a dividend of $9.80, but management expects to reduce the payout by 4 percent per year indefinitely. If you require a return of 9.5...

See Answer

Q: In addition to the five factors discussed in the chapter,

In addition to the five factors discussed in the chapter, dividends also affect the price of an option. The Black-Scholes option pricing model with dividends is:All of the variables are the same as th...

See Answer

Q: You have found the following stock quote for RJW Enterprises,

You have found the following stock quote for RJW Enterprises, Inc., in the financial pages of today’s newspaper. What was the closing price for this stock that appeared in yesterday&...

See Answer

Q: A7X Corp. just paid a dividend of $1.55 per share.

A7X Corp. just paid a dividend of $1.55 per share. The dividends are expected to grow at 21 percent for the next eight years and then level off to a growth rate of 3.5 percent indefinitely. If the req...

See Answer

Q: Navel County Choppers, Inc., is experiencing rapid growth. The company

Navel County Choppers, Inc., is experiencing rapid growth. The company expects dividends to grow at 18 percent per year for the next 11 years before leveling off at 4 percent into perpetuity. The requ...

See Answer

Q: Domergue Corp. currently has an EPS of $3.76, and the

Domergue Corp. currently has an EPS of $3.76, and the benchmark PE for the company is 21. Earnings are expected to grow at 5.1 percent per year.a. What is your estimate of the current stock price?b. W...

See Answer

Q: You have found the following historical information for the Daniela

You have found the following historical information for the Daniela Company over the past four years:

See Answer

Q: In the previous problem, we assumed that the stock had

In the previous problem, we assumed that the stock had a single stock price for the year. However, if you look at stock prices over any year, you will find a high and low stock price for the year. Ins...

See Answer

Q: Consider a project to supply Detroit with 30,000 tons of

Consider a project to supply Detroit with 30,000 tons of machine screws annually for automobile production. You will need an initial $4.3 million investment in threading equipment to get the project s...

See Answer

Q: You’ve observed the following returns on Crash-n-Burn Computer’s stock over

You’ve observed the following returns on Crash-n-Burn Computer’s stock over the past five years: 8 percent, −15 percent, 19 percent, 31 percent, and 21 percent.a. What was the arithmetic average retur...

See Answer

Q: You have just won the lottery and will receive $1,500,000

You have just won the lottery and will receive $1,500,000 in one year. You will receive payments for 30 years, and the payments will increase by 2.7 percent per year. If the appropriate discount rate...

See Answer

Q: Meyer & Co. expects its EBIT to be $97,000 every

Meyer & Co. expects its EBIT to be $97,000 every year forever. The firm can borrow at 8 percent. The company currently has no debt, and its cost of equity is 13 percent. If the tax rate is 24 percent,...

See Answer

Q: In Problem 14, what is the cost of equity after

In Problem 14, what is the cost of equity after recapitalization? What is the WACC? What are the implications for the firm’s capital structure decision?Problem 14:Meyer & Co. expects its EBIT to be $9...

See Answer

Q: Tool Manufacturing has an expected EBIT of $51,000 in perpetuity

Tool Manufacturing has an expected EBIT of $51,000 in perpetuity and a tax rate of 21 percent. The firm has $126,000 in outstanding debt at an interest rate of 5.35 percent, and its unlevered cost of...

See Answer

Q: Repeat parts (a) and (b) in Problem 1 assuming the

Repeat parts (a) and (b) in Problem 1 assuming the company has a tax rate of 21 percent, a market-to-book ratio of 1.0, and the stock price remains constant.(a) and (b) in Problem 1:a. Calculate earni...

See Answer

Q: Suppose the company in Problem 1 has a market-to-book ratio

Suppose the company in Problem 1 has a market-to-book ratio of 1.0 and the stock price remains constant.Problem 1:Ghost, Inc., has no debt outstanding and a total market value of $185,000. Earnings be...

See Answer

Q: The present value of the following cash flow stream is

The present value of the following cash flow stream is $7,500 when discounted at 9 percent annually. What is the value of the missing cash flow? Year …………………………………………………………………….. Cash Flow1 …………………………...

See Answer

Q: Round Hammer is comparing two different capital structures: An all-equity

Round Hammer is comparing two different capital structures: An all-equity plan (Plan I) and a levered plan (Plan II). Under Plan I, the company would have 180,000 shares of stock outstanding. Under Pl...

See Answer

Q: In Problem 4, use M&M Proposition I to find the

In Problem 4, use M&M Proposition I to find the price per share of equity under each of the two proposed plans. What is the value of the firm?Problem 4:Round Hammer is comparing two different capital...

See Answer

Q: Bellwood Corp. is comparing two different capital structures. Plan I

Bellwood Corp. is comparing two different capital structures. Plan I would result in 12,700 shares of stock and $109,250 in debt. Plan II would result in 9,800 shares of stock and $247,000 in debt.The...

See Answer

Q: First Simple Bank pays 6.4 percent simple interest on its

First Simple Bank pays 6.4 percent simple interest on its investment accounts. If First Complex Bank pays interest on its accounts compounded annually, what rate should the bank set if it wants to mat...

See Answer

Q: Ignoring taxes in Problem 6, what is the price per

Ignoring taxes in Problem 6, what is the price per share of equity under Plan I? Plan II? What principle is illustrated by your answers?Problem 6:Bellwood Corp. is comparing two different capital stru...

See Answer

Q: Your job pays you only once a year for all

Your job pays you only once a year for all the work you did over the previous 12 months. Today, December 31, you just received your salary of $55,000 and you plan to spend all of it. However, you want...

See Answer

Q: FCOJ, Inc., a prominent consumer products firm, is debating whether

FCOJ, Inc., a prominent consumer products firm, is debating whether to convert its all-equity capital structure to one that is 30 percent debt. Currently, there are 5,800 shares outstanding, and the p...

See Answer

Q: ABC Co. and XYZ Co. are identical firms in all

ABC Co. and XYZ Co. are identical firms in all respects except for their capital structure. ABC is all-equity financed with $720,000 in stock. XYZ uses both stock and perpetual debt; its stock is wort...

See Answer

Q: You own 1,000 shares of stock in Avondale Corporation. You

You own 1,000 shares of stock in Avondale Corporation. You will receive a $3.15 per share dividend in one year. In two years, the company will pay a liquidating dividend of $57 per share. The required...

See Answer

Q: In Problem 10, suppose you want only $1,500 total in

In Problem 10, suppose you want only $1,500 total in dividends the first year. What will your homemade dividend be in two years?Problem 10:You own 1,000 shares of stock in Avondale Corporation. You wi...

See Answer

Q: Awake Corporation is evaluating an extra dividend versus a share

Awake Corporation is evaluating an extra dividend versus a share repurchase. In either case, $17,500 would be spent. Current earnings are $1.89 per share, and the stock currently sells for $64 per sha...

See Answer

Q: The Gecko Company and the Gordon Company are two firms

The Gecko Company and the Gordon Company are two firms that have the same business risk but different dividend policies. Gecko pays no dividend, whereas Gordon has an expected dividend yield of 2.9 pe...

See Answer

Q: You just won the TVM Lottery. You will receive $1

You just won the TVM Lottery. You will receive $1 million today plus another 10 annual payments that increase by $375,000 per year. Thus, in one year, you receive $1.375 million. In two years, you get...

See Answer

Q: As discussed in the text, in the absence of market

As discussed in the text, in the absence of market imperfections and tax effects, we would expect the share price to decline by the amount of the dividend payment when the stock goes ex dividend. Once...

See Answer

Q: What are the portfolio weights for a portfolio that has

What are the portfolio weights for a portfolio that has 115 shares of Stock A that sell for $43 per share and 180 shares of Stock B that sell for $19 per share?

See Answer

Q: National Business Machine Co. (NBM) has $4 million of extra

National Business Machine Co. (NBM) has $4 million of extra cash after taxes have been paid. NBM has two choices to make use of this cash. One alternative is to invest the cash in financial assets. Th...

See Answer

Q: After completing its capital spending for the year, Carlson Manufacturing

After completing its capital spending for the year, Carlson Manufacturing has $1,000 extra cash. Carlson’s managers must choose between investing the cash in Treasury bonds that yield 3 percent or pay...

See Answer

Q: An investment offers $4,350 per year for 15 years, with

An investment offers $4,350 per year for 15 years, with the first payment occurring one year from now. If the required return is 6 percent, what is the value of the investment? What would the value be...

See Answer

Q: Change Corporation expects an EBIT of $31,200 every year forever.

Change Corporation expects an EBIT of $31,200 every year forever. The company currently has no debt, and its cost of equity is 11 percent.a. What is the current value of the company?b. Suppose the com...

See Answer

Q: The Day Company and the Knight Company are identical in

The Day Company and the Knight Company are identical in every respect except that Day is not levered. Financial information for the two firms appears in the following table. All earnings streams are p...

See Answer

Q: The owners’ equity accounts for Vidi International are shown here:

The owners’ equity accounts for Vidi International are shown here:Common stock ($.50 par value) ……………………………………………………… $ 25,000Capital surplus ………………………………………………………………………………. 215,000Retained earnings …...

See Answer

Q: The balance sheet for Sinking Ship Corp. is shown here

The balance sheet for Sinking Ship Corp. is shown here in market value terms. There are 14,000 shares of stock outstanding.The company has declared a dividend of $1.30 per share. The stock goes ex div...

See Answer

Q: In Problem 5, suppose the company has announced it is

In Problem 5, suppose the company has announced it is going to repurchase $18,200 worth of stock. What effect will this transaction have on the equity of the firm? How many shares will be outstanding?...

See Answer

Q: The market value balance sheet for Bobaflex Manufacturing is shown

The market value balance sheet for Bobaflex Manufacturing is shown here. The company has declared a 25 percent stock dividend. The stock goes ex dividend tomorrow (the chronology for a stock dividend...

See Answer

Q: The company with the common equity accounts shown here has

The company with the common equity accounts shown here has declared a 15 percent stock dividend when the market value of its stock is $53 per share. What effects will the distribution of the stock div...

See Answer

Q: Consider the following information:

Consider the following information:b. What is the variance of this portfolio? The standard deviation?

See Answer

Q: You have just purchased a new warehouse. To finance the

You have just purchased a new warehouse. To finance the purchase, you’ve arranged for a 30- year mortgage loan for 80 percent of the $3,400,000 purchase price. The monthly payment on this loan will be...

See Answer

Q: In Problem 8, suppose the company instead decides on a

In Problem 8, suppose the company instead decides on a four-for-one stock split. The firm’s 65 cent per-share cash dividend on the new (postsplit) shares represents an increase of 10...

See Answer

Q: Ginger, Inc., has declared a $5.35 per share dividend. Suppose

Ginger, Inc., has declared a $5.35 per share dividend. Suppose capital gains are not taxed, but dividends are taxed at 15 percent. New IRS regulations require that taxes be withheld at the time the di...

See Answer

Q: If you put up $41,000 today in exchange for a

If you put up $41,000 today in exchange for a 5.1 percent, 15-year annuity, what will the annual cash flow be?

See Answer

Q: The following is the sales budget for Profit, Inc., for

The following is the sales budget for Profit, Inc., for the first quarter of 2018:Credit sales are collected as follows:65 percent in the month of the sale20 percent in the month after the sale15 perc...

See Answer

Q: Here are some important figures from the budget of Nashville

Here are some important figures from the budget of Nashville Nougats, Inc., for the second quarter of 2018:The company predicts that 5 percent of its credit sales will never be collected, 35 percent o...

See Answer

Q: Below are the most recent balance sheets for Country Kettles,

Below are the most recent balance sheets for Country Kettles, Inc. Excluding accumulated depreciation, determine whether each item is a source or a use of cash and the amount:,,,

See Answer

Q: Cori’s Corp. has an equity value of $13,315. Long-term debt

Cori’s Corp. has an equity value of $13,315. Long-term debt is $8,200. Net working capital, other than cash, is $2,750. Fixed assets are $17,380. How much cash does the company have? If current liabil...

See Answer

Q: For the company in Problem 2, show how the equity

For the company in Problem 2, show how the equity accounts will change if:Problem 2:The owners’ equity accounts for Vidi International are shown here:Common stock ($.50 par value) ……………………………………………………...

See Answer

Q: Simmons Mineral Operations, Inc. (SMO), currently has 530,000 shares of

Simmons Mineral Operations, Inc. (SMO), currently has 530,000 shares of stock outstanding that sell for $68 per share. Assuming no market imperfections or tax effects exist, what will the share price...

See Answer

Q: You own a stock portfolio invested 20 percent in Stock

You own a stock portfolio invested 20 percent in Stock Q, 30 percent in Stock R, 35 percent in Stock S, and 15 percent in Stock T. The betas for these four stocks are .79, 1.23, 1.13, and 1.36, respec...

See Answer

Q: Morning Jolt Coffee Company has projected the following quarterly sales

Morning Jolt Coffee Company has projected the following quarterly sales amounts for the coming year:a. Accounts receivable at the beginning of the year are $365. The company has a 45-day collection pe...

See Answer

Q: Consider the following financial statement information for the Newk Corporation:

Consider the following financial statement information for the Newk Corporation:Calculate the operating and cash cycles. How do you interpret your answer?

See Answer

Q: Consider a firm with a contract to sell an asset

Consider a firm with a contract to sell an asset for $145,000 four years from now. The asset costs $91,700 to produce today. Given a relevant discount rate of 11 percent per year, will the firm make a...

See Answer

Q: Your firm has an average collection period of 31 days.

Your firm has an average collection period of 31 days. Current practice is to factor all receivables immediately at a discount of 1.25 percent. What is the effective cost of borrowing in this case? As...

See Answer

Q: Your company will generate $47,000 in annual revenue each year

Your company will generate $47,000 in annual revenue each year for the next seven years from a new information database. If the appropriate interest rate is 7.1 percent, what is the present value of t...

See Answer

Q: Sexton Corp. has projected the following sales for the coming

Sexton Corp. has projected the following sales for the coming year:a. Calculate payments to suppliers assuming that the company places orders during each quarter equal to 30 percent of projected sales...

See Answer

Q: The Torrey Pine Corporation’s purchases from suppliers in a quarter

The Torrey Pine Corporation’s purchases from suppliers in a quarter are equal to 75 percent of the next quarter’s forecast sales. The payables period is 60 days. Wa...

See Answer

Q: In a typical month, the Monk Corporation receives 80 checks

In a typical month, the Monk Corporation receives 80 checks totaling $113,000. These are delayed four days on average. What is the average daily float? Assume 30 days in a month

See Answer

Q: No More Books Corporation has an agreement with Floyd Bank

No More Books Corporation has an agreement with Floyd Bank whereby the bank handles $4.5 million in collections per day and requires a $340,000 compensating balance. No More Books is contemplating can...

See Answer

Q: Bird’s Eye Treehouses, Inc., a Kentucky company, has determined that

Bird’s Eye Treehouses, Inc., a Kentucky company, has determined that a majority of its customers are located in the Pennsylvania area. It therefore is considering using a lockbox system offered by a b...

See Answer

Q: You own a portfolio equally invested in a risk-free asset

You own a portfolio equally invested in a risk-free asset and two stocks. If one of the stocks has a beta of 1.17 and the total portfolio is equally as risky as the market, what must the beta be for t...

See Answer

Q: Cow Chips, Inc., a large fertilizer distributor based in California,

Cow Chips, Inc., a large fertilizer distributor based in California, is planning to use a lockbox system to speed up collections from its customers located on the East Coast. A Philadelphia-area bank...

See Answer

Q: A bank offers your firm a revolving credit arrangement for

A bank offers your firm a revolving credit arrangement for up to $50 million at an interest rate of 1.65 percent per quarter. The bank also requires you to maintain a compensating balance of 5 percent...

See Answer

Q: Wildcat, Inc., has estimated sales (in millions) for the next

Wildcat, Inc., has estimated sales (in millions) for the next four quarters as follows:Sales for the first quarter of the following year are projected at $180 million. Accounts receivable at the begin...

See Answer

Q: Rework Problem 15 assuming:

Rework Problem 15 assuming:Problem 15:Wildcat, Inc., has estimated sales (in millions) for the next four quarters as follows:Sales for the first quarter of the following year are projected at $180 mil...

See Answer

Q: What is the value today of $4,400 per year, at

What is the value today of $4,400 per year, at a discount rate of 8.3 percent, if the first payment is received 6 years from today and the last payment is received 20 years from today?

See Answer

Q: If you deposit $4,500 at the end of each of

If you deposit $4,500 at the end of each of the next 20 years into an account paying 9.7 percent interest, how much money will you have in the account in 20 years? How much will you have if you make d...

See Answer

Q: In a world of corporate taxes only, show that the

In a world of corporate taxes only, show that the WACC can be written as WACC = RU × [1 − TC(D/V)]

See Answer

Q: Each business day, on average, a company writes checks totaling

Each business day, on average, a company writes checks totaling $17,000 to pay its suppliers. The usual clearing time for the checks is four days. Meanwhile, the company is receiving payments from its...

See Answer

Q: A stock has a beta of 1.15, the expected return

A stock has a beta of 1.15, the expected return on the market is 10.3 percent, and the risk-free rate is 3.1 percent. What must the expected return on this stock be?

See Answer

Q: Assume a firm’s debt is risk-free, so that the cost

Assume a firm’s debt is risk-free, so that the cost of debt equals the risk-free rate, Rf. Define βA as the firm’s asset beta—that is, the systematic risk of the firm’s assets. Define βE to be the bet...

See Answer

Q: Suppose a firm’s business operations are such that they mirror

Suppose a firm’s business operations are such that they mirror movements in the economy as a whole very closely; that is, the firm’s asset beta is 1.0. Use the result of Problem 21 to find the equity...

See Answer

Q: Purple Feet Wine, Inc., receives an average of $17,500 in

Purple Feet Wine, Inc., receives an average of $17,500 in checks per day. The delay in clearing is typically three days. The current interest rate is .017 percent per day.a. What is the company’s floa...

See Answer

Q: Your neighbor goes to the post office once a month

Your neighbor goes to the post office once a month and picks up two checks, one for $10,700 and one for $4,600. The larger check takes four days to clear after it is deposited; the smaller one takes t...

See Answer

Q: Your firm has an average receipt size of $125. A

Your firm has an average receipt size of $125. A bank has approached you concerning a lockbox service that will decrease your total collection time by two days. You typically receive 5,100 checks per...

See Answer

Q: A mail-order firm processes 5,300 checks per month. Of these,

A mail-order firm processes 5,300 checks per month. Of these, 60 percent are for $47 and 40 percent are for $79. The $47 checks are delayed two days on average; the $79 checks are delayed three days o...

See Answer

Q: Frostbite Thermal wear has a zero coupon bond issue outstanding

Frostbite Thermal wear has a zero coupon bond issue outstanding with a face value of $23,000 that matures in one year. The current market value of the firm’s assets is $26,200. The standard deviation...

See Answer

Q: A 15-year annuity pays $1,475 per month, and payments are

A 15-year annuity pays $1,475 per month, and payments are made at the end of each month. If the interest rate is 9 percent compounded monthly for the first seven years, and 6 percent compounded monthl...

See Answer

Q: Paper Submarine Manufacturing is investigating a lockbox system to reduce

Paper Submarine Manufacturing is investigating a lockbox system to reduce its collection time. It has determined the following:Average number of payments per day ……………………………………………………. 485Average value...

See Answer

Q: It takes Cookie Cutter Modular Homes, Inc., about six days

It takes Cookie Cutter Modular Homes, Inc., about six days to receive and deposit checks from customers. The company’s management is considering a lockbox system to reduce the firm’s collection times....

See Answer

Q: A stock has an expected return of 10.2 percent, the

A stock has an expected return of 10.2 percent, the risk-free rate is 3.9 percent, and the market risk premium is 7.2 percent. What must the beta of this stock be?

See Answer

Q: Every two weeks, No More Pencils, Inc., disburses checks that

Every two weeks, No More Pencils, Inc., disburses checks that average $107,000 and take seven days to clear. How much interest can the company earn annually if it delays transfer of funds from an inte...

See Answer

Q: Sanchez, Inc., is considering a change in its cash-only sales

Sanchez, Inc., is considering a change in its cash-only sales policy. The new terms of sale would be net one month. Based on the following information, determine if the company should proceed or not....

See Answer

Q: Provenza Manufacturing uses 3,400 switch assemblies per week and then

Provenza Manufacturing uses 3,400 switch assemblies per week and then reorders another 3,400. If the relevant carrying cost per switch assembly is $7.45, and the fixed order cost is $1,100, is the com...

See Answer

Q: The Trektronics store begins each week with 450 phasers in

The Trektronics store begins each week with 450 phasers in stock. This stock is depleted each week and reordered. If the carrying cost per phaser is $34 per year and the fixed order cost is $130, what...

See Answer

Q: You’ve worked out a line of credit arrangement that allows

You’ve worked out a line of credit arrangement that allows you to borrow up to $40 million at any time. The interest rate is .36 percent per month. In addition, 4 percent of the amount that you borrow...

See Answer

Q: The Snedecker Corporation is considering a change in its cash-only

The Snedecker Corporation is considering a change in its cash-only policy. The new terms would be net one period. Based on the following information, determine if the company should proceed or not. Th...

See Answer

Q: Veni, Inc., currently has an all-cash credit policy. It is

Veni, Inc., currently has an all-cash credit policy. It is considering making a change in the credit policy by going to terms of net 30 days. Based on the following information, what do you recommend?...

See Answer

Q: You want to have $60,000 in your savings account 12

You want to have $60,000 in your savings account 12 years from now, and you’re prepared to make equal annual deposits into the account at the end of each year. If the account pays 6.4 percent interest...

See Answer

Q: The Silver Spokes Bicycle Shop has decided to offer credit

The Silver Spokes Bicycle Shop has decided to offer credit to its customers during the spring selling season. Sales are expected to be 125 bicycles. The average cost to the shop of a bicycle is $750....

See Answer

Q: Assume that your company does not anticipate paying taxes for

Assume that your company does not anticipate paying taxes for the next several years. What are the cash flows from leasing in this case?

See Answer

Q: A stock has an expected return of 10.45 percent,

A stock has an expected return of 10.45 percent, its beta is .93, and the risk-free rate is 3.6 percent. What must the expected return on the market be?

See Answer

Q: You have your choice of two investment accounts. Investment A

You have your choice of two investment accounts. Investment A is a 13-year annuity that features end-of-month $1,250 payments and has an interest rate of 7.5 percent compounded monthly. Investment B i...

See Answer

Q: In Problem 14, what is the break-even quantity for the

In Problem 14, what is the break-even quantity for the new credit policy?Problem 14:The Snedecker Corporation is considering a change in its cash-only policy. The new terms would be net one period. Ba...

See Answer

Q: Skye Flyer, Inc., has weekly credit sales of $21,900, and

Skye Flyer, Inc., has weekly credit sales of $21,900, and the average collection period is 33 days. What is the average accounts receivable figure?

See Answer

Q: A firm offers terms of 1/10, net 30. What effective

A firm offers terms of 1/10, net 30. What effective annual interest rate does the firm earn when a customer does not take the discount? Without doing any calculations, explain what will happen to this...

See Answer

Q: Starset, Inc., has an average collection period of 27 days.

Starset, Inc., has an average collection period of 27 days. Its average daily investment in receivables is $46,300. What are annual credit sales? What is the receivables turnover? Assume 365 days per...

See Answer

Q: Essence of Skunk Fragrances, Ltd., sells 7,900 units of its

Essence of Skunk Fragrances, Ltd., sells 7,900 units of its perfume collection each year at a price per unit of $385. All sales are on credit with terms of 1/10, net 40. The discount is taken by 65 pe...

See Answer

Q: The Arizona Bay Corporation sells on credit terms of net

The Arizona Bay Corporation sells on credit terms of net 30. Its accounts are, on average, five days past due. If annual credit sales are $8.35 million, what is the company’s balance sheet amount in a...

See Answer

Q: Air Spares is a wholesaler that stocks engine components and

Air Spares is a wholesaler that stocks engine components and test equipment for the commercial aircraft industry. A new customer has placed an order for eight high-bypass turbine engines, which increa...

See Answer

Q: Prescott Bank offers you a five-year loan for $75,000 at

Prescott Bank offers you a five-year loan for $75,000 at an annual interest rate of 6.8 percent. What will your annual loan payment be?

See Answer

Q: You place an order for 300 units of inventory at

You place an order for 300 units of inventory at a unit price of $140. The supplier offers terms of 1/10, net 30. a. How long do you have to pay before the account is overdue? If you take the full per...

See Answer

Q: Aria Acoustics, Inc. (AAI), projects unit sales for a new

Aria Acoustics, Inc. (AAI), projects unit sales for a new seven-octave voice emulation implant as follows:Year ………………………………………………………………………………………….. Unit Sales1 …………………………………………………………………………………………………….....

See Answer

Q: Suppose the spot and six-month forward rates on the Norwegian

Suppose the spot and six-month forward rates on the Norwegian krone are Kr 8.39 and Kr 8.48, respectively. The annual risk-free rate in the United States is 3.8 percent, and the annual risk-free rate...

See Answer

Q: The Red Zeppelin Corporation has annual sales of $34 million.

The Red Zeppelin Corporation has annual sales of $34 million. The average collection period is 28 days. What is the average investment in accounts receivable as shown on the balance sheet? Assume 365...

See Answer

Q: Given an interest rate of 5.3 percent per year, what

Given an interest rate of 5.3 percent per year, what is the value at date t = 7 of a perpetual stream of $6,400 payments that begins at date t = 15?

See Answer

Q: Kyoto Joe, Inc., sells earnings forecasts for Japanese securities. Its

Kyoto Joe, Inc., sells earnings forecasts for Japanese securities. Its credit terms are 2/10, net 30. Based on experience, 70 percent of all customers will take the discount.a. What is the average col...

See Answer

Q: Suppose the spot exchange rate for the Canadian dollar is

Suppose the spot exchange rate for the Canadian dollar is Can$1.29 and the six-month forward rate is Can$1.31.a. Which is worth more, a U.S. dollar or a Canadian dollar?b. Assuming absolute PPP holds,...

See Answer

Q: Suppose the Japanese yen exchange rate is ¥114 = $1,

Suppose the Japanese yen exchange rate is ¥114 = $1, and the British pound exchange rate is £1 = $1.26.a. What is the cross-rate in terms of yen per pound?b. Suppose the cross-rate is ¥147 = £1. Is th...

See Answer

Q: Use Figure 21.1 to answer the following questions: Suppose interest

Use Figure 21.1 to answer the following questions: Suppose interest rate parity holds, and the current six-month risk-free rate in the United States is 1.3 percent. What must the six-month risk free r...

See Answer

Q: The treasurer of a major U.S. firm has $30 million

The treasurer of a major U.S. firm has $30 million to invest for three months. The interest rate in the United States is .28 percent per month. The interest rate in Great Britain is .31 percent per mo...

See Answer

Q: Refer to Table 23.1 in the text to answer this

Refer to Table 23.1 in the text to answer this question. Suppose you purchase a March 2017 cocoa futures contract this day at the last price of the day. What will your profit or loss be if cocoa price...

See Answer

Q: A stock is currently priced at $47. A call option

A stock is currently priced at $47. A call option with an expiration of one year has an exercise price of $50. The risk-free rate is 12 percent per year, compounded continuously, and the standard devi...

See Answer

Q: In Problem 27, suppose you’re confident about your own projections,

In Problem 27, suppose you’re confident about your own projections, but you’re a little unsure about Detroit’s actual machine screw requirement. What is the sensitivity of the project OCF to changes i...

See Answer

Q: A stock has an expected return of 11.85 percent, its

A stock has an expected return of 11.85 percent, its beta is 1.24, and the expected return on the market is 10.2 percent. What must the risk-free rate be?

See Answer

Q: In calculating insurance premiums, the actuarially fair insurance premium is

In calculating insurance premiums, the actuarially fair insurance premium is the premium that results in a zero NPV for both the insured and the insurer. As such, the present value of the expected los...

See Answer

Q: You observe that the inflation rate in the United States

You observe that the inflation rate in the United States is 3.5 percent per year and that T-bills currently yield 4.1 percent annually. Using the approximate international Fisher effect, what do you e...

See Answer

Q: Suppose the spot and three-month forward rates for the yen

Suppose the spot and three-month forward rates for the yen are ¥113.65 and ¥113.18, respectively. a. Is the yen expected to get stronger or weaker?b. What would you estimate is the difference between...

See Answer

Q: Suppose the spot exchange rate for the Hungarian forint is

Suppose the spot exchange rate for the Hungarian forint is HUF 289.97. The inflation rate in the United States will be 2.9 percent per year. It will be 4.5 percent in Hungary. What do you predict the...

See Answer

Q: A local finance company quotes an interest rate of 17.1

A local finance company quotes an interest rate of 17.1 percent on one-year loans. So, if you borrow $20,000, the interest for the year will be $3,420. Because you must repay a total of $23,420 in one...

See Answer

Q: Lakonishok Equipment has an investment opportunity in Europe. The

Lakonishok Equipment has an investment opportunity in Europe. The project costs €10.5 million and is expected to produce cash flows of €1.7 million in Year 1, €2.4 million in Year 2, and €3.3 million...

See Answer

Q: You are evaluating a proposed expansion of an existing subsidiary

You are evaluating a proposed expansion of an existing subsidiary located in Switzerland. The cost of the expansion would be SF 13.8 million. The cash flows from the project would be SF 3.9 million pe...

See Answer

Q: Atreides International has operations in Arrakis. The balance sheet for

Atreides International has operations in Arrakis. The balance sheet for this division in Arrakeen solaris shows assets of 38,000 solaris, debt in the amount of 12,000 solaris, and equity of 26,000 sol...

See Answer

Q: In Problem 16, assume the equity increases by 1,250 solaris

In Problem 16, assume the equity increases by 1,250 solaris due to retained earnings. If the exchange rate at the end of the year is 1.54 solaris per dollar, what does the balance sheet look like?Prob...

See Answer

Q: Asset W has an expected return of 11.8 percent and

Asset W has an expected return of 11.8 percent and a beta of 1.10. If the risk-free rate is 3.3 percent, complete the following table for portfolios of Asset W and a risk-free asset. Illustrate the re...

See Answer

Q: In Problem 14, what is the break-even price per unit

In Problem 14, what is the break-even price per unit that should be charged under the new credit policy? Assume that the sales figure under the new policy is 3,310 units and all other values remain th...

See Answer

Q: Yan Yan Corp. has a $2,000 par value bond outstanding

Yan Yan Corp. has a $2,000 par value bond outstanding with a coupon rate of 4.4 percent paid semiannually and 13 years to maturity. The yield to maturity of the bond is 4.8 percent. What is the price...

See Answer

Q: In Problem 15, what is the break-even price per unit

In Problem 15, what is the break-even price per unit under the new credit policy? Assume all other values remain the same.Problem 15:Veni, Inc., currently has an all-cash credit policy. It is consider...

See Answer

Q: Refer to Table 23.1 in the text to answer this

Refer to Table 23.1 in the text to answer this question. Suppose you sell five March 2017 silver futures contracts this day at the last price of the day. What will your profit or loss be if silver pri...

See Answer

Q: Assuming a world of corporate taxes only, show that the

Assuming a world of corporate taxes only, show that the cost of equity, RE, is as given in the chapter by M&M Proposition II with corporate taxes

See Answer

Q: Solar Engines manufactures solar engines for tractor-trailers. Given the fuel

Solar Engines manufactures solar engines for tractor-trailers. Given the fuel savings available, new orders for 125 units have been made by customers requesting credit. The variable cost is $6,900 per...

See Answer

Q: In Problem 21, assume that the probability of default is

In Problem 21, assume that the probability of default is 15 percent. Should the orders be filled now? Assume the number of repeat customers is affected by the defaults. In other words, 30 percent of t...

See Answer

Q: A five-year annuity of 10 $5,900 semiannual payments will begin

A five-year annuity of 10 $5,900 semiannual payments will begin 9 years from now, with the first payment coming 9.5 years from now. If the discount rate is 8 percent compounded monthly, what is the va...

See Answer

Q: Refer to Table 23.2 in the text to answer this

Refer to Table 23.2 in the text to answer this question. Suppose you purchase the May 2017 call option on corn futures with a strike price of $3.85. Assume you purchased the option at the last price....

See Answer

Q: Suppose a financial manager buys call options on 50,000 barrels

Suppose a financial manager buys call options on 50,000 barrels of oil with an exercise price of $57 per barrel. She simultaneously sells a put option on 50,000 barrels of oil with the same exercise p...

See Answer

Q: Fuente, Inc., has identified an investment project with the following

Fuente, Inc., has identified an investment project with the following cash flows. If the discount rate is 8 percent, what is the future value of these cash flows in Year 4? What is the future value at...

See Answer

Q: Refer to Table 23.2 in the text to answer this

Refer to Table 23.2 in the text to answer this question. Suppose you purchase the May 2017 put option on corn futures with a strike price of $3.80. Assume your purchase was at the last price. What is...

See Answer

Q: Suppose your company has a building worth $165 million. Because

Suppose your company has a building worth $165 million. Because it is located in a high-risk area for natural disasters, the probability of a total loss in any particular year is 1.15 percent. What is...

See Answer

Q: Union Local School District has a bond outstanding with a

Union Local School District has a bond outstanding with a coupon rate of 2.8 percent paid semiannually and 16 years to maturity. The yield to maturity on this bond is 3.4 percent, and the bond has a p...

See Answer

Q: Refer to Table 23.1 in the text to answer this

Refer to Table 23.1 in the text to answer this question. Suppose today is February 10, 2017, and your firm produces breakfast cereal and needs 145,000 bushels of corn in May 2017 for an upcoming promo...

See Answer

Q: Suppose the current exchange rate for the Polish zloty is

Suppose the current exchange rate for the Polish zloty is Z 4.04. The expected exchange rate in three years is Z 4.13. What is the difference in the annual inflation rates for the United States and Po...

See Answer

Q: Suppose your company imports computer motherboards from Singapore. The exchange

Suppose your company imports computer motherboards from Singapore. The exchange rate is given in Figure 21.1. You have just placed an order for 30,000 motherboards at a cost to you of 218.50 Singapore...

See Answer

Q: T-bills currently yield 3.4 percent. Stock in Deadwood Manufacturing is

T-bills currently yield 3.4 percent. Stock in Deadwood Manufacturing is currently selling for $58 per share. There is no possibility that the stock will be worth less than $50 per share in one year.a....

See Answer

Q: The following facts apply to a convertible bond making semiannual

The following facts apply to a convertible bond making semiannual payments:Conversion price …………………………………………………………………… $37/shareCoupon rate …………………………………………………………………………………. 2.6%Par value …………………………………...

See Answer

Q: You have been hired to value a new 30-year callable,

You have been hired to value a new 30-year callable, convertible bond. The bond has a coupon rate of 2.7 percent, payable semiannually, and its face value is $1,000. The conversion price is $54, and t...

See Answer

Q: Suppose you are going to receive $13,500 per year for

Suppose you are going to receive $13,500 per year for five years. The appropriate interest rate is 6.8 percent. a. What is the present value of the payments if they are in the form of an ordinary annu...

See Answer

Q: Stock Y has a beta of 1.2 and an expected

Stock Y has a beta of 1.2 and an expected return of 11.1 percent. Stock Z has a beta of .80 and an expected return of 7.85 percent. If the risk-free rate is 2.4 percent and the market risk premium is...

See Answer

Q: A bond with 20 detachable warrants has just been offered

A bond with 20 detachable warrants has just been offered for sale at $1,000. The bond matures in 20 years and has an annual coupon of $24. Each warrant gives the owner the right to purchase two shares...

See Answer

Q: Your company is deciding whether to invest in a new

Your company is deciding whether to invest in a new machine. The new machine will increase cash flow by $275,000 per year. You believe the technology used in the machine has a 10-year life; in other w...

See Answer

Q: We are examining a new project. We expect to sell

We are examining a new project. We expect to sell 7,100 units per year at $56 net cash flow apiece for the next 10 years. In other words, the annual cash flow is projected to be $56 × 7,100 = $397,600...

See Answer

Q: If Treasury bills are currently paying 4.7 percent and the

If Treasury bills are currently paying 4.7 percent and the inflation rate is 2.2 percent, what is the approximate real rate of interest? The exact real rate?

See Answer

Q: In Problem 14, suppose you think it is likely that

In Problem 14, suppose you think it is likely that expected sales will be revised upward to 10,800 units if the first year is a success and revised downward to 3,900 units if the first year is not a s...

See Answer

Q: In Problem 15, suppose the scale of the project can

In Problem 15, suppose the scale of the project can be doubled in one year in the sense that twice as many units can be produced and sold. Naturally, expansion would be desirable only if the project i...

See Answer

Q: Suppose a share of stock sells for $63. The risk-free

Suppose a share of stock sells for $63. The risk-free rate is 5 percent, and the stock price in one year will be either $70 or $80.a. What is the value of a call option with an exercise price of $70?b...

See Answer

Q: Which of the following two sets of relationships, at time

Which of the following two sets of relationships, at time of issuance for convertible bonds, is more typical? Why?,,,

See Answer

Q: Campbell, Inc., has a $1,000 face value convertible bond issue

Campbell, Inc., has a $1,000 face value convertible bond issue that is currently selling in the market for $960. Each bond is exchangeable at any time for 18 shares of the company’s stock. The convert...

See Answer

Q: Use the option quote information shown here to answer the

Use the option quote information shown here to answer the questions that follow. The stock is currently selling for $85.a. Are the call options in the money? What is the intrinsic value of an RWJ Corp...

See Answer

Q: In the previous problem, what would the risk-free rate have

In the previous problem, what would the risk-free rate have to be for the two stocks to be correctly priced?Previous problem:Stock Y has a beta of 1.2 and an expected return of 11.1 percent. Stock Z h...

See Answer

Q: Liberty Products, Inc., is considering a new product launch. The

Liberty Products, Inc., is considering a new product launch. The firm expects to have annual operating cash flow of $5.3 million for the next eight years. The company uses a discount rate of 11 percen...

See Answer

Q: You want to buy a new sports car from Muscle

You want to buy a new sports car from Muscle Motors for $57,500. The contract is in the form of a 60-month annuity due at an APR of 5.9 percent. What will your monthly payment be?

See Answer

Q: You have been hired to value a new 25-year callable,

You have been hired to value a new 25-year callable, convertible bond. The bond has a coupon rate of 2.3 percent, payable annually. The conversion price is $68, and the stock currently sells for $27.8...

See Answer

Q: Consider the following project of Hand Clapper, Inc. The company

Consider the following project of Hand Clapper, Inc. The company is considering a four-year project to manufacture clap-command garage door openers. This project requires an initial investment of $14...

See Answer

Q: Suppose the real rate is 2.1 percent and the inflation

Suppose the real rate is 2.1 percent and the inflation rate is 3.4 percent. What rate would you expect to see on a Treasury bill?

See Answer

Q: Minder Industries stock has a beta of 1.08. The company

Minder Industries stock has a beta of 1.08. The company just paid a dividend of $.65, and the dividends are expected to grow at 4 percent. The expected return on the market is 10.5 percent, and Treasu...

See Answer

Q: Pearl Corp. is expected to have an EBIT of $1.8

Pearl Corp. is expected to have an EBIT of $1.8 million next year. Depreciation, the increase in net working capital, and capital spending are expected to be $155,000, $75,000, and $115,000, respectiv...

See Answer

Q: You have looked at the current financial statements for Reigle

You have looked at the current financial statements for Reigle Homes, Co. The company has an EBIT of $3.15 million this year. Depreciation, the increase in net working capital, and capital spending we...

See Answer

Q: Use the option quote information shown here to answer the

Use the option quote information shown here to answer the questions that follow. The stock is currently selling for $40.a. Suppose you buy 10 contracts of the February 38 call option. How much will yo...

See Answer

Q: The price of Chive Corp. stock will be either $67

The price of Chive Corp. stock will be either $67 or $91 at the end of the year. Call options are available with one year to expiration. T-bills currently yield 4 percent.a. Suppose the current price...

See Answer

Q: You own a portfolio that has $3,480 invested in Stock

You own a portfolio that has $3,480 invested in Stock A and $7,430 invested in Stock B. If the expected returns on these stocks are 8 percent and 11 percent, respectively, what is the expected return...

See Answer

Q: The price of Cilantro, Inc., stock will be either $70

The price of Cilantro, Inc., stock will be either $70 or $90 at the end of the year. Call options are available with one year to expiration. T-bills currently yield 6 percent.a. Suppose the current pr...

See Answer

Q: A one-year call option contract on Cheesy Poofs Co. stock

A one-year call option contract on Cheesy Poofs Co. stock sells for $845. In one year, the stock will be worth $64 or $81 per share. The exercise price on the call option is $70. What is the current v...

See Answer

Q: Rackin Pinion Corporation’s assets are currently worth $1,065. In one

Rackin Pinion Corporation’s assets are currently worth $1,065. In one year, they will be worth either $1,000 or $1,340. The risk-free interest rate is 3.9 percent. Suppose the company has an outstandi...

See Answer

Q: Prepare an amortization schedule for a five-year loan of $71,500.

Prepare an amortization schedule for a five-year loan of $71,500. The interest rate is 7 percent per year, and the loan calls for equal annual payments. How much interest is paid in the third year? Ho...

See Answer

Q: Buckeye Industries has a bond issue with a face value

Buckeye Industries has a bond issue with a face value of $1,000 that is coming due in one year. The value of the company’s assets is currently $1,040. Urban Meyer, the CEO, believes that the assets in...

See Answer

Q: An investment offers a total return of 12.3 percent over

An investment offers a total return of 12.3 percent over the coming year. Janice Yellen thinks the total real return on this investment will be only 8 percent. What does Janice believe the inflation r...

See Answer

Q: A $1,000 par convertible debenture has a conversion price for

A $1,000 par convertible debenture has a conversion price for common stock of $27 per share. With the common stock selling at $31, what is the conversion value of the bond?

See Answer

Q: If you have $1,275 today, how much will it be

If you have $1,275 today, how much will it be worth in six years at 8 percent per year compounded continuously?

See Answer

Q: In Problem 9, suppose you wanted the option to sell

In Problem 9, suppose you wanted the option to sell the land to the buyer in one year. Assuming all the facts are the same, describe the transaction that would occur today. What is the price of the tr...

See Answer

Q: A call option with an exercise price of $25 and

A call option with an exercise price of $25 and four months to expiration has a price of $2.75. The stock is currently priced at $23.80, and the risk-free rate is 2.5 percent per year, compounded cont...

See Answer

Q: A stock has a beta of 1.12 and an expected

A stock has a beta of 1.12 and an expected return of 10.8 percent. A risk-free asset currently earns 2.7 percent.

See Answer

Q: A call option matures in six months. The underlying stock

A call option matures in six months. The underlying stock price is $75, and the stock’s return has a standard deviation of 20 percent per year. The risk-free rate is 4 percent per year, compounded con...

See Answer

Q: A call option has an exercise price of $60 and

A call option has an exercise price of $60 and matures in six months. The current stock price is $64, and the risk-free rate is 5 percent per year, compounded continuously. What is the price of the ca...

See Answer

Q: If you need $20,000 in 12 years, how much will

If you need $20,000 in 12 years, how much will you need to deposit today if you can earn 9 percent per year compounded continuously?

See Answer

Q: A stock is currently selling for $67 per share. A

A stock is currently selling for $67 per share. A call option with an exercise price of $70 sells for $3.21 and expires in three months. If the risk-free rate of interest is 2.6 percent per year, comp...

See Answer

Q: A put option that expires in six months with an

A put option that expires in six months with an exercise price of $45 sells for $2.34. The stock is currently priced at $48, and the risk-free rate is 3.5 percent per year, compounded continuously. Wh...

See Answer

Q: Rework Problem 55 assuming that the loan agreement calls for

Rework Problem 55 assuming that the loan agreement calls for a principal reduction of $14,300 every year instead of equal annual payments.Data from Problem 55:Prepare an amortization schedule for a fi...

See Answer

Q: Say you own an asset that had a total return

Say you own an asset that had a total return last year of 11.65 percent. If the inflation rate last year was 2.75 percent, what was your real return?

See Answer

Q: A put option and a call option with an exercise

A put option and a call option with an exercise price of $70 and three months to expiration sell for $1.30 and $6.25, respectively. If the risk-free rate is 3.1 percent per year, compounded continuous...

See Answer

Q: A put option and call option with an exercise price

A put option and call option with an exercise price of $50 expire in four months and sell for $5.99 and $8.64, respectively. If the stock is currently priced at $52.27, what is the annual continuously...

See Answer

Q: What are the prices of a call option and a

What are the prices of a call option and a put option with the following characteristics?,,,

See Answer

Q: Assume that the historical return on large-company stocks is a

Assume that the historical return on large-company stocks is a predictor of the future returns. What return would you estimate for large-company stocks over the next year? The next 10 years? 20 years?...

See Answer

Q: What are the deltas of a call option and a

What are the deltas of a call option and a put option with the following characteristics? What does the delta of the option tell you?,,,

See Answer

Q: You own a lot in Key West, Florida, that is

You own a lot in Key West, Florida, that is currently unused. Similar lots have recently sold for $1,250,000. Over the past five years, the price of land in the area has increased 7 percent per year,...

See Answer

Q: Pearl, Inc., has offered $228 million cash for all of

Pearl, Inc., has offered $228 million cash for all of the common stock in Jam Corporation. Based on recent market information, Jam is worth $214 million as an independent operation. If the merger make...

See Answer

Q: Consider the following premerger information about Firm A and Firm

Consider the following premerger information about Firm A and Firm B:Assume that Firm A acquires Firm B via an exchange of stock at a price of $49 for each share of B’s stock. Both F...

See Answer

Q: You are given the following information concerning options on a

You are given the following information concerning options on a particular stock:a. What is the intrinsic value of the call option? Of the put option?b. What is the time value of the call option? Of t...

See Answer

Q: Fly-By-Night Couriers is analyzing the possible acquisition of Flash-in-the-Pan Restaurants.

Fly-By-Night Couriers is analyzing the possible acquisition of Flash-in-the-Pan Restaurants.Neither firm has debt. The forecasts of Fly-By-Night show that the purchase would increase its annual aftert...

See Answer

Q: Consider the following premerger information about Firm X and Firm

Consider the following premerger information about Firm X and Firm Y:Assume that Firm X acquires Firm Y by issuing new long-term debt for all the shares outstanding at a merger premium of $6 per share...

See Answer

Q: Locate the Treasury issue in Figure 7.4 maturing in May

Locate the Treasury issue in Figure 7.4 maturing in May 2038. What is its coupon rate? What is its bid price? What was the previous day’s asked price? Assume a par value of $10,000....

See Answer

Q: Bilbo Baggins wants to save money to meet three objectives.

Bilbo Baggins wants to save money to meet three objectives. First, he would like to be able to retire 30 years from now with retirement income of $17,500 per month for 25 years, with the first payment...

See Answer

Q: Assume that the following balance sheets are stated at book

Assume that the following balance sheets are stated at book value. Suppose that Meat Co. purchases Loaf, Inc.The fair market value of Loaf’s fixed assets is $9,800 versus the $6,900...

See Answer

Q: In Problem 20, McGilla Golf would like to know the

In Problem 20, McGilla Golf would like to know the sensitivity of NPV to changes in the price of the new clubs and the quantity of new clubs sold. What is the sensitivity of the NPV to each of these v...

See Answer

Q: Silver Enterprises has acquired All Gold Mining in a merger

Silver Enterprises has acquired All Gold Mining in a merger transaction. Construct the balance sheet for the new corporation if the merger is treated as a purchase of interests for accounting purposes...

See Answer

Q: Penn Corp. is analyzing the possible acquisition of Teller Company.

Penn Corp. is analyzing the possible acquisition of Teller Company. Both firms have no debt. Penn believes the acquisition will increase its total aftertax annual cash flows by $1.6 million indefinite...

See Answer

Q: Three Guys Burgers, Inc., has offered $16.5 million for all

Three Guys Burgers, Inc., has offered $16.5 million for all of the common stock in Two Guys Fries, Corp. The current market capitalization of Two Guys as an independent company is $13.4 million. Assum...

See Answer

Q: The shareholders of Bread Company have voted in favor of

The shareholders of Bread Company have voted in favor of a buyout offer from Butter Corporation. Information about each firm is given here:Bread’s shareholders will receive one share...

See Answer

Q: Consider the following premerger information about a bidding firm (Firm

Consider the following premerger information about a bidding firm (Firm B) and a target firm (Firm T). Assume that both firms have no debt outstanding.Firm B has estimated that the value of the synerg...

See Answer

Q: In Problem 8, are the shareholders of Firm T better

In Problem 8, are the shareholders of Firm T better off with the cash offer or the stock offer? At what exchange ratio of B shares to T shares would the shareholders in T be indifferent between the tw...

See Answer

Q: Sunset Boards is a small company that manufactures and sells

Sunset Boards is a small company that manufactures and sells surfboards in Malibu. Tad Marks, the founder of the company, is in charge of the design and sale of the surfboards, but his background is i...

See Answer

Q: You have recently been hired by Swan Motors, Inc. (SMI),

You have recently been hired by Swan Motors, Inc. (SMI), in its relatively new treasury management department. SMI was founded eight years ago by Joe Swan. Joe found a method to manufacture a cheaper...

See Answer

Q: Assume that the tax rate is 21 percent. You can

Assume that the tax rate is 21 percent. You can borrow at 8 percent before taxes. Should you lease or buy?

See Answer

Q: Mark Sexton and Todd Story have been discussing the future

Mark Sexton and Todd Story have been discussing the future of S&S Air. The company has been experiencing fast growth, and the two see only clear skies in the company’s future. However, the fast growth...

See Answer

Q: Consider the following information about three stocks:

Consider the following information about three stocks:b. If the expected T-bill rate is 3.80 percent, what is the expected risk premium on the portfolio?c. If the expected inflation rate is 3.30 perce...

See Answer

Q: After deciding to buy a new car, you can either

After deciding to buy a new car, you can either lease the car or purchase it on a three-year loan. The car you wish to buy costs $43,000. The dealer has a special leasing arrangement where you pay $4,...

See Answer

Q: Stephenson Real Estate Company was founded 25 years ago by

Stephenson Real Estate Company was founded 25 years ago by the current CEO, Robert Stephenson. The company purchases real estate, including land and buildings, and rents the property to tenants. The c...

See Answer

Q: You have recently been hired by Piepkorn Manufacturing to work

You have recently been hired by Piepkorn Manufacturing to work in the newly established treasury department. Piepkorn Manufacturing is a small company that produces cardboard boxes in a variety of siz...

See Answer

Q: Webb Corporation was founded 20 years ago by its president,

Webb Corporation was founded 20 years ago by its president, Bryan Webb. The company originally began as a mail-order company, but it has grown rapidly in recent years, in large part due to its website...

See Answer

Q: Sterling Wyatt, the president of Howlett Industries, has been exploring

Sterling Wyatt, the president of Howlett Industries, has been exploring ways of improving the company’s financial performance. Howlett manufactures and sells office equipment to reta...

See Answer

Q: Joi Chatman recently received her finance degree and has decided

Joi Chatman recently received her finance degree and has decided to enter the mortgage broker business. Rather than working for someone else, she will open her own shop. Her cousin Mike has approached...

See Answer

Q: S&S Air is preparing its first public securities offering. In

S&S Air is preparing its first public securities offering. In consultation with Renata Harper of underwriter Raines and Warren, Chris Guthrie decided that a convertible bond with a 20-year maturity wa...

See Answer

Q: As a new graduate, you’ve taken a management position with

As a new graduate, you’ve taken a management position with Exotic Cuisines, Inc., a restaurant chain that just went public last year. The company’s restaurants specialize in exotic main dishes, using...

See Answer

Q: Birdie Golf, Inc., has been in merger talks with Hybrid

Birdie Golf, Inc., has been in merger talks with Hybrid Golf Company for the past six months. After several rounds of negotiations, the offer under discussion is a cash offer of $185 million for Hybri...

See Answer

Q: Suppose Sunburn Sunscreen and Frostbite Thermal wear in the previous

Suppose Sunburn Sunscreen and Frostbite Thermal wear in the previous problems have decided to merge. Because the two companies have seasonal sales, the combined firm’s return on assets will have a sta...

See Answer

Q: Rework Problems 1 and 2 assuming the ending share price

Rework Problems 1 and 2 assuming the ending share price is $58.Data from Problem 2:In Problem 1, what was the dividend yield? The capital gains yield?Data from Problem 1:Suppose a stock had an initial...

See Answer

Q: You want to create a portfolio equally as risky as

You want to create a portfolio equally as risky as the market, and you have $1,000,000 to invest. Given this information, fill in the rest of the following table:,,,

See Answer

Q: Warf Computers has decided to proceed with the manufacture and

Warf Computers has decided to proceed with the manufacture and distribution of the virtual keyboard (VK) the company has developed. To undertake this venture, the company needs to obtain equipment for...

See Answer

Q: First City Bank pays 9 percent simple interest on its

First City Bank pays 9 percent simple interest on its savings account balances, whereas Second City Bank pays 9 percent interest compounded annually. If you made a deposit of $7,500 in each bank, how...

See Answer

Q: In Problem 4, over what range of lease payments will

In Problem 4, over what range of lease payments will the lease be profitable for both parties?Problem 4:Assume that your company does not anticipate paying taxes for the next several years. What are t...

See Answer

Q: An All-Pro defensive lineman is in contract negotiations. The team

An All-Pro defensive lineman is in contract negotiations. The team has offered the following salary structure:Time ……………………………………………………………………………..…….. Salary0 ……………………………………………………………………………….. $8,400,0...

See Answer

Q: For each of the following, compute the future value:

For each of the following, compute the future value:

See Answer

Q: For each of the following, compute the present value:

For each of the following, compute the present value:

See Answer

Q: Solve for the unknown interest rate in each of the

Solve for the unknown interest rate in each of the following:

See Answer

Q: Solve for the unknown number of years in each of

Solve for the unknown number of years in each of the following:

See Answer

Q: Assume the total cost of a college education will be

Assume the total cost of a college education will be $345,000 when your child enters college in 18 years. You presently have $73,000 to invest. What annual rate of interest must you earn on your inves...

See Answer

Q: At 6.1 percent interest, how long does it take to

At 6.1 percent interest, how long does it take to double your money? To quadruple it?

See Answer

Q: A proposed cost-saving device has an installed cost of $735,000.

A proposed cost-saving device has an installed cost of $735,000. The device will be used in a five-year project but is classified as three-year MACRS property for tax purposes. The required initial ne...

See Answer

Q: In the previous problem, suppose a sales associate told you

In the previous problem, suppose a sales associate told you the policy costs $800,000. At what interest rate would this be a fair deal?Data from Problem 10:The Maybe Pay Life Insurance Co. is trying t...

See Answer

Q: Locate the Treasury bond in Figure 7.4 maturing in February

Locate the Treasury bond in Figure 7.4 maturing in February 2040. Is this a premium or a discount bond? What is its current yield? What is its yield to maturity? What is the bid-ask spread in dollars?...

See Answer

Q: According to the Census Bureau, in October 2016, the average

According to the Census Bureau, in October 2016, the average house price in the United States was $354,900. In October 2000, the average price was $215,100. What was the annual increase in the price o...

See Answer

Q: You’re trying to save to buy a new $245,000 Ferrari.

You’re trying to save to buy a new $245,000 Ferrari. You have $50,000 today that can be invested at your bank. The bank pays 4.3 percent annual interest on its accounts. How long will it be before you...

See Answer

Q: Imprudential, Inc., has an unfunded pension liability of $415 million

Imprudential, Inc., has an unfunded pension liability of $415 million that must be paid in 20 years. To assess the value of the firm’s stock, financial analysts want to discount this liability back to...

See Answer

Q: You have just received notification that you have won the

You have just received notification that you have won the $2 million first prize in the Centennial Lottery. However, the prize will be awarded on your 100th birthday (assuming you’re around to collect...

See Answer

Q: This question illustrates what is known as discount interest. Imagine

This question illustrates what is known as discount interest. Imagine you are discussing a loan with a somewhat unscrupulous lender. You want to borrow $25,000 for one year. The interest rate is 14.9...

See Answer

Q: Your coin collection contains fifty 1952 silver dollars. If your

Your coin collection contains fifty 1952 silver dollars. If your grandparents purchased them for their face value when they were new, how much will your collection be worth when you retire in 2067, as...

See Answer

Q: In 1895, the first U.S. Open Golf Championship was held.

In 1895, the first U.S. Open Golf Championship was held. The winner’s prize money was $150. In 2016, the winner’s check was $1,800,000. What was the percentage increase per year in the winner’s check...

See Answer

Q: The “Brasher doubloon,” which was featured in the plot of

The “Brasher doubloon,” which was featured in the plot of the Raymond Chandler novel, The High Window, was sold at auction in 2014 for $4,582,500. The coin had a face value of $15 when it was first is...

See Answer

Q: You have $100,000 to invest in a portfolio containing Stock

You have $100,000 to invest in a portfolio containing Stock X and Stock Y. Your goal is to create a portfolio that has an expected return of 12.7 percent. If Stock X has an expected return of 11.4 per...

See Answer

Q: Refer back to the Series EE savings bonds we discussed

Refer back to the Series EE savings bonds we discussed at the very beginning of the chapter.a. Assuming you purchased a $50 face value bond, what is the exact rate of return you would earn if you held...

See Answer

Q: Suppose you are still committed to owning a $245,000 Ferrari

Suppose you are still committed to owning a $245,000 Ferrari (see Problem 9). If you believe your mutual fund can achieve an annual rate of return of 11.2 percent and you want to buy the car in 9 year...

See Answer

Q: Bond X is a premium bond making semiannual payments. The

Bond X is a premium bond making semiannual payments. The bond pays a coupon rate of 7.4 percent, has a YTM of 6.8 percent, and has 13 years to maturity. Bond Y is a discount bond making semiannual pay...

See Answer

Q: You have just made your first $5,500 contribution to your

You have just made your first $5,500 contribution to your retirement account. Assuming you earn a return of 10 percent per year and make no additional contributions, what will your account be worth wh...

See Answer

Q: You are scheduled to receive $20,000 in two years. When

You are scheduled to receive $20,000 in two years. When you receive it, you will invest it for six more years at 6.8 percent per year. How much will you have in eight years?

See Answer

Q: You expect to receive $10,000 at graduation in two years.

You expect to receive $10,000 at graduation in two years. You plan on investing it at 9 percent until you have $60,000. How long will you wait from now?

See Answer

Q: The Johnson Company sells 2,400 pairs of running shoes per

The Johnson Company sells 2,400 pairs of running shoes per month at a cash price of $99 per pair. The firm is considering a new policy that involves 30 days’ credit and an increase in price to $100 pe...

See Answer

Q: Silicon Wafers, Inc. (SWI), is debating whether or not to

Silicon Wafers, Inc. (SWI), is debating whether or not to extend credit to a particular customer. SWI’s products, primarily used in the manufacture of semiconductors, currently sell for $975 per unit....

See Answer

Q: You are serving on a jury. A plaintiff is suing

You are serving on a jury. A plaintiff is suing the city for injuries sustained after a freak street sweeper accident. In the trial, doctors testified that it will be five years before the plaintiff i...

See Answer

Q: Consider the following information about two alternative credit strategies:

Consider the following information about two alternative credit strategies:The higher cost per unit reflects the expense associated with credit orders, and the higher price per unit reflects the exist...

See Answer

Q: You receive a credit card application from Shady Banks Savings

You receive a credit card application from Shady Banks Savings and Loan offering an introductory rate of 1.25 percent per year, compounded monthly for the first six months, increasing thereafter to 17...

See Answer

Q: Suppose a corporation currently sells Q units per month for

Suppose a corporation currently sells Q units per month for a cash-only price of P. Under a new credit policy that allows one month’s credit, the quantity sold will be Q′ and the price per unit will b...

See Answer

Q: All Night, Inc., uses a Miller-Orr cash management approach with

All Night, Inc., uses a Miller-Orr cash management approach with a lower limit of $43,000, an upper limit of $125,000, and a target balance of $80,000. Explain what each of these points represents; th...

See Answer

Q: Slap Shot Corporation has a fixed cost of $40 associated

Slap Shot Corporation has a fixed cost of $40 associated with buying and selling marketable securities. The interest rate is currently .013 percent per day, and the firm has estimated that the standar...

See Answer

Q: Both Bond Sam and Bond Dave have 7.3 percent coupons,

Both Bond Sam and Bond Dave have 7.3 percent coupons, make semiannual payments, and are priced at par value. Bond Sam has three years to maturity, whereas Bond Dave has 20 years to maturity. If intere...

See Answer

Q: Based on the Miller-Orr model, describe what will happen to

Based on the Miller-Orr model, describe what will happen to the lower limit, the upper limit, and the spread (the distance between the two) if the variation in net cash flow grows. Give an intuitive e...

See Answer

Q: The variance of the daily cash flows for the Pele

The variance of the daily cash flows for the Pele Bicycle Shop is $890,000. The opportunity cost to the firm of holding cash is 4.1 percent per year. What should the target cash level and the upper li...

See Answer

Q: Rise Against Corporation has determined that its target cash balance

Rise Against Corporation has determined that its target cash balance if it uses the BAT model is $5,100. The total cash needed for the year is $31,000, and the order cost is $10. What interest rate mu...

See Answer

Q: Given the following information, calculate the target cash balance using

Given the following information, calculate the target cash balance using the BAT model:Annual interest rate ………………………………………………………………………. 4.5%Fixed order cost ………………………………………………………………………………. $25Total c...

See Answer

Q: White Whale Corporation has an average daily cash balance of

White Whale Corporation has an average daily cash balance of $1,700. Total cash needed for the year is $64,000. The interest rate is 5 percent, and replenishing the cash costs $8 each time. What are t...

See Answer

Q: Debit and Credit Bookkeepers needs a total of $21,000 in

Debit and Credit Bookkeepers needs a total of $21,000 in cash during the year for transactions and other purposes. Whenever cash runs low, it sells $1,500 in securities and transfers in the cash. The...

See Answer

Q: Consider the following information about Stocks I and II:

Consider the following information about Stocks I and II:The market risk premium is 7 percent, and the risk-free rate is 3.5 percent. Which stock has the most systematic risk? Which one has the most u...

See Answer

Q: You are looking at a one-year loan of $10,000. The

You are looking at a one-year loan of $10,000. The interest rate is quoted as 9.8 percent plus 2 points. A point on a loan is 1 percent (one percentage point) of the loan amount. Quotes similar to thi...

See Answer

Q: The All Day Company is currently holding $690,000 in cash.

The All Day Company is currently holding $690,000 in cash. It projects that over the next year its cash outflows will exceed cash inflows by $140,000 per month. How much of the current cash holdings s...

See Answer

Q: Chris Guthrie was recently hired by S&S Air, Inc., to

Chris Guthrie was recently hired by S&S Air, Inc., to assist the company with its financial planning and to evaluate the company’s performance. Chris graduated from college five...

See Answer

Q: After Chris completed the ratio analysis for S&S Air (see

After Chris completed the ratio analysis for S&S Air (see Chapter 3), Mark and Todd approached him about planning for next year’s sales. The company had historically used little...

See Answer

Q: Investment X offers to pay you $4,200 per year for

Investment X offers to pay you $4,200 per year for eight years, whereas Investment Y offers to pay you $6,100 per year for five years. Which of these cash flow streams has the higher present value if...

See Answer

Q: Ben Bates graduated from college six years ago with a

Ben Bates graduated from college six years ago with a finance undergraduate degree. Although he is satisfied with his current job, his goal is to become an investment banker. He feels that an MBA degr...

See Answer

Q: Ragan, Inc., was founded nine years ago by brother and

Ragan, Inc., was founded nine years ago by brother and sister Carrington and Genevieve Ragan. The company manufactures and installs commercial heating, ventilation, and cooling (HVAC) units. Ragan, In...

See Answer

Q: Seth Bullock, the owner of Bullock Gold Mining, is evaluating

Seth Bullock, the owner of Bullock Gold Mining, is evaluating a new gold mine in South Dakota. Dan Dority, the company’s geologist, has just finished his analysis of the mine site. He has estimated th...

See Answer

Q: Conch Republic Electronics is a midsized electronics manufacturer located in

Conch Republic Electronics is a midsized electronics manufacturer located in Key West, Florida. The company president is Shelley Couts, who inherited the company. When it was founded over 70 years ago...

See Answer

Q: Shelley Couts, the owner of Conch Republic Electronics, has received

Shelley Couts, the owner of Conch Republic Electronics, has received the capital budgeting analysis from Jay McCanless for the new smartphone the company is considering. Shelley is pleased with the re...

See Answer

Q: Suppose you observe the following situation:

Suppose you observe the following situation:

See Answer

Q: "You recently graduated from college, and your job search led

"You recently graduated from college, and your job search led you to S&S Air. Because you felt the company’s business was taking off, you accepted a job offer. The first day on t...

See Answer

Q: Joey Moss, a recent finance graduate, has just begun his

Joey Moss, a recent finance graduate, has just begun his job with the investment firm of Covili and Wyatt. Paul Covili, one of the firm’s founders, has been talking to Joey about the firm’s investment...

See Answer

Q: The interest rate on a one-year loan is quoted as

The interest rate on a one-year loan is quoted as 12 percent plus 3 points (see the previous problem). What is the EAR? Is your answer affected by the loan amount?

See Answer

Q: You are buying a house and will borrow $225,000 on

You are buying a house and will borrow $225,000 on a 30-year fixed rate mortgage with monthly payments to finance the purchase. Your loan officer has offered you a mortgage with an APR of 4.3 percent....

See Answer

Q: In the previous problem, suppose that you believe that you

In the previous problem, suppose that you believe that you will only live in the house for eight years before selling the house and buying another house. This means that in eight years, you will pay o...

See Answer

Q: Bond J has a coupon rate of 3 percent. Bond

Bond J has a coupon rate of 3 percent. Bond K has a coupon rate of 9 percent. Both bonds have 14 years to maturity, make semiannual payments, and have a YTM of 6 percent. If interest rates suddenly ri...

See Answer

Q: Two banks in the area offer 30-year, $275,000 mortgages at

Two banks in the area offer 30-year, $275,000 mortgages at 5.1 percent and charge a $4,300 loan application fee. However, the application fee charged by Insecurity Bank and Trust is refundable if the...

See Answer

Q: This problem illustrates a deceptive way of quoting interest rates

This problem illustrates a deceptive way of quoting interest rates called add-on interest. Imagine that you see an advertisement for Crazy Judy’s Stereo City that reads something like this: “$1,000 In...

See Answer

Q: You have successfully started and operated a company for the

You have successfully started and operated a company for the past 10 years. You have decided that it is time to sell your company and spend time on the beaches of Hawaii. A potential buyer is interest...

See Answer

Q: Rework Problem 1 assuming that the scanner will be depreciated

Rework Problem 1 assuming that the scanner will be depreciated as three-year property under MACRS (see Chapter 10 for the depreciation allowances).Problem 1:Assume that the tax rate is 21 percent. You...

See Answer

Q: Suppose you observe the following situation:

Suppose you observe the following situation:a. Calculate the expected return on each stock.b. Assuming the capital asset pricing model holds and Stock A’s beta is greater than Stock...

See Answer

Q: Your Christmas ski vacation was great, but it unfortunately ran

Your Christmas ski vacation was great, but it unfortunately ran a bit over budget. All is not lost: You just received an offer in the mail to transfer your $15,000 balance from your current credit car...

See Answer

Q: An insurance company is offering a new policy to its

An insurance company is offering a new policy to its customers. Typically, the policy is bought by a parent or grandparent for a child at the child’s birth. The purchaser (say, the parent) makes the f...

See Answer

Q: You have just arranged for a $2,350,000 mortgage to finance

You have just arranged for a $2,350,000 mortgage to finance the purchase of a large tract of land. The mortgage has an APR of 5.2 percent, and it calls for monthly payments over the next 30 years. How...

See Answer

Q: A financial planning service offers a college savings program.

A financial planning service offers a college savings program. The plan calls for you to make six annual payments of $15,000 each, with the first payment occurring today, your child’s 12th birthday. B...

See Answer

Q: Your financial planner offers you two different investment plans. Plan

Your financial planner offers you two different investment plans. Plan X is an annual perpetuity of $35,000 per year. Plan Y is an annuity for 15 years and an annual payment of $47,000. Both plans wil...

See Answer

Q: What is the value of an investment that pays $25,000

What is the value of an investment that pays $25,000 every other year forever, if the first payment occurs one year from today and the discount rate is 7 percent compounded daily? What is the value to...

See Answer

Q: Workman Software has 6.4 percent coupon bonds on the market

Workman Software has 6.4 percent coupon bonds on the market with 18 years to maturity. The bonds make semiannual payments and currently sell for 94.31 percent of par. What is the current yield on the...

See Answer

Q: Twist Corp. has a current accounts receivable balance of $537,810.

Twist Corp. has a current accounts receivable balance of $537,810. Credit sales for the year just ended were $5,473,640. What is the receivables turnover? The days’ sales in receivables? How long did...

See Answer

Q: You have 45 years left until retirement and want to

You have 45 years left until retirement and want to retire with $4 million. Your salary is paid annually, and you will receive $50,000 at the end of the current year. Your salary will increase at 3 pe...

See Answer

Q: A check-cashing store is in the business of making personal

A check-cashing store is in the business of making personal loans to walk-up customers. The store makes only one-week loans at 6.8 percent interest per week. a. What APR must the store report to its c...

See Answer

Q: Use the results of Problem 25 to find the accounting,

Use the results of Problem 25 to find the accounting, cash, and financial break-even quantities for the company in Problem 27.Data from Problem 27:Consider a project to supply Detroit with 30,000 tons...

See Answer

Q: The King Corporation has ending inventory of $386,735, and cost

The King Corporation has ending inventory of $386,735, and cost of goods sold for the year just ended was $4,981,315. What is the inventory turnover? The days’ sales in inventory? How long on average...

See Answer

Q: The Wildcat Oil Company is trying to decide whether to

The Wildcat Oil Company is trying to decide whether to lease or buy a new computer-assisted drilling system for its oil exploration business. Management has decided that it must use the system to stay...

See Answer

Q: Queen, Inc., has a total debt ratio of .46. What

Queen, Inc., has a total debt ratio of .46. What is its debt-equity ratio? What is its equity multiplier?

See Answer

Q: Makers Corp. had additions to retained earnings for the year

Makers Corp. had additions to retained earnings for the year just ended of $415,000. The firm paid out $220,000 in cash dividends, and it has ending total equity of $5.6 million. If the company curren...

See Answer

Q: If Roten Rooters, Inc., has an equity multiplier of 1.27,

If Roten Rooters, Inc., has an equity multiplier of 1.27, total asset turnover of 2.10, and a profit margin of 6.1 percent, what is its ROE?

See Answer

Q: Jack Corp. has a profit margin of 6.4 percent, total

Jack Corp. has a profit margin of 6.4 percent, total asset turnover of 1.77, and ROE of 15.84 percent. What is this firm’s debt-equity ratio?

See Answer

Q: Based only on the following information for Thrice Corp., did

Based only on the following information for Thrice Corp., did cash go up or down? By how much? Classify each event as a source or use of cash.Decrease in inventory ………………………………………………………………………….. $375D...

See Answer

Q: Chamberlain Co. wants to issue new 20-year bonds for some

Chamberlain Co. wants to issue new 20-year bonds for some much-needed expansion projects. The company currently has 6 percent coupon bonds on the market that sell for $1,083, make semiannual payments,...

See Answer

Q: Consider the following simplified financial statements for the Wims Corporation

Consider the following simplified financial statements for the Wims Corporation (assuming no income taxes):

See Answer

Q: The balance sheet for the Heir Jordan Corporation follows. Based

The balance sheet for the Heir Jordan Corporation follows. Based on this information and the income statement in the previous problem, supply the missing information using the percentage of sales appr...

See Answer

Q: You own a portfolio that is invested 35 percent in

You own a portfolio that is invested 35 percent in Stock X, 20 percent in Stock Y, and 45 percent in Stock Z. The expected returns on these three stocks are 9 percent, 15 percent, and 12 percent, resp...

See Answer

Q: From the previous two questions, prepare a pro forma balance

From the previous two questions, prepare a pro forma balance sheet showing EFN, assuming an increase in sales of 15 percent, no new external debt or equity financing, and a constant payout ratio.Data...

See Answer

Q: If A7X Co. has an ROA of 7.6 percent and

If A7X Co. has an ROA of 7.6 percent and a payout ratio of 25 percent, what is its internal growth rate?

See Answer

Q: If Synyster Corp. has an ROE of 14.7 percent and

If Synyster Corp. has an ROE of 14.7 percent and a payout ratio of 30 percent, what is itss ustainable growth rate?

See Answer

Q: The Wildcat Oil Company is trying to decide whether to

The Wildcat Oil Company is trying to decide whether to lease or buy a new computer-assisted drilling system for its oil exploration business. Management has decided that it must use the system to stay...

See Answer

Q: Based on the following information, calculate the sustainable growth rate

Based on the following information, calculate the sustainable growth rate for Kaleb’s Heavy Equipment:,,,

See Answer

Q: Assuming the following ratios are constant, what is the sustainable

Assuming the following ratios are constant, what is the sustainable growth rate?,,,

See Answer

Q: Hodgkiss Mfg., Inc., is currently operating at only 91 percent

Hodgkiss Mfg., Inc., is currently operating at only 91 percent of fixed asset capacity. Current sales are $715,000. How fast can sales grow before any new fixed assets are needed?

See Answer

Q: For the company in Problem 16, suppose fixed assets are

For the company in Problem 16, suppose fixed assets are $520,000 and sales are projected to grow to $790,000. How much in new fixed assets are required to support this growth in sales? Assume the comp...

See Answer

Q: You purchase a bond with an invoice price of $948.

You purchase a bond with an invoice price of $948. The bond has a coupon rate of 5.9 percent, and there are four months to the next semiannual coupon date. What is the clean price of the bond?

See Answer

Q: Ramble On Co. wishes to maintain a growth rate of

Ramble On Co. wishes to maintain a growth rate of 12 percent per year, a debt-equity ratio of .90, and a dividend payout ratio of 25 percent. The ratio of total assets to sales is constant at 0.85. Wh...

See Answer

Q: Use the results of Problem 26 to find the degree

Use the results of Problem 26 to find the degree of operating leverage for the company in Problem 27 at the base-case output level of 30,000 tons. How does this number compare to the sensitivity figur...

See Answer

Q: A firm wishes to maintain an internal growth rate of

A firm wishes to maintain an internal growth rate of 7.1 percent and a dividend payout ratio of 25 percent. The current profit margin is 6.5 percent, and the firm uses no external financing sources. W...

See Answer

Q: In Question 1, assume the company pays out half of

In Question 1, assume the company pays out half of net income in the form of a cash dividend. Costs and assets vary with sales, but debt and equity do not. Prepare the pro forma statements and determi...

See Answer

Q: Based on the following information, calculate the sustainable growth rate

Based on the following information, calculate the sustainable growth rate for Hendrix Guitars, Inc.:,,,

See Answer

Q: You’ve collected the following information about Molino, Inc.:

You’ve collected the following information about Molino, Inc.:What is the sustainable growth rate for the company? If it does grow at this rate, how much new borrowing will take plac...

See Answer

Q: Calculate the internal growth rate for the company in Problem

Calculate the internal growth rate for the company in Problem 22. Now calculate the internal growth rate using ROA × b for both beginning of period and end of period total assets. What do you observe?...

See Answer

Q: You purchase a bond with a coupon rate of 5.3

You purchase a bond with a coupon rate of 5.3 percent and a clean price of $951. If the next semiannual coupon payment is due in two months, what is the invoice price?

See Answer

Q: Excey Corp. has 8 percent coupon bonds making annual payments

Excey Corp. has 8 percent coupon bonds making annual payments with a YTM of 7.2 percent. The current yield on these bonds is 7.55 percent. How many years do these bonds have left until they mature?

See Answer

Q: Find the EAR in each of the following cases:

Find the EAR in each of the following cases:

See Answer

Q: A company has a single zero coupon bond outstanding that

A company has a single zero coupon bond outstanding that matures in five years with a face value of $17.5 million. The current value of the company’s assets is $15.9 million, and the standard deviatio...

See Answer

Q: Suppose the following bond quotes for IOU Corporation appear in

Suppose the following bond quotes for IOU Corporation appear in the financial page of today’s newspaper. Assume the bond has a face value of $2,000 and the current date is April 19,...

See Answer

Q: Suppose you bought a bond with an annual coupon of

Suppose you bought a bond with an annual coupon of 7 percent one year ago for $1,010. The bond sells for $985 today.a. Assuming a $1,000 face value, what was your total dollar return on this investmen...

See Answer

Q: You have $10,000 to invest in a stock portfolio. Your

You have $10,000 to invest in a stock portfolio. Your choices are Stock X with an expected return of 12.1 percent and Stock Y with an expected return of 9.8 percent. If your goal is to create a portfo...

See Answer

Q: If the appropriate discount rate for the following cash flows

If the appropriate discount rate for the following cash flows is 9 percent compounded quarterly, what is the present value of the cash flows? Year …………………………….……………………. Cash Flow1 …….……………………………………………...

See Answer

Q: Imagination Dragons Corporation needs to raise funds to finance a

Imagination Dragons Corporation needs to raise funds to finance a plant expansion, and it has decided to issue 25-year zero coupon bonds with a par value of $1,000 each to raise the money. The require...

See Answer

Q: Suppose your company needs to raise $53 million and you

Suppose your company needs to raise $53 million and you want to issue 20-year bonds for this purpose. Assume the required return on your bond issue will be 5.3 percent, and you’re evaluating two issue...

See Answer

Q: Even though most corporate bonds in the United States make

Even though most corporate bonds in the United States make coupon payments semiannually, bonds issued elsewhere often have annual coupon payments. Suppose a German company issues a bond with a par val...

See Answer

Q: You are looking at an investment that has an effective

You are looking at an investment that has an effective annual rate of 11.6 percent. What is the effective semiannual return? The effective quarterly return? The effective monthly return?

See Answer

Q: You want to have $2.5 million in real dollars in

You want to have $2.5 million in real dollars in an account when you retire in 40 years. The nominal return on your investment is 10.3 percent and the inflation rate is 3.7 percent. What real amount m...

See Answer

Q: Bond P is a premium bond with a coupon rate

Bond P is a premium bond with a coupon rate of 9 percent. Bond D has a coupon rate of 5 percent and is currently selling at a discount. Both bonds make annual payments, have a YTM of 7 percent, and ha...

See Answer

Q: The YTM on a bond is the interest rate you

The YTM on a bond is the interest rate you earn on your investment if interest rates don’t change. If you actually sell the bond before it matures, your realized return is known as the holding period...

See Answer

Q: Find the APR, or stated rate, in each of the

Find the APR, or stated rate, in each of the following cases:

See Answer

Q: Jallouk Corporation has two different bonds currently outstanding. Bond M

Jallouk Corporation has two different bonds currently outstanding. Bond M has a face value of $20,000 and matures in 20 years. The bond makes no payments for the first six years, then pays $900 every...

See Answer

Q: Based on the following information, calculate the expected return:

Based on the following information, calculate the expected return:,,,

See Answer

Q: Zevon Industries has a zero coupon bond issue that matures

Zevon Industries has a zero coupon bond issue that matures in two years with a face value of $40,000. The current value of the company’s assets is $26,700, and the standard deviation of the return on...

See Answer

Q: At one point, certain U.S. Treasury bonds were callable. Consider

At one point, certain U.S. Treasury bonds were callable. Consider the prices in the following three Treasury issues as of May 15, 2017:

See Answer

Q: A Japanese company has a bond outstanding that sells for

A Japanese company has a bond outstanding that sells for 105.43 percent of its ¥100,000 par value. The bond has a coupon rate of 3.4 percent paid annually and matures in 16 years. What is the yield to...

See Answer

Q: Gabriele Enterprises has bonds on the market making annual payments,

Gabriele Enterprises has bonds on the market making annual payments, with eight years to maturity, a par value of $1,000, and selling for $948. At this price, the bonds yield 5.1 percent. What must th...

See Answer

Q: Weismann Co. issued 15-year bonds a year ago at a

Weismann Co. issued 15-year bonds a year ago at a coupon rate of 4.9 percent. The bonds make semiannual payments and have a par value of $1,000. If the YTM on these bonds is 4.5 percent, what is the c...

See Answer

Q: West Corp. issued 25-year bonds two years ago at a

West Corp. issued 25-year bonds two years ago at a coupon rate of 5.3 percent. The bonds make semiannual payments. If these bonds currently sell for 105 percent of par value, what is the YTM?

See Answer

Q: McConnell Corporation has bonds on the market with 14.5 years

McConnell Corporation has bonds on the market with 14.5 years to maturity, a YTM of 5.3 percent, a par value of $1,000, and a current price of $1,045. The bonds make semiannual payments. What must the...

See Answer

Q: You find a zero coupon bond with a par value

You find a zero coupon bond with a par value of $10,000 and 17 years to maturity. If the yield to maturity on this bond is 4.2 percent, what is the price of the bond? Assume semiannual compounding per...

See Answer

Q: The Jackson-Timberlake Wardrobe Co. just paid a dividend of $2.15

The Jackson-Timberlake Wardrobe Co. just paid a dividend of $2.15 per share on its stock. The dividends are expected to grow at a constant rate of 4 percent per year indefinitely. If investors require...

See Answer

Q: First National Bank charges 13.1 percent compounded monthly on its

First National Bank charges 13.1 percent compounded monthly on its business loans. First United Bank charges 13.4 percent compounded semiannually. As a potential borrower, which bank would you go to f...

See Answer

Q: Based on the following information, calculate the expected return:

Based on the following information, calculate the expected return:,,,

See Answer

Q: After successfully completing your corporate finance class, you feel the

After successfully completing your corporate finance class, you feel the next challenge ahead is to serve on the board of directors of Schenkel Enterprises. Unfortunately, you will be the only person...

See Answer

Q: In the previous problem, assume that the company uses cumulative

In the previous problem, assume that the company uses cumulative voting, and there are four seats in the current election. How much will it cost you to buy a seat now?Previous problem:After successful...

See Answer

Q: Colosseum Corp. has a zero coupon bond that matures in

Colosseum Corp. has a zero coupon bond that matures in five years with a face value of $65,000. The current value of the company’s assets is $62,000, and the standard deviation of its return on assets...

See Answer

Q: The Perfect Rose Co. has earnings of $3.18 per share.

The Perfect Rose Co. has earnings of $3.18 per share. The benchmark PE for the company is 18. What stock price would you consider appropriate? What if the benchmark PE were 21?

See Answer

Q: TwitterMe, Inc., is a new company and currently has negative

TwitterMe, Inc., is a new company and currently has negative earnings. The company’s sales are $2.1 million and there are 130,000 shares outstanding. If the benchmark price-sales ratio is 4.3, what is...

See Answer

Q: Moody Farms just paid a dividend of $2.65 on its

Moody Farms just paid a dividend of $2.65 on its stock. The growth rate in dividends is expected to be a constant 3.8 percent per year indefinitely. Investors require a return of 15 percent for the fi...

See Answer

Q: Metallica Bearings, Inc., is a young start-up company. No dividends

Metallica Bearings, Inc., is a young start-up company. No dividends will be paid on the stock over the next nine years because the firm needs to plow back its earnings to fuel growth. The company will...

See Answer

Q: Maurer, Inc., has an odd dividend policy. The company has

Maurer, Inc., has an odd dividend policy. The company has just paid a dividend of $2.75 per share and has announced that it will increase the dividend by $4.50 per share for each of the next five year...

See Answer

Q: Lohn Corporation is expected to pay the following dividends over

Lohn Corporation is expected to pay the following dividends over the next four years: $13, $9, $6, and $2.75. Afterward, the company pledges to maintain a constant 5 percent growth rate in dividends f...

See Answer

Q: Synovec Co. is growing quickly. Dividends are expected to grow

Synovec Co. is growing quickly. Dividends are expected to grow at a rate of 30 percent for the next three years, with the growth rate falling off to a constant 4 percent thereafter. If the required re...

See Answer

Q: To solve the bid price problem presented in the text,

To solve the bid price problem presented in the text, we set the project NPV equal to zero and found the required price using the definition of OCF. Thus the bid price represents a financial breakeven...

See Answer

Q: Elliott Credit Corp. wants to earn an effective annual return

Elliott Credit Corp. wants to earn an effective annual return on its consumer loans of 17.1 percent per year. The bank uses daily compounding on its loans. What interest rate is the bank required by l...

See Answer

Q: Mobray Corp. is experiencing rapid growth. Dividends are expected to

Mobray Corp. is experiencing rapid growth. Dividends are expected to grow at 25 percent per year during the next three years, 15 percent over the following year, and then 6 percent per year indefinite...

See Answer

Q: The next dividend payment by Savitz, Inc., will be $2.34

The next dividend payment by Savitz, Inc., will be $2.34 per share. The dividends are anticipated to maintain a growth rate of 4.5 percent forever. If the stock currently sells for $37 per share, what...

See Answer

Q: Antiques R Us is a mature manufacturing firm. The company

Antiques R Us is a mature manufacturing firm. The company just paid a dividend of $9.80, but management expects to reduce the payout by 4 percent per year indefinitely. If you require a return of 9.5...

See Answer

Q: In addition to the five factors discussed in the chapter,

In addition to the five factors discussed in the chapter, dividends also affect the price of an option. The Black-Scholes option pricing model with dividends is:All of the variables are the same as th...

See Answer

Q: You have found the following stock quote for RJW Enterprises,

You have found the following stock quote for RJW Enterprises, Inc., in the financial pages of today’s newspaper. What was the closing price for this stock that appeared in yesterday&...

See Answer

Q: A7X Corp. just paid a dividend of $1.55 per share.

A7X Corp. just paid a dividend of $1.55 per share. The dividends are expected to grow at 21 percent for the next eight years and then level off to a growth rate of 3.5 percent indefinitely. If the req...

See Answer

Q: Navel County Choppers, Inc., is experiencing rapid growth. The company

Navel County Choppers, Inc., is experiencing rapid growth. The company expects dividends to grow at 18 percent per year for the next 11 years before leveling off at 4 percent into perpetuity. The requ...

See Answer

Q: Domergue Corp. currently has an EPS of $3.76, and the

Domergue Corp. currently has an EPS of $3.76, and the benchmark PE for the company is 21. Earnings are expected to grow at 5.1 percent per year.a. What is your estimate of the current stock price?b. W...

See Answer

Q: You have found the following historical information for the Daniela

You have found the following historical information for the Daniela Company over the past four years:

See Answer

Q: Suppose the firm in Problem 16 is considering two mutually

Suppose the firm in Problem 16 is considering two mutually exclusive investments. Project A has an NPV of $1,900, and Project B has an NPV of $2,800. As the result of taking Project A, the standard de...

See Answer

Q: In the previous problem, we assumed that the stock had

In the previous problem, we assumed that the stock had a single stock price for the year. However, if you look at stock prices over any year, you will find a high and low stock price for the year. Ins...

See Answer

Q: Based on the following information, calculate the expected return:

Based on the following information, calculate the expected return:

See Answer

Q: A portfolio is invested 25 percent in Stock G, 55

A portfolio is invested 25 percent in Stock G, 55 percent in Stock J, and 20 percent in Stock K. The expected returns on these stocks are 11 percent, 9 percent, and 15 percent, respectively. What is t...

See Answer

Q: Consider the following information:

Consider the following information:

See Answer

Q: The Drogon Co. just issued a dividend of $2.80 per

The Drogon Co. just issued a dividend of $2.80 per share on its common stock. The company is expected to maintain a constant 4.5 percent growth rate in its dividends indefinitely. If the stock sells f...

See Answer

Q: Lannister Manufacturing has a target debt-equity ratio of .55. Its

Lannister Manufacturing has a target debt-equity ratio of .55. Its cost of equity is 11 percent, and its cost of debt is 6 percent. If the tax rate is 21 percent, what is the company’s WACC?

See Answer

Q: What was the average annual return on large-company stocks from

What was the average annual return on large-company stocks from 1926 through 2016: a. In nominal terms?b. In real terms?

See Answer

Q: Fama’s Llamas has a weighted average cost of capital of

Fama’s Llamas has a weighted average cost of capital of 7.9 percent. The company’s cost of equity is 11 percent, and its pretax cost of debt is 5.8 percent. The tax rate is 25 percent. What is the com...

See Answer

Q: Dinklage Corp. has 7 million shares of common stock outstanding.

Dinklage Corp. has 7 million shares of common stock outstanding. The current share price is $68, and the book value per share is $8. The company also has two bond issues outstanding. The first bond is...

See Answer

Q: In Problem 12, suppose the most recent dividend was $3.25

In Problem 12, suppose the most recent dividend was $3.25 and the dividend growth rate is 5 percent. Assume that the overall cost of debt is the weighted average of that implied by the two outstanding...

See Answer

Q: Starset, Inc., has a target debt-equity ratio of .85. Its

Starset, Inc., has a target debt-equity ratio of .85. Its WACC is 9.1 percent, and the tax rate is 23 percent.a. If the company’s cost of equity is 14 percent, what is its pretax cost of debt?b. If in...

See Answer

Q: Given the following information for Watson Power Co., find the

Given the following information for Watson Power Co., find the WACC. Assume the company’s tax rate is 21 percent.Debt: …………………….. 15,000 bonds with a 5.8 percent coupon outstanding, $1,000 par value,...

See Answer

Q: You are planning to save for retirement over the next

You are planning to save for retirement over the next 30 years. To do this, you will invest $750 per month in a stock account and $250 per month in a bond account. The return of the stock account is e...

See Answer

Q: What would the lease payment have to be for both

What would the lease payment have to be for both lessor and lessee to be indifferent about the lease?

See Answer

Q: Your company has been approached to bid on a contract

Your company has been approached to bid on a contract to sell 4,800 voice recognition (VR) computer keyboards per year for four years. Due to technological improvements, beyond that time they will be...

See Answer

Q: The Rhaegel Corporation’s common stock has a beta of 1.07.

The Rhaegel Corporation’s common stock has a beta of 1.07. If the risk-free rate is 3.5 percent and the expected return on the market is 10 percent, what is the company’s cost of equity capital?

See Answer

Q: Stock in Daenerys Industries has a beta of 1.05. The

Stock in Daenerys Industries has a beta of 1.05. The market risk premium is 7 percent, and T-bills are currently yielding 3.4 percent. The company’s most recent dividend was $2.35 per share, and divid...

See Answer

Q: In the previous problem, suppose the projections given for price,

In the previous problem, suppose the projections given for price, quantity, variable costs, and fixed costs are all accurate to within ±10 percent. Calculate the best-case and worst-case NPV figures.P...

See Answer

Q: Suppose Stark, Ltd., just issued a dividend of $2.51 per

Suppose Stark, Ltd., just issued a dividend of $2.51 per share on its common stock. The company paid dividends of $2.01, $2.17, $2.25, and $2.36 per share in the last four years. If the stock currentl...

See Answer

Q: Holdup Bank has an issue of preferred stock with a

Holdup Bank has an issue of preferred stock with a stated dividend of $4.25 that just sold for $93 per share. What is the bank’s cost of preferred stock?

See Answer

Q: Viserion, Inc., is trying to determine its cost of debt.

Viserion, Inc., is trying to determine its cost of debt. The firm has a debt issue outstanding with 23 years to maturity that is quoted at 103 percent of face value. The issue makes semiannual payment...

See Answer

Q: Jiminy’s Cricket Farm issued a 30-year, 6 percent semiannual bond

Jiminy’s Cricket Farm issued a 30-year, 6 percent semiannual bond three years ago. The bond currently sells for 93 percent of its face value. The company’s tax rate is 22 percent.a. What is the pretax...

See Answer

Q: For the firm in Problem 7, suppose the book value

For the firm in Problem 7, suppose the book value of the debt issue is $95 million. In addition, the company has a second debt issue on the market, a zero coupon bond with eight years left to maturity...

See Answer

Q: Targaryen Corporation has a target capital structure of 70 percent

Targaryen Corporation has a target capital structure of 70 percent common stock, 5 percent preferred stock, and 25 percent debt. Its cost of equity is 10 percent, the cost of preferred stock is 5 perc...

See Answer

Q: You have an investment that will pay you 0.67 percent

You have an investment that will pay you 0.67 percent per month. How much will you have per dollar invested in one year? In two years?

See Answer

Q: Leah, Inc., is proposing a rights offering. Presently there are

Leah, Inc., is proposing a rights offering. Presently there are 375,000 shares outstanding at $67 each. There will be 50,000 new shares offered at $58 each.a. What is the new market value of the compa...

See Answer

Q: The Metallica Heavy Metal Mining (MHMM) Corporation wants to diversify

The Metallica Heavy Metal Mining (MHMM) Corporation wants to diversify its operations. Some recent financial information for the company is shown here:Stock price …………………………………………………………………………………………… $...

See Answer

Q: Suppose we are thinking about replacing an old computer with

Suppose we are thinking about replacing an old computer with a new one. The old one cost us $1,560,000; the new one will cost $1,872,000. The new machine will be depreciated straight-line to zero over...

See Answer

Q: In the previous problem, suppose the fixed asset actually qualifies

In the previous problem, suppose the fixed asset actually qualifies for 100 percent bonus depreciation in the first year. What is the new NPV?Previous problem:You have been hired as a consultant for P...

See Answer

Q: In Problem 10, what would the ROE on the investment

In Problem 10, what would the ROE on the investment have to be if we wanted the price after the offering to be $75 per share? (Assume the PE ratio remains constant.) What is the NPV of this investment...

See Answer

Q: Bell Hill Mfg. is considering a rights offer. The company

Bell Hill Mfg. is considering a rights offer. The company has determined that the ex-rights price would be $63. The current price is $68 per share, and there are 26 million shares outstanding. The rig...

See Answer

Q: Show that the value of a right just prior to

Show that the value of a right just prior to expiration can be written as:Value of a right=PRO PX = (PRO Ps) / (N + 1)where PRO, PS, and PX stand for the rights-on price, the subscription price, and t...

See Answer

Q: Prahm Corp. wants to raise $4.7 million via a rights

Prahm Corp. wants to raise $4.7 million via a rights offering. The company currently has 530,000 shares of common stock outstanding that sell for $55 per share. Its underwriter has set a subscription...

See Answer

Q: Knight Inventory Systems, Inc., has announced a rights offer. The

Knight Inventory Systems, Inc., has announced a rights offer. The company has announced that it will take four rights to buy a new share in the offering at a subscription price of $35. At the close of...

See Answer

Q: Titan Mining Corporation has 7.5 million shares of common stock

Titan Mining Corporation has 7.5 million shares of common stock outstanding, 250,000 shares of 4.2 percent preferred stock outstanding, and 140,000 bonds with a semiannual coupon of 5.1 percent outsta...

See Answer

Q: An all-equity firm is considering the following projects:

An all-equity firm is considering the following projects:The T-bill rate is 4 percent, and the expected return on the market is 11 percent.a. Which projects have a higher expected return than the firm...

See Answer

Q: You want to be a millionaire when you retire in

You want to be a millionaire when you retire in 40 years. How much do you have to save each month if you can earn an annual return of 9.7 percent? How much do you have to save each month if you wait 1...

See Answer

Q: Suppose your company needs $24 million to build a new

Suppose your company needs $24 million to build a new assembly line. Your target debt-equity ratio is .75. The flotation cost for new equity is 7 percent, but the flotation cost for debt is only 3 per...

See Answer

Q: Cully Company needs to raise $80 million to start a

Cully Company needs to raise $80 million to start a new project and will raise the money by selling new bonds. The company will generate no internal equity for the foreseeable future. The company has...

See Answer

Q: Using the following returns, calculate the arithmetic average returns, the

Using the following returns, calculate the arithmetic average returns, the variances, and the standard deviations for X and Y.

See Answer

Q: The Clifford Corporation has announced a rights offer to raise

The Clifford Corporation has announced a rights offer to raise $35 million for a new journal, the Journal of Financial Excess. This journal will review potential articles after the author pays a nonre...

See Answer

Q: You want to borrow $115,000 from your local bank to

You want to borrow $115,000 from your local bank to buy a new sailboat. You can afford to make monthly payments of $2,250, but no more. Assuming monthly compounding, what is the highest rate you can a...

See Answer

Q: Sommer, Inc., is considering a project that will result in

Sommer, Inc., is considering a project that will result in initial aftertax cash savings of $2.3 million at the end of the first year, and these savings will grow at a rate of 2 percent per year indef...

See Answer

Q: Being Human, Inc., recently issued new securities to finance a

Being Human, Inc., recently issued new securities to finance a new TV show. The project cost $35 million, and the company paid $2.2 million in flotation costs. In addition, the equity issued had a flo...

See Answer

Q: Ying Import has several bond issues outstanding, each making semiannual

Ying Import has several bond issues outstanding, each making semiannual interest payments. The bonds are listed in the following table. If the corporate tax rate is 22 percent, what is the aftertax co...

See Answer

Q: Red Shoe Co. has concluded that additional equity financing will

Red Shoe Co. has concluded that additional equity financing will be needed to expand operations and that the needed funds will be best obtained through a rights offering. It has correctly determined t...

See Answer

Q: The Woods Co. and the Spieth Co. have both announced

The Woods Co. and the Spieth Co. have both announced IPOs at $40 per share. One of these is undervalued by $9, and the other is overvalued by $4, but you have no way of knowing which is which. You pla...

See Answer

Q: The Whistling Straits Corporation needs to raise $60 million to

The Whistling Straits Corporation needs to raise $60 million to finance its expansion into new markets. The company will sell new shares of equity via a general cash offering to raise the needed funds...

See Answer

Q: The Maybe Pay Life Insurance Co. is trying to sell

The Maybe Pay Life Insurance Co. is trying to sell you an investment policy that will pay you and your heirs $35,000 per year forever. If the required return on this investment is 4.7 percent, how muc...

See Answer

Q: Given the following information for Bowie Pizza Co., calculate the

Given the following information for Bowie Pizza Co., calculate the depreciation expense: Sales = $64,000; Costs = $30,700; Addition to retained earnings = $5,700; Dividends paid = $1,980; Interest exp...

See Answer

Q: Refer to Table 12.1 in the text and look at

Refer to Table 12.1 in the text and look at the period from 1970 through 1975. Data from Table 12.1:a. Calculate the arithmetic average returns for large-company stocks and T-bills over this period.b....

See Answer

Q: In Problem 5, if the SEC filing fee and associated

In Problem 5, if the SEC filing fee and associated administrative expenses of the offering are $1.2 million, how many shares need to be sold?Problem 5:The Whistling Straits Corporation needs to raise...

See Answer

Q: The Raven Co. has just gone public. Under a firm

The Raven Co. has just gone public. Under a firm commitment agreement, Raven received $21.39 for each of the 20 million shares sold. The initial offering price was $23 per share, and the stock rose to...

See Answer

Q: Nemesis, Inc., has 165,000 shares of stock outstanding. Each share

Nemesis, Inc., has 165,000 shares of stock outstanding. Each share is worth $77, so the company’s market value of equity is $12,705,000. Suppose the firm issues 30,000 new shares at the following pric...

See Answer

Q: Wayne, Inc., wishes to expand its facilities. The company currently

Wayne, Inc., wishes to expand its facilities. The company currently has 6 million shares outstanding and no debt. The stock sells for $64 per share, but the book value per share is $19. Net income is...

See Answer

Q: You need a 30-year, fixed-rate mortgage to buy a new

You need a 30-year, fixed-rate mortgage to buy a new home for $235,000. Your mortgage bank will lend you the money at an APR of 5.35 percent for this 360-month loan. However, you can afford monthly pa...

See Answer

Q: Ghost, Inc., has no debt outstanding and a total market

Ghost, Inc., has no debt outstanding and a total market value of $185,000. Earnings before interest and taxes, EBIT, are projected to be $29,000 if economic conditions are normal. If there is strong e...

See Answer

Q: Thrice Corp. uses no debt. The weighted average cost of

Thrice Corp. uses no debt. The weighted average cost of capital is 8.4 percent. If the current market value of the equity is $16.3 million and there are no taxes, what is EBIT?

See Answer

Q: In Problem 10, suppose the corporate tax rate is 22

In Problem 10, suppose the corporate tax rate is 22 percent. What is EBIT in this case? What is the WACC? Explain.Problem 10:Thrice Corp. uses no debt. The weighted average cost of capital is 8.4 perc...

See Answer

Q: Blitz Industries has a debt-equity ratio of 1.25. Its WACC

Blitz Industries has a debt-equity ratio of 1.25. Its WACC is 8.3 percent, and its cost of debt is 5.1 percent. The corporate tax rate is 21 percent.a. What is the company’s cost of equity capital?b....

See Answer

Q: Citee Corp. has no debt but can borrow at 6.1

Citee Corp. has no debt but can borrow at 6.1 percent. The firm’s WACC is currently 9.4 percent, and the tax rate is 21 percent.a. What is the company’s cost of equity?b. If the firm converts to 25 pe...

See Answer

Q: After living in a university residence for one year, Mary‐

After living in a university residence for one year, Mary‐Beth decides to rent an apartment for the remaining three years of her degree. She has found a nice location that will cost $550 per month. Re...

See Answer

Q: You are examining the economy of a very small, completely isolated

You are examining the economy of a very small, completely isolated island nation. There are only three people on this island: Fred, Robinson, and Friday. Fred owns a house valued at $1,000 and owes Fr...

See Answer

Q: After losing money playing online poker, Scott visits a loan shark

After losing money playing online poker, Scott visits a loan shark for a $750 loan. To avoid a visit from the “collection agency,” he will have to repay $800 in just one week. a. What is the nominal i...

See Answer

Q: Josephine needs to borrow $ 180,000 to purchase her new

Josephine needs to borrow $ 180,000 to purchase her new house in Yarmouth, Nova Scotia. She would like to pay off the mortgage in 20 years, making monthly payments. For the initial three‐year term, Pr...

See Answer

Q: A lakefront house in Kingston, Ontario, is for sale with

A lakefront house in Kingston, Ontario, is for sale with an asking price of $499,000. The real estate market has been quite active, so the house will almost certainly attract several offers, and may s...

See Answer

Q: Timmy sets himself a goal of amassing $1 million in his

Timmy sets himself a goal of amassing $1 million in his retirement fund by the time he turns 61. He begins saving $ 3,000 at the end of each year, starting on his 21st birthday (40 years of saving). a...

See Answer

Q: Paul and Maria want to have enough money to travel around the

Paul and Maria want to have enough money to travel around the world when they retire. They both just turned 30 and will retire when they turn 60. They earn a total of $ 9,000 after taxes each month. T...

See Answer

Q: Use the following information to create a revised forecast of the year

Use the following information to create a revised forecast of the year 3 balance sheet for Finns ’ Fridges. Cash will increase by the forecast EBITDA amount (see Practice Problem 29); it will be reduc...

See Answer

Q: A firm has just declared that its dividend next year will be

A firm has just declared that its dividend next year will be $3 per share. That rate of payment will continue for an additional four years, after which the dividends will fall back to their usual $2 p...

See Answer

Q: GG Co. shows the following information on its financial statements:

GG Co. shows the following information on its financial statements: interest‐bearing debt is $900,000; shareholders ’ equity (SE) is $2,500,000; sales are $5,050,000; net income is $685,750; dividends...

See Answer

Q: A 65‐year‐old man intends to use his retirement

A 65‐year‐old man intends to use his retirement funds to purchase an annuity from a life insurance company. Given the amount of money the man has available to invest, the insurance company is able to...

See Answer

Q: Two friends, Abe and Betty, are planning for their retirement

Two friends, Abe and Betty, are planning for their retirement. Both are 20 years old and plan on retiring in 40 years with $1 million each. Betty plans on making annual deposits beginning in one year...

See Answer

Q: List the four major financial sectors in the financial system and discuss

List the four major financial sectors in the financial system and discuss how they relate to one another.

See Answer

Q: You are planning on buying your first home and need to borrow

You are planning on buying your first home and need to borrow $ 250,000 from the bank. The manager offers you two mortgages: the long option will take 25 years to be paid off, and your annual payments...

See Answer

Q: What is a corporate spread?

What is a corporate spread?

See Answer

Q: 1. Which of the following statements concerning bonds is incorrect?

1. Which of the following statements concerning bonds is incorrect? a. They involve blended payments of principal and interest. b. They have a fixed maturity date, at which time the issuer repays the...

See Answer

Q: 1. Which of the following statements is incorrect? a

1. Which of the following statements is incorrect? a. An ordinary annuity has payments at the end of each year. b. An annuity due has payments at the beginning of each year. c. A perpetuity is conside...

See Answer

Q: 1. Which of the following statements about consistent financial analysis is

1. Which of the following statements about consistent financial analysis is correct? a. Accounting standards are different across countries. b. If the input data are the same, the ratios for companies...

See Answer

Q: 1. What is the total amount accumulated after six years if

1. What is the total amount accumulated after six years if someone invests $ 1,000 today with a simple annual interest rate of 8 percent? How about with a compound annual interest rate of 8 percent? a...

See Answer

Q: 1. Which of the following is correct? a.

1. Which of the following is correct? a. IFRS are the primary accounting standards for publicly accountable enterprises in Canada. b. Canadian private companies are required to use IFRS. c. Canadian G...

See Answer

Q: 1. Which of the following is not classified as cash flow

1. Which of the following is not classified as cash flow from financing? a. issuance of long‐term debt b. repurchase of capital stock c. payment of dividends d. purchase of inventory 2. Which of the...

See Answer

Q: 1. Which of the following businesses is the least likely to

1. Which of the following businesses is the least likely to be operated as a partnership? a. accounting firm b. doctors ’ office c. lawyers ’ office d. steel foundry 2. Which of the following statemen...

See Answer

Q: Explain what is meant by the matching principle. How is this

Explain what is meant by the matching principle. How is this principle related to the use of accrual accounting?

See Answer

Q: Explain how banks, pension funds, insurance firms, and mutual

Explain how banks, pension funds, insurance firms, and mutual funds work in the financial system.

See Answer

Q: What are some of the key corporate financing decisions made by firms

What are some of the key corporate financing decisions made by firms?

See Answer

Q: What does IFRS stand for? What types of Canadian companies must

What does IFRS stand for? What types of Canadian companies must prepare their financial statements in accordance with IFRS (or U.S. GAAP)?

See Answer

Q: Identify the major sources of financing used by: (a

Identify the major sources of financing used by: (a) governments and (b) businesses.

See Answer

Q: Who is responsible for the preparation of a company’s financial statements?

Who is responsible for the preparation of a company’s financial statements?

See Answer

Q: What complications arise when dealing with mortgage loans in Canada?

What complications arise when dealing with mortgage loans in Canada?

See Answer

Q: Identify the main components of a firm’s balance sheet and income statement

Identify the main components of a firm’s balance sheet and income statement.

See Answer

Q: What are secondary market transactions? How do secondary markets facilitate the

What are secondary market transactions? How do secondary markets facilitate the primary markets?

See Answer

Q: Find Finns ’ Fridges ’ return on equity (ROE) for

Find Finns ’ Fridges ’ return on equity (ROE) for years 1 and 2, using the owners ’ equity figure at the end of each year. Did this ratio improve or get worse between year 1 and year 2?

See Answer

Q: State three of the most basic principles of IFRS.

State three of the most basic principles of IFRS.

See Answer

Q: Calculate the cash price of the following bond, sold on September

Calculate the cash price of the following bond, sold on September 21: par = $1,000; coupon rate = 4 percent, paid on January 1 and July 1; quoted price = $956. Explain why the cash price is different...

See Answer

Q: Describe why financial and market intermediaries exist in our financial system.

Describe why financial and market intermediaries exist in our financial system.

See Answer

Q: The following values are the spread for corporate bond yields.

The following values are the spread for corporate bond yields. a . One‐year T‐bills are trading with a YTM of 6 percent. What yield would you expect to find on A&ac...

See Answer

Q: Using the Fisher relationship, calculate the exact real interest rate and

Using the Fisher relationship, calculate the exact real interest rate and the approximate real rate, given a Tbill rate of 8 percent and an expected inflation rate of 3.6 percent.

See Answer

Q: In the DuPont system, there are two components of ROA.

In the DuPont system, there are two components of ROA. Determine whether efficiency or productivity (or both) is responsible for the increase in ROA for Finns ’ Fridges from year 1 to year 2.

See Answer

Q: Calculate the price of a bond with FV of $1,

Calculate the price of a bond with FV of $1,000, a coupon rate of 8 percent (paid semi‐annually), and five years to maturity when: a . k b = 10 percent. b . kb = 8 percent. c . kb = 6 percent.

See Answer

Q: Suppose that, several years ago, the Canadian government issued three

Suppose that, several years ago, the Canadian government issued three very similar bonds; each has a $1,000 face value and a 10‐percent coupon rate and will mature in five years. The only difference b...

See Answer

Q: The following is data for two bonds at a time when the

The following is data for two bonds at a time when the market yield is 7 percent. These bonds are otherwise identical (FV = $1,000, five years to maturity, semi‐annual coupon payment...

See Answer

Q: It is now May 1, 2015, and Peter has just

It is now May 1, 2015, and Peter has just purchased a five‐year U.S. government bond (FV = $1,000) with a quoted price of 93.863. This bond has a 6‐percent coupon rate, and the last semi‐annual coupon...

See Answer

Q: a . Suppose the Finns believe they can increase revenues to $

a . Suppose the Finns believe they can increase revenues to $2,600 in year 3. Use this figure and the percentage of sales balance sheet (Practice Problem 27) to forecast the company ’ s balance sheet...

See Answer

Q: Why does money have a “time value”?

Why does money have a “time value”?

See Answer

Q: 1. Which of the following is true about finance?

1. Which of the following is true about finance? a. Finance is the study of how and under what terms savings (money) are allocated between lenders and borrowers. b. Finance is different from economics...

See Answer

Q: List and define the four major forms of business organization.

List and define the four major forms of business organization.

See Answer

Q: What is the primary objective of financial reporting under IFRS?

What is the primary objective of financial reporting under IFRS?

See Answer

Q: Why do income statements differ from tax statements? What is the

Why do income statements differ from tax statements? What is the major difference?

See Answer

Q: When market interest rates are above the coupon rate on a bond

When market interest rates are above the coupon rate on a bond, is it a premium or discount bond?

See Answer

Q: What are the major provisions of SOX?

What are the major provisions of SOX?

See Answer

Q: Explain how to calculate the effective rate for any period.

Explain how to calculate the effective rate for any period.

See Answer

Q: How is the balance sheet related to the income statement?

How is the balance sheet related to the income statement?

See Answer

Q: Explain why a firm cannot claim CCA recapture and a terminal loss

Explain why a firm cannot claim CCA recapture and a terminal loss for the same asset class in the same year.

See Answer

Q: How do floaters and real return bonds provide protection against inflation?

How do floaters and real return bonds provide protection against inflation?

See Answer

Q: What is the difference between a positive and a negative covenant provision

What is the difference between a positive and a negative covenant provision?

See Answer

Q: Identify and briefly describe the three main channels of savings.

Identify and briefly describe the three main channels of savings.

See Answer

Q: State the main differences and similarities between sole proprietorships and partnerships.

State the main differences and similarities between sole proprietorships and partnerships.

See Answer

Q: Distinguish between market and financial intermediaries.

Distinguish between market and financial intermediaries.

See Answer

Q: Explain the cost imposed on society if firms become too big to

Explain the cost imposed on society if firms become too big to fail, and discuss whether the government should break up large firms when they pose such risks.

See Answer

Q: Should the Government allow one of the Big Six Canadian banks to

Should the Government allow one of the Big Six Canadian banks to fail if it loses money on its loan portfolio?

See Answer

Q: Describe the nature of the basic owner-manager agency relationship.

Describe the nature of the basic owner-manager agency relationship.

See Answer

Q: Define agency costs and describe both types.

Define agency costs and describe both types.

See Answer

Q: What is the primary goal of the corporation?

What is the primary goal of the corporation?

See Answer

Q: What role does the board of directors serve?

What role does the board of directors serve?

See Answer

Q: What are the main advantages and disadvantages of the corporation structure?

What are the main advantages and disadvantages of the corporation structure?

See Answer

Q: What form of investment income has the highest tax rate in Canada

What form of investment income has the highest tax rate in Canada?

See Answer

Q: What is the day count convention in Canada and the United States

What is the day count convention in Canada and the United States?

See Answer

Q: Describe the causes of a “credit crunch.”

Describe the causes of a “credit crunch.”

See Answer

Q: Explain how to calculate the present value and future value of an

Explain how to calculate the present value and future value of an ordinary annuity and an annuity due.

See Answer

Q: Why is the present value of $1 million in 50 years’

Why is the present value of $1 million in 50 years’ time worth very little today?

See Answer

Q: Who prescribes GAAP for U.S. companies?

Who prescribes GAAP for U.S. companies?

See Answer

Q: What time-value-of-money formula do we need

What time-value-of-money formula do we need to value a bond?

See Answer

Q: Define “perpetuity”.

Define “perpetuity”.

See Answer

Q: Describe the difference between positive and negative bond covenants.

Describe the difference between positive and negative bond covenants.

See Answer

Q: State the relationship between market rates and bond prices.

State the relationship between market rates and bond prices.

See Answer

Q: Calculate the price of the following bond: FV = $1

Calculate the price of the following bond: FV = $1,000; coupon rate = 6 percent, paid semi-annually; market rate = 5 percent; term to maturity = 10 years.

See Answer

Q: Describe the relationship between bond interest rate risk and the coupon rate

Describe the relationship between bond interest rate risk and the coupon rate, the market yield, and the term to maturity.

See Answer

Q: Summarize the main characteristics of corporations.

Summarize the main characteristics of corporations.

See Answer

Q: State the statutory responsibilities of directors that are described in the Canada

State the statutory responsibilities of directors that are described in the Canada Business Corporations Act.

See Answer

Q: A bond is currently trading at $841.70. It

A bond is currently trading at $841.70. It has 15 years to maturity. If you require a rate of return of 12 percent, what should be the bond ’ s coupon rate if the bond pays semi‐annual coupons?

See Answer

Q: Trustco Income Fund is an income trust whose units trade on the

Trustco Income Fund is an income trust whose units trade on the Toronto Stock Exchange. On October 31, 2006, just before the Government of Canada announced new taxes for businesses organized as trusts...

See Answer

Q: Suppose the inflation rate in Canada, as measured by the CPI

Suppose the inflation rate in Canada, as measured by the CPI, has been averaging 3.5 percent in recent years. The most recent Bank of Canada announcement indicates that it expects 3.2‐percent inflatio...

See Answer

Q: Khalil’ s summer job has given him $ 1,200 more

Khalil’ s summer job has given him $ 1,200 more than he needs for his tuition this year. The local bank pays simple interest at a rate of 0.5 percent per month. How much interest will he earn in one y...

See Answer

Q: A new Internet bank pays compound interest of 0.5 percent

A new Internet bank pays compound interest of 0.5 percent per month on deposits. How much interest will Khalil’ s summer savings of $ 1,200 earn in one year with this online bank account?

See Answer

Q: When you hired Dan to manage your business, you agreed to

When you hired Dan to manage your business, you agreed to pay him a bonus of 10 percent of profits at the end of each year. The company now has a choice between two projects (it can take on only one o...

See Answer

Q: If a bond‐rating agency downgrades the rating of a bond

If a bond‐rating agency downgrades the rating of a bond, how will it affect the price of that bond?

See Answer

Q: On the advice of a friend, Gilda invests $ 20,

On the advice of a friend, Gilda invests $ 20,000 in a mutual fund that has earned 10 percent per year, on average, in recent years. If this rate of return continues, determine how much her investment...

See Answer

Q: When Jon graduates in three years, he wants to throw a

When Jon graduates in three years, he wants to throw a big party, which will cost $800. To have this amount available, how much does he have to invest today if he can earn a compound return of 5 perce...

See Answer

Q: Grace, a retired librarian, would like to donate some money

Grace, a retired librarian, would like to donate some money to her alma mater to endow a $ 5,000 annual scholarship. The university will manage the funds and expects to earn 3 percent per year. How mu...

See Answer

Q: Describe the two major types of secondary markets.

Describe the two major types of secondary markets.

See Answer

Q: List the major jobs available in the financial industry.

List the major jobs available in the financial industry.

See Answer

Q: List the correct bookkeeping entries when a firm sells $50,

List the correct bookkeeping entries when a firm sells $50,000 worth of inventories for $80,000 using credit sales. (Ignore the tax effect.)

See Answer

Q: Calculate the price change for a 1-percent decrease in market

Calculate the price change for a 1-percent decrease in market yield for the following bond: par = $1,000; coupon rate = 6 percent, paid semi-annually; market yield = 6 percent; term to maturity = 10 y...

See Answer

Q: Based on the figures in practice problems 17 and 18, how

Based on the figures in practice problems 17 and 18, how much money did the shareholders actually invest in the firm (i.e., what is the value of the capital stock)? Use the following information to an...

See Answer

Q: An 8 percent annual coupon bond with 12 years left to maturity

An 8 percent annual coupon bond with 12 years left to maturity is selling for $928. What is the YTM of the bond?

See Answer

Q: A nine‐year, 6.5‐percent coupon bond

A nine‐year, 6.5‐percent coupon bond is selling for 106.2 percent of par. What is the bond ’ s market yield if it makes semi‐annual coupon payments?

See Answer

Q: The forecast for retained earnings (Practice Problem 31) changes the

The forecast for retained earnings (Practice Problem 31) changes the year 3 forecast for total liabilities and owners ’ equity to $4,770. With total assets forecast to be $5,177, determine how much ex...

See Answer

Q: Corine ’ s Candies Inc. registered a gross profit margin of

Corine ’ s Candies Inc. registered a gross profit margin of 75 percent on sales of $16 million in 2016. What would the company ’s income statement show for the cost of goods sold?

See Answer

Q: A company is contemplating issuing new 15‐year bonds at par

A company is contemplating issuing new 15‐year bonds at par. The company currently has 5.5 percent coupon bonds on the market selling for $936. The bonds pay semi‐annual interest and have 15 years unt...

See Answer

Q: Suppose Finns’ Fridges is subject to corporate income tax at a rate

Suppose Finns’ Fridges is subject to corporate income tax at a rate of 40 percent. What will the company ’s net income be after tax?

See Answer

Q: Janice borrowed $100,000 from friends and family to start

Janice borrowed $100,000 from friends and family to start her company (a sole proprietorship). Business has been poor recently, and Janice has decided to cease operations and liquidate the firm. She e...

See Answer

Q: Calculate the bank discount yield on a 92‐day U.

Calculate the bank discount yield on a 92‐day U.S. T‐bill that is currently quoted at $97.75. Find the bond equivalent yield on a 92‐day Canadian T‐bill with the same quoted price.

See Answer

Q: a . What is the value of a 10‐year,

a . What is the value of a 10‐year, zero coupon bond with a face value of $1,000 when the market rate is 8 percent? b . Calculate the YTM of the above zero coupon bond if the current price is $760.

See Answer

Q: A 90‐day U.S. T‐bill has

A 90‐day U.S. T‐bill has a bank discount yield (kBDY) of 4.673 percent. Find the quoted price. Find the bond equivalent yield (kBEY) on a 90‐day Canadian T‐bill with the same quoted price.

See Answer

Q: Altech Inc. has a convertible bond with a face value of

Altech Inc. has a convertible bond with a face value of $1,000 and coupon rate of 6 percent. The bond will mature in 10 years, and its current price is $950. The bond can be converted at any time into...

See Answer

Q: A bond has a yield to maturity of 8 percent and a

A bond has a yield to maturity of 8 percent and a current yield of 6 percent. Is the bond trading at par, at a premium, or at a discount? What can you say about the coupon rate?

See Answer

Q: Use two approaches to determine ROE in year 2. (Hint

Use two approaches to determine ROE in year 2. (Hint: one approach is from the definition and the other is to use the DuPont system.)

See Answer

Q: The present value of a dollar to be received one year from

The present value of a dollar to be received one year from today is 0.927644. The present value of a dollar to be received two years from today is 0.854172. What is the price of a bond that pays an an...

See Answer

Q: Determine Excelsior Inc. ’ s efficiency ratio of times interest earned

Determine Excelsior Inc. ’ s efficiency ratio of times interest earned in year 2.

See Answer

Q: Determine the company ’ s efficiency ratios including gross profit margin and

Determine the company ’ s efficiency ratios including gross profit margin and operating margin in year 2. Explain the differences.

See Answer

Q: Determine productivity ratios including receivables turnover and average collection period in year

Determine productivity ratios including receivables turnover and average collection period in year 2.

See Answer

Q: Describe the main advantages and disadvantages of sole proprietorships and partnerships.

Describe the main advantages and disadvantages of sole proprietorships and partnerships.

See Answer

Q: Suppose Janice obtains only $93,000 when she sells all

Suppose Janice obtains only $93,000 when she sells all the assets of the firm described in Practice Problem 18. How much money would the debt holders receive if the business were a corporation? If it...

See Answer

Q: Determine productivity ratios including inventory turnover and average days revenue in inventory

Determine productivity ratios including inventory turnover and average days revenue in inventory in year 2.

See Answer

Q: Determine net fixed asset turnover in year 2. /

Determine net fixed asset turnover in year 2.

See Answer

Q: David and Douglas invested $500 each to capitalize Finns ’ Fridges

David and Douglas invested $500 each to capitalize Finns ’ Fridges. To allow for future flexibility (such as selling shares to other investors), they placed a “par value” of $10 on each share; thus ea...

See Answer

Q: Determine valuation ratios including book value per share, dividend yield,

Determine valuation ratios including book value per share, dividend yield, and dividend payout for year 2.

See Answer

Q: If Excelsior Inc. keeps the same dividend payout ratio, what

If Excelsior Inc. keeps the same dividend payout ratio, what are the expected total dividends in year 3 if the sales growth rate is 5 percent?

See Answer

Q: What are the firm’ s changes in net working capital in year

What are the firm’ s changes in net working capital in year 2?

See Answer

Q: What sales growth rate must Excelsior Inc. achieve if it is

What sales growth rate must Excelsior Inc. achieve if it is to have a cash surplus?

See Answer

Q: If Excelsior Inc. ’ s expected sales growth rate is 5

If Excelsior Inc. ’ s expected sales growth rate is 5 percent, determine the external financing required. Will the corporation have a cash surplus or deficit?

See Answer

Q: The firm ’ s dividend payout ratio is 30 percent. What

The firm ’ s dividend payout ratio is 30 percent. What was the firm ’ s year 2 net income?

See Answer

Q: Using the net income and earnings per common share (EPS)

Using the net income and earnings per common share (EPS) figures from Canadian Pacific ’ s (CP ’ s) income statement (Figure 3‐5), determine how many shares (approximately) the company had outstanding...

See Answer

Q: List the four areas of conflict of interest between shareholders and managers

List the four areas of conflict of interest between shareholders and managers.

See Answer

Q: What was the growth rate of sales at CP in 2013 and

What was the growth rate of sales at CP in 2013 and 2014? Did the sales growth rate increase or decrease?

See Answer

Q: Based on the balance sheet you created, how much working capital

Based on the balance sheet you created, how much working capital does Finns ’ Fridges have?

See Answer

Q: Suppose that Finns’ Fridges actually pays $270 in dividends in year

Suppose that Finns’ Fridges actually pays $270 in dividends in year 3. Determine the value of the retained earnings account at the end of year 3 based on the forecast net income calculated in Practice...

See Answer

Q: In 2015, a firm ’ s revenue is $100,

In 2015, a firm ’ s revenue is $100,000, cost of sales is $40,000, rent is$15,000, depreciation is$3,000, and interest paid is $2,000. Its tax rate is 35 percent. a. Construct an income statement base...

See Answer

Q: What is the firm’ s net working capital in year 2?

What is the firm’ s net working capital in year 2?

See Answer

Q: For GG. Co., calculate the degree of total leverage (

For GG. Co., calculate the degree of total leverage (DTL) and break‐even point of sales at which the firm covers all its operating and fixed costs, given the following information: sales are $5,050,00...

See Answer

Q: Calculate the degree of total leverage (DTL) and break‐

Calculate the degree of total leverage (DTL) and break‐even point for a company, given the following information: sales are $400,000; variable cost is $130,000; net income is $180,000; tax rate ( T )...

See Answer

Q: The firm has 200 million shares outstanding. What is the firm

The firm has 200 million shares outstanding. What is the firm ’ s year 2 earnings per share?

See Answer

Q: Using the DuPont system, what are Excelsior Inc. ’ s

Using the DuPont system, what are Excelsior Inc. ’ s net profit margin, asset turnover, and leverage ratio in year 2?

See Answer

Q: Use the total “Dividends paid” figure from the firm ’

Use the total “Dividends paid” figure from the firm ’ s cash flow statement (Figure 3‐6) and your answer to Practice Problem 40 to estimate the dividends per common share outstanding for 2014.

See Answer

Q: The large competitor firm mentioned in Practice Problem 19 had net operating

The large competitor firm mentioned in Practice Problem 19 had net operating income of $4.426 million and sales of $30.16 million in its most recent accounting period. Find the operating margin for th...

See Answer

Q: One key part of ROE in the DuPont system is the return

One key part of ROE in the DuPont system is the return on assets (ROA). Find the ROA for Finns ’ Fridges for both years and determine if it is increasing or decreasing.

See Answer

Q: A firm ’ s net earnings are $85 million and it

A firm ’ s net earnings are $85 million and it has 60 million shares outstanding. Determine its earnings per share.

See Answer

Q: The balance sheet for a small firm shows total assets of $

The balance sheet for a small firm shows total assets of $529,500 and total liabilities of $379,000. What is the shareholders ’ equity?

See Answer

Q: Mrs. Kwan lives in Ontario. Her annual salary is $

Mrs. Kwan lives in Ontario. Her annual salary is $225,000. What is the marginal tax on her salary?

See Answer

Q: The balance sheet for a small corporation shows total assets of $

The balance sheet for a small corporation shows total assets of $525,600, common equity of $136,000, and retained earnings of $75,000. Calculate the total liabilities.

See Answer

Q: A firm had retained earnings of $18,000 at

A firm had retained earnings of $18,000 at the beginning of the year. Its net income for the year was $14,300, and its dividend payout ratio is 30 percent. What are its retained earnings at the end o...

See Answer

Q: Bank A pays 7.25 percent interest compounded semi‐annually

Bank A pays 7.25 percent interest compounded semi‐annually, Bank B pays 7.2 percent compounded quarterly, and Bank C pays 7.15 percent compounded monthly. Which bank pays the highest effective annual...

See Answer

Q: Jimmie is buying a new car. His bank quotes a rate

Jimmie is buying a new car. His bank quotes a rate of 9.5 percent per year for a car loan. Calculate the effective annual rate if the compounding occurs: a. annually b. quarterly c. monthly

See Answer

Q: If Alysha puts $ 50,000 in a savings account paying

If Alysha puts $ 50,000 in a savings account paying 6 percent per year, determine how much money she will have in total at the end of the first year if interest is compounded: a. annually b. monthly c...

See Answer

Q: Public corporations have no fixed lifespan; as such, they are

Public corporations have no fixed lifespan; as such, they are often viewed as entities that will pay dividends to their shareholders in perpetuity. Suppose a firm pays a dividend of $2 per share every...

See Answer

Q: What is the objective behind the stock option plan of executives?

What is the objective behind the stock option plan of executives? In reality, does it achieve this objective?

See Answer

Q: Mary‐Beth is planning to live in a university residence for

Mary‐Beth is planning to live in a university residence for three years while completing her degree. The annual cost for food and lodging is $ 6,500 and must be paid at the start of each school year....

See Answer

Q: At the age of 10, Felix decided that he wanted to

At the age of 10, Felix decided that he wanted to attend a very prestigious (and expensive) university. How much will his parents have to save each year to accumulate $ 40,000 by the time Felix needs...

See Answer

Q: An investment promises to pay you $100 per year starting in

An investment promises to pay you $100 per year starting in one year. The cash flow from the investment is expected to increase by 3 percent per year forever. If alternative investments of similar ris...

See Answer

Q: Jack is 28 years old now and plans to retire in 35

Jack is 28 years old now and plans to retire in 35 years. He works in a local bank and has an annual after‐tax income of $ 45,000. His expected annual expenditure is $36,000, and the rest of his incom...

See Answer

Q: Tommy has a goal of amassing $1 million by the time

Tommy has a goal of amassing $1 million by the time he retires. However, there always seems to be a reason not to save money, so he put it off for many years. Finally, with just 15 years before his re...

See Answer

Q: Use the definition of the leverage ratio in the DuPont system to

Use the definition of the leverage ratio in the DuPont system to determine if Finns ’ Fridges has become more or less leveraged between year 1 and year 2.

See Answer

Q: Grace, a retired librarian, would like to donate some money

Grace, a retired librarian, would like to donate some money to her alma mater to endow a $ 5,000 annual scholarship. The first scholarship will be awarded in five years. The university will manage the...

See Answer

Q: The most recent financial statements for a large Canadian furniture and appliance

The most recent financial statements for a large Canadian furniture and appliance rental chain show that its debt ratio was 0.256 and its debt‐to‐equity ratio (D/E) was 0.073. At the end of year 2, wa...

See Answer

Q: At the end of 2015, Corine ’ s Candies Inc.

At the end of 2015, Corine ’ s Candies Inc. had total shareholders ’ equity of $13.8 million. In 2016, the company had net income of $5.2 million and paid out half this amount in dividends, resulting...

See Answer

Q: A bank is currently offering a savings account paying an interest rate

A bank is currently offering a savings account paying an interest rate of 9 percent compounded quarterly. It would like to offer another account, with the same effective annual rate, but compounded mo...

See Answer

Q: List the basic areas of capital budgeting.

List the basic areas of capital budgeting.

See Answer

Q: Calculate the effective annual rates for the following: a.

Calculate the effective annual rates for the following: a. 24%, compounded daily b. 24%, compounded quarterly c. 24%, compounded every four months d. 24%, compounded semi‐annually e. 24%, compounded c...

See Answer

Q: Jane’s parents save $ 1,000 per year for 17 years

Jane’s parents save $ 1,000 per year for 17 years to pay for her university tuition costs. They deposit the money into a Registered Education Savings Plan (RESP) account so that no tax is payable on t...

See Answer

Q: GG Inc. just bought a computer for $4,000

GG Inc. just bought a computer for $4,000. It belongs to asset class 45 and has a CCA rate of 45 percent. Calculate the first‐year and second‐year CCA expenses. (Assume this computer is the only asset...

See Answer

Q: The firm in Practice Problem 17 had retained earnings of $15

The firm in Practice Problem 17 had retained earnings of $15,000 at the beginning of the year. Its net income for the year was $7,500, and it paid out $4,000 in dividends. What are its retained earnin...

See Answer

Q: Shirley has been offered two perpetuities: Grow and Shrink. Grow

Shirley has been offered two perpetuities: Grow and Shrink. Grow promises her $100 in one year and an annual cash flow that will increase by 4 percent per year forever. Shrink, in contrast, promises h...

See Answer

Q: An investment promises to pay you $100 per year starting immediately

An investment promises to pay you $100 per year starting immediately. The cash flow from the investment is expected to increase by 3 percent per year forever. If alternative investments of similar ris...

See Answer

Q: Dmitri Chekov has made an investment of $25,000 that

Dmitri Chekov has made an investment of $25,000 that promises to pay him 8 percent simple interest per year for 10 years. Determine how much interest he will earn in the: a. first year b. ninth year

See Answer

Q: After a summer of travelling (and not working), a student

After a summer of travelling (and not working), a student finds himself $ 1,500 short for this year’ s tuition fees. His parents have agreed to lend him the money for three years at a simple interest...

See Answer

Q: Your sister has been forced to borrow money to pay her tuition

Your sister has been forced to borrow money to pay her tuition this year. If she makes annual interest payments on the loan at year end for the next three years, and the loan is for $ 2,500 at a simpl...

See Answer

Q: History tells us that a group of Dutch colonists purchased the island

History tells us that a group of Dutch colonists purchased the island of Manhattan from the Native American residents in 1626. Payment was made with wampum (likely glass beads and trinkets), which had...

See Answer

Q: List the basic areas of financial management.

List the basic areas of financial management.

See Answer

Q: David has been awarded a scholarship that will pay $ 5,

David has been awarded a scholarship that will pay $ 5,000 one year from now. However, he really needs the money today and has decided to take out a loan. If the interest rate is 6 percent, how much c...

See Answer

Q: When bonds sell above their par value, is the yield to

When bonds sell above their par value, is the yield to maturity greater or less than the coupon rate?

See Answer

Q: If market interest rates go up, what happens to bond prices

If market interest rates go up, what happens to bond prices?

See Answer

Q: Explain how to compute future values and present values when using compound

Explain how to compute future values and present values when using compound interest.

See Answer

Q: Veda has to choose between two investments that have the same cost

Veda has to choose between two investments that have the same cost today. Both investments will ultimately pay $ 1,300 but at different times, as shown in the table below. If Veda does not choose one...

See Answer

Q: Explain how timelines can be used to break a complicated time value

Explain how timelines can be used to break a complicated time value of money problem into manageable components.

See Answer

Q: Why does compound interest result in higher future values than simple

Why does compound interest result in higher future values than simple interest?

See Answer

Q: Suppose that GG Co. would like to grow its sales by

Suppose that GG Co. would like to grow its sales by 25 percent, which is greater than its sustainable growth rate (see Practice Problem 56). If all the other financial information remains unchanged, h...

See Answer

Q: Describe the two key decision areas with respect to the financial management

Describe the two key decision areas with respect to the financial management of assets?

See Answer

Q: What are the two key topics covered in the study of corporate

What are the two key topics covered in the study of corporate finance?

See Answer

Q: List the basic questions related to corporate financing.

List the basic questions related to corporate financing.

See Answer

Q: Distinguish among the various types of financial assets.

Distinguish among the various types of financial assets.

See Answer

Q: What is the relationship between FVIFs and PVIFs? Why does this

What is the relationship between FVIFs and PVIFs? Why does this make sense?

See Answer

Q: Explain how to evaluate a growing perpetuity.

Explain how to evaluate a growing perpetuity.

See Answer

Q: Explain how to calculate the present value of a growing annuity.

Explain how to calculate the present value of a growing annuity.

See Answer

Q: Distinguish between primary and secondary markets.

Distinguish between primary and secondary markets.

See Answer

Q: Xiang wishes to have $1 million in 25 years. He

Xiang wishes to have $1 million in 25 years. He cannot afford to make large deposits at the moment; however, he believes that he will be able to increase his deposits by 4 percent per year for the nex...

See Answer

Q: How does the expected rate of inflation affect nominal interest rates?

How does the expected rate of inflation affect nominal interest rates?

See Answer

Q: Why do interest rates on different-maturity Canada bonds differ?

Why do interest rates on different-maturity Canada bonds differ?

See Answer

Q: Why do interest rates differ between Canada and the United States?

Why do interest rates differ between Canada and the United States?

See Answer

Q: Explain how loan and mortgage payments can be determined using annuity concepts

Explain how loan and mortgage payments can be determined using annuity concepts.

See Answer

Q: The following two bonds are identical (FV = $1,

The following two bonds are identical (FV = $1,000, 8‐ percent coupon rate paid semi‐annually), except that they mature at different times.

See Answer

Q: Differentiate between debits and credits with respect to assets and liabilities.

Differentiate between debits and credits with respect to assets and liabilities.

See Answer

Q: What is an “opportunity cost”?

What is an “opportunity cost”?

See Answer

Q: Which types of bonds have more interest rate risk: short-

Which types of bonds have more interest rate risk: short-term or long-term bonds?

See Answer

Q: Why is there no simple analytical formula for the yield to maturity

Why is there no simple analytical formula for the yield to maturity?

See Answer

Q: Suppose that a 6‐percent, annual‐pay, Government

Suppose that a 6‐percent, annual‐pay, Government of Canada bond that matures in two years has a yield to maturity of 6.75 percent. If inflation is expected to be 2.5 percent per year over the next two...

See Answer

Q: If you have a retractable bond, under what conditions will you

If you have a retractable bond, under what conditions will you exercise your right to sell the bond back to the bond issuer?

See Answer

Q: The rowboats Randy purchased (see Practice Problem 25) are a

The rowboats Randy purchased (see Practice Problem 25) are a class 7 asset, so they have a CCA rate of 15 percent. Determine the amount of the capital cost allowance for each of the four years the boa...

See Answer

Q: How do U.S. bank discount yields differ from bond

How do U.S. bank discount yields differ from bond equivalent yields?

See Answer

Q: Why is a 6 percent U.S. mortgage not the

Why is a 6 percent U.S. mortgage not the same as a 6 percent Canadian mortgage?

See Answer

Q: How does the formula for determining the price of a T-

How does the formula for determining the price of a T-bill resemble the formula for determining the price of a zero coupon bond? Why is this so?

See Answer

Q: Summarize the main responsibilities of the finance function (including CFO,

Summarize the main responsibilities of the finance function (including CFO, treasurer, and controller) in a non-financial company.

See Answer

Q: Why can effective rates often be very different from quoted rates?

Why can effective rates often be very different from quoted rates?

See Answer

Q: What happens to the net income figure when a firm’s accountants make

What happens to the net income figure when a firm’s accountants make more aggressive accounting assumptions? Briefly explain.

See Answer

Q: How do cash flow statements alleviate the impact of most major accounting

How do cash flow statements alleviate the impact of most major accounting assumptions?

See Answer

Q: What is finance?

What is finance?

See Answer

Q: Distinguish between real and financial assets.

Distinguish between real and financial assets.

See Answer

Q: Explain how simple interest payments are determined.

Explain how simple interest payments are determined.

See Answer

Q: Why does simple interest take into account the time value of money

Why does simple interest take into account the time value of money?

See Answer

Q: Which sector or sectors of the economy are net providers of financing

Which sector or sectors of the economy are net providers of financing and which are the net users of financing?

See Answer

Q: In what ways are bonds different from mortgages?

In what ways are bonds different from mortgages?

See Answer

Q: What is a bond indenture?

What is a bond indenture?

See Answer

Q: As the CFO of your company, it falls to you to

As the CFO of your company, it falls to you to make the final decision on large expenditures. Recently, your controller has proposed purchasing a new computer system at a cost of $50,000. He believes...

See Answer

Q: Is the yield to call always greater than the yield to maturity

Is the yield to call always greater than the yield to maturity?

See Answer

Q: Why would firms prefer to receive dividend income and make interest payments

Why would firms prefer to receive dividend income and make interest payments rather than make dividend payments and receive interest payments?

See Answer

Q: Explain how to calculate the CCA expense for an asset class in

Explain how to calculate the CCA expense for an asset class in a given year.

See Answer

Q: How is a traditional bond structured?

How is a traditional bond structured?

See Answer

Q: Discuss how the three most important types of financial intermediaries operate.

Discuss how the three most important types of financial intermediaries operate.

See Answer

Q: Explain why global financial markets are so important to Canadians.

Explain why global financial markets are so important to Canadians.

See Answer

Q: Identify and briefly describe the two major stock markets in the United

Identify and briefly describe the two major stock markets in the United States.

See Answer

Q: Explain briefly why events in the United States affected countries around the

Explain briefly why events in the United States affected countries around the world so drastically.

See Answer

Q: Calculate book value per share, dividend yield, dividend payout,

Calculate book value per share, dividend yield, dividend payout, market‐to‐book ratio, earnings per share, and price‐to‐earnings ratio given the following information: shareholders’ equity is $945,000...

See Answer

Q: Stephen has learned that his great‐aunt intends to give him

Stephen has learned that his great‐aunt intends to give him $ 5,000 each year he is studying at university. Tuition must be paid in advance, so Stephen would like to receive his payments at the beginn...

See Answer

Q: How are trusts distinct from corporations?

How are trusts distinct from corporations?

See Answer

Q: You bought a bond last year for $102.50 and

You bought a bond last year for $102.50 and just sold it for $98.50. What has happened to the interest rate over that period?

See Answer

Q: Use the average dividend payout ratio from years 1 and 2,

Use the average dividend payout ratio from years 1 and 2, and the forecast net income figure from Practice Problem 29, to estimate the total amount of dividends that will be paid by the company in yea...

See Answer

Q: Construct a balance sheet and income statement for the business, assuming

Construct a balance sheet and income statement for the business, assuming no income tax.

See Answer

Q: For each of the following YTM figures, calculate the price and

For each of the following YTM figures, calculate the price and current yield for a two‐year, 7‐percent, annual‐pay bond with a face value of $1,000. a . YTM = 6 percent b . YTM = 7 percent c . YTM = 8...

See Answer

Q: For each of the following YTM figures, calculate the price and

For each of the following YTM figures, calculate the price and current yield for a 10‐year, 5‐percent, semiannual‐ pay bond with a face value of $1,000. a . YTM = 4 percent b . YTM = 5 percent c . YTM...

See Answer

Q: The shares of Corine ’ s Candies Inc. are currently trading

The shares of Corine ’ s Candies Inc. are currently trading at $18.20. There are four million shares outstanding. The company ’ s 2016 net income was $5.2 million. Find the market value of equity for...

See Answer

Q: At maturity, each of the following zero coupon bonds (pure

At maturity, each of the following zero coupon bonds (pure discount bonds) will be worth $1,000. For the following each bond, fill in the missing quantity in table. Assume semi‐annua...

See Answer

Q: : The managers of Corine ’ s Candies like to use the

The managers of Corine ’ s Candies like to use the EBITDA multiple to value the firm. EBITDA was approximately $10 million in 2016. Use the market value of equity from Practice Problem 34 and a debt...

See Answer

Q: In year 1, a firm had cash and cash equivalents of

In year 1, a firm had cash and cash equivalents of $150,000, accounts receivable of $35,000, and inventories of $23,000. In year 2, it had cash and cash equivalents of $80,000, accounts receivable of...

See Answer

Q: We can calculate cash flow from operations (CFO) as net

We can calculate cash flow from operations (CFO) as net income + non‐cash expenses + change in working capital ignoring cash. In year 2, the change in working capital for Finns ’ Fridges was -$25. Fin...

See Answer

Q: How have management compensation schemes been designed to better align owner-

How have management compensation schemes been designed to better align owner-manager interests? How well have these schemes performed in this regard?

See Answer

Q: Find the operating margin for Finns ’ Fridges for both years (

Find the operating margin for Finns ’ Fridges for both years (you may assume that the net operating income is equal to the firm ’ s EBIT). Was there an increase or a decrease in the operating margin,...

See Answer

Q: Calculate the fixed asset turnover for Finns ’ Fridges for years 1

Calculate the fixed asset turnover for Finns ’ Fridges for years 1 and 2 (note that net fixed assets correspond to “property and equipment (net)” on the company ’ s statement of financial position). H...

See Answer

Q: Explain yield to maturity in terms of the spot rate.

Explain yield to maturity in terms of the spot rate.

See Answer

Q: Determine leverage ratios, including debt ratio and debt‐equity ratio

Determine leverage ratios, including debt ratio and debt‐equity ratio, in both years. Has Excelsior Inc. improved on its leverage ratios in year 2?

See Answer

Q: You are a financial advisor and one of your clients comes to

You are a financial advisor and one of your clients comes to you with a convertible bond that has a coupon rate of 8 percent. The market interest rate is 6 percent. The share price of the company that...

See Answer

Q: Adam has saved C$1,000 and plans to go

Adam has saved C$1,000 and plans to go surfing in Australia next summer. He won’t need the money for a year, so he decides to invest it. Adam could invest the money in Canada, where a T‐bill will earn...

See Answer

Q: Omar ’s business purchased several pieces of machinery some time ago for

Omar ’s business purchased several pieces of machinery some time ago for $25,000. At the beginning of the current year, this pool of assets had a UCC of $15,000. During the year, Omar decided to sell...

See Answer

Q: Suppose firms A and B have identical revenues and operating expenses,

Suppose firms A and B have identical revenues and operating expenses, so that each has earnings before amortization and taxes of exactly $1 million. Both firms will report amortization of $250,000 on...

See Answer

Q: What is the apparent tax rate (tax paid as a percentage

What is the apparent tax rate (tax paid as a percentage of net income) for firms A and B in Practice Problem 46?

See Answer

Q: Determine liquidity ratios including working capital ratio, current ratio, and

Determine liquidity ratios including working capital ratio, current ratio, and quick ratio for year 2. Explain the differences among the ratios.

See Answer

Q: What is moral hazard and why did it become the buzz word

What is moral hazard and why did it become the buzz word of the 2008 financial crisis?

See Answer

Q: Suppose Prince Rupert Fly ‘ n ’ Fish Inc. (see

Suppose Prince Rupert Fly ‘ n ’ Fish Inc. (see Practice Problem 48) decides to sell its first aircraft for $50,000 in year 2 (purchased for $90,000 in year 1). As before, the second plane costs $100,0...

See Answer

Q: Tina ‘ s Business Inc. bought machines some time ago for

Tina ‘ s Business Inc. bought machines some time ago for $25,000. She decided to sell all the assets from this pool at the end of this year. The pool of assets had a UCC of $5,000 before the sale, and...

See Answer

Q: Determine the price‐earnings ratio, market‐to book ratio

Determine the price‐earnings ratio, market‐to book ratio, and EBITDA ratio for year 2.

See Answer

Q: Prince Rupert Fly ‘n ’ Fish Inc. purchases one small

Prince Rupert Fly ‘n ’ Fish Inc. purchases one small plane in its first year of business for $90,000. In year 2, it purchases another plane for $100,000. Find the UCC at the end of year 3 if the CCA r...

See Answer

Q: At the end of its most recent fiscal period, the large

At the end of its most recent fiscal period, the large appliance rental company mentioned in Practice Problem 19 had a working capital ratio of 4.3 percent and a current ratio of 18.2 percent. Calcula...

See Answer

Q: The Finn brothers are planning their third year of operations. As

The Finn brothers are planning their third year of operations. As a first step in the process, create a “percentage of sales” statement of financial position for Finns ’ Fridges as of the end of year...

See Answer

Q: To achieve the target level of revenues in year 3 ($2

To achieve the target level of revenues in year 3 ($2,600), Finns ’ Fridges will have to buy some more equipment. This will increase the amortization expense to $1,422. Selling costs will be the same...

See Answer

Q: Corine ’ s Candies Inc. paid dividends of $1 million

Corine ’ s Candies Inc. paid dividends of $1 million during2015. However, the company needed extra cash to open new stores, so it issued $1.4 million in new stock. What was Corine ’ s cash flow from f...

See Answer

Q: Other candy‐making firms have an average forward P/E

Other candy‐making firms have an average forward P/E ratio of 12 at this time. With a share price of $18.20, what are the expected 2017 EPS for Corine ’ s Candies if its forward P/E ratio is the same...

See Answer

Q: Based on CP’s balance sheet (Figure 3‐4), the

Based on CP’s balance sheet (Figure 3‐4), the firm ’s total current assets changed between the two years. Which component of the current assets changed the most? By what dollar amount and percentage d...

See Answer

Q: How do callable bonds differ from retractable and extendable bonds?

How do callable bonds differ from retractable and extendable bonds?

See Answer

Q: Use the income statement for CP (Figure 3‐5)

Use the income statement for CP (Figure 3‐5) to calculate the average tax rate (tax paid as a percentage of net income) in 2014.

See Answer

Q: Amanda would like to borrow $ 50,000 to pay one

Amanda would like to borrow $ 50,000 to pay one year ’s tuition at a private U.S. university. She would like to make quarterly payments and finish repaying the loan in five years. If the bank is quoti...

See Answer

Q: Wilma would like to borrow $ 250,000 to start her

Wilma would like to borrow $ 250,000 to start her own business. She would like to make monthly payments to repay the loan in 10 years. If the bank is quoting her a rate of 6 percent compounded quarter...

See Answer

Q: Roger has his eye on a new car that will cost $

Roger has his eye on a new car that will cost $ 20,000. He has $ 15,000 in his savings account, earning interest at a rate of 0.5 percent per month. a. How long (to the nearest month) will it be befor...

See Answer

Q: How many years will it take for an investment to double in

How many years will it take for an investment to double in value if the rate of return is 9 percent and compounding occurs: a. annually? b. quarterly?

See Answer

Q: Céline has just won a lottery. She will receive a payment

Céline has just won a lottery. She will receive a payment of $ 6,000 at the end of each year for nine years. As an alternative, she can choose an immediate payment of $ 50,000. a. Which alternative sh...

See Answer

Q: Use the year 2 financial statements for Finns ’ Fridges to determine

Use the year 2 financial statements for Finns ’ Fridges to determine the company ’ s sustainable growth rate.

See Answer

Q: Alysha has decided to use $ 50,000 in savings to

Alysha has decided to use $ 50,000 in savings to make a down payment on a house. She will live in the house for the next two years while still at university and then sell it when she graduates. The ba...

See Answer

Q: Jimmie wishes to buy a new car that will cost $29

Jimmie wishes to buy a new car that will cost $29,000. a. How much will his monthly car payments be if he obtains a loan that is amortized over 60 months, and the nominal interest rate is 8.5 percent...

See Answer

Q: Michelle is offered a loan of $ 30,000 that requires

Michelle is offered a loan of $ 30,000 that requires 60 monthly payments of $ 622.75. What is the effective annual interest rate on this loan? What would the quoted rate be?

See Answer

Q: How do sinking funds work?

How do sinking funds work?

See Answer

Q: Five years ago, Franklin borrowed $ 250,000 to purchase

Five years ago, Franklin borrowed $ 250,000 to purchase a house in Sandy Lake. At the time, the quoted rate on the mortgage was 6 percent, the amortization period was 25 years, the term was five years...

See Answer

Q: a. Determine the month‐end payment for a $ 200

a. Determine the month‐end payment for a $ 200,000, 10‐year loan with an interest rate of 12 percent, compounded monthly, assuming there is no down payment. b. Calculate the outstanding loan amount af...

See Answer

Q: Investor A just turned 20 years old and currently has no investments

Investor A just turned 20 years old and currently has no investments. She plans to invest $ 5,500 at the end of each year for eight years, beginning in five years. The rate of return on her investment...

See Answer

Q: Tina ’ s Business Inc. purchases one machine in its first

Tina ’ s Business Inc. purchases one machine in its first year of business for $1,000,000. In year 2, it purchases another machine for $500,000. The CCA rate for these assets is 20 percent. a. Find th...

See Answer

Q: Calculate the receivables turnover, inventory turnover, and average collection period

Calculate the receivables turnover, inventory turnover, and average collection period for a firm, given the following accounting data: accounts receivable are $600,000; accounts payable are $305,000;...

See Answer

Q: Estimate cash flow from operations for KER Inc. using the following

Estimate cash flow from operations for KER Inc. using the following information.

See Answer

Q: Suppose firms A and B have identical revenues and operating expenses,

Suppose firms A and B have identical revenues and operating expenses, so that each has earnings before amortization and taxes of $10 million. Both firms will report amortization of $1 million on their...

See Answer

Q: Kash Kow Inc. pays out all its after‐tax earnings

Kash Kow Inc. pays out all its after‐tax earnings to shareholders in the form of dividends. Suppose that in 2015 the company earned $1 per share before tax. Corporate income tax was paid at a rate of...

See Answer

Q: David Finn notices that the local appliance store is now charging $

David Finn notices that the local appliance store is now charging $210 for the same model of refrigerator his company bought for $200. Given that Finns ’ Fridges purchased 25 of these refrigerators, w...

See Answer

Q: The Finn brothers are concerned that the financial statements for Finns ’

The Finn brothers are concerned that the financial statements for Finns ’ Fridges don ’ t reflect the true state of affairs. The company ’ s 25 customers were charged $10 for rent at the end of each m...

See Answer

Q: What are the scope and purpose of the auditor’s opinion?

What are the scope and purpose of the auditor’s opinion?

See Answer

Q: Calculate cash flow from operations for Tina’ s Business Inc. using

Calculate cash flow from operations for Tina’ s Business Inc. using the following information.

See Answer

Q: Consider a bond with five years to maturity, FV of $

Consider a bond with five years to maturity, FV of $1,000, and a coupon rate of 6.5 percent (semi‐annual payments). a . Calculate the price of this bond if the market yield is: i) 7.75 percent; ii) 5....

See Answer

Q: A zero coupon bond has a par value of $1,

A zero coupon bond has a par value of $1,000 and will mature in eight years. a . Calculate the current price of this bond if the market yield is: i) 7.75 percent; ii) 5.25 percent. b . In each case, c...

See Answer

Q: A bond with semi‐annual coupons at a rate of 10

A bond with semi‐annual coupons at a rate of 10 percent will mature in one year. If the bond’s price is $1,010, use the trial‐and‐error method to find the YTM. Check your answer by using a financial c...

See Answer

Q: Sapna would like to receive a real return of 5 percent per

Sapna would like to receive a real return of 5 percent per year on a bond investment at a time when the expected inflation rate is 2.5 percent. How much would she be willing to pay for a bond maturing...

See Answer

Q: The Slice & Dice Investment Co. needs some help understanding the

The Slice & Dice Investment Co. needs some help understanding the intricacies of bond pricing. It has observed the following prices for zero coupon bonds that have no risk of default: a . How much...

See Answer

Q: Bower is a Canadian investor. He noticed that the euro spot

Bower is a Canadian investor. He noticed that the euro spot rate is currently quoted at C$1.4768 pereuro. The European interest rate is 6 percent on one‐year T‐bills, and the one‐year interest rate in...

See Answer

Q: A bond that matures in 10 years is callable in three years

A bond that matures in 10 years is callable in three years at a call price of $1,025. The bond has a semiannual coupon rate of 8 percent. If the YTM is 7.3 percent and the YTC is 6.92 percent, what is...

See Answer

Q: A 12‐year, 7.5‐percent bond is

A 12‐year, 7.5‐percent bond is callable in four years at a call price of $1,045. If the bond pays semi‐annual coupons and is selling for $1,038, what is the YTM and YTC of the bond? Is this bond likel...

See Answer

Q: A firm borrowed $3 million and paid 10 percent interest this

A firm borrowed $3 million and paid 10 percent interest this year. It also paid a dividend of $1 per share on 500,000 shares outstanding. What is the firm ’ s cash flow from financing?

See Answer

Q: Demonstrate how to solve a typical retirement problem.

Demonstrate how to solve a typical retirement problem.

See Answer

Q: Randy ’s Rowboats Ltd. purchased and began to use its first

Randy ’s Rowboats Ltd. purchased and began to use its first six rowboats for a total cost of $2,400. Randy believes the boats can be used for four years, providing the company with equal value each ye...

See Answer

Q: Franklin is trying to decide whether or not to take a philosophy

Franklin is trying to decide whether or not to take a philosophy course next semester. He finds the topic interesting, but being a business student, he wants to measure the cost of taking the course....

See Answer

Q: Muriel would like to support the education of her favourite grand‐

Muriel would like to support the education of her favourite grand‐nephew, Stephen, who plans to begin university in three years. How much will Muriel have to invest today, at 5 percent, to be able to...

See Answer

Q: You have just won $50 million in a lottery and are

You have just won $50 million in a lottery and are offered two options: receive $40 million today or receive $5 million per year for the next 10 years. At what interest rate are you indifferent to the...

See Answer

Q: Felix will need $ 10,000 per year for four years

Felix will need $ 10,000 per year for four years to pay for tuition. How much will Felix ’ s parents have to invest at the end of each year for the eight years before he begins his studies if their sa...

See Answer

Q: A 20‐year semi-annual bond has just been issued

A 20‐year semi-annual bond has just been issued with its coupon rate set at the current market yield of 6 percent. How much would the price of the bond change (in percentage terms) if the market yield...

See Answer

Q: An investment promises to pay you $100 per year starting in

An investment promises to pay you $100 per year starting in five years. The cash flow from the investment is expected to increase by 3 percent per year forever. If alternative investments of similar r...

See Answer

Q: To start a new business, Su Mei intends to borrow $

To start a new business, Su Mei intends to borrow $ 25,000 from a local bank. If the bank asks her to repay the loan in five equal annual instalments of $6,935.24, determine the bank’s effective annua...

See Answer

Q: The Business Development Bank is willing to loan Su Mei the $

The Business Development Bank is willing to loan Su Mei the $ 25,000 she needs to start her new business. The loan will require monthly payments of $ 556.11 over five years. a. What is the effective m...

See Answer

Q: Describe the compensation structure of top Canadian executives.

Describe the compensation structure of top Canadian executives.

See Answer

Q: What is the difference between the covariance and the correlation coefficient?

What is the difference between the covariance and the correlation coefficient?

See Answer

Q: An investor purchased 600 shares of stock A at $23 per

An investor purchased 600 shares of stock A at $23 per share and 1,000 shares of stock B at $34 per share one year ago. Stock A and stock B paid quarterly dividends of $1 per share and $1.50 per share...

See Answer

Q: Apex Financial Ltd. is concerned about the impact of errors in

Apex Financial Ltd. is concerned about the impact of errors in its estimates of the future dividend payout ratio for Barnett Steel Corporation. Assume that the current dividend is $1, ROE is fixed at...

See Answer

Q: What is the “bigger fool theorem” of valuation?

What is the “bigger fool theorem” of valuation?

See Answer

Q: How can we estimate future growth rates?

How can we estimate future growth rates?

See Answer

Q: Why is share value based on the present value of expected future

Why is share value based on the present value of expected future dividends?

See Answer

Q: Why does an increase in the expected dividend growth rate increase share

Why does an increase in the expected dividend growth rate increase share prices?

See Answer

Q: Why can’t the expected growth rate exceed the investor’s required return in

Why can’t the expected growth rate exceed the investor’s required return in the constant growth model?

See Answer

Q: Dillon Mechanical Inc. ’ s first dividend of $2 per

Dillon Mechanical Inc. ’ s first dividend of $2 per share is expected to be paid six years from today. From then on, dividends will grow by 10 percent per year for five years. After five years, the gr...

See Answer

Q: TelTec Inc. stock is expected to sell for $10 per

TelTec Inc. stock is expected to sell for $10 per share four years from now. TelTec has just paid a dividend of 50 cents per share. Dividends are expected to grow at a rate of 5 percent per year for t...

See Answer

Q: JINX Ltd. had earnings per share of $5 as of

JINX Ltd. had earnings per share of $5 as of December 31, 2015, but paid no dividends. Earnings were expected to grow at 15 percent per year for the following five years. JINX Ltd. will start paying d...

See Answer

Q: TelTec Inc. has a patent that will expire in two years

TelTec Inc. has a patent that will expire in two years. The firm is expected to grow at 10 percent for the next two years and dividends will be paid at year end. It just paid a dividend of $1. After t...

See Answer

Q: In Practice Problem 40, the correlation coefficient ρ AB is 0

In Practice Problem 40, the correlation coefficient ρ AB is 0.4 and the standard deviations of stock A and stock B are 28 percent and 15 percent, respectively. Calculate the standard deviation of the...

See Answer

Q: INV Design Ltd. just paid a dividend of $4 and

INV Design Ltd. just paid a dividend of $4 and its current earnings per share is $6. The current T‐bill rate is 3.5 percent and INV ’ s risk premium is 10 percent. The net profit margin, asset turnove...

See Answer

Q: Dillon Mechanical Ltd.’ s preferred shares have a par value

Dillon Mechanical Ltd.’ s preferred shares have a par value of $50, a dividend rate of 7 percent, and trade at a price of $70. Sherwood Inc.’s preferred shares have a par value of $60, have a dividend...

See Answer

Q: Apex Financial Ltd. is interested in investing in Scion Systems Inc

Apex Financial Ltd. is interested in investing in Scion Systems Inc. Scion’s current dividend is $5.50 and its shares are selling for $40. The required rate of return for firms like Scion is 8 percent...

See Answer

Q: As part of your duties at Apex Financial Ltd. you have

As part of your duties at Apex Financial Ltd. you have been asked to review the analysis carried out by a rival company—Prime Group—of the WX Media Company. WX has had a constant P/E ratio for the pas...

See Answer

Q: Selkirk Inc. has an expected profit margin of 10 percent,

Selkirk Inc. has an expected profit margin of 10 percent, turnover ratio of 1.8, and a leverage ratio of 0.3. The leading EPS is $2.50 and the firm uses a dividend payout ratio of 35 percent. The requ...

See Answer

Q: Apex Financial Ltd. has completed a fundamental analysis of Spark Energy

Apex Financial Ltd. has completed a fundamental analysis of Spark Energy Inc. Spark Energy is a young company and expects to invest heavily in facilities and research and development during the next f...

See Answer

Q: Global Systems Inc. has just paid $2.40 in

Global Systems Inc. has just paid $2.40 in dividends D0 $2.40 . The firm is expected to continue paying dividends in perpetuity. a. Suppose that the dividends are constant (Di = $2.40 for all i) . Wh...

See Answer

Q: Barchuk Mining Inc.’s share is currently selling for $120

Barchuk Mining Inc.’s share is currently selling for $120. The current dividend is $5 and the required rate of return is 10 percent. What is the expected dividend growth rate?

See Answer

Q: Larch Foods Inc.’ s current dividend is $5.

Larch Foods Inc.’ s current dividend is $5. Dividends are expected to decline by 4 percent per year for the next three years, and then remain constant thereafter. The required rate of return for this...

See Answer

Q: Describe the characteristics of preferred shares.

Describe the characteristics of preferred shares.

See Answer

Q: FinCorp Inc. is exploring the risk of different portfolio allocations between

FinCorp Inc. is exploring the risk of different portfolio allocations between two stocks. Complete the following table.

See Answer

Q: Describe the constant growth DDM valuation method.

Describe the constant growth DDM valuation method.

See Answer

Q: Describe how to estimate the present value of growth opportunities (PVGO

Describe how to estimate the present value of growth opportunities (PVGO) and what it represents.

See Answer

Q: Fill in the missing information in the following table: /

Fill in the missing information in the following table:

See Answer

Q: The preferred shares of Chinook Electrical Co. have a par value

The preferred shares of Chinook Electrical Co. have a par value of $100 and a dividend rate of 8 percent. The current price is $105. If the risk‐free rate is 2.5 percent, what is the risk premium asso...

See Answer

Q: Fill in the missing information in the following table: /

Fill in the missing information in the following table:

See Answer

Q: ToolWerks Company is expected to earn $12 million next year.

ToolWerks Company is expected to earn $12 million next year. There are 4 million shares outstanding and the company uses a dividend payout ratio of 30 percent. The required rate of return for companie...

See Answer

Q: Oak Furniture Company’s most recent earnings were $300,000.

Oak Furniture Company’s most recent earnings were $300,000. From these earnings, it paid dividends on common equity totalling $175,000. There are 50,000 common shares outstanding. The ROE for Oak Furn...

See Answer

Q: List the elements needed for the calculation of a share price using

List the elements needed for the calculation of a share price using the constant growth DDM.

See Answer

Q: FinCorp Inc. purchased a stock for $48. It expects

FinCorp Inc. purchased a stock for $48. It expects to receive a dividend of $4 in one year and to sell the stock immediately afterwards. a. If the sale price is $65, what is the expected one‐year hold...

See Answer

Q: Determine the present value of growth opportunities for a company with a

Determine the present value of growth opportunities for a company with a leading EPS of $1.85, a required rate of return of 8 percent, and a current stock price of $50.

See Answer

Q: You are interested in using short selling to increase the possible returns

You are interested in using short selling to increase the possible returns from your portfolio. 20 You have short sold $200 of ABC and invested $1,200 in DEF. The following data are available on ABC a...

See Answer

Q: List three reasons why one firm may have a higher leading P

List three reasons why one firm may have a higher leading P/E ratio than a comparable firm.

See Answer

Q: What are some of the key assumptions that must be made when

What are some of the key assumptions that must be made when applying the valuation concepts discussed in this chapter to an actual valuation situation?

See Answer

Q: What other relative valuation multiples are useful in valuation?

What other relative valuation multiples are useful in valuation?

See Answer

Q: How do equity shareholders exert their influence over a company?

How do equity shareholders exert their influence over a company?

See Answer

Q: What are the two main components of the required rate of return

What are the two main components of the required rate of return on equity securities?

See Answer

Q: State the relationship that the required rate of return, the expected

State the relationship that the required rate of return, the expected growth rate, and expected dividends have with the market share price, according to the constant growth DDM.

See Answer

Q: Star Corporation has issued $1 million in preferred shares to investors

Star Corporation has issued $1 million in preferred shares to investors with a 6.75 percent annual dividend rate on a par value of $100. Assuming the firm pays dividends indefinitely and the required...

See Answer

Q: Calculate the leading P/E ratio, given the following information

Calculate the leading P/E ratio, given the following information: retention ratio = 0.4, required rate of return = 10 percent, expected growth rate = 6 percent.

See Answer

Q: Karlyle Inc. has just paid a dividend of $4.

Karlyle Inc. has just paid a dividend of $4. An analyst forecasts annual dividend growth of 9 percent for the next five years; then dividends will decrease by 1 percent per year in perpetuity. The req...

See Answer

Q: 1. Jason bought 46,000 shares of CTB Inc.

1. Jason bought 46,000 shares of CTB Inc. on January 12, 2015. At that time, CTB Inc. had 2 million common shares outstanding. Calculate the portion of CTB Inc. that Jason owns. a. 2.3 percent b. 1.4...

See Answer

Q: The expected return of ABC is 15 percent, and the expected

The expected return of ABC is 15 percent, and the expected return of DEF is 23 percent. Their standard deviations are 10 percent and 23 percent, respectively, and the correlation coefficient between t...

See Answer

Q: 1. Park Recreational Vehicles Ltd. shares are currently selling for

1. Park Recreational Vehicles Ltd. shares are currently selling for $37.50 each. You bought 200 shares one year ago at $34 and received dividend payments of $1.50 per share. What was your total dollar...

See Answer

Q: What drives P/E ratios?

What drives P/E ratios?

See Answer

Q: Why do P/E ratios differ even between comparable firms?

Why do P/E ratios differ even between comparable firms?

See Answer

Q: How are multiples linked to a discounted cash flow valuation?

How are multiples linked to a discounted cash flow valuation?

See Answer

Q: Explain the efficient market hypothesis (EMH).

Explain the efficient market hypothesis (EMH).

See Answer

Q: Describe the various forms of EMH.

Describe the various forms of EMH.

See Answer

Q: The manager of Quest Adventures Ltd. is puzzled. Analysts are

The manager of Quest Adventures Ltd. is puzzled. Analysts are saying that the future prospects for his company are poor because the stock price has dropped 5 percent. a. Explain to the manager the rel...

See Answer

Q: Describe two common tests for the weak form and for the semi

Describe two common tests for the weak form and for the semi-strong form of the EMH.

See Answer

Q: State the main assumptions required for the existence of efficient markets.

State the main assumptions required for the existence of efficient markets.

See Answer

Q: What is the momentum effect? What form of the EMH does

What is the momentum effect? What form of the EMH does it contradict?

See Answer

Q: Using the following information, calculate the expected return and standard deviation

Using the following information, calculate the expected return and standard deviation of a portfolio with 50 percent in ABC and 50 percent in DEF. Then calculate the expected return and standard devia...

See Answer

Q: Elvira, the CEO of AT Pharmaceutical Ltd., has hired Dome

Elvira, the CEO of AT Pharmaceutical Ltd., has hired Dome Financial Inc. to advise her on issuing new stock. Her company will need to issue more stock soon to finance the development of a new product—...

See Answer

Q: You are on the board of directors of Marlin Company. The

You are on the board of directors of Marlin Company. The stock price of Marlin has suddenly increased by 20 percent, and the CEO has come to the board asking for a substantial pay increase. The CEO ar...

See Answer

Q: On Monday, the stock of Wicker Company was trading at $

On Monday, the stock of Wicker Company was trading at $25. The CEO was satisfied with this price as it reflected the prospects of the firm (future dividend growth and required rate of return). The fir...

See Answer

Q: Assume that the information system is so advanced that the market,

Assume that the information system is so advanced that the market, as confirmed by numerous unbiased studies, is efficient. Investment firms therefore decide to retire all portfolio managers and finan...

See Answer

Q: Distinguish between loss aversion and risk aversion.

Distinguish between loss aversion and risk aversion.

See Answer

Q: On the morning of March 15, Decker Marketing Inc. announced

On the morning of March 15, Decker Marketing Inc. announced that it would pay its first dividend of $5 this year. The ex-dividend date will be July 3. a. If the announcement is a complete surprise to...

See Answer

Q: On Monday evening, Codina Model Steam Engine Company announced that it

On Monday evening, Codina Model Steam Engine Company announced that it would be restating its financial statements for the past five years, and its CFO was arrested for fraud. Interpret the following...

See Answer

Q: A news story about United Airline’s decision to seek protection from creditors

A news story about United Airline’s decision to seek protection from creditors was originally published December 10, 2002 by the Chicago Tribune. The story was recirculated by a Florida news service i...

See Answer

Q: Would you expect investors to be more overconfident during a boom or

Would you expect investors to be more overconfident during a boom or a recession? Why?

See Answer

Q: How would you expect increasing access to the Internet to affect investor

How would you expect increasing access to the Internet to affect investor behaviour?

See Answer

Q: You are interested in two stocks: Alcon and Beldon. Both

You are interested in two stocks: Alcon and Beldon. Both stocks have an expected return of 8 percent. The standard deviation of Alcon is 3 percent, and the standard deviation of Beldon is 5 percent. Y...

See Answer

Q: Is the weak form EMH well supported by empirical evidence? Discuss

Is the weak form EMH well supported by empirical evidence? Discuss any exceptions.

See Answer

Q: Is the semi-strong form EMH well supported by empirical evidence

Is the semi-strong form EMH well supported by empirical evidence? Discuss any exceptions.

See Answer

Q: Is the strong form EMH well supported by empirical evidence? Discuss

Is the strong form EMH well supported by empirical evidence? Discuss any exceptions.

See Answer

Q: Contrast behavioural finance with the traditional view.

Contrast behavioural finance with the traditional view.

See Answer

Q: Explain why behavioural flaws could result in investors holding portfolios that are

Explain why behavioural flaws could result in investors holding portfolios that are not as predicted by modern portfolio theory.

See Answer

Q: Explain why behavioural traits can cause asset price bubbles.

Explain why behavioural traits can cause asset price bubbles.

See Answer

Q: What are the main implications of the EMH for investors? For

What are the main implications of the EMH for investors? For corporate officers?

See Answer

Q: Identify the types of inefficiency (allocational, operational, or informational

Identify the types of inefficiency (allocational, operational, or informational) described below: a. The Lower Red River stock market is characterized by very high transaction costs, and due to freque...

See Answer

Q: Explain whether each of the following is an example of informational efficiency

Explain whether each of the following is an example of informational efficiency. a. Every time my broker tells me to buy, the stock price subsequently goes down. Every time my broker tells me to sell,...

See Answer

Q: Parker Investments Inc. has just completed an investigation of strong form

Parker Investments Inc. has just completed an investigation of strong form efficiency in the Canadian stock market and has concluded that its evidence is statistically significant but not economically...

See Answer

Q: You are interested in two stocks: Alcon and Beldon. Both

You are interested in two stocks: Alcon and Beldon. Both stocks have a standard deviation of 8 percent. The expected return of Alcon is 10 percent, and the expected return of Beldon is 20 percent. You...

See Answer

Q: Which type of analyst, buy side or sell side, is

Which type of analyst, buy side or sell side, is more likely to “sell” their recommendation to the public?

See Answer

Q: Summarize the empirical conclusions regarding the three forms of the EMH.

Summarize the empirical conclusions regarding the three forms of the EMH.

See Answer

Q: State four important implications of the EMH for investors and two implications

State four important implications of the EMH for investors and two implications for corporate officers.

See Answer

Q: 1. Which of the following statements about an efficient market is

1. Which of the following statements about an efficient market is false ? a. Prices fully and accurately reflect all available information. b. Prices reflect information about a firm’s future plans. c...

See Answer

Q: 1. Which of the following statements about strong form EMH is

1. Which of the following statements about strong form EMH is false? a. It encompasses both the weak and semi-strong EMH. b. It is the most flexible form of market efficiency. c. It states that prices...

See Answer

Q: Explain the difference between an insurance contract and a credit default swap

Explain the difference between an insurance contract and a credit default swap.

See Answer

Q: How and why did AIG fail?

How and why did AIG fail?

See Answer

Q: Why would making CDSs an exchange-listed product have avoided the

Why would making CDSs an exchange-listed product have avoided the collapse of AIG and averted the 2008–9 financial crisis?

See Answer

Q: Complete the following table. The underlying asset is ounces of gold

Complete the following table. The underlying asset is ounces of gold. Assume no arbitrage.

See Answer

Q: The Health Bracelet Company will need 1 ,000 kilograms of copper

The Health Bracelet Company will need 1 ,000 kilograms of copper in one year and is trying to decide between buying the copper on the spot market or using a forward contract. The spot price of copper...

See Answer

Q: FinCorp Inc. wishes to examine the effect of correlation on the

FinCorp Inc. wishes to examine the effect of correlation on the efficient frontier that can be created by investing in ABC and FGI. The expected return of ABC is 6 percent, with a standard deviation o...

See Answer

Q: Suppose the spot exchange rate is C$1.4665 per

Suppose the spot exchange rate is C$1.4665 per euro, while the six‐month forward rate is C$1.50 per euro. What will be the profit for an investor who assumes a €100,000 long position in the forward co...

See Answer

Q: Assume an investor takes a €100,000 short position in

Assume an investor takes a €100,000 short position in the six‐month euro forward contract with forward rate of C$1.50 per euro. Determine the investor ’ s profit (loss) if the spot rate in six months...

See Answer

Q: The spot exchange rate is C$1.4665 per euro

The spot exchange rate is C$1.4665 per euro, while the six‐month forward rate is C$1.50 per euro. Suppose a firm has to pay a foreign supplier €100,000 in six months and decides to eliminate its forei...

See Answer

Q: The spot exchange rate is C$1.4665 per euro

The spot exchange rate is C$1.4665 per euro, while the six‐month forward rate is C$1.50 per euro. Suppose a firm expects to receive €100,000 in six months from a foreign customer and decides to elimin...

See Answer

Q: CanComp has a contract to deliver a large computer system to a

CanComp has a contract to deliver a large computer system to a South African company in one year and would like to hedge the currency risk. CanComp will receive payment of R3.5 million (the currency o...

See Answer

Q: Explain the difference between forwards and futures.

Explain the difference between forwards and futures.

See Answer

Q: Explain basis risk and the advantage of forward contracts over future contracts

Explain basis risk and the advantage of forward contracts over future contracts in minimizing basis risk.

See Answer

Q: Ethel and Egbert have decided to invest in the futures market.

Ethel and Egbert have decided to invest in the futures market. Both entered into 1,000 futures contracts, which required a $30,000 initial margin. The maintenance margin for each investor is $22,500....

See Answer

Q: An investor enters into a long position in 50,000 futures

An investor enters into a long position in 50,000 futures contracts that require a $50,000 initial margin and have a maintenance margin that is 75 percent of this amount. The futures price associated...

See Answer

Q: An investor enters into a short position in 50,000 futures

An investor enters into a short position in 50,000 futures contracts that require a $50,000 initial margin and have a maintenance margin that is 75 percent of this amount. The futures price associated...

See Answer

Q: You have observed the following monthly returns for ABC and DEF.

You have observed the following monthly returns for ABC and DEF. a. Graph the relationship between the weight in ABC and the portfolio returns (restrict all weights to be greater than or equal to zero...

See Answer

Q: Ethel decided to invest in the futures market. She entered a

Ethel decided to invest in the futures market. She entered a long position in 1,000 futures contracts that require a $30,000 initial margin. The maintenance margin is $22,500. Assume that Ethel deposi...

See Answer

Q: Aqua Boat Company recently issued floating rate debt. The rate is

Aqua Boat Company recently issued floating rate debt. The rate is L IBOR 3percent, reset semi‐annually. Compost Earth Company has recently issued fixed rate debt. The rate is 5 perce...

See Answer

Q: David says, “CDS is essentially the same as buying default

David says, “CDS is essentially the same as buying default insurance on the risky corporate bond, where the buyer pays for the default protection and the seller sells the protection.” Thus he conclude...

See Answer

Q: You are in the process of developing forecasts of short‐term

You are in the process of developing forecasts of short‐term interest rates. In order to determine a bond trading strategy, you want to determine the market’ s shor...

See Answer

Q: Calculate the F1, F 2, F 3 given the following

Calculate the F1, F 2, F 3 given the following interest rates on zero coupon bonds:

See Answer

Q: On a given day, a new futures contract starts to trade

On a given day, a new futures contract starts to trade. There are five participants in the market labelled as investors V, W, X, Y, and Z. Four sets of trades for the day were as follows: Investor V b...

See Answer

Q: certain group of futures traders may be interested only in the financial

certain group of futures traders may be interested only in the financial exposure of the underlying asset. If this is the case, do you think the open interest will increase or decrease as the futures...

See Answer

Q: Why do forward contracts involve credit risk for banks?

Why do forward contracts involve credit risk for banks?

See Answer

Q: When would a speculator assume a long position in a forward contract

When would a speculator assume a long position in a forward contract on an underlying asset? When would a speculator assume a short position?

See Answer

Q: When would a hedger assume a long position in a forward contract

When would a hedger assume a long position in a forward contract on an underlying asset? When would a hedger assume a short position?

See Answer

Q: Explain when to use the arithmetic mean and when to use the

Explain when to use the arithmetic mean and when to use the geometric mean to describe a return series.

See Answer

Q: FinCorp Inc. wants to examine a “real” efficient frontier

FinCorp Inc. wants to examine a “real” efficient frontier involving BlackBerry (BB.TO) and the Royal Bank (RY.TO). a. Using monthly data for these two companies from January 2011 to December 2011, gra...

See Answer

Q: Describe the process of marking to market for futures contracts.

Describe the process of marking to market for futures contracts.

See Answer

Q: Describe open interest with respect to futures contracts.

Describe open interest with respect to futures contracts.

See Answer

Q: Explain why two counterparties would enter into an interest rate swap even

Explain why two counterparties would enter into an interest rate swap even when one has an absolute financing advantage in both the fixed and the floating rate markets.

See Answer

Q: Describe how total return swaps work.

Describe how total return swaps work.

See Answer

Q: If the trading volume of a futures contract has been increasing over

If the trading volume of a futures contract has been increasing over the past 5 days, does this mean that the open interest has also been increasing?

See Answer

Q: CanComp, a Canadian computer manufacturer, will be delivering a large

CanComp, a Canadian computer manufacturer, will be delivering a large computer system to a German firm in six months. CanComp expects to receive payment of US$1.5 million at that time. Currently the s...

See Answer

Q: Bert, the business reporter for the Sidney Driftwood, a small

Bert, the business reporter for the Sidney Driftwood, a small newspaper, has contacted you for information about the oil market. Provide responses to his questions below with arbitrage opportunities....

See Answer

Q: Joyce and Anthony are in the process of renewing their mortgages.

Joyce and Anthony are in the process of renewing their mortgages. Each mortgage is an interest‐only mortgage (i.e., the borrower pays only interest and has a balloon payment at the end) for $100,000....

See Answer

Q: Suppose ABC Inc. can borrow at a fixed rate of 9

Suppose ABC Inc. can borrow at a fixed rate of 9.5 percent or a floating rate of L IBOR 1percent. DEF Inc. can borrow at a fixed rate of 12 percent or a floating rate of L IBOR 1.5percent. Supposing t...

See Answer

Q: CanGold Mining Company borrowed €100 million in France at an annual

CanGold Mining Company borrowed €100 million in France at an annual interest rate of 3.5 percent. The principal plus interest is due in one year. CanGold used the funds to purchase machine parts in Ge...

See Answer

Q: Why is the expected return on a portfolio a weighted average of

Why is the expected return on a portfolio a weighted average of the expected returns of the underlying securities?

See Answer

Q: Angela, a new investor, has contacted you with a question

Angela, a new investor, has contacted you with a question about the swap market. Provide a response to her question: “A swap agreement allows two companies to swap payments. Presumably, both parties b...

See Answer

Q: At the end of the current year, you observe the following

At the end of the current year, you observe the following data about Government of Canada pure discount bonds (zero coupon bonds): Bond issue: A Years to maturity: 1 YTM%: 5 B...

See Answer

Q: Over a given day, there were 10,000 trades made

Over a given day, there were 10,000 trades made for corn futures (i.e., 10,000 buyers and 10,000 sellers). A total of 2,000 of these trades were by buyers entering into new positions and 9,000 of thes...

See Answer

Q: Discuss three different strategies you can follow to invest in Canadian securities

Discuss three different strategies you can follow to invest in Canadian securities to obtain a return over a three-year period.

See Answer

Q: If the yield curve is upward (downward/inverted) where

If the yield curve is upward (downward/inverted) where is the market expecting short-term interest rates to go?

See Answer

Q: What is the relationship among spot rates, forward rates, and

What is the relationship among spot rates, forward rates, and the cost of carry?

See Answer

Q: Define initial margin, maintenance margin, margin call, open interest

Define initial margin, maintenance margin, margin call, open interest, and notional amount.

See Answer

Q: Explain what is meant by “marked to market”.

Explain what is meant by “marked to market”.

See Answer

Q: What is basis risk? Why is it important for hedgers?

What is basis risk? Why is it important for hedgers?

See Answer

Q: Compare and contrast forwards and futures.

Compare and contrast forwards and futures.

See Answer

Q: Why is portfolio standard deviation not a weighted average of the standard

Why is portfolio standard deviation not a weighted average of the standard deviations of the underlying securities?

See Answer

Q: 1. Which of the following statements is false? a

1. Which of the following statements is false? a. A spot price is a price today for immediate delivery. b. If a Canadian firm has to pay U.S. dollars in the future, it worries about the potential depr...

See Answer

Q: 1. Which of the following statements concerning Government of Canada bond

1. Which of the following statements concerning Government of Canada bond futures is false? a. The contract price is quoted per $100. b. A maximum position limit is set to prevent a single dominant ho...

See Answer

Q: Explain how plain vanilla interest rate swaps are structured and what purpose

Explain how plain vanilla interest rate swaps are structured and what purpose they serve.

See Answer

Q: Explain how currency swaps are structured and how they can be used

Explain how currency swaps are structured and how they can be used for hedging purposes.

See Answer

Q: Why does it make sense that interest rate swaps involve an exchange

Why does it make sense that interest rate swaps involve an exchange of net payments, while currency swaps exchange all cash flows?

See Answer

Q: Simon manages a large bond portfolio and wishes to hedge against interest

Simon manages a large bond portfolio and wishes to hedge against interest rate risk. His portfolio includes Government of Canada bonds and high‐grade Canadian corporate bonds. The correlation between...

See Answer

Q: You have received the following incomplete information about a set of currency

You have received the following incomplete information about a set of currency forwards. All the forwards are for C$1,000 in one year. Complete the following table.

See Answer

Q: How do you form the minimum variance frontier in the two-

How do you form the minimum variance frontier in the two-security case?

See Answer

Q: Why is the efficient frontier bowed?

Why is the efficient frontier bowed?

See Answer

Q: What is an unattainable portfolio, and what is a dominated portfolio

What is an unattainable portfolio, and what is a dominated portfolio?

See Answer

Q: Why is all risk removed in a two-security portfolio if

Why is all risk removed in a two-security portfolio if the securities are perfectly negatively correlated?

See Answer

Q: What assumptions about investors underlie Markowitz’s theories regarding efficient portfolios?

What assumptions about investors underlie Markowitz’s theories regarding efficient portfolios?

See Answer

Q: Why might a scenario-based estimate be more accurate for a

Why might a scenario-based estimate be more accurate for a short-run expected return estimate than a historical AM estimate?

See Answer

Q: You have observed the following returns: 18 percent, −15

You have observed the following returns: 18 percent, −15 percent, 8 percent, 6 percent, and −12 percent. a. Calculate the geometric mean return. b. Calculate the arithmetic mean return. c. Calculate t...

See Answer

Q: You observed the following daily returns for two companies, ABC and

You observed the following daily returns for two companies, ABC and DEF. a. Calculate the following for each stock: i. Five‐day cumulative return ii. Geometric mean daily return iii...

See Answer

Q: FinCorp Inc. is exploring different portfolio allocations between two stocks.

FinCorp Inc. is exploring different portfolio allocations between two stocks. Complete the following table.

See Answer

Q: Your portfolio consists of two securities: Transcomm and MidCap. The

Your portfolio consists of two securities: Transcomm and MidCap. The expected return for Transcomm is 15 percent, while for MidCap it is 5 percent. The standard deviation is 6 percent for Transcomm an...

See Answer

Q: FinCorp Inc. is exploring the risk of different portfolio allocations between

FinCorp Inc. is exploring the risk of different portfolio allocations between two stocks. Complete the following table.

See Answer

Q: You have the following return data on six stocks: /

You have the following return data on six stocks: a. Graph the returns of each stock (ABC, DEF, GHI, JKL, and MNO) against the returns of XYZ. b. Based on the five graphs, which stocks are positively...

See Answer

Q: On January 1, FinCorp Inc. published the following forecasts for

On January 1, FinCorp Inc. published the following forecasts for the economy: During the year you observed quarterly returns of 2 percent, −5 percent, 3 percent, and 8 percent. a. Ca...

See Answer

Q: On January 1, FinCorp Inc. completed its analysis of the

On January 1, FinCorp Inc. completed its analysis of the prospects for the Geriatric Toy Store and concluded that there was a 25‐percent chance the stock price would be $150 in one year and a 75‐perce...

See Answer

Q: Is the zero-risk portfolio described in Question 4 generally equally

Is the zero-risk portfolio described in Question 4 generally equally weighted in both securities? Explain.

See Answer

Q: You wish to combine two stocks, Peledon and Mexcor, into

You wish to combine two stocks, Peledon and Mexcor, into a portfolio with a standard deviation of 6 percent. The expected return of Peledon is 2 percent with a standard deviation of 1 percent. The exp...

See Answer

Q: The expected return of ABC is 16 percent, and the expected

The expected return of ABC is 16 percent, and the expected return of DEF is 27 percent. Their standard deviations are 10 percent and 23 percent, respectively. If a portfolio is composed of 40 percent...

See Answer

Q: At the beginning of the year you bought 300 shares of Lycel

At the beginning of the year you bought 300 shares of Lycel Ltd. at $84 each. During the year you received dividends of $780. At the end of the year the stock is trading for $87 and you decide to sell...

See Answer

Q: At the beginning of last year you invested $24,000

At the beginning of last year you invested $24,000 in 1,500 shares of Goran Products Inc. During the year you received $3,750 as a dividend. At the end of the year you sold the shares for $15 each. Ca...

See Answer

Q: Calculate the annual arithmetic mean and geometric mean return on the following

Calculate the annual arithmetic mean and geometric mean return on the following security, and state which method is more appropriate for the situation: purchase price = $30; first‐year dividend = $5;...

See Answer

Q: Calculate the ex post standard deviation of returns for the following:

Calculate the ex post standard deviation of returns for the following: 50 percent, 30 percent, 20 percent, 35 percent, 55 percent.

See Answer

Q: An investor owns a portfolio of $60,000 that

An investor owns a portfolio of $60,000 that contains $15,000 in stock A, with an expected return of 12 percent; $20,000 in bonds, with an expected return of 8 percent; and the rest in stock B, with...

See Answer

Q: As an analyst for FinCorp Inc., you are responsible for many

As an analyst for FinCorp Inc., you are responsible for many firms, including ADFC. Currently you have a “hold” recommendation on ADFC. 19 The current price of ADFC is $140. You have conducted an exte...

See Answer

Q: FinCorp Inc. has been using the services of San Bernadino Brokerage

FinCorp Inc. has been using the services of San Bernadino Brokerage Company (SBBC) for the past six months. SBBC has informed FinCorp Inc. that the geometric mean monthly return was 6 percent and that...

See Answer

Q: You wish to combine two stocks, Encor and Maestro, into

You wish to combine two stocks, Encor and Maestro, into a portfolio with an expected return of 16 percent. The expected return of Encor is 2 percent with a standard deviation of 1 percent. The expecte...

See Answer

Q: Why is it logical to believe that international diversification will provide benefits

Why is it logical to believe that international diversification will provide benefits to investors?

See Answer

Q: Calculate the covariance and correlation coefficient between the two securities of a

Calculate the covariance and correlation coefficient between the two securities of a portfolio that has 60 percent in stock X (with an expected return of 40 percent and a standard deviation of 12 perc...

See Answer

Q: Using the following information, calculate the expected return and the standard

Using the following information, calculate the expected return and the standard deviation of ABC.

See Answer

Q: Five years ago, your dad bought 250 shares of ABC for

Five years ago, your dad bought 250 shares of ABC for $6 each and 300 shares of DEF for $7.50 each. He has now given you all his shares, when both stocks are trading at $8. What are the weights of the...

See Answer

Q: What is naïve diversification?

What is naïve diversification?

See Answer

Q: What is the difference between diversifiable and non-diversifiable risk?

What is the difference between diversifiable and non-diversifiable risk?

See Answer

Q: Why is the range sometimes a poor measure of risk?

Why is the range sometimes a poor measure of risk?

See Answer

Q: What is the difference between estimating a scenario-based (probability

What is the difference between estimating a scenario-based (probability) estimate of risk versus a historic data-based estimate of risk?

See Answer

Q: Why would we sometimes want to use scenario based risk measures rather

Why would we sometimes want to use scenario based risk measures rather than the standard deviation of actual returns over a long time period?

See Answer

Q: 1. Calculate the capital gain return for a stock that was

1. Calculate the capital gain return for a stock that was purchased at $32 one year ago and is now worth $34. It paid four quarterly dividends of $1.50 per share each throughout the year. a. 9.75 perc...

See Answer

Q: State three of the most important assumptions underlying Markowitz’s notion of efficient

State three of the most important assumptions underlying Markowitz’s notion of efficient portfolios.

See Answer

Q: Why do the income and capital gains component of the total return

Why do the income and capital gains component of the total return differ between common shares and bonds?

See Answer

Q: What is the difference between ex ante and ex post returns?

What is the difference between ex ante and ex post returns?

See Answer

Q: Why is the GM return a better estimate of long run investment

Why is the GM return a better estimate of long run investment performance than the AM return?

See Answer

Q: Why is beta a measure of market risk for a security?

Why is beta a measure of market risk for a security?

See Answer

Q: Assuming the CAPM holds, if the expected return on a diversified

Assuming the CAPM holds, if the expected return on a diversified portfolio lies above the CML, should an investor buy or sell the portfolio?

See Answer

Q: What is a characteristic line, and why is it useful?

What is a characteristic line, and why is it useful?

See Answer

Q: What is the slope of the CML, and why can it

What is the slope of the CML, and why can it be reviewed as the market price of risk for efficient portfolios according to the CML?

See Answer

Q: If a security’s correlation with the market return increases, will its

If a security’s correlation with the market return increases, will its beta get larger or smaller?

See Answer

Q: If the market risk premium increases will securities become over or under

If the market risk premium increases will securities become over or under valued?

See Answer

Q: For the following decisions, indicate if they are consistent with risk

For the following decisions, indicate if they are consistent with risk aversion or risk loving. a. Buying a lottery ticket b. Buying fire insurance on your house c. Jaywalking on St. Catherine Street...

See Answer

Q: FinCorp Inc. conducted an extensive analysis of the economy and concluded

FinCorp Inc. conducted an extensive analysis of the economy and concluded that the probability of a recession next year is 30 percent, the probability of a boom is 45 percent, and the probability of a...

See Answer

Q: Today, you observe the market portfolio has an expected return of

Today, you observe the market portfolio has an expected return of 13 percent, with a standard deviation of 7 percent. The risk‐free rate is 2 percent. If only the risk‐free rate increases (i.e., there...

See Answer

Q: State three of the assumptions underlying the capital asset pricing model (

State three of the assumptions underlying the capital asset pricing model (CAPM).

See Answer

Q: How do you determine if a portfolio or security is undervalued,

How do you determine if a portfolio or security is undervalued, correctly valued, or overvalued?

See Answer

Q: What is the beta of the following? a. Risk

What is the beta of the following? a. Risk‐free asset b. Market portfolio

See Answer

Q: If a security’ s total risk (variance) increases, does

If a security’ s total risk (variance) increases, does that mean the beta must have increased? Explain.

See Answer

Q: Which security, A, B, or C, will provide

Which security, A, B, or C, will provide the greatest return per unit of risk when combined with the risk free asset with a 5 percent rate of return? ERA = 20 percent, σA = 5 percent ERB = 25 percent,...

See Answer

Q: The current price of a stock is $20. It is

The current price of a stock is $20. It is expected to rise to $22 in one year and pay an annual dividend of $1 during the year. The RF is 4 percent, the ER M is 10 percent, and the stock’s beta is 1....

See Answer

Q: The idea behind CAPM is that investors should not be compensated for

The idea behind CAPM is that investors should not be compensated for diversifiable risk. Why not?

See Answer

Q: The risk‐free rate is 5 percent, the risk premium

The risk‐free rate is 5 percent, the risk premium is 6 percent, and stock A has an expected return of 15.5 percent. What is the beta of stock A?

See Answer

Q: State Roll’ s critique concerning the CAPM.

State Roll’ s critique concerning the CAPM.

See Answer

Q: On Monday you invested $182 in Dettall Ltd. Dettall has

On Monday you invested $182 in Dettall Ltd. Dettall has earned daily returns of −8 percent, 18 percent, −30 percent, 6 percent, 7 percent, and −5 percent. What is the value of your investment at the e...

See Answer

Q: You are following five different stocks and need to issue a recommendation

You are following five different stocks and need to issue a recommendation (buy, hold, or sell) to your customers. The market return is 10 percent, with a standard deviation of 15 percent. The risk&ac...

See Answer

Q: Suppose you have a portfolio that has $100 in stock A

Suppose you have a portfolio that has $100 in stock A with a beta of 0.9, $400 in stock B with a beta of 1.2, and $300 in the risk‐free asset. You have another $200 to invest. You wish to achieve a be...

See Answer

Q: Portfolio A has a beta of 0.82. Portfolio B

Portfolio A has a beta of 0.82. Portfolio B has a beta of 1.05. RF is 3 percent and the market risk premium is 6 percent. Calculate the required rate of return of A and B. If the expected rate of retu...

See Answer

Q: TrenStar Inc. is planning to offer several investments to investors and

TrenStar Inc. is planning to offer several investments to investors and is in the process of designing its marketing materials. Each investment’ s value in the future will be related...

See Answer

Q: TrenStar Inc. has five clients with different risk and return preferences

TrenStar Inc. has five clients with different risk and return preferences. The market portfolio has an expected return of 12 percent, with a standard deviation of 16 percent. The riskâ€...

See Answer

Q: A lawyer prosecuting a lawsuit against The Brokerage Company has hired you

A lawyer prosecuting a lawsuit against The Brokerage Company has hired you to conduct an investigation into the advice the company has been giving its clients. You observe that clients have invested i...

See Answer

Q: Three of your friends (Jean, Evan, and Lee)

Three of your friends (Jean, Evan, and Lee) are having an argument about investments and, because you have taken this course, have come to you for advice. The possible investments are set out in the f...

See Answer

Q: Obtain monthly returns for BlackBerry, the Royal Bank of Canada,

Obtain monthly returns for BlackBerry, the Royal Bank of Canada, and the S&P/TSX Composite Index for January to December 2015. (Note: Monthly historical prices, adjusted for dividends, are available f...

See Answer

Q: Your client is confused. He owns shares in the Whistler Snow

Your client is confused. He owns shares in the Whistler Snow‐Making Company (WSMC) and wants you to explain your recommendation. Both of you agree on the following: WSMC has an expected return of 12 p...

See Answer

Q: You are forecasting the returns for PVC Company, a plumbing supply

You are forecasting the returns for PVC Company, a plumbing supply company, which pays a current dividend of $10. The dividend is expected to grow at a rate of 3 percent. You have identified two publi...

See Answer

Q: To achieve a zero standard deviation for a portfolio, calculate the

To achieve a zero standard deviation for a portfolio, calculate the weights of stock A and stock B, assuming the correlation coefficient is −1.

See Answer

Q: Describe some of the criticisms of the CAPM, including Roll’s critique

Describe some of the criticisms of the CAPM, including Roll’s critique.

See Answer

Q: Briefly describe the strengths and weaknesses of the Fama-French model

Briefly describe the strengths and weaknesses of the Fama-French model and the APT.

See Answer

Q: Why is the CAPM called a single-factor model?

Why is the CAPM called a single-factor model?

See Answer

Q: Jackie borrowed $1,000 at the risk‐free rate

Jackie borrowed $1,000 at the risk‐free rate of 6 percent. She invested the borrowed money and her own money of $2,500 in a portfolio with a 15‐percent rate of return and a 25‐percent standard deviati...

See Answer

Q: Calculate the missing values for the following five efficient portfolios. The

Calculate the missing values for the following five efficient portfolios. The expected return on the market is 8 percent, with a standard deviation of 5 percent, and the risk‐free ra...

See Answer

Q: TrenStar Inc. obtained the following incomplete information from ABC Company and

TrenStar Inc. obtained the following incomplete information from ABC Company and has given you the task of completing the table.

See Answer

Q: Stock FM has a standard deviation of 28 percent and a correlation

Stock FM has a standard deviation of 28 percent and a correlation coefficient of 0.7 with market returns. The standard deviation of market return is 16 percent, and the expected return is 13.5 percent...

See Answer

Q: Estimate the beta of the following stock: market risk premium =

Estimate the beta of the following stock: market risk premium = 25 percent, RF = 6 percent, P0 = $10, expected dividend at the end of the year = $2.50, P = $12.50 . Assume the market is in equilibrium...

See Answer

Q: You are valuing the Vancouver Rain‐Making Company (VRM)

You are valuing the Vancouver Rain‐Making Company (VRM) and need to calculate the following: a. Required rate of return (assume the market risk premium is 8 percent, the risk‐free rate is 3 percent, a...

See Answer

Q: Which of the following are examples of systematic (market) risks

Which of the following are examples of systematic (market) risks? Which are examples of unsystematic (unique) risks? a. Inflation risk b. CFO’ s fraudulent activities c. Changes in interest rates d. P...

See Answer

Q: Data on the daily performance of Carraway Corporation have been partially completed

Data on the daily performance of Carraway Corporation have been partially completed in the following table. Fill in the missing data. The closing price on one day is assumed to be the opening price fo...

See Answer

Q: Determine the beta of QTax based on the following information:

Determine the beta of QTax based on the following information: • Market expected return is 9 percent; standard deviation is 12 percent • R isk‐free rate is 3 percent • Current dividend is $4 • Dividen...

See Answer

Q: Assuming CAPM is valid, can we have a situation where stock

Assuming CAPM is valid, can we have a situation where stock A has a required rate of return of 15 percent and a beta of 1.4, and stock B has a required rate of return of 20 percent and beta of 1.2?

See Answer

Q: Stock A has a beta of 1.8 and an expected

Stock A has a beta of 1.8 and an expected return of 20 percent. Stock B has a beta of 1.2 and an expected return of 14 percent. If CAPM holds, what should the return on the market and the risk‐free ra...

See Answer

Q: The expected return on stock A is 15 percent. The expected

The expected return on stock A is 15 percent. The expected return on stock B is 9 percent. Assuming CAPM holds, if the beta of stock A is higher than the beta of stock B by 0.4, what should the risk p...

See Answer

Q: The variance of the market returns is 0.0576, and

The variance of the market returns is 0.0576, and the covariance of the returns on ABC stock and the market is 0.09504. If the risk‐free rate is 5 percent and the market risk premium is 8 percent, wha...

See Answer

Q: You invested $100,000 in the following stocks:

You invested $100,000 in the following stocks: If the risk‐free rate is 5 percent and the market risk premium is 8 percent, what is the expected return on your portfolio?

See Answer

Q: Four risk factors, F 1 , F 2 , F 3

Four risk factors, F 1 , F 2 , F 3 , and F 4 , have been identified to determine the required rate of return, as follows: ERi a0 bi1 F1 bi2 F2 bi3 F3 bi4 F4, where a0 , is the expected return on a sec...

See Answer

Q: 1. What is the expected return and standard deviation of a

1. What is the expected return and standard deviation of a portfolio consisting of $7,500 invested in a risk free asset with an 8‐percent rate of return, and $2,500 invested in a risky security with a...

See Answer

Q: When is the expected return equal to the required return?

When is the expected return equal to the required return?

See Answer

Q: Why is the Sharpe ratio frequently referred to as a ‘risk

Why is the Sharpe ratio frequently referred to as a ‘risk-adjusted’ measure of performance?

See Answer

Q: FinCorp Inc. is interested in the tradeoff between investing in two

FinCorp Inc. is interested in the tradeoff between investing in two stocks, ABC and DEF. The expected return on ABC is 6 percent and on DEF is 18 percent. a. Graph the relationship between the expecte...

See Answer

Q: What is risk aversion and how do we know investors are risk

What is risk aversion and how do we know investors are risk averse?

See Answer

Q: What is the risk of a portfolio consisting of a risk-

What is the risk of a portfolio consisting of a risk-free asset and a risky security?

See Answer

Q: Why is the tangent portfolio so important?

Why is the tangent portfolio so important?

See Answer

Q: How do we generate a portfolio with a higher expected rate of

How do we generate a portfolio with a higher expected rate of return than that on the tangent portfolio?

See Answer

Q: Why can the P/E ratio be viewed as a type

Why can the P/E ratio be viewed as a type of payback period?

See Answer

Q: In what ways are preferred shares different from bonds?

In what ways are preferred shares different from bonds?

See Answer

Q: How is a traditional preferred share valued?

How is a traditional preferred share valued?

See Answer

Q: How can we estimate the investor’s required rate of return for a

How can we estimate the investor’s required rate of return for a traditional preferred share?

See Answer

Q: FinCorp Inc. purchased a stock for $50. It expects

FinCorp Inc. purchased a stock for $50. It expects to hold the stock for two years, receive a dividend of $1.50 at the end of each year, and sell the stock immediately after receiving the second divid...

See Answer

Q: You have just been to see your broker at Acclaim Capital Inc

You have just been to see your broker at Acclaim Capital Inc. for advice about investing in the Empire Bank. The broker indicates that the Empire Bank has three different types of securities: debt, pr...

See Answer

Q: Calculate the correlation coefficient ( ρ AB ) for the following situation

Calculate the correlation coefficient ( ρ AB ) for the following situation:

See Answer

Q: Ibis Company is expected to pay a $1.50 dividend

Ibis Company is expected to pay a $1.50 dividend next year. Dividends are expected to grow at 3 percent forever and the required rate of return is 7 percent. a. What is the price of Ibis today? b. Wha...

See Answer

Q: Parker Imports Ltd. is expected to pay a $2 dividend

Parker Imports Ltd. is expected to pay a $2 dividend in one year. The required rate of return is 9 percent. The firm uses a dividend payout ratio of 25 percent. Calculate the leading P/E ratio in the...

See Answer

Q: Spinnaker Supplies Ltd. currently doesn’t pay any dividends but is expected

Spinnaker Supplies Ltd. currently doesn’t pay any dividends but is expected to start paying dividends in five years. The first dividend is expected to be $1.80 and is expected to grow at 4.5 percent t...

See Answer

Q: Latta Incorporated has announced an annual dividend of $5.00

Latta Incorporated has announced an annual dividend of $5.00. The firm has zero growth and the required rate of return for this type of firm is 10 percent. Assuming that the ex‐dividend date is Januar...

See Answer

Q: OK Natural Foods’ current dividend is $5. You expect the

OK Natural Foods’ current dividend is $5. You expect the growth rate to be 0 percent for years 1 to 5, and 2 percent for years 6 to infinity. The required rate of return on this firm’s equity is 10 pe...

See Answer

Q: Peak’s Organic Foods’ current dividend is $4. You expect the

Peak’s Organic Foods’ current dividend is $4. You expect the growth rate to be 10 percent for years 1 to 5, and 3 percent from years 6 to infinity. The required rate of return on this firm’s equity is...

See Answer

Q: Peele Clothiers Ltd. ’ s current dividend is $3.

Peele Clothiers Ltd. ’ s current dividend is $3.60. Dividends are expected to grow by 9 percent for years 1 to 3, 6 percent for years 4 to 7, and 2 percent thereafter. The required rate of return on t...

See Answer

Q: Prime Tire’s current dividend is $4. Dividends are expected to

Prime Tire’s current dividend is $4. Dividends are expected to grow by 25 percent for years 1 to 3 and 10 percent thereafter. The required rate of return on the stock is 15 percent. What is Prime’s cu...

See Answer

Q: Investors demand a rate of return of 15 percent on Sweet Life

Investors demand a rate of return of 15 percent on Sweet Life Food Inc. ’ s common shares. These shares are currently trading at $20 per share. Dividend payout for the current year is expected to be $...

See Answer

Q: Next year Dillon Mechanical Inc ’s EPS is expected to be $

Next year Dillon Mechanical Inc ’s EPS is expected to be $4. The firm is not expected to pay any dividends for the next four years. In year 5, a dividend of $1.50 is expected and subsequent dividends...

See Answer

Q: How does the existence of asymmetric information lead to market inefficiencies?

How does the existence of asymmetric information lead to market inefficiencies?

See Answer

Q: A firm plans to either lease a piece of equipment or purchase

A firm plans to either lease a piece of equipment or purchase it. The upfront purchase price is $900,000, and it is depreciated at $90,000 per year for tax purposes. The equipment could be sold in nin...

See Answer

Q: Bert has just been hired by your company as a summer co

Bert has just been hired by your company as a summer co‐op student and has been assigned to assist you. Bert is puzzled about why your company is calculating IRR and payback periods for investment pro...

See Answer

Q: State the drawbacks of the payback period and discounted payback period.

State the drawbacks of the payback period and discounted payback period.

See Answer

Q: Firms A and B are competing for a project. The potential

Firms A and B are competing for a project. The potential client has provided the following information on a hypothetical project: initial cost is $500,000; building renovation is $600,000; and the bui...

See Answer

Q: An investor has observed that BathGate Company, a shareholder wealth–

An investor has observed that BathGate Company, a shareholder wealth–maximizing company, has just made an investment that appears to have a negative NPV. The investor is very puzzled about why a compa...

See Answer

Q: Briefly describe the main factors that affect a call option’s value,

Briefly describe the main factors that affect a call option’s value, and how they affect the value.

See Answer

Q: Summarize all the cash flows that cannot be used in the capital

Summarize all the cash flows that cannot be used in the capital budgeting process and explain the reasons.

See Answer

Q: LargeCo has a capital budget of $100 million to invest in

LargeCo has a capital budget of $100 million to invest in projects. It has evaluated six independent projects and the results of the analysis are summarized in the following table. a. If the company w...

See Answer

Q: Complete the following table. /

Complete the following table.

See Answer

Q: Mr. Kent, one of your clients, has been reading

Mr. Kent, one of your clients, has been reading his daughter ’ s finance textbook and has a question about options. He says: “If I buy a call option, I have the right to buy the asset at the strike pr...

See Answer

Q: The current price of a stock is $107. You expect

The current price of a stock is $107. You expect the price of the stock to be in the range of $105 to $115 in the next period. The call option with an exercise price of $115 is $1.30; the put option w...

See Answer

Q: A firm plans to either purchase or lease a machine that costs

A firm plans to either purchase or lease a machine that costs $300,000 and is subject to a 20‐percent CCA rate, using the declining balance method. The required lease payments are $35,000 at the begin...

See Answer

Q: You have just been appointed manager of the equity portfolio of a

You have just been appointed manager of the equity portfolio of a large pension plan. The portfolio has a current value of $100 billion and is well diversified; consequently, it has a beta close to on...

See Answer

Q: In the next period, the economy can be in either state

In the next period, the economy can be in either state 1 or state 2, with a probability of 50 percent. You may trade any combination, including fractions of shares, of the following three securities:...

See Answer

Q: Xiang Zhu, a client of FinCorp Inc., has phoned you

Xiang Zhu, a client of FinCorp Inc., has phoned you with a question. She has been reading a finance textbook and cannot understand how to use the binomial option pricing model to value a call option....

See Answer

Q: Explain how the purchase method gives rise to goodwill.

Explain how the purchase method gives rise to goodwill.

See Answer

Q: What is so different about evaluating FDI compared to domestic projects?

What is so different about evaluating FDI compared to domestic projects?

See Answer

Q: Name some unique risks that may arise when evaluating FDI.

Name some unique risks that may arise when evaluating FDI.

See Answer

Q: Assume that BigCo’s cost of capital for all the projects is 10

Assume that BigCo’s cost of capital for all the projects is 10 percent. Calculate the NPV, IRR, payback period, discounted payback, and profitability index for each project in Table...

See Answer

Q: a. If the firm is not capital constrained and the projects

a. If the firm is not capital constrained and the projects in Table 1 are independent, which projects should the firm undertake using the following criteria? i. NPV ii. IRR iii. Payback period iv. Dis...

See Answer

Q: a. If the firm is not capital constrained and the projects

a. If the firm is not capital constrained and the projects in Table 1 are mutually exclusive, which project should the firm undertake using the following criteria? i. NPV ii. IRR iii. Payback period i...

See Answer

Q: Using projects A to F in Table 1, construct BigCo’s investment

Using projects A to F in Table 1, construct BigCo’s investment opportunity schedule. If BigCo has $9,000 (millions) available for investment, which projects should it undertake (assume all projects ar...

See Answer

Q: Describe how CCA recapture and CCA terminal losses occur and their tax

Describe how CCA recapture and CCA terminal losses occur and their tax treatment.

See Answer

Q: The CFO of BigCo is concerned about the sensitivity of his decisions

The CFO of BigCo is concerned about the sensitivity of his decisions to the choice of discount rate. For projects A, C, and E, plot the NPV profiles on the same graph. Does the NPV ranking of the thre...

See Answer

Q: Calculate the crossover rate for projects B and C from Table 1

Calculate the crossover rate for projects B and C from Table 1.

See Answer

Q: You have conducted an analysis of BigCo and have found that the

You have conducted an analysis of BigCo and have found that the firm is made up of two different divisions: SatellitesRUs (a satellite launching service) and a bank. Projects A to G are all related to...

See Answer

Q: The CEO of BigCo has just bought a fancy financial calculator and

The CEO of BigCo has just bought a fancy financial calculator and calculated the IRR and NPV of Project D from Table 1. He is utterly confused. His calculator is telling him that the IRR is 20.72 perc...

See Answer

Q: Cutler Compacts will generate cash flows of $30,000 in

Cutler Compacts will generate cash flows of $30,000 in year 1 and $65,000 in year 2. However, if it makes an immediate investment of $20,000, it can instead expect to have cash streams of $55,000 in t...

See Answer

Q: Elaine is evaluating two investments—investment 1 has a profitability index

Elaine is evaluating two investments—investment 1 has a profitability index (PI) of 2.4 while investment 2 has a PI of 1.2. As these investments are mutually exclusive, Elaine is recommending investme...

See Answer

Q: Daria is evaluating two investments—investment 1 will produce cash flows

Daria is evaluating two investments—investment 1 will produce cash flows for the next 5 years and has an NPV of $1,000. Investment 2 will produce cash flows for the next 15 years and has an NPV of $70...

See Answer

Q: What is the crossover rate ( r ) given the following?

What is the crossover rate ( r ) given the following? Project A: C F0 $25,000; C F1 $6,600; C F2 $10,200; C F3 $19,800 P roject B: C F0 $20,000; C F1 $5,400; C F2 $7,800; C F3 $16,500

See Answer

Q: If the NPV of a project is $5,360 and

If the NPV of a project is $5,360 and its after‐tax initial investment is $12,050, what is its PI? Should the firm accept the project? Does the PI yield the same decision as the NPV? (Assume all the c...

See Answer

Q: SK Inc. has two projects as follows: /

SK Inc. has two projects as follows: If SK set 2.6 years as a cut‐off period for screening projects, which projects will be selected, using the payback period method?

See Answer

Q: What is the difference between sensitivity analysis and scenario analysis?

What is the difference between sensitivity analysis and scenario analysis?

See Answer

Q: Which project(s) will be selected if the company uses

Which project(s) will be selected if the company uses the discounted payback period method in Practice Problem 34 and the discount rate is 12 percent?

See Answer

Q: State the decision rules for NPV, IRR, PI, and

State the decision rules for NPV, IRR, PI, and the discounted payback period. List two possible consequences of using IRR.

See Answer

Q: SK Inc. has a project that requires a $60,

SK Inc. has a project that requires a $60,000 after‐tax initial investment and produces these after‐tax cash flows at each year‐end: $20,000; $22,000; −$8,000; $38,050; $55,000; and $16,000. The appro...

See Answer

Q: Calculate the NPV and IRR of the following project and check whether

Calculate the NPV and IRR of the following project and check whether they produce the same decision. After‐tax initial investment is $66,777; after‐tax cash flows at each of the following six year‐end...

See Answer

Q: You have two mutually exclusive projects: / Irrespective of

You have two mutually exclusive projects: Irrespective of the project, the discount rate is 10 percent. Calculate the payback and discounted payback periods for the projects. Which one will you consid...

See Answer

Q: What are independent projects? What are mutually exclusive projects?

What are independent projects? What are mutually exclusive projects?

See Answer

Q: For each pair of investment opportunities, indicate if they are more

For each pair of investment opportunities, indicate if they are more likely to be mutually exclusive or independent projects. Explain your choices. a. Cruise line: i. Build a cruise ship to carry 10,0...

See Answer

Q: What are the practical difficulties when attempting to apply the NPV evaluation

What are the practical difficulties when attempting to apply the NPV evaluation process to foreign investments?

See Answer

Q: For the following decisions, indicate whether they are examples of a

For the following decisions, indicate whether they are examples of a bottom‐up analysis or a top‐down analysis: a. Replacing the printing press at a newspaper b. A...

See Answer

Q: Your truck has a market value of $60,000.

Your truck has a market value of $60,000. You can sell it to your brother, who agreed to buy it now and pay $75,000 three years from now, or you can sell it to your cousin who agreed to pay you $65,00...

See Answer

Q: Explain how real option valuation (ROV) is applied.

Explain how real option valuation (ROV) is applied.

See Answer

Q: You are interested in an investment where the initial investment is $

You are interested in an investment where the initial investment is $118,000 and your required cost of capital is 12 percent. Cash inflows from this project are expected to be $11,400 at the end of th...

See Answer

Q: Name one condition under which the discounted payback period will be equal

Name one condition under which the discounted payback period will be equal to the payback period.

See Answer

Q: Using initial cost of I 0 , and annual cash inflows of

Using initial cost of I 0 , and annual cash inflows of R over N period, express R in terms of I 0 and N such that the payback period is equal to its life.

See Answer

Q: Using NPV, should you invest in a project where the initial

Using NPV, should you invest in a project where the initial cash outflow is $24,000 and the cash inflow in the first year is $1,600 and “grows” at a rate of 3 percent thereafter? Assume cost of capita...

See Answer

Q: A bond is currently trading at par, which is $1

A bond is currently trading at par, which is $1,000. If the bond pays an annual coupon rate of 10 percent, calculate the IRR of this bond.

See Answer

Q: Financial statements for Canadian public corporations are available from corporate websites or

Financial statements for Canadian public corporations are available from corporate websites or from the System for Electronic Document Analysis and Retrieval (SEDAR) at www.sedar.com. a. Review a copy...

See Answer

Q: White River Manufacturing Company has just signed several leases and has hired

White River Manufacturing Company has just signed several leases and has hired FinCorp Inc. to do the initial classification of the leases as operating or financial for accounting purposes. White Rive...

See Answer

Q: Igor, the intern at FinCorp Inc., has just presented your

Igor, the intern at FinCorp Inc., has just presented your boss, Mr. Cabinet, with his valuation of Kitchen Gadget Company (KGC). Igor has identified two other companies, Kitchen Widgets and Kitchen Th...

See Answer

Q: Nash Business School is considering whether to lease or buy shuttle buses

Nash Business School is considering whether to lease or buy shuttle buses for students travelling between its two campuses. The buses cost $2 million, with a CCA rate of 20 percent. The lease payments...

See Answer

Q: In late 2006, Canada Post began issuing “permanent” stamps

In late 2006, Canada Post began issuing “permanent” stamps. Purchased at the current first‐class letter rate these stamps may be used indefinitely even if the price of stamps increases. Arthur Ponzare...

See Answer

Q: Explain how offering memorandums differ from prospectuses and how exempt markets differ

Explain how offering memorandums differ from prospectuses and how exempt markets differ from public markets.

See Answer

Q: You are in the process of interviewing for a promotion at FinCorp

You are in the process of interviewing for a promotion at FinCorp Inc. and have to identify the type of security based on the payoff diagrams below. All options expire on January 15, 201x, and the und...

See Answer

Q: Mr. Cabinet, your boss at FinCorp Inc., prefers to

Mr. Cabinet, your boss at FinCorp Inc., prefers to have his information presented visually. At his request, graph the payoffs (intrinsic values) and profits at expiration for the following option inve...

See Answer

Q: Describe three major categories of exempt purchasers.

Describe three major categories of exempt purchasers.

See Answer

Q: Describe the basic stages of IPOs.

Describe the basic stages of IPOs.

See Answer

Q: Jackie would like to borrow $125,000 to expand her

Jackie would like to borrow $125,000 to expand her small business, but needs to understand the impact of the 6-percent interest payments. Last year, her company did not pay any interest and had total...

See Answer

Q: The cost (interest rate) of the loan Jackie needs for

The cost (interest rate) of the loan Jackie needs for her business is 6 percent per year. Given that the company s net income will fall by less than the amount of interest paid (see Practice Problem 2...

See Answer

Q: Describe four different types of public offerings.

Describe four different types of public offerings.

See Answer

Q: Compare the payoff of a call option and the underlying security.

Compare the payoff of a call option and the underlying security. Show that the price of a call option must always be less than the value of the underlying security.

See Answer

Q: Collingwood Corp.’s bank is willing to provide it with a

Collingwood Corp.’s bank is willing to provide it with a 10‐year term loan for $25 million. The annual payments on this loan will be $2.65 million, and there is a “bullet” payment of $25 million at ma...

See Answer

Q: List and briefly describe the six basic factors used to determine a

List and briefly describe the six basic factors used to determine a DBRS rating.

See Answer

Q: What is a reverse takeover and a backdoor listing?

What is a reverse takeover and a backdoor listing?

See Answer

Q: Complete the following table. /

Complete the following table.

See Answer

Q: Richards & Co. Analysts have provided the following partially completed table

Richards & Co. Analysts have provided the following partially completed table of information about different securities. All options are written on XCT, a nondividend‐ paying sto...

See Answer

Q: If the IPO for Finns’ Fridges (see Practice Problem 28)

If the IPO for Finns’ Fridges (see Practice Problem 28) goes well, the contract with the investment bankers has a “green‐shoe” clause that permits them to sell 15 percent more shares than originally p...

See Answer

Q: Niagara Vineyards and Winery needs to raise $4 million in new

Niagara Vineyards and Winery needs to raise $4 million in new equity. a. If the costs of the share issue are estimated to be 6 percent of gross proceeds, how large does the offering need to be? How mu...

See Answer

Q: Complete the following table. /

Complete the following table.

See Answer

Q: Which of the following items, relating to working capital, would

Which of the following items, relating to working capital, would be considered a cash inflow or outflow when evaluating a project (and why)? a. Increase in inventory b. Increase in accounts payable c....

See Answer

Q: In most markets, you are not permitted to short sell if

In most markets, you are not permitted to short sell if the stock price has fallen. That is, you can only short sell on an “uptick.” Using put‐call parity, show that you can replicate the cash flows o...

See Answer

Q: QBV, a non‐dividend‐paying stock, is currently

QBV, a non‐dividend‐paying stock, is currently trading for $80 a share. There is a 25‐percent chance that the stock will trade for $65 in one year, and a 75‐percent chance that the price will increase...

See Answer

Q: QBV, a non‐dividend‐paying stock, is currently

QBV, a non‐dividend‐paying stock, is currently trading for $100 a share. There is a 25‐percent chance that the stock will trade for $85 in one year, and a 75‐percent chance that the price will increas...

See Answer

Q: a. Describe how CCA expenses change through the life of a

a. Describe how CCA expenses change through the life of a project. b. Given C0 = $250,000; CCA rate = 20%; tax rate = 40%; and year 2 operating income = $150,000, calculate the cash flow in year 2.

See Answer

Q: What are some of the more important issues arising from the fact

What are some of the more important issues arising from the fact that securities regulation is a provincial and territorial, but not a federal, responsibility in Canada?

See Answer

Q: QBV, a non‐dividend‐paying stock, is currently

QBV, a non‐dividend‐paying stock, is currently trading for $100 a share. There is a 25‐percent chance that the stock will trade for $85 in one year, and a 75‐percent chance that the price will increas...

See Answer

Q: DPG, a non‐dividend‐paying stock, is currently

DPG, a non‐dividend‐paying stock, is currently trading for $150 a share. There is a 30‐percent chance that the stock will trade for $125 in one year, and a 70‐percent chance that the price will increa...

See Answer

Q: Jensen’s Juice Bar is considering purchasing a new blender. Indicate which

Jensen’s Juice Bar is considering purchasing a new blender. Indicate which of the following statements is a relevant consideration in the new-blender decision. a. Last year, Jensen’s spent $500 on a n...

See Answer

Q: Briefly state all the factors that affect the value of a call

Briefly state all the factors that affect the value of a call option and a put option.

See Answer

Q: Your boss has observed that the call options on XCT and BRG

Your boss has observed that the call options on XCT and BRG are trading at different prices. Both options have the same strike price and the same time to expiration. Provide two possible explanations...

See Answer

Q: Mr. Cabinet is interested in the payoffs to combinations of options

Mr. Cabinet is interested in the payoffs to combinations of options. Graph the intrinsic values of the following portfolios (all options expire on the same day and are written on the same non dividend...

See Answer

Q: You have observed that a very smart and successful investor has bought

You have observed that a very smart and successful investor has bought a call and a put on the S&P/TSX Index. The options have the same strike prices and expire on the same day. What does the smart in...

See Answer

Q: You observe the following data on different options (all are European

You observe the following data on different options (all are European options with the same exercise date, and all are written on the same underlying security). What are the profits you would earn for...

See Answer

Q: Marcel owns 12 percent of Steam Forge Company (SFC). SFC

Marcel owns 12 percent of Steam Forge Company (SFC). SFC trades on the Toronto Stock Exchange and has been the subject of a takeover attempt by Iron Forge Company (IFC). Assuming that Marcel is the on...

See Answer

Q: Julius is a shareholder in a public corporation, which has recently

Julius is a shareholder in a public corporation, which has recently acquired another company, and the consequences for the bidding firm have been catastrophic. Julius is suing the bidder’s board of di...

See Answer

Q: How is goodwill treated for accounting purposes in Canada and the United

How is goodwill treated for accounting purposes in Canada and the United States?

See Answer

Q: Your boss is very puzzled by the finance courses in his MBA

Your boss is very puzzled by the finance courses in his MBA program. He has learned that “cash flow is king,” but notices that the capital budgeting problems spend a lot of time and effort dealing wit...

See Answer

Q: Describe limit orders and market orders.

Describe limit orders and market orders.

See Answer

Q: What is the “quiet period”?

What is the “quiet period”?

See Answer

Q: Describe the overallotment or “green‐shoe” option.

Describe the overallotment or “green‐shoe” option.

See Answer

Q: What is a “road show”?

What is a “road show”?

See Answer

Q: On a one‐year loan of $10,000,

On a one‐year loan of $10,000, a bank charges interest at 8 percent. The bank also charges an application fee of $75 to cover processing expenses. What is the effective interest cost (annual rate) bei...

See Answer

Q: Calculate the price of a 91‐day T‐bill if

Calculate the price of a 91‐day T‐bill if the face value is $1 million and the quoted interest rate is 3.8 percent. Round to the nearest dollar.

See Answer

Q: Calculate the value of the one‐month CP given the following

Calculate the value of the one‐month CP given the following: par value is $2 million, promised yield is 4%, probability of not defaulting is 97%, recovery rate is zero, and required return is 14%. Rou...

See Answer

Q: Discuss the important characteristics of money market debt instruments. How are

Discuss the important characteristics of money market debt instruments. How are these characteristics important to money market participants?

See Answer

Q: State the five major areas in which the Ontario Securities Commission (

State the five major areas in which the Ontario Securities Commission (OSC) is involved.

See Answer

Q: Suppose you are going to enter into a six‐year,

Suppose you are going to enter into a six‐year, $25,000 financial lease that requires monthly payments based on an 9‐percent lease rate. Alternatively, you could borrow $25,000 via a six‐year loan tha...

See Answer

Q: State three types of distributions of securities determined by the OSC.

State three types of distributions of securities determined by the OSC.

See Answer

Q: In your job as treasurer of Collingwood Corp., you have to

In your job as treasurer of Collingwood Corp., you have to arrange a line of credit for the firm. The following is taken from the company ’ s balance sheet. The bank will provide cre...

See Answer

Q: Why do corporations issue long‐term bonds, knowing that interest

Why do corporations issue long‐term bonds, knowing that interest rate risk is higher for longer‐term bonds?

See Answer

Q: Richards & Co. Analysts has provided FinCorp Inc. with incomplete

Richards & Co. Analysts has provided FinCorp Inc. with incomplete information, and it is your job to fill in the missing information in the table below.

See Answer

Q: Fill in the missing information in the following table for a non

Fill in the missing information in the following table for a non‐dividend‐paying stock and European call options.

See Answer

Q: What are continuous disclosure requirements?

What are continuous disclosure requirements?

See Answer

Q: Back in their college days, David and Douglas Finn started renting

Back in their college days, David and Douglas Finn started renting refrigerators to other students for use in their dormitory rooms. Over the years, Finns’ Fridges has grown and financed its operation...

See Answer

Q: Complete the following table. Assume that the asset class is left

Complete the following table. Assume that the asset class is left open, and/or the salvage value is less than the UCC for the entire class.

See Answer

Q: What is the price of a put option with a strike price

What is the price of a put option with a strike price of $50 and six months to maturity when the stock price is currently trading at $45? Assume the stock‐price variance is 0.5 and the risk‐free rate...

See Answer

Q: Determine the call price ( C ), given the following information:

Determine the call price ( C ), given the following information: stock price ( S) $36, s trike price (X) $32, r isk-free rate (r) 5% , t 2 years, 20%.

See Answer

Q: List and briefly describe the four basic stages of the IPO process

List and briefly describe the four basic stages of the IPO process.

See Answer

Q: Paolo, the CEO of Paola Bros Inc., wants to have

Paolo, the CEO of Paola Bros Inc., wants to have a fleet of electric cars. These electric cars cost $1.5 million. For tax purposes, assume that these cars will depreciate at a rate of $160,000 per yea...

See Answer

Q: You are given the following information: C0 = $300,

You are given the following information: C0 = $300,000; CCA rate (d) = 0.3; T = 0.4; RF = 4.5%; project beta = 1.2; market risk premium = 10%; SVn = $35,000; UCCn = $55,000. This project has a 5-year...

See Answer

Q: You are given the following information: CFBT = $215,

You are given the following information: CFBT = $215,000; T = 40%; this project will last for eight years. The project has a 2.5-percent extra risk premium compared with the firm ’ s cost of capital....

See Answer

Q: Calculate the present value of the operating cash flows if the revenue

Calculate the present value of the operating cash flows if the revenue of a project grows at 5 percent, while expense grows at 4 percent, given that Revenue1 = $15,000 and Expense 1 = $7,000. Assume t...

See Answer

Q: Calculate the put price (P), according to put‐call

Calculate the put price (P), according to put‐call parity, given the information in Practice Problem 29.

See Answer

Q: What is the strike price (X) if PU is $

What is the strike price (X) if PU is $50, PD is $42, and the hedge ratio ( h) is 2?

See Answer

Q: Assume stock XYZ does not pay dividends and has a market value

Assume stock XYZ does not pay dividends and has a market value of $98 per share. There is a 60‐percent chance that the stock will trade for $130 in one year, and a 40‐ percent chance that it will trad...

See Answer

Q: Does put‐call parity hold for the following? Riskfree rate

Does put‐call parity hold for the following? Riskfree rate = 5%, P0 = $13, C0 = $10, stock price (S) = $30, t = 4 years, strike price (X) = $33. If not, what is the put price according to put‐call par...

See Answer

Q: A bidder paid $1,500 for a target. The

A bidder paid $1,500 for a target. The target’s market asset is $2,000 and market liability is $1,250. What is the goodwill created during the acquisition?

See Answer

Q: Prepare a schedule of the annual capital cost allowance for a major

Prepare a schedule of the annual capital cost allowance for a major project. The initial investment is $80,000, the tax rate is 40 percent, and the CCA rate is 30 percent. Determine the amount of CCA...

See Answer

Q: Describe the market or disaster “out” clause.

Describe the market or disaster “out” clause.

See Answer

Q: In Practice Problem 24, what would the lease payment have to

In Practice Problem 24, what would the lease payment have to be for Paolo to be indifferent about whether the company buys or leases the electric cars?

See Answer

Q: Why is it usually more precise to use nominal cash flows and

Why is it usually more precise to use nominal cash flows and nominal discount rates when evaluating projects?

See Answer

Q: Why might inflation affect cash inflows differently from the way it would

Why might inflation affect cash inflows differently from the way it would affect cash outflows?

See Answer

Q: What is the difference between independent and mutually exclusive projects?

What is the difference between independent and mutually exclusive projects?

See Answer

Q: How can we compare two choices, one involving a wooden bridge

How can we compare two choices, one involving a wooden bridge lasting 10 years and another involving a steel bridge lasting 25 years that costs more?

See Answer

Q: How should we treat taxes and inflation when determining the present value

How should we treat taxes and inflation when determining the present value of future cash flows?

See Answer

Q: What do we mean by incremental cash flows?

What do we mean by incremental cash flows?

See Answer

Q: What are externalities and opportunity costs?

What are externalities and opportunity costs?

See Answer

Q: Why do we not deduct interest costs from the cash flows to

Why do we not deduct interest costs from the cash flows to be discounted?

See Answer

Q: What goes into a confidentiality agreement and why do people sign them

What goes into a confidentiality agreement and why do people sign them?

See Answer

Q: What is due diligence?

What is due diligence?

See Answer

Q: In Practice Problem 15, assuming that Nash Business School has an

In Practice Problem 15, assuming that Nash Business School has an effective tax rate of 40 percent, should the shuttle buses be bought or leased?

See Answer

Q: What is a shareholder rights plan?

What is a shareholder rights plan?

See Answer

Q: What are some standard takeover defences?

What are some standard takeover defences?

See Answer

Q: When is it best to mount a hostile bid?

When is it best to mount a hostile bid?

See Answer

Q: Why is the payback period a poor evaluation technique?

Why is the payback period a poor evaluation technique?

See Answer

Q: What discount rate do we use to determine the NPV of a

What discount rate do we use to determine the NPV of a project and why?

See Answer

Q: Why do we sometimes get multiple IRRs for a project?

Why do we sometimes get multiple IRRs for a project?

See Answer

Q: What are the reinvestment rate assumptions underlying NPV and IRR?

What are the reinvestment rate assumptions underlying NPV and IRR?

See Answer

Q: What is the crossover rate?

What is the crossover rate?

See Answer

Q: Is the PI rule consistent with the NPV rule?

Is the PI rule consistent with the NPV rule?

See Answer

Q: How does the analysis change when the lease is a financial lease

How does the analysis change when the lease is a financial lease?

See Answer

Q: Michael M. specializes in buying high‐risk commercial paper;

Michael M. specializes in buying high‐risk commercial paper; his required return on these investments is 14 percent per year. He is considering buying some 60‐day paper from Collingwood Corp. with a p...

See Answer

Q: Why does the initial cash outlay often exceed the purchase price of

Why does the initial cash outlay often exceed the purchase price of an asset?

See Answer

Q: Explain why the valuation by components approach can save computational time and

Explain why the valuation by components approach can save computational time and still lead to the correct answer.

See Answer

Q: How do taxes affect the annual cash flows and terminal cash flows

How do taxes affect the annual cash flows and terminal cash flows of an investment project?

See Answer

Q: Explain how to calculate comparisons in the lease-versus-buy

Explain how to calculate comparisons in the lease-versus-buy decision when the lease in question is an operating lease.

See Answer

Q: Distinguish debt from equity.

Distinguish debt from equity.

See Answer

Q: Explain how to estimate the after-tax cost of debt.

Explain how to estimate the after-tax cost of debt.

See Answer

Q: What three characteristics does the CRA look for to determine whether interest

What three characteristics does the CRA look for to determine whether interest payments are tax deductible?

See Answer

Q: What is the difference between value and price?

What is the difference between value and price?

See Answer

Q: What is fair market value?

What is fair market value?

See Answer

Q: What key multiples are used in valuing companies?

What key multiples are used in valuing companies?

See Answer

Q: Collingwood Corp. is able to issue its 60‐day commercial

Collingwood Corp. is able to issue its 60‐day commercial paper at par with a promised yield of 10 percent per year. The current T‐bill yield is 6 percent per year (or 1 percent for the 60‐day period)....

See Answer

Q: Why do differing capital structures cause problems with using P/E

Why do differing capital structures cause problems with using P/E multiples?

See Answer

Q: What is free cash flow?

What is free cash flow?

See Answer

Q: When does EPS increase when using a share swap?

When does EPS increase when using a share swap?

See Answer

Q: What is an amalgamation?

What is an amalgamation?

See Answer

Q: What is the majority of the minority rule?

What is the majority of the minority rule?

See Answer

Q: What insights can be gained by using sensitivity analysis, scenario analysis

What insights can be gained by using sensitivity analysis, scenario analysis, and NPV break-even analysis?

See Answer

Q: What limitations of scenario analysis does the real option valuation approach address

What limitations of scenario analysis does the real option valuation approach address?

See Answer

Q: What is the difference between an acquisition and a merger?

What is the difference between an acquisition and a merger?

See Answer

Q: Explain how interest is received on most money market instruments.

Explain how interest is received on most money market instruments.

See Answer

Q: Contrast treasury bills, commercial paper, and BAs in terms of

Contrast treasury bills, commercial paper, and BAs in terms of who issues them, their basic structure and default risk, and the yields they provide.

See Answer

Q: When Collingwood Corp. issued its 60‐day commercial paper the

When Collingwood Corp. issued its 60‐day commercial paper the promised yield was 10 percent, whereas the 60‐day T‐bill yield was 6 percent. There is a 1-percent chance that Collingwood will default on...

See Answer

Q: Define yield spreads and explain how they arise.

Define yield spreads and explain how they arise.

See Answer

Q: How can the Black-Scholes equation be used to price options

How can the Black-Scholes equation be used to price options?

See Answer

Q: What is measured by each of the five Greeks discussed in this

What is measured by each of the five Greeks discussed in this section?

See Answer

Q: Discuss any differences in the evaluation of a replacement decision versus the

Discuss any differences in the evaluation of a replacement decision versus the evaluation of an expansion decision.

See Answer

Q: Contrast the payoff from a put option with that from a call

Contrast the payoff from a put option with that from a call option.

See Answer

Q: Explain how to estimate the intrinsic value and time value for a

Explain how to estimate the intrinsic value and time value for a put option.

See Answer

Q: Briefly describe the main factors that affect a put or a call

Briefly describe the main factors that affect a put or a call option’s value, and explain how they affect the value of each.

See Answer

Q: Illustrate how to combine the four basic option positions to create a

Illustrate how to combine the four basic option positions to create a variety of net payoff positions.

See Answer

Q: Explain why the put-call parity relationship should hold if markets

Explain why the put-call parity relationship should hold if markets are efficient.

See Answer

Q: Explain how to synthetically create long and short positions in calls,

Explain how to synthetically create long and short positions in calls, puts, and the underlying assets using put-call parity.

See Answer

Q: You are a risk arbitrageur and you observe the following information about

You are a risk arbitrageur and you observe the following information about a deal: the current price of the target is $22 per share and the current price of the bidder is $16 per share. The bidder is...

See Answer

Q: Explain why the lock-up period is an important consideration for

Explain why the lock-up period is an important consideration for investors, especially for issues that are still largely held by insiders.

See Answer

Q: How do continuous disclosure requirements protect investors?

How do continuous disclosure requirements protect investors?

See Answer

Q: Briefly explain why short-form prospectuses are permitted by regulators for

Briefly explain why short-form prospectuses are permitted by regulators for a large percentage of seasoned issues, and explain why they have led to the growth in popularity of bought deals.

See Answer

Q: Where are options traded?

Where are options traded?

See Answer

Q: How are implied volatilities calculated? What information do they provide?

How are implied volatilities calculated? What information do they provide?

See Answer

Q: What real options have you been given over the past year and

What real options have you been given over the past year and how valuable were they? What factors do you think influenced your valuation of them?

See Answer

Q: What tax benefits can occur in an M&A?

What tax benefits can occur in an M&A?

See Answer

Q: What is the empirical record on the success of M&As

What is the empirical record on the success of M&As in the 1990s?

See Answer

Q: What is SVAR and why do managers prefer to finance with shares

What is SVAR and why do managers prefer to finance with shares than cash?

See Answer

Q: Define mortgage bonds, secured debentures, unsecured debentures, and subordinated

Define mortgage bonds, secured debentures, unsecured debentures, and subordinated debt.

See Answer

Q: If the interest rate for non‐fraudulent bonds is 8 percent

If the interest rate for non‐fraudulent bonds is 8 percent, and chances are that one out of eight bonds is fraudulent, what is the interest rate based on a one‐year investment and assuming the market...

See Answer

Q: Discuss the rationale for including debt covenants in a public issue.

Discuss the rationale for including debt covenants in a public issue.

See Answer

Q: Briefly describe the negative pledge and cross-default clauses.

Briefly describe the negative pledge and cross-default clauses.

See Answer

Q: What is the difference between an operating and a financial lease?

What is the difference between an operating and a financial lease?

See Answer

Q: What type of leases do chartered banks normally make?

What type of leases do chartered banks normally make?

See Answer

Q: What is a sale and leaseback agreement (SLB)?

What is a sale and leaseback agreement (SLB)?

See Answer

Q: Why are leases often more flexible than a borrow-purchase option

Why are leases often more flexible than a borrow-purchase option?

See Answer

Q: Why do you think that the major market for leasing is often

Why do you think that the major market for leasing is often SMEs, rather than large corporations?

See Answer

Q: If you were opening a copy centre, do you think you

If you were opening a copy centre, do you think you would lease or borrow to buy the equipment and why?

See Answer

Q: 1. Which of the following statements about debt is incorrect?

1. Which of the following statements about debt is incorrect? a. Interest payments and principal payments are fixed commitments. b. Interest payments are not tax deductible. c. Bond holders are paid a...

See Answer

Q: 1. Which of the following is not one of the three

1. Which of the following is not one of the three types of merger? a. Vertical M&A b. Horizontal M&A c. Proxy contest d. Conglomerate 2. Which of the following M&As is valid? a. VA T $400,000; VA $20...

See Answer

Q: Assume two bonds in the market—bond A and bond B

Assume two bonds in the market—bond A and bond B—have the same rating and the same YTM. Discuss three reasons that might make one bond preferable to the other.

See Answer

Q: 1. Which of the following statements about a call option is

1. Which of the following statements about a call option is false? a. A call option is the right, not the obligation, to buy the underlying asset. b. A call option is in the money if the asset price i...

See Answer

Q: 1. What will probably happen if a firm does not invest

1. What will probably happen if a firm does not invest effectively? a. The firm could still maintain its competitive advantage. b. The cost of capital of the firm will be unchanged. d. The short‐term...

See Answer

Q: 1. Which of the following statements about IRR and NPV is

1. Which of the following statements about IRR and NPV is incorrect? a. NPV and IRR yield the same ranking when evaluating projects. b. NPV assumes that cash flows are reinvested at the cost of capita...

See Answer

Q: 1. When making capital expenditure decisions, firms should not consider

1. When making capital expenditure decisions, firms should not consider which of the following? a. After-tax incremental cash flows b. Additional working capital requirements c. Sunk costs d. Salvage...

See Answer

Q: 1. Which of the following statements is false? a

1. Which of the following statements is false? a. CCA recapture occurs when the salvage value is greater than the ending UCC for the asset or asset class. b. Capital gains occur when the salvage value...

See Answer

Q: 1. Which of the following statements about takeovers is false?

1. Which of the following statements about takeovers is false? a. Mergers create a new firm, while acquisitions do not. b. Both mergers and acquisitions require two‐thirds votes from both firms. c. In...

See Answer

Q: 1. Which of the following statements about an operating lease is

1. Which of the following statements about an operating lease is false ? a. The lessor is responsible for maintaining the asset. b. The lessee is responsible for maintaining the asset. c. An operating...

See Answer

Q: 1. Which of the following statements about due diligence is false

1. Which of the following statements about due diligence is false? a. It is designed to ensure the legitimacy of securities offered to the public. b. It is designed to ensure that there is no misleadi...

See Answer

Q: What is the difference between vertical and horizontal mergers?

What is the difference between vertical and horizontal mergers?

See Answer

Q: What is an extension M&A, an overcapacity M&

What is an extension M&A, an overcapacity M&A, and a geographic roll-up M&A?

See Answer

Q: Lansdowne Ltd. needs to raise $20 million and intends to

Lansdowne Ltd. needs to raise $20 million and intends to sell additional shares. The company ’ s existing shares are trading on the Toronto Stock Exchange for $54. However, the investment dealer hired...

See Answer

Q: What financial synergies are possible in an M&A transaction?

What financial synergies are possible in an M&A transaction?

See Answer

Q: What is a tender?

What is a tender?

See Answer

Q: What is a takeover circular?

What is a takeover circular?

See Answer

Q: What is a creeping takeover?

What is a creeping takeover?

See Answer

Q: Why is it reasonable to assume that most firms will have a

Why is it reasonable to assume that most firms will have a banking relationship?

See Answer

Q: GiS Inc. now has the following two projects available:

GiS Inc. now has the following two projects available: Assume that R F 4%, r isk premium 8%, and b eta 1.25. Use the chain replication approach to determine which project GiS Inc. should choose if the...

See Answer

Q: Why can increases in interest rates not be used to solve the

Why can increases in interest rates not be used to solve the “lemons problem” in markets?

See Answer

Q: Why are securities legislation and corporate laws essential for markets to perform

Why are securities legislation and corporate laws essential for markets to perform properly?

See Answer

Q: A firm is considering two mutually exclusive projects, as follows.

A firm is considering two mutually exclusive projects, as follows. Determine which project should be accepted if the discount rate is 15 percent. Use the chain replication approach. Assume both projec...

See Answer

Q: Differentiate investment-grade debt from junk debt.

Differentiate investment-grade debt from junk debt.

See Answer

Q: List and briefly describe some possible reasons for the existence of IPO

List and briefly describe some possible reasons for the existence of IPO underpricing.

See Answer

Q: The little company you and your friend started in your parents ’

The little company you and your friend started in your parents ’ garage has grown so much that you are now ready to take the firm public. In your discussions with one of the top investment dealers, yo...

See Answer

Q: What complications arise when firms are rationed in terms of their available

What complications arise when firms are rationed in terms of their available capital budget?

See Answer

Q: Explain how firms should decide which projects to accept and which to

Explain how firms should decide which projects to accept and which to reject when capital rationing exists.

See Answer

Q: Briefly describe the main factors DBRS considers in determining its debt ratings

Briefly describe the main factors DBRS considers in determining its debt ratings.

See Answer

Q: Briefly summarize the evidence regarding how well debt ratings work.

Briefly summarize the evidence regarding how well debt ratings work.

See Answer

Q: Explain why the payoff from a call option is non-linear

Explain why the payoff from a call option is non-linear.

See Answer

Q: Explain how to estimate the intrinsic value and time value for a

Explain how to estimate the intrinsic value and time value for a call option.

See Answer

Q: What are irrevocable investment decisions? Why are they important for capital

What are irrevocable investment decisions? Why are they important for capital budgeting?

See Answer

Q: Contrast top down and bottom up analysis.

Contrast top down and bottom up analysis.

See Answer

Q: Briefly describe three motivations for leasing.

Briefly describe three motivations for leasing.

See Answer

Q: State the three basic tests the CRA uses to ensure interest payments

State the three basic tests the CRA uses to ensure interest payments are tax deductible.

See Answer

Q: Pills4u.com and Drugs‐R‐Us Co. both

Pills4u.com and Drugs‐R‐Us Co. both sell prescription medications over the Internet. Each company has recently announced an IPO at $20 per share. At this price, one of the companies is undervalued by...

See Answer

Q: Determine the selling price of a Government of Canada treasury bill that

Determine the selling price of a Government of Canada treasury bill that has a quoted annual interest rate of 1.2 percent and will mature in 90 days. Assume a par value of $1,000.

See Answer

Q: Collingwood Corp’s 60‐day commercial paper has a promised yield of

Collingwood Corp’s 60‐day commercial paper has a promised yield of 10 percent per year, but the expected yield is just 1.5 percent due to the risk of default. If the current 60‐day T‐bill yield is 1 p...

See Answer

Q: As the newly appointed treasurer for Collingwood Corp., you have to

As the newly appointed treasurer for Collingwood Corp., you have to decide how to raise $25 million in short‐term financing. You believe you could issue commercial paper with a promised yield of 10 pe...

See Answer

Q: Collingwood Corp. has a revolving line of credit on which it

Collingwood Corp. has a revolving line of credit on which it owes $25 million. One of the restrictions imposed with this financing arrangement is that the company must maintain a minimum interest cove...

See Answer

Q: An investment has the following cash inflows: $2,500

An investment has the following cash inflows: $2,500 at the end of the first year, $2,000 at the end of the second year, and $1,500 at the end of the third year. What is the discounted payback period...

See Answer

Q: Briefly explain the pure play method for estimating beta.

Briefly explain the pure play method for estimating beta.

See Answer

Q: FinCorp Inc. has both a call option and a put option

FinCorp Inc. has both a call option and a put option with exercise prices of $50. Both expire in one year. The call is currently selling for $10 per share, while the put is currently selling for $2 pe...

See Answer

Q: What is the lock‐up period?

What is the lock‐up period?

See Answer

Q: Calculate the initial cash flows (CF0) for the following projects

Calculate the initial cash flows (CF0) for the following projects. Which project has a larger CF0? a. Project A: equipment purchase price = $195,000; installation cost = $4,500; extra working capital...

See Answer

Q: What is the waiting period?

What is the waiting period?

See Answer

Q: Sous‐Chef Inc. is an employment agency that specializes in

Sous‐Chef Inc. is an employment agency that specializes in the restaurant industry. The company intends to sell 800,000 shares in its IPO and the investment dealers working on the is...

See Answer

Q: Winnipeg Water & Gas Co. recently issued a series of bonds

Winnipeg Water & Gas Co. recently issued a series of bonds; the gross proceeds were $25 million. The underwriting fees were 2.8 percent, and additional issuance costs were $150,000. How much did the c...

See Answer

Q: In Practice Problem 29, if the time value is $5

In Practice Problem 29, if the time value is $5, calculate the intrinsic value.

See Answer

Q: Rather than take a term loan from the bank, Collingwood Corp

Rather than take a term loan from the bank, Collingwood Corp. has decided to issue $25 million of 10‐year bonds. DBRS has assigned a rating of “BB” to this bond issue. a. Determine the probability tha...

See Answer

Q: Sometimes, bonds are completely worthless when a company defaults on payments

Sometimes, bonds are completely worthless when a company defaults on payments. However, in practice, bonds typically have some market value (recovery rate) even after a default. Collingwood’s bonds ar...

See Answer

Q: What is a hedge ratio?

What is a hedge ratio?

See Answer

Q: Why don’t the probabilities of going up and down affect the options

Why don’t the probabilities of going up and down affect the options value?

See Answer

Q: Calculate the market price of the firm’ s common shares using a

Calculate the market price of the firm’ s common shares using a relative valuation approach. (Round your answer to one decimal.)

See Answer

Q: Calculate the after-tax operating cash flow NPV break-even

Calculate the after-tax operating cash flow NPV break-even point for the project described in Practice Problem 48 by using a 12-percent discount rate. Also, the asset class will be closed at the end o...

See Answer

Q: Which type of lease, operating or financial, gives a higher

Which type of lease, operating or financial, gives a higher asset turnover ratio?

See Answer

Q: Why are prospectuses so important for public market issues?

Why are prospectuses so important for public market issues?

See Answer

Q: The balance sheets of a bidder and target companies are as follows

The balance sheets of a bidder and target companies are as follows: The tax rate for both companies is 25 percent. The acquisition will be accounted for using the purchase method. Prior to the acquisi...

See Answer

Q: Based on the cash flows given below, calculate the PI of

Based on the cash flows given below, calculate the PI of a project that has a required rate of return of 15 percent. Also, indicate whether the project should be accepted. Year 0: −$90,000 Year 1: $20...

See Answer

Q: GG Inc. is now considering replacing some old equipment. The

GG Inc. is now considering replacing some old equipment. The market price of the old equipment is $50,000 and the salvage value at the end of five years is $15,000. The new equipment will cost $100,00...

See Answer

Q: Why do banks typically impose debt covenants on their borrowing customers?

Why do banks typically impose debt covenants on their borrowing customers?

See Answer

Q: Complete the following balance sheet for the post‐merger firm B

Complete the following balance sheet for the post‐merger firm B‐T. The bidder acquired the target for $8,000 in cash.

See Answer

Q: Java Cafe’s tax rate is 45 percent and the appropriate discount rate

Java Cafe’s tax rate is 45 percent and the appropriate discount rate is 8 percent. It is considering another project. Each asset class consists only of the project asset and will be terminated at the...

See Answer

Q: Java Cafe’s tax rate is 40 percent and the appropriate discount rate

Java Cafe’s tax rate is 40 percent and the appropriate discount rate is 12 percent. It is considering another project. Each asset class consists only of the project asset and will be terminated at the...

See Answer

Q: KRZ Company has hired you to help evaluate several projects. The

KRZ Company has hired you to help evaluate several projects. The firm ’ s tax rate is 40 percent and the appropriate discount rate is 10 percent. Each asset class is small and will be terminated at th...

See Answer

Q: KRZ Company has hired you to help evaluate several projects. The

KRZ Company has hired you to help evaluate several projects. The firm’s tax rate is 40 percent and the appropriate discount rate is 15 percent. Each asset class is small and will be terminated at the...

See Answer

Q: You are trying to decide whether or not to go to graduate

You are trying to decide whether or not to go to graduate school. If you get a job right after you get your bachelor’s degree, you expect to earn $40,000 a year, and you expect your salary to increase...

See Answer

Q: Briefly discuss the possible motivation for firms to enter into IPOs,

Briefly discuss the possible motivation for firms to enter into IPOs, and relate these motivations to the five stages of firm development discussed by Myers (1999).

See Answer

Q: You are trying to decide whether to continue renting an apartment or

You are trying to decide whether to continue renting an apartment or to buy a house. In 20 years, you plan on leaving Canada and moving to a warm tropical island and would like to have as much money a...

See Answer

Q: Brigid Co. has the following potential project: Machine price =

Brigid Co. has the following potential project: Machine price = $1,800,000; additional inventory requirement = $150,000. Cash flows will be generated at year end. Rev1 = $400,000 and grows at 5 percen...

See Answer

Q: Calculate the NPV in Practice Problem 45 assuming a best case of

Calculate the NPV in Practice Problem 45 assuming a best case of the following: project life = 10 years; project beta = 0.8; SVn = $100,000; Rev1 = $500,000.

See Answer

Q: You have been hired as consultants to XrayGlasses Corporation (XGC).

You have been hired as consultants to XrayGlasses Corporation (XGC). XGC is in the process of deciding whether to invest in a new production facility. The new facility will enable it to produce and se...

See Answer

Q: GG Inc. has a project that requires purchases of capital assets

GG Inc. has a project that requires purchases of capital assets costing $60,000 and additional raw material inventory of $3,000. Shipping and installation costs are $1,800. GG Inc. estimated that the...

See Answer

Q: Repeat Practice Problem 48 assuming that the project would generate annual revenue

Repeat Practice Problem 48 assuming that the project would generate annual revenue of $70,000 and annual costs of $40,000 for six years. Also, assume the asset class will remain open.

See Answer

Q: Repeat Practice Problem 48 assuming that the project would generate annual revenue

Repeat Practice Problem 48 assuming that the project would generate annual revenue of $70,000 and annual costs of $40,000 for six years. Also, the asset class will be closed at the end of six years.

See Answer

Q: Calculate the NPV of the project described in Practice Problem 49,

Calculate the NPV of the project described in Practice Problem 49, but assume that the discount rate has changed based on the following information: RF = 3.4%; project beta = 1.2; the market risk prem...

See Answer

Q: The analysis of a two‐division company (DV2) has

The analysis of a two‐division company (DV2) has indicated that the beta of the entire company is 1.35. The company is 100‐percent equity funded. The company has two divisions: Major League TV (MLTV)...

See Answer

Q: You are evaluating a project for a small manufacturing firm. The

You are evaluating a project for a small manufacturing firm. The firm has provided the following data: the initial cost of the project is $2,500; the CCA rate is 10 percent; the tax rate is 25 percent...

See Answer

Q: A firm decides to enter into a lease agreement. The lease

A firm decides to enter into a lease agreement. The lease term is five years, while the economic life of the asset is six years. The annual lease payment is $12,000 at the beginning of each year, and...

See Answer

Q: AK Radio has hired a consultant to help in the assessment of

AK Radio has hired a consultant to help in the assessment of a project to launch a satellite to deliver a 24/7 infomercial radio station to the world. The satellite costs $400 million and has a CCA ra...

See Answer

Q: The CFO of CanGold Company is considering investing in a gold mine

The CFO of CanGold Company is considering investing in a gold mine in Mongolia. The mine will cost $200 million to get into production and will last for one year. At the end of one year, it is expecte...

See Answer

Q: A consultant has presented the following statement to the board of directors

A consultant has presented the following statement to the board of directors of BigCo: When comparing two mutually exclusive projects, we only need to consider the NPV. When the two projects have diff...

See Answer

Q: Consider the following scenario and calculate the beginning UCC, CCA,

Consider the following scenario and calculate the beginning UCC, CCA, ending UCC, CCA tax shield, and after-tax incremental cash flow for each and every year of the project.. Assume the asset class is...

See Answer

Q: MedCo, a large manufacturing company, currently uses a large printing

MedCo, a large manufacturing company, currently uses a large printing press in its operations and is considering two replacements: the PDX341 and PDW581. The PDX costs $400,000 and has annual maintena...

See Answer

Q: BathGate Group has just completed its analysis of a project. The

BathGate Group has just completed its analysis of a project. The CFO has presented the following information to the board of directors: The initial cost of the project is $15,000. Sales are expected t...

See Answer

Q: Solve Practice Problem 59 using EANPV.

Solve Practice Problem 59 using EANPV.

See Answer

Q: Refer to the information for BigCo Manufacturing Company in Table 1.

Refer to the information for BigCo Manufacturing Company in Table 1. BigCo is debating whether to invest in Project H (a three‐year project) or Project D. Project H has cash flow of −$3,500, $1,800, $...

See Answer

Q: Redo Practice Problem 61 using the EANPV approach.

Redo Practice Problem 61 using the EANPV approach.

See Answer

Q: Assume that SK Inc. has a capital budget of $250

Assume that SK Inc. has a capital budget of $250,000. In addition, it has the following projects for evaluation. Determine which project(s) should be chosen, assuming k is 15 percent.

See Answer

Q: Estimate the change in NI, CFO, and CFF if the

Estimate the change in NI, CFO, and CFF if the economic life of the lease described in Practice Problem 16 is seven years instead of six years.

See Answer

Q: Project X has a cost of capital of 10 percent and the

Project X has a cost of capital of 10 percent and the following cash flows: investment of $20,000 in year 0, cash inflows of $8,500, $6,400, and $11,200 in years 1, 2, and 3. a. What is the IRR? What...

See Answer

Q: Malcolm, a very junior reporter, has asked for your help

Malcolm, a very junior reporter, has asked for your help with his first article for a major national newspaper. He has provided you with the following excerpt from his article and would like your comm...

See Answer

Q: Longlife Company is considering an investment in Ponce Leon Mineral Baths.

Longlife Company is considering an investment in Ponce Leon Mineral Baths. The investment has the same risk characteristics as the firm. It is assumed that all cash flows are perpetuities and that the...

See Answer

Q: Westlake Corp. has a capital structure that has 60‐ percent

Westlake Corp. has a capital structure that has 60‐ percent debt at a cost of 10 percent and 40‐percent equity. Westlake’s stock has a beta of 1.6, market risk premium of 7 percent, and risk‐free rate...

See Answer

Q: A project has an NPV of $55,000. Calculate

A project has an NPV of $55,000. Calculate the cost of capital of this project if it generates the following cash flows for six years after an initial investment of $200,000: Year 1: $40,000 Year 2: $...

See Answer

Q: GiS Inc. has the following four projects on hand:

GiS Inc. has the following four projects on hand: Note the cash flow in the table is accumulative. Assume that R F = 5%, ER M = 12%, firm‐beta = 1.2, after‐tax cost...

See Answer

Q: You are considering buying a machine that will cost you $15

You are considering buying a machine that will cost you $15,000. There will be a maintenance cost of $1,200 at the beginning of each year, and the machine will generate cash flows of $5,750 over the n...

See Answer

Q: KRZ Company ’ s tax rate is 40 percent and the appropriate

KRZ Company ’ s tax rate is 40 percent and the appropriate discount rate is 10 percent. It is considering a project. Each asset class is large and continues after the project terminates. KRZ is not ca...

See Answer

Q: KRZ Company’s tax rate is 40 percent and the appropriate discount rate

KRZ Company’s tax rate is 40 percent and the appropriate discount rate is 8 percent. It is considering a project. Each asset class is large and continues after the project terminates. KRZ is not capit...

See Answer

Q: Java Cafe’s tax rate is 45 percent and the appropriate discount rate

Java Cafe’s tax rate is 45 percent and the appropriate discount rate is 8 percent. It is considering a project. Each asset class consists only of the project asset and will be terminated at the end of...

See Answer

Q: What are the changes in current assets, longterm assets, current

What are the changes in current assets, longterm assets, current liabilities, and long‐term liabilities at the end of the first year in Practice Problem 16?

See Answer

Q: Calculate the trailing and forward P/E ratios using the price

Calculate the trailing and forward P/E ratios using the price calculated in Practice Problem 30. Assume a 6‐percent earnings growth. (Round your answer to one decimal.)

See Answer

Q: Calculate the trailing and forward P/E ratios using the following

Calculate the trailing and forward P/E ratios using the following assumptions: RF = 3.5 percent; β = 1.12; market risk premium = 5 percent; dividends and earnings grow at 6 percent indefinitely; and t...

See Answer

Q: Calculate the market value of the firm given the following additional information

Calculate the market value of the firm given the following additional information: cost of equity is 8.25 percent, free cash flow to equity grows at 6 percent indefinitely; total debt outstanding = $1...

See Answer

Q: In Practice Problem 33, if the free cash flow to equity

In Practice Problem 33, if the free cash flow to equity grows at 8 percent for the first two years and then grows at 5 percent indefinitely, what is the market value of the firm now? (Round your answe...

See Answer

Q: ABC Inc. is planning to purchase DEF Inc. in one

ABC Inc. is planning to purchase DEF Inc. in one of two ways: (1) by paying $24 per share in cash; or (2) by giving DEF ’ s shareholders two shares in the new combined firm ABC‐DEF for each share of D...

See Answer

Q: Calculate the post‐merger EPS and market value of equity,

Calculate the post‐merger EPS and market value of equity, assuming no synergies arise in the following acquisition settled in cash. Analyze the difference, if there is any. Further,...

See Answer

Q: A firm has the option of borrowing $2.5 million

A firm has the option of borrowing $2.5 million through a 12‐year loan with monthly payments based on a 7.5-percent lending rate, or entering into a 12-year, $2.5‐ million financial lease arrangement...

See Answer

Q: A used car that currently costs $25,000 will have

A used car that currently costs $25,000 will have a market value of $8,000 in four years. As a student, you cannot afford to pay $25,000, but you want to have a car while you are going to university f...

See Answer

Q: Carla is the CEO of Superior Sausage Company (a Canadian firm

Carla is the CEO of Superior Sausage Company (a Canadian firm, listed on the Toronto Stock Exchange) and believes that the best way for the company to grow is through acquisitions. She has identified...

See Answer

Q: Why is IPO underpricing less severe in Canada than it is in

Why is IPO underpricing less severe in Canada than it is in the United States? What causes underpricing?

See Answer

Q: Expedic Utility Corp. needs to increase its electricity production capacity.

Expedic Utility Corp. needs to increase its electricity production capacity. It is interested in a slightly used reactor located in Ontario. It has been offered two alternatives: buy the reactor for $...

See Answer

Q: Explain how to create a risk-free portfolio from the stock

Explain how to create a risk-free portfolio from the stock and the option’s payoff.

See Answer

Q: What are risk-neutral probabilities?

What are risk-neutral probabilities?

See Answer

Q: Briefly describe operating LCs, revolving LCs, and term loans.

Briefly describe operating LCs, revolving LCs, and term loans.

See Answer

Q: What are the cash flow from operations and the free cash flow

What are the cash flow from operations and the free cash flow implications of an operating versus a financial lease?

See Answer

Q: Charles Zhang, the owner of a small moving company, has

Charles Zhang, the owner of a small moving company, has decided that economic conditions are perfect for him to expand his business. Such an expansion will require him to buy five new moving trucks at...

See Answer

Q: Briefly explain “the Greeks” (delta, theta, Vega

Briefly explain “the Greeks” (delta, theta, Vega, and rho) in option pricing.

See Answer

Q: How and why do we adjust the discount rate for multi-

How and why do we adjust the discount rate for multi-divisional firms?

See Answer

Q: What mistakes can occur if firms do not make the appropriate adjustments

What mistakes can occur if firms do not make the appropriate adjustments?

See Answer

Q: List and briefly describe the two types of takeovers.

List and briefly describe the two types of takeovers.

See Answer

Q: The Bynum Private Equity group has just made a tender offer for

The Bynum Private Equity group has just made a tender offer for, at most, 60 percent of Vendall Company. Vendall has 1,000 shares outstanding. Mr. VanDuun is a shareholder of Vendall and has tendered...

See Answer

Q: Sharon McKee has been appointed finance minister and is convinced that the

Sharon McKee has been appointed finance minister and is convinced that the leasing business is just a way to avoid paying taxes. She asked her department officials to evaluate the cash flow effects fo...

See Answer

Q: Describe the process of a friendly acquisition.

Describe the process of a friendly acquisition.

See Answer

Q: Briefly describe three common defensive tactics against a takeover and the difference

Briefly describe three common defensive tactics against a takeover and the difference between U.S. and Canadian practice on poison pills.

See Answer

Q: Describe the possible market price change after a tender offer and the

Describe the possible market price change after a tender offer and the possible reasons.

See Answer

Q: Describe horizontal M&A, vertical M&A, and

Describe horizontal M&A, vertical M&A, and conglomerate.

See Answer

Q: Describe the economies of scale of M&A.

Describe the economies of scale of M&A.

See Answer

Q: What is the difference between a tangible asset and an intangible asset

What is the difference between a tangible asset and an intangible asset?

See Answer

Q: In what ways is DCF capex analysis similar to valuing common shares

In what ways is DCF capex analysis similar to valuing common shares, and in what ways is it different?

See Answer

Q: Describe the financing synergies of M&A.

Describe the financing synergies of M&A.

See Answer

Q: Describe the empirical evidence on the actual gains or losses resulting from

Describe the empirical evidence on the actual gains or losses resulting from mergers.

See Answer

Q: Describe the purchase method in accounting for acquisitions.

Describe the purchase method in accounting for acquisitions.

See Answer

Q: Identify the costs and benefits of holding inventory.

Identify the costs and benefits of holding inventory.

See Answer

Q: State four main motives firms have to hold cash.

State four main motives firms have to hold cash.

See Answer

Q: Use two different methods to calculate ROI given the following information for

Use two different methods to calculate ROI given the following information for Brandon Corp. Sales = $400,000 ; Cost of goods sold = $120,000; Interest cost = $20,000; TC = 25%. The firm ’ s debt (B =...

See Answer

Q: Calculate the fixed burden coverage and cashflow- to-debt ratio

Calculate the fixed burden coverage and cashflow- to-debt ratio given the following: EBIT = $680,000; depreciation and amortization = $66,000; preferred dividend = $40,000; sinking fund payments = $15...

See Answer

Q: Calculate Altman’ s Z score for Home Depot in fiscal year 2011

Calculate Altman’ s Z score for Home Depot in fiscal year 2011 (as of January 29, 2012) and then compare it with Moody ’ s rating chart. Is this company an IG or no...

See Answer

Q: Explain the elements of Altman’s Z score as used in Practice Problem

Explain the elements of Altman’s Z score as used in Practice Problem 22. Problem 22. Calculate Altman’s Z score for Home Depot in fiscal year 2011 (as of January 29...

See Answer

Q: Calculate the EPS indifference EBIT* level given the following information.

Calculate the EPS indifference EBIT* level given the following information. The corporate tax rate is 20 percent. Under a 75‐percent D/E ratio, the number of common shares outstanding is 30,000; pre‐t...

See Answer

Q: In the M&M no‐tax world, calculate the

In the M&M no‐tax world, calculate the value of the levered firm (V L ). Cost of unlevered equity (KU) = 16%; cost of debt (KD) = 8%; debt (D) = $500,000; and NI = $480,000. What is the cost of levere...

See Answer

Q: With reference to the M&M irrelevance theorem, calculate the

With reference to the M&M irrelevance theorem, calculate the market value of an unlevered firm (U) and an identical risk‐levered firm (L). The expected EBIT of U $2.1 million, which will remain consta...

See Answer

Q: In the M&M tax world, what is the value

In the M&M tax world, what is the value of levered firm (V L ) in Practice Problem 25 if the tax rate is 30 percent? What is the cost of levered equity? What is the market risk premium (MRP) given U 1...

See Answer

Q: In the M&M tax world, calculate the value of

In the M&M tax world, calculate the value of the unlevered firm (U) and the identical risk-levered firm (L). Corporate tax rate = 30%; perpetual EBIT for U and L = $3 million; cost of capital of U = 1...

See Answer

Q: What important issues does the static trade‐off model ignore?

What important issues does the static trade‐off model ignore?

See Answer

Q: What are captive finance companies?

What are captive finance companies?

See Answer

Q: Describe the factors that can affect a firm ’ s capital structure

Describe the factors that can affect a firm ’ s capital structure in practice.

See Answer

Q: State the two possible ways bankruptcy can occur. What is the

State the two possible ways bankruptcy can occur. What is the role of a monitor appointed by the court under the Companies’ Creditors Arrangement Act ( CCAA )? Compare the difference of recognizing fi...

See Answer

Q: Describe the relationship between the debt ratio and firm value when we

Describe the relationship between the debt ratio and firm value when we consider the existence of bankruptcy costs. How do you view the agency costs when bankruptcy occurs?

See Answer

Q: Gus Fitzgerald, a local shipping tycoon, is very confused.

Gus Fitzgerald, a local shipping tycoon, is very confused. He has issued stock to finance a positive NPV investment. He expected the stock price to rise, as positive NPV projects are supposed to incre...

See Answer

Q: Your cousin has just started his MBA and is confused. He

Your cousin has just started his MBA and is confused. He understands that without taxes, capital structure is irrelevant. He also understands that with taxes, firms should use 100‐percent debt. Howeve...

See Answer

Q: Define business risk and financial risk.

Define business risk and financial risk.

See Answer

Q: How does financial leverage affect the relationship between ROI and ROE?

How does financial leverage affect the relationship between ROI and ROE?

See Answer

Q: What are the three rules of leverage?

What are the three rules of leverage?

See Answer

Q: Describe how we determine the ROE and EPS indifference points for a

Describe how we determine the ROE and EPS indifference points for a firm based on various financing alternatives, and explain why this analysis provides the firm with useful information.

See Answer

Q: Explain the impact of financial distress and agency costs on M&

Explain the impact of financial distress and agency costs on M&M’s conclusions regarding capital structure.

See Answer

Q: What are some of the advantages of carrying inventories?

What are some of the advantages of carrying inventories?

See Answer

Q: Why can the firm’s debt be viewed as the exercise price to

Why can the firm’s debt be viewed as the exercise price to the shareholders’ option to purchase the firm?

See Answer

Q: Explain the static trade-off theory.

Explain the static trade-off theory.

See Answer

Q: Explain how ratios may be used to assess a company’s ability to

Explain how ratios may be used to assess a company’s ability to assume more debt.

See Answer

Q: What is Altman’s Z score and what does it measure?

What is Altman’s Z score and what does it measure?

See Answer

Q: A firm is going to finance a new project 100 percent with

A firm is going to finance a new project 100 percent with debt, through a new bond issue. Since the firm is using only debt to finance the project, the NPV of the project should be calculated using th...

See Answer

Q: Montreal Brokers, a small brokerage firm and PEI tronics, a

Montreal Brokers, a small brokerage firm and PEI tronics, a software development company, are both separately considering developing and marketing a new software package. Neither party is aware that t...

See Answer

Q: Suppose a firm uses a constant WACC to calculate the NPV of

Suppose a firm uses a constant WACC to calculate the NPV of all of its capital budgeting projects, rather than adjusting for the risk of the individual projects. What errors will the firm make in its...

See Answer

Q: Why is the earnings yield not usually an adequate measure of the

Why is the earnings yield not usually an adequate measure of the investor’s required return on equity?

See Answer

Q: How are the ROE and Ke related to a firm’s growth opportunities

How are the ROE and Ke related to a firm’s growth opportunities and its M/B ratio?

See Answer

Q: How can we relate the existence of multiple growth stages to four

How can we relate the existence of multiple growth stages to four commonly used firm classifications?

See Answer

Q: What are some of the disadvantages of carrying inventories?

What are some of the disadvantages of carrying inventories?

See Answer

Q: Describe the Fed model and how it may be used to estimate

Describe the Fed model and how it may be used to estimate the required rate of return of the market as a whole.

See Answer

Q: How do flotation costs affect the cost of capital sources for a

How do flotation costs affect the cost of capital sources for a firm?

See Answer

Q: Explain how to estimate the cost of debt and preferred equity for

Explain how to estimate the cost of debt and preferred equity for a firm.

See Answer

Q: A firm has the following capital structure based on market values:

A firm has the following capital structure based on market values: equity 60 percent and debt 40 percent. The current yield on government T‐bills is 3 percent, the expected return on the market portfo...

See Answer

Q: A firm has the following balance sheet items: /

A firm has the following balance sheet items: The before‐tax interest cost on new 15‐year debt would be 7 percent, and each $1,000 bond would net the firm $972 afte...

See Answer

Q: A company can issue new 20‐year bonds at par that

A company can issue new 20‐year bonds at par that pay 6‐percent annual coupons. The net proceeds to the firm (after taxes) will be 96 percent of par value. They estimate that new preferred shares prov...

See Answer

Q: a. Kitchener Consumer Products plans to issue 25‐year bonds

a. Kitchener Consumer Products plans to issue 25‐year bonds with an 7‐percent coupon rate, with coupons paid semi‐annually and a par value of $1,000. After tax flotation costs (issuing and underwritin...

See Answer

Q: A firm wishes to raise funds in the following proportions: 20

A firm wishes to raise funds in the following proportions: 20‐percent debt, 20‐percent P/S, and 60‐percent CE (common equity). Assume the cost of internally generated funds is 15 percent. Annual after...

See Answer

Q: Explain how we can use the constant growth DDM to estimate the

Explain how we can use the constant growth DDM to estimate the cost of firms’ internal common equity, as well as the cost of new common share issues.

See Answer

Q: Explain the relationship among ROE, retention rates, and firm growth

Explain the relationship among ROE, retention rates, and firm growth.

See Answer

Q: Describe the four inventory management approaches.

Describe the four inventory management approaches.

See Answer

Q: Name and discuss the four criteria used by DBRS to classify a

Name and discuss the four criteria used by DBRS to classify a security as debt versus equity.

See Answer

Q: Define the following types of hybrids: income bonds, commodity bonds

Define the following types of hybrids: income bonds, commodity bonds, real return bonds, original issue discount bonds, LYONs, ARCs, preferred securities, and COINS.

See Answer

Q: Relate the costs of various financing options to their equity-like

Relate the costs of various financing options to their equity-like characteristics.

See Answer

Q: State the three basic rights of common shareholders.

State the three basic rights of common shareholders.

See Answer

Q: 1. Which of the following does not appear in the share

1. Which of the following does not appear in the share structure of a firm? a. Preferred shares b. Common shares c. Limited voting shares d. None of the above 2. Which of the following statements abo...

See Answer

Q: Discuss three reasons why firms issue preferred shares.

Discuss three reasons why firms issue preferred shares.

See Answer

Q: A firm has just filed for bankruptcy and is likely to be

A firm has just filed for bankruptcy and is likely to be liquidated. The creditors, such as equipment suppliers and employees, are owed $1.5 million. a. Determine how much the equity holders will rece...

See Answer

Q: With the savings from your summer job you were able to buy

With the savings from your summer job you were able to buy 500 shares of a hot new Internet company last year. A few months after your purchase, the company was low on cash and needed to raise more eq...

See Answer

Q: When a firm needed to raise capital to expand, the founder

When a firm needed to raise capital to expand, the founder was concerned about losing control of the firm if he sold too many shares. The solution devised by his investment banker was to create two di...

See Answer

Q: A firm has 50 million common shares outstanding, on which it

A firm has 50 million common shares outstanding, on which it pays a quarterly dividend of $0.25 per share. The firm ’ s capital structure also includes two million cumulative preferred shares with a $...

See Answer

Q: Explain how commercial paper differs from bankers’ acceptances.

Explain how commercial paper differs from bankers’ acceptances.

See Answer

Q: Collingwood Corp. has decided to invest some of its excess cash

Collingwood Corp. has decided to invest some of its excess cash in straight preferred shares issued by other companies. It will earn a yield of 4.5 percent on the $10‐million investment. How much net...

See Answer

Q: The common shares of a firm are currently trading at $15

The common shares of a firm are currently trading at $15, while its preferred shares trade at par (see Practice Problem 13). Calculate the conversion premium on the preferred shares. What does this pr...

See Answer

Q: What are the differences between call options and warrants?

What are the differences between call options and warrants?

See Answer

Q: Calculate the conversion price and conversion value of the convertible bonds given

Calculate the conversion price and conversion value of the convertible bonds given the following: selling price $102; each bond is convertible into 5 common shares; current common share price $18. Wil...

See Answer

Q: Jack and Jill Inc. very nearly tumbled into bankruptcy last year

Jack and Jill Inc. very nearly tumbled into bankruptcy last year. To refinance the firm, the firm issued $30 million worth of 30‐year income bonds. These bonds have an 8‐percent coupon that is payable...

See Answer

Q: In December 2015, Collingwood Corp. decided to issue 100,

In December 2015, Collingwood Corp. decided to issue 100,000 convertible bonds, maturing in December 2025. The bonds have a face value of $1,000 and promise an annual coupon payment of 5.75 percent. T...

See Answer

Q: Calculate the payoff of fully exercising warrants given the following: 950

Calculate the payoff of fully exercising warrants given the following: 950,000 existing shares are outstanding; 150,000 warrants are outstanding and are exercisable at $10. The firm is valued at $10 m...

See Answer

Q: Orion’s Belt Mining Co. has 12 million common shares outstanding,

Orion’s Belt Mining Co. has 12 million common shares outstanding, which are currently trading for $5. In addition, the company has issued two million share purchase warrants with a strike price of $4....

See Answer

Q: A firm’s common shares currently trade at $20 per share.

A firm’s common shares currently trade at $20 per share. The firm has warrants outstanding that entitle the holder to purchase two shares at an exercise price of $18 per share. The expiry date is two...

See Answer

Q: Straight preferred shares issued by a firm have a discount rate of

Straight preferred shares issued by a firm have a discount rate of 8 percent per year, whereas these shares are yielding 4 percent on the $25 par value. The conversion value of these shares is calcula...

See Answer

Q: Montreal Brewers is going to issue $100 million of 90‐

Montreal Brewers is going to issue $100 million of 90‐day commercial paper for net proceeds of $99 million. Montreal Brewers must maintain a $100 million credit line, on which it must pay a standby fe...

See Answer

Q: A firm has just issued convertible preferred shares with a $100

A firm has just issued convertible preferred shares with a $100 par value. The conversion price for these shares is $20 (per common share). What is the conversion ratio?

See Answer

Q: A firm has just issued convertible preferred shares with a call feature

A firm has just issued convertible preferred shares with a call feature that permits the firm to repurchase the shares at par value (or, in effect, force the conversion into common shares). Usually, t...

See Answer

Q: Why are preferred shares sometimes called hybrid securities?

Why are preferred shares sometimes called hybrid securities?

See Answer

Q: What are the basic rights associated with equity securities? How do

What are the basic rights associated with equity securities? How do these differ across different categories or classes of equities?

See Answer

Q: Why do voting rights affect the prices of some common shares and

Why do voting rights affect the prices of some common shares and not others?

See Answer

Q: Why is dividend income preferred by both corporations and individual investors?

Why is dividend income preferred by both corporations and individual investors?

See Answer

Q: Explain why issuing debt or preferred shares with warrants attached or issuing

Explain why issuing debt or preferred shares with warrants attached or issuing convertible bonds or convertible preferred shares, may represent attractive sources of financing for higher-risk firms.

See Answer

Q: Define and explain how to determine the following for a convertible:

Define and explain how to determine the following for a convertible: conversion price, conversion value, straight bond value, floor value, and convertible premium.

See Answer

Q: Briefly describe the following types of preferred shares: straight, retractable

Briefly describe the following types of preferred shares: straight, retractable, and floating rate.

See Answer

Q: a. Calculate the effective annual cost of forgoing the discount from

a. Calculate the effective annual cost of forgoing the discount from credit terms of 2/15 net 60. The selling price is $800. b. Another supplier offers $820 on credit terms of net 90. If you could fin...

See Answer

Q: Why would you want a cumulative feature when purchasing preferred shares?

Why would you want a cumulative feature when purchasing preferred shares?

See Answer

Q: 1. Which of the following statements is false? a

1. Which of the following statements is false? a. Financing total assets is called the financial structure decision. b. Capital structure is how invested capital is financed. c. The financial structur...

See Answer

Q: Explain how we can use the CAPM to estimate the cost of

Explain how we can use the CAPM to estimate the cost of common equity.

See Answer

Q: Explain why beta estimates are “period specific” and outline the

Explain why beta estimates are “period specific” and outline the potential problems that may arise. Allude to problems with recent beta estimates.

See Answer

Q: Explain the importance of using the WACC as a hurdle rate for

Explain the importance of using the WACC as a hurdle rate for making investment decisions.

See Answer

Q: Why does the MCC suddenly jump up and become expensive?

Why does the MCC suddenly jump up and become expensive?

See Answer

Q: What is the cost of equity (K e ) given RF

What is the cost of equity (K e ) given RF 3%, beta ( ) 1.4, expected market return ( ERM) 10%?

See Answer

Q: What is the market price and market‐to‐book ratio

What is the market price and market‐to‐book ratio, assuming the firm’s stock is a perpetuity and all earnings are paid out as cash dividends (i.e., the retention rate is zero)?

See Answer

Q: State the two rules of financial leverage.

State the two rules of financial leverage.

See Answer

Q: In order for the M&M irrelevance theorem to hold,

In order for the M&M irrelevance theorem to hold, what key assumptions must be met?

See Answer

Q: What are special purpose vehicles (SPVs)? What is the main

What are special purpose vehicles (SPVs)? What is the main advantage of SPVs? List a few forms of credit enhancement that are critical to SPVs.

See Answer

Q: Explain how the static trade‐off model can be used to

Explain how the static trade‐off model can be used to find an optimal capital structure.

See Answer

Q: What is the pecking order according to Myers’ argument?

What is the pecking order according to Myers’ argument?

See Answer

Q: Summarize the main factors you need to consider if the CFO of

Summarize the main factors you need to consider if the CFO of your firm asks you to evaluate your firm’s capital structure.

See Answer

Q: Explain four of the most important factors influencing capital structure decisions as

Explain four of the most important factors influencing capital structure decisions as indicated in the survey results and how they relate to the conceptual discussion of an optimal capital structure....

See Answer

Q: Distinguish between operating and financial leverage.

Distinguish between operating and financial leverage.

See Answer

Q: Why do we say that equity holders bear the brunt of the

Why do we say that equity holders bear the brunt of the effects of leverage?

See Answer

Q: Why is the weighted average cost of capital (WACC) so

Why is the weighted average cost of capital (WACC) so important?

See Answer

Q: What are the steps involved in estimating a firm’s WACC?

What are the steps involved in estimating a firm’s WACC?

See Answer

Q: How can we estimate the market value of common equity, preferred

How can we estimate the market value of common equity, preferred equity, and long-term debt?

See Answer

Q: In the M&M no‐tax world, an unlevered

In the M&M no‐tax world, an unlevered firm has a cost of equity of 12 percent and expected EBIT of $480,000. The firm decided to issue $3 million of debt at a cost of 8 percent to finance a project, w...

See Answer

Q: Suppose Sio Inc. has 45 days of accounts receivable (AR

Suppose Sio Inc. has 45 days of accounts receivable (AR) of $900,000 on its books. A factor offers a 45‐day AR loan equal to 90 percent of AR. The quoted interest rate is 6 percent, and there is a com...

See Answer

Q: Susan and Celia are twins but have very different attitudes toward debt

Susan and Celia are twins but have very different attitudes toward debt. Susan believes that firms should have a D/E ratio of 0.2 while Celia believes that the D/E ratio should be 1.1. Both sisters ha...

See Answer

Q: The Saskatchewan Botanicals Company expects a free cash flow of $1

The Saskatchewan Botanicals Company expects a free cash flow of $1.08 million every year forever. Saskatchewan Botanicals currently has no debt, and its cost of equity is 18 percent. The corporate tax...

See Answer

Q: Athabascan Drilling is currently unlevered and is valued at $10 million

Athabascan Drilling is currently unlevered and is valued at $10 million. The company is considering including debt in its capital structure and wants to know the likely impact on its value and cost of...

See Answer

Q: Straightforward Theatre Company has an EBIT of $1.2 million

Straightforward Theatre Company has an EBIT of $1.2 million per year. The WACC of the firm is 10 percent and the before‐tax cost of debt is 5 percent. The debt is risk free and all cash flows are perp...

See Answer

Q: Calculate invested capital and before‐tax ROI.

Calculate invested capital and before‐tax ROI.

See Answer

Q: Provide two reasons why the cost of a security to a company

Provide two reasons why the cost of a security to a company differs from its required return in capital markets.

See Answer

Q: A firm’s market values of equity and debt are $750,

A firm’s market values of equity and debt are $750,000 and $250,000, respectively. The before-tax cost of debt 6%; RF 3%; b eta ( ) 1.08; the market risk premium 8%; and the tax rate 25%. Calculate th...

See Answer

Q: What is V Fed if the expected earnings per share on the

What is V Fed if the expected earnings per share on the S&P 500 is $23.50 and the long‐term U.S. bond rate is 4.75 percent?

See Answer

Q: Rocky Mountain Depot just announced its EPS of $5. Retention

Rocky Mountain Depot just announced its EPS of $5. Retention rate (b) 0.4. The earnings are expected to grow at 10 percent for one year and then at 5 percent indefinitely. Given that Ke 16 percent, wh...

See Answer

Q: Calculate the cost of issuing new equity for a firm, assuming

Calculate the cost of issuing new equity for a firm, assuming issue costs are 6 percent of the share price after taxes; market price per share $50; current dividend $3.75; and the constant growth rate...

See Answer

Q: What is the difference between a bank operating line of credit and

What is the difference between a bank operating line of credit and a traditional loan?

See Answer

Q: What is the effective annual cost if a firm issues $5

What is the effective annual cost if a firm issues $5 million face value of 90‐day commercial paper for net proceeds of $4.85 million? The firm pays a standby fee of 0.1 percent on the face value.

See Answer

Q: A firm has common shares outstanding with a discount rate of 10

A firm has common shares outstanding with a discount rate of 10.5 percent. The current market price is $25. The company just paid a dividend of $1.20 per share. What is the per‐share implied growth ra...

See Answer

Q: A firm’s earnings and dividends are expected to grow at a constant

A firm’s earnings and dividends are expected to grow at a constant rate indefinitely, and it is expected to pay a dividend of $9 per share next year. Expected EPS and BVPS next year are $12 and $50, r...

See Answer

Q: Calculate the cost of equity using the constant growth DDM given the

Calculate the cost of equity using the constant growth DDM given the following: current dividend $3; payout ratio 0.5 (assume it is not changing); ROE 12%; and the current market price of the stock $2...

See Answer

Q: Calculate PVGO, PVEO, and P 0 given the following information

Calculate PVGO, PVEO, and P 0 given the following information: ROE1 15%; ROE2 20%; further investment (Inv) $200; BVPS $25; and Ke 12%. Is this firm a star? If not, what is it according to Boston Cons...

See Answer

Q: What are the main determinants of capital structure?

What are the main determinants of capital structure?

See Answer

Q: 1. What is the invested capital given the following? Accounts

1. What is the invested capital given the following? Accounts receivable = $50,000; current assets = $200,000; total assets = $700,000; shareholders’ equity = $450,000; accounts payable = $10,000; sho...

See Answer

Q: Explain how the existence of informational asymmetries and agency problems may lead

Explain how the existence of informational asymmetries and agency problems may lead firms to follow a pecking order to financing.

See Answer

Q: Calculate ROE and ROI given the following./

Calculate ROE and ROI given the following.

See Answer

Q: Explain how and under what assumptions M&M show that dividends

Explain how and under what assumptions M&M show that dividends are irrelevant.

See Answer

Q: Explain the relationship between M&M’s argument and the use of

Explain the relationship between M&M’s argument and the use of a residual dividend policy.

See Answer

Q: There are two suppliers of one input for a factory. Supplier

There are two suppliers of one input for a factory. Supplier A offers a selling price of $1,000 with terms of 1/10 net 30, while Supplier B offers $1,100 with 3/10 net 60. Which supplier offers the lo...

See Answer

Q: Briefly describe the notion of homemade dividends as it relates to M

Briefly describe the notion of homemade dividends as it relates to M&M’s irrelevancy argument.

See Answer

Q: 1. Dividend‐payout ratio is defined as: a

1. Dividend‐payout ratio is defined as: a. the dividend yield plus the capital gains yield. b. dividends per share divided by earnings per share. c. dividends per share divided by income per share. d....

See Answer

Q: A firm has a dividend yield of 3.8 percent and

A firm has a dividend yield of 3.8 percent and a payout ratio of 36 percent. If its earnings are $22 million and there are 6 million shares outstanding, what is the price per share?

See Answer

Q: What is the market price per share if the next period’s dividend

What is the market price per share if the next period’s dividend = $2.50, P 1 = $30, and K = 16%?

See Answer

Q: According to equity market capitalization, what is the cost of capital

According to equity market capitalization, what is the cost of capital for the stock of the following firm? Current market value of the equity is $1.8 million with 150,000 shares outstanding. The stoc...

See Answer

Q: Explain why dividend policy will be relevant in the presence of transactions

Explain why dividend policy will be relevant in the presence of transactions costs, informational asymmetry and agency problems, and taxes.

See Answer

Q: Describe split shares, and explain what their popularity implies about investor

Describe split shares, and explain what their popularity implies about investor preferences for dividends in the real world.

See Answer

Q: Currently a firm has an operating cash flow of $42 million

Currently a firm has an operating cash flow of $42 million and there is a promising project available, which costs $30 million. There are 5 million shares outstanding with a current price of $64 per s...

See Answer

Q: A firm follows a strict residual dividend policy. This firm will

A firm follows a strict residual dividend policy. This firm will have profits of $800,000 this year. After screening all available investment projects, the firm has decided to take three out of the 10...

See Answer

Q: Explain why firms do not simply pay out dividends as a fixed

Explain why firms do not simply pay out dividends as a fixed portion of their profits. What do most firms do in terms of dividend policy?

See Answer

Q: A company receives an average of $100,000 in cheques

A company receives an average of $100,000 in cheques per day from its customers. It takes the company an average of five days to receive and deposit these cheques. The company is considering a lockbox...

See Answer

Q: Investor A ’ s personal tax rate is 30 percent while Investor

Investor A ’ s personal tax rate is 30 percent while Investor B ’ s is 22 percent. Investor A owns 1,000 shares of SNS Company and receives an annual dividend of $1.75 per share. Investor B owns 1,000...

See Answer

Q: What is the relationship between the break-even sales growth rate

What is the relationship between the break-even sales growth rate and a firm’s collection policy, payables policy, and inventory policy?

See Answer

Q: Why does cash flow from operations increase if the firm speeds up

Why does cash flow from operations increase if the firm speeds up the collection of receivables, delays paying its bills, or increases its inventory turnover ratio?

See Answer

Q: 1. Which of the following is not a warning sign of

1. Which of the following is not a warning sign of potential liquidity problems? a. Declines in working capital and daily cash flows b. Increases in accounts receivable and longer collection periods c...

See Answer

Q: What is the difference between profit and cash flow from operations?

What is the difference between profit and cash flow from operations?

See Answer

Q: Why should all firms prepare a cash budget?

Why should all firms prepare a cash budget?

See Answer

Q: Explain how trade credit allows firms to use their suppliers as sources

Explain how trade credit allows firms to use their suppliers as sources of short‐term funds.

See Answer

Q: Why do firms hold cash?

Why do firms hold cash?

See Answer

Q: What is float and why is it important to the firm?

What is float and why is it important to the firm?

See Answer

Q: ABC Inc. currently grants no credit, but it is considering

ABC Inc. currently grants no credit, but it is considering offering new credit terms of net 30. As a result, the price of its product will increase by $2 per unit. The original price per unit is $50....

See Answer

Q: Suppose that ABC Inc. (see Practice Problem 30) switches

Suppose that ABC Inc. (see Practice Problem 30) switches to 3/10 net 30 from net 30. It is 80 percent of customers will take advantage of the discount, while the remaining 20 percent will pay on day 3...

See Answer

Q: EastShore Inc. has an ACP of 60 days and daily credit

EastShore Inc. has an ACP of 60 days and daily credit sales of $75,000. A factor offers a 60‐day accounts receivable loan equal to 90 percent of accounts receivable. The quoted interest rate is 10 per...

See Answer

Q: ABC Inc. grants credit terms of net 25. It is

ABC Inc. grants credit terms of net 25. It is considering a new policy that involves more stringent credit terms: net 20. As a result, the price of its product will stay the same at $45. The expected...

See Answer

Q: Calculate the effective annual cost of issuing 270‐day BAs at

Calculate the effective annual cost of issuing 270‐day BAs at a quoted rate of 5 percent with a face value of $10 million. The bank charges a 0.6 percent stamping fee.

See Answer

Q: A firm engaged a one‐year, monthly pay, $

A firm engaged a one‐year, monthly pay, $100,000 line of credit at 9 percent plus a 0.25 percent commitment fee on the unused portion of the line. The firm used 75 percent of the line for the first ha...

See Answer

Q: Calculate the effective annual cost of a one‐year $2

Calculate the effective annual cost of a one‐year $2‐million operating line of credit. The firm borrowed $1.2 million for the first four months of the year and reduced the loan amount to $500,000 for...

See Answer

Q: What additional services does a factor provide over a bank?

What additional services does a factor provide over a bank?

See Answer

Q: What is the effective annual cost if a firm issues $10

What is the effective annual cost if a firm issues $10 million of 180‐day BAs at a quoted rate of 5.5 percent, and the bank charges it a 0.4 percent stamping fee? Compare the effective annual cost of...

See Answer

Q: Why is trade credit different from bank credit?

Why is trade credit different from bank credit?

See Answer

Q: What are the four C’s of credit?

What are the four C’s of credit?

See Answer

Q: What does 2/10 net 30 mean, and what is

What does 2/10 net 30 mean, and what is the implicit interest cost?

See Answer

Q: What is an aged accounts receivable report?

What is an aged accounts receivable report?

See Answer

Q: What are the limitations of the current ratio and the quick ratio

What are the limitations of the current ratio and the quick ratio as measures of working capital management?

See Answer

Q: What are the operating cycle and the cash conversion cycle, and

What are the operating cycle and the cash conversion cycle, and how are they related to working capital policy?

See Answer

Q: What are three major sources of float? What are some common

What are three major sources of float? What are some common methods that address float?

See Answer

Q: Explain the function of a factor in working capital management.

Explain the function of a factor in working capital management.

See Answer

Q: What is the purpose of credit analysis and how is it accomplished

What is the purpose of credit analysis and how is it accomplished?

See Answer

Q: What is the difference between a BA and commercial paper?

What is the difference between a BA and commercial paper?

See Answer

Q: When a firm is deciding whether or not to extend credit to

When a firm is deciding whether or not to extend credit to an applicant, what two things need to be established about the applicant?

See Answer

Q: Habitant Maple Syrup Sweets Company just issued $25 million of 180

Habitant Maple Syrup Sweets Company just issued $25 million of 180‐day commercial paper for net proceeds of $24.5 million. What is the commercial paper’s quoted yield?

See Answer

Q: Why is the efficient utilization of net working capital important?

Why is the efficient utilization of net working capital important?

See Answer

Q: Describe the types of cash inflows and outflows in a cash budget

Describe the types of cash inflows and outflows in a cash budget.

See Answer

Q: What is the connection between cash budgets and pro forma financial statements

What is the connection between cash budgets and pro forma financial statements?

See Answer

Q: For 2012, Ontario Manufacturing Company provided the following accounting information:

For 2012, Ontario Manufacturing Company provided the following accounting information: a. Which of the above are sources of funds? b. Which of the above are uses of funds? c. What is the overall incre...

See Answer

Q: Calculate the ending cash balance of the current month for the following

Calculate the ending cash balance of the current month for the following opening cash balance and current month’s cash flows:

See Answer

Q: Would each of the following changes increase or decrease the break‐

Would each of the following changes increase or decrease the break‐even sales growth rate? a. An increase in the amount of inventory held b. An increase in the proportion of invoices that are paid imm...

See Answer

Q: a. What can you conclude if a firm’s planned sales growth

a. What can you conclude if a firm’s planned sales growth exceeds its break‐even sales growth rate? b. What can you conclude if a firm’s planned sales growth is less than its break‐even sales growth r...

See Answer

Q: Explain the difference between the receivables turnover ratio and the average collection

Explain the difference between the receivables turnover ratio and the average collection period (ACP).

See Answer

Q: What are the drawbacks to using the turnover ratio to measure inventory

What are the drawbacks to using the turnover ratio to measure inventory policy?

See Answer

Q: Discuss measures businesses can implement to improve working capital management.

Discuss measures businesses can implement to improve working capital management.

See Answer

Q: Explain the difference between the operating cycle (OC) and the

Explain the difference between the operating cycle (OC) and the cash conversion cycle (CCC).

See Answer

Q: Explain the difference between the quick ratio and the current ratio.

Explain the difference between the quick ratio and the current ratio.

See Answer

Q: Great Northern Manufacturing presently has net working capital of $100,

Great Northern Manufacturing presently has net working capital of $100,000 and sales of $125,000. It is considering entering into a new project that would increase next year’s sales by $15,000. The pr...

See Answer

Q: Manitoba Services is considering undertaking a new order that would cause its

Manitoba Services is considering undertaking a new order that would cause its average days of revenues in payables (ADRP) to decrease from 58 days to 48 days, while its average collection period (ACP)...

See Answer

Q: Determine the operating cycle and cash conversion cycle for a company with

Determine the operating cycle and cash conversion cycle for a company with inventory turnover of 6.25 times per year, receivables turnover of 8 times per year, and an average days of revenues in payab...

See Answer

Q: Use the information in the financial statements below to answer the following

Use the information in the financial statements below to answer the following. a. Calculate the current ratio, quick ratio, and net working capital. b. Calculate the inventory turnover (using revenues...

See Answer

Q: Christmas Inc. is a wholesaler of Christmas decorations and wrapping paper

Christmas Inc. is a wholesaler of Christmas decorations and wrapping paper. It is a seasonal business and, due to the timing of cash inflows and outflows, it frequently experiences a cash shortfall in...

See Answer

Q: A firm has estimated its sales, purchases from suppliers, and

A firm has estimated its sales, purchases from suppliers, and wages and miscellaneous operating cash outlays for the first four months of next year as follows: It estimates that 50 percent of its sale...

See Answer

Q: A firm collects 60 percent of its monthly sales immediately and the

A firm collects 60 percent of its monthly sales immediately and the rest a month later; its production costs are 80 percent of sales, and it holds two months of sales in inventory. It pays 40 percent...

See Answer

Q: What is the cost of 3/15 net 60 trade credit

What is the cost of 3/15 net 60 trade credit?

See Answer

Q: Atlantic Transport collects 75 percent of its monthly sales immediately and the

Atlantic Transport collects 75 percent of its monthly sales immediately and the rest at the end of the month; has production costs that are 60 percent of sales; pays 50 percent of its bills immediatel...

See Answer

Q: MB Corporation has a receivables turnover of 10, an inventory turnover

MB Corporation has a receivables turnover of 10, an inventory turnover of 15, and a payables turnover of 5. Calculate its cash conversion cycle. What does a negative cash conversion cycle tell you abo...

See Answer

Q: Explain the “bird in the hand” argument about dividends.

Explain the “bird in the hand” argument about dividends.

See Answer

Q: Reconcile the predictions of M&M with Gordon’s arguments about dividend

Reconcile the predictions of M&M with Gordon’s arguments about dividend policy.

See Answer

Q: State the characteristics of sound net working capital management.

State the characteristics of sound net working capital management.

See Answer

Q: How is net working capital calculated?

How is net working capital calculated?

See Answer

Q: Calculate the cash conversion cycle (CCC) for the company examined

Calculate the cash conversion cycle (CCC) for the company examined in questions 4 to 6. Round to the nearest day.

See Answer

Q: Why can share repurchases be viewed as an alternative to paying a

Why can share repurchases be viewed as an alternative to paying a cash dividend?

See Answer

Q: What factors may influence a firm’s decision to enter into share repurchases

What factors may influence a firm’s decision to enter into share repurchases?

See Answer

Q: Describe possible reactions from the market of the following dividend payout changes

Describe possible reactions from the market of the following dividend payout changes. a. Dividend initiation b. Dividend increase c. Dividend decrease

See Answer

Q: Why do securitizations require credit enhancements?

Why do securitizations require credit enhancements?

See Answer

Q: A company has announced an increase in its quarterly dividend from $

A company has announced an increase in its quarterly dividend from $0.30 to $0.42 per share. If an investor who owns 1,500 shares is in the 25 percent tax bracket, calculate the amount by which the in...

See Answer

Q: List the main reasons why firms repurchase shares.

List the main reasons why firms repurchase shares.

See Answer

Q: What obvious question arises when we examine historical patterns in aggregate dividend

What obvious question arises when we examine historical patterns in aggregate dividend payouts?

See Answer

Q: What obvious question arises when we examine cross-sectional patterns in

What obvious question arises when we examine cross-sectional patterns in the dividend payouts of individual companies?

See Answer

Q: Define four important dates that arise with respect to dividend payments.

Define four important dates that arise with respect to dividend payments.

See Answer

Q: Explain the similarities and differences of DRIPs, stock dividends, and

Explain the similarities and differences of DRIPs, stock dividends, and stock splits.

See Answer

Q: What does real-world evidence imply about how firms manage their

What does real-world evidence imply about how firms manage their dividend payments?

See Answer

Q: Describe the difference between a stock dividend and a cash dividend plus

Describe the difference between a stock dividend and a cash dividend plus a DRIP.

See Answer

Q: Explain the implications of M&M’s homemade dividend argument.

Explain the implications of M&M’s homemade dividend argument.

See Answer

Q: Discuss the “bird in the hand” argument in support of

Discuss the “bird in the hand” argument in support of dividend payments.

See Answer

Q: Explain the transactions motive.

Explain the transactions motive.

See Answer

Q: According to John Lintner, what is the adjustment factor ß?

According to John Lintner, what is the adjustment factor ß? Interpret the calculated adjustment factor.

See Answer

Q: How do personal taxes affect preference for high versus low dividend yields

How do personal taxes affect preference for high versus low dividend yields?

See Answer

Q: Describe the implication of releasing the assumptions of no transactions costs.

Describe the implication of releasing the assumptions of no transactions costs.

See Answer

Q: CGC Company is considering its dividend policy. Currently CGC pays no

CGC Company is considering its dividend policy. Currently CGC pays no dividends, has cash flows from operations of $10 million per year (perpetual), and needs $8 million for capital expenditures. The...

See Answer

Q: A firm has one million shares outstanding. After‐tax earnings

A firm has one million shares outstanding. After‐tax earnings have been constant at $8 per share. The firm pays out all earnings in dividends at the end of each year. The shareholders’ required rate o...

See Answer

Q: Assume that the shareholders of a firm pay a net tax of

Assume that the shareholders of a firm pay a net tax of 30 percent on cash dividends received. After‐tax earnings have been constant at $10 per share. The firm pays out all earnings in dividends at th...

See Answer

Q: A firm’s next‐period market value of equity is $3

A firm’s next‐period market value of equity is $3 million and there are 100,000 shares outstanding, with K = 12%. a. What is the current stock price if the firm pays $600,000 in cash dividends? b. Wha...

See Answer

Q: MCC Corporation currently has cash flow from operations of $10 million

MCC Corporation currently has cash flow from operations of $10 million, capital expenditures of $8 million, and pays a dividend of $2 million (all are perpetuities). The firm has no growth prospects o...

See Answer

Q: A dividend‐paying company has a current dividend yield of 8

A dividend‐paying company has a current dividend yield of 8 percent and a stock price of $100. The company has paid the same dividend for the past 15 years and it is not expected to change. Alice beli...

See Answer

Q: The current stock price of Abacus is $50. For the

The current stock price of Abacus is $50. For the past 20 years, the firm has paid an annual dividend of $5. On July 26, it announced a dividend of $6 payable on September 10 to shareholders of record...

See Answer

Q: State the principle of the optimal cash balance.

State the principle of the optimal cash balance.

See Answer

Q: Kumar expected his firm to earn $1,000 per year

Kumar expected his firm to earn $1,000 per year forever, with no growth. Given a cost of capital of 10 percent, the value of the firm is $10,000. Kumar identified a new project, which costs $1,000 but...

See Answer

Q: State the assumptions underlying the M&M irrelevance theory.

State the assumptions underlying the M&M irrelevance theory.

See Answer

Q: Explain the importance of this theory.

Explain the importance of this theory.

See Answer

Q: What is the basic argument that M&M use to arrive

What is the basic argument that M&M use to arrive at the irrelevancy result?

See Answer

Q: In this ideal M&M world, what will affect firm

In this ideal M&M world, what will affect firm value?

See Answer

Q: How do taxes affect the M&M argument?

How do taxes affect the M&M argument?

See Answer

Q: What are the practical difficulties associated with the implications of M&

What are the practical difficulties associated with the implications of M&M’s corporate tax model?

See Answer

Q: Calculate ROE if R OI 15%, R D 10%, B

Calculate ROE if R OI 15%, R D 10%, B $300,000, S E $500,000, and T 30%. Identify the business risk and financial risk.

See Answer

Q: What is the intercept and slope of the financial leverage (ROE

What is the intercept and slope of the financial leverage (ROE‐ROI) line in Practice Problem 17? Explain the meaning of the slope.

See Answer

Q: What is the ROE indifference point of these two financing strategies of

What is the ROE indifference point of these two financing strategies of Arctic Inc.: Strategy 1: debt-equity ratio = 0.6. Strategy 2: debt-equity ratio = 1.5? Arctic Inc.’s tax rate is 25 perc...

See Answer

Q: What is the most important difference between a corporation and all other

What is the most important difference between a corporation and all other organizational forms?

See Answer

Q: Corporate managers work for the owners of the corporation. Consequently,

Corporate managers work for the owners of the corporation. Consequently, they should make decisions that are in the interests of the owners, rather than their own. What strategies are available to sha...

See Answer

Q: You are thinking of retiring. Your retirement plan will pay you

You are thinking of retiring. Your retirement plan will pay you either $250,000 immediately on retirement or $350,000 five years after the date of your retirement. Which alternative should you choose...

See Answer

Q: Your son has been accepted into college. This college guarantees that

Your son has been accepted into college. This college guarantees that your son’s tuition will not increase for the four years he attends college. The first $10,000 tuition payment is due in six months...

See Answer

Q: You make monthly payments on your mortgage. It has a quoted

You make monthly payments on your mortgage. It has a quoted APR of 5% (monthly compounding). What percentage of the outstanding principal do you pay in interest each month?

See Answer

Q: Oppenheimer Bank is offering a 30-year mortgage with an EAR

Oppenheimer Bank is offering a 30-year mortgage with an EAR of 53⁄8%. If you plan to borrow $150,000, what will your monthly payment be?

See Answer

Q: You have just purchased a home and taken out a $500

You have just purchased a home and taken out a $500,000 mortgage. The mortgage has a 30-year term with monthly payments and an APR of 6%. a. How much will you pay in interest, and how much will you pa...

See Answer

Q: Your mortgage has 25 years left, and has an APR of

Your mortgage has 25 years left, and has an APR of 7.625% with monthly payments of $1449. a. What is the outstanding balance? b. Suppose you cannot make the mortgage payment and you are in danger of l...

See Answer

Q: Oppenheimer Bank is offering a 30-year mortgage with an APR

Oppenheimer Bank is offering a 30-year mortgage with an APR of 5.25% based on monthly compounding. With this mortgage your monthly payments would be $2,000 per month. In addition, Oppenheimer Bank off...

See Answer

Q: Your friend tells you he has a very simple trick for shortening

Your friend tells you he has a very simple trick for shortening the time it takes to repay your mortgage by one-third: Use your holiday bonus to make an extra payment on January 1 of each year (that i...

See Answer

Q: You need a new car and the dealer has offered you a

You need a new car and the dealer has offered you a price of $20,000, with the following payment options: (a) pay cash and receive a $2000 rebate, or (b) pay a $5000 down payment and finance the rest...

See Answer

Q: Can the nominal interest rate available to an investor be significantly negative

Can the nominal interest rate available to an investor be significantly negative? Can the real interest rate be negative? Explain.

See Answer

Q: Suppose that in 2016, Global launches an aggressive marketing campaign that

Suppose that in 2016, Global launches an aggressive marketing campaign that boosts sales by 15%. However, their operating margin falls from 5.57% to 4.50%. Suppose that they have no other income, inte...

See Answer

Q: Consider a project that requires an initial investment of $100,

Consider a project that requires an initial investment of $100,000 and will produce a single cash flow of $150,000 in five years. a. What is the NPV of this project if the five-year interest rate is 5...

See Answer

Q: Suppose the term structure of risk-free interest rates is as

Suppose the term structure of risk-free interest rates is as shown below: a. Calculate the present value of an investment that pays $1000 in two years and $2000 in five years for certain. b. Calcula...

See Answer

Q: Many academic institutions offer a sabbatical policy. Every seven years a

Many academic institutions offer a sabbatical policy. Every seven years a professor is given a year free of teaching and other administrative responsibilities at full pay. For a professor earning $70,...

See Answer

Q: Using the term structure in Problem 29, what is the present

Using the term structure in Problem 29, what is the present value of an investment that pays $100 at the end of each of years 1, 2, and 3? If you wanted to value this investment correctly using the an...

See Answer

Q: What is the shape of the yield curve given the term structure

What is the shape of the yield curve given the term structure in Problem 29? What expectations are investors likely to have about future interest rates? Data from Problem 29: Suppose the term struct...

See Answer

Q: Suppose the current one-year interest rate is 6%. One

Suppose the current one-year interest rate is 6%. One year from now, you believe the economy will start to slow and the one-year interest rate will fall to 5%. In two years, you expect the economy to...

See Answer

Q: Figure 5.4 shows that Johnson and Johnson’s five-year

Figure 5.4 shows that Johnson and Johnson’s five-year borrowing rate is 1.9% and Xerox’s is 4.0%. Which would you prefer? $500 from Johnson and Johnson paid today o...

See Answer

Q: Your best taxable investment opportunity has an EAR of 4%. Your

Your best taxable investment opportunity has an EAR of 4%. Your best tax-free investment opportunity has an EAR of 3%. If your tax rate is 30%, which opportunity provides the higher after-tax interest...

See Answer

Q: Your uncle Fred just purchased a new boat. He brags to

Your uncle Fred just purchased a new boat. He brags to you about the low 7% interest rate (APR, monthly compounding) he obtained from the dealer. The rate is even lower than the rate he could have obt...

See Answer

Q: You are enrolling in an MBA program. To pay your tuition

You are enrolling in an MBA program. To pay your tuition, you can either take out a standard student loan (so the interest payments are not tax deductible) with an EAR of 51⁄2% or you can use a tax-de...

See Answer

Q: You are the CEO of a company and you are considering entering

You are the CEO of a company and you are considering entering into an agreement to have your company buy another company. You think the price might be too high, but you will be the CEO of the combined...

See Answer

Q: Your best friend consults you for investment advice. You learn that

Your best friend consults you for investment advice. You learn that his tax rate is 35%, and he has the following current investments and debts: A car loan with an outstanding balance of $5000 and a 4...

See Answer

Q: Suppose you have outstanding debt with an 8% interest rate that

Suppose you have outstanding debt with an 8% interest rate that can be repaid any time, and the interest rate on U.S. Treasuries is only 5%. You plan to repay your debt using any cash that you don’t i...

See Answer

Q: In the summer of 2008, at Heathrow Airport in London,

In the summer of 2008, at Heathrow Airport in London, Best of the best (BB), a private company, offered a lottery to win a Ferrari or 90,000 British pounds, equivalent at the time to about $180,000. B...

See Answer

Q: Your firm is considering the purchase of a new office phone system

Your firm is considering the purchase of a new office phone system. You can either pay $32,000 now, or $1000 per month for 36 months. a. Suppose your firm currently borrows at a rate of 6% per year (A...

See Answer

Q: After reading the Novy-Marx and Rauh article (see the

After reading the Novy-Marx and Rauh article (see the Common Mistake Box on page 161), you decide to compute the total obligation of the state you live in. After some research you determine that your...

See Answer

Q: You are considering moving your money to a new bank offering a

You are considering moving your money to a new bank offering a one-year CD that pays an 8% APR with monthly compounding. Your current bank’s manager offers to match the rate you have been offered. The...

See Answer

Q: You can earn $50 in interest on a $1000 deposit

You can earn $50 in interest on a $1000 deposit for eight months. If the EAR is the same regardless of the length of the investment, determine how much interest you will earn on a $1000 deposit for a....

See Answer

Q: Suppose you invest $100 in a bank account, and five

Suppose you invest $100 in a bank account, and five years later it has grown to $134.39. a. What APR did you receive, if the interest was compounded semiannually? b. What APR did you receive if the in...

See Answer

Q: A 30-year bond with a face value of $1000

A 30-year bond with a face value of $1000 has a coupon rate of 5.5%, with semiannual payments. a. What is the coupon payment for this bond? b. Draw the cash flows for the bond on a timeline.

See Answer

Q: Suppose a seven-year, $1000 bond with an 8

Suppose a seven-year, $1000 bond with an 8% coupon rate and semiannual coupons is trading with a yield to maturity of 6.75%. a. Is this bond currently trading at a discount, at par, or at a premium? E...

See Answer

Q: Are hostile takeovers necessarily bad for firms or their investors? Explain

Are hostile takeovers necessarily bad for firms or their investors? Explain.

See Answer

Q: Suppose that Ally Financial Inc. issued a bond with 10 years

Suppose that Ally Financial Inc. issued a bond with 10 years until maturity, a face value of $1000, and a coupon rate of 7% (annual payments). The yield to maturity on this bond when it was issued was...

See Answer

Q: Consider the following bonds: / a.

Consider the following bonds: a. What is the percentage change in the price of each bond if its yield to maturity falls from 6% to 5%? b. Which of the bonds A–D is most sensitive t...

See Answer

Q: Suppose you purchase a 30-year Treasury bond with a 5

Suppose you purchase a 30-year Treasury bond with a 5% annual coupon, initially trading at par. In 10 years’ time, the bond’s yield to maturity has risen to 7% (EAR). a. If you sell the bond now, what...

See Answer

Q: Suppose the current yield on a one-year, zero coupon

Suppose the current yield on a one-year, zero coupon bond is 3%, while the yield on a five-year, zero coupon bond is 5%. Neither bond has any risk of default. Suppose you plan to invest for one year....

See Answer

Q: What is the price today of a two-year, default

What is the price today of a two-year, default-free security with a face value of $1000 and an annual coupon rate of 6%? Does this bond trade at a discount, at par, or at a premium? Data for Problem...

See Answer

Q: What is the price of a five-year, zero-

What is the price of a five-year, zero-coupon, default-free security with a face value of $1000? Data for Problem 18:

See Answer

Q: What is the price of a three-year, default-

What is the price of a three-year, default-free security with a face value of $1000 and an annual coupon rate of 4%? What is the yield to maturity for this bond? Data for Problem 19:

See Answer

Q: What is the maturity of a default-free security with annual

What is the maturity of a default-free security with annual coupon payments and a yield to maturity of 4%? Why? Data for Problem 20:

See Answer

Q: Consider a four-year, default-free security with annual

Consider a four-year, default-free security with annual coupon payments and a face value of $1000 that is issued at par. What is the coupon rate of this bond? Data for Problem 21:

See Answer

Q: Consider a five-year, default-free bond with annual

Consider a five-year, default-free bond with annual coupons of 5% and a face value of $1000. a. Without doing any calculations, determine whether this bond is trading at a premium or at a discount. Ex...

See Answer

Q: What is the difference between a public and a private corporation?

What is the difference between a public and a private corporation?

See Answer

Q: Prices of zero-coupon, default-free securities with face

Prices of zero-coupon, default-free securities with face values of $1000 are summarized in the following table: Suppose you observe that a three-year, default-free security with an annual coupon rat...

See Answer

Q: Assume there are four default-free bonds with the following prices

Assume there are four default-free bonds with the following prices and future cash flows: Do these bonds present an arbitrage opportunity? If so, how would you take advantage of this opportunity? If...

See Answer

Q: Suppose you are given the following information about the default-free

Suppose you are given the following information about the default-free, coupon-paying yield curve: a. Use arbitrage to determine the yield to maturity of a two-year, zero-coupon bond. b. What is the...

See Answer

Q: Explain why the expected return of a corporate bond does not equal

Explain why the expected return of a corporate bond does not equal its yield to maturity.

See Answer

Q: In the Data Case in Chapter 5, we suggested using the

In the Data Case in Chapter 5, we suggested using the yield on Florida Sate bonds to estimate the State of Florida’s cost of capital. Why might this estimate overstate the actual cost of capital?

See Answer

Q: Grummon Corporation has issued zero-coupon corporate bonds with a five

Grummon Corporation has issued zero-coupon corporate bonds with a five-year maturity. Investors believe there is a 20% chance that Grummon will default on these bonds. If Grummon does default, investo...

See Answer

Q: The following table summarizes prices of various default-free, zero

The following table summarizes prices of various default-free, zero-coupon bonds (expressed as a percentage of face value): a. Compute the yield to maturity for each bond. b. Plot the zero-coupon yi...

See Answer

Q: The Isabelle Corporation rents prom dresses in its stores across the southern

The Isabelle Corporation rents prom dresses in its stores across the southern United States. It has just issued a five-year, zero-coupon corporate bond at a price of $74. You have purchased this bond...

See Answer

Q: What does it mean for a country to “inflate away”

What does it mean for a country to “inflate away” its debt? Why might this be costly for investors even if the country does not default?

See Answer

Q: Suppose the yield on German government bonds is 1%, while the

Suppose the yield on German government bonds is 1%, while the yield on Spanish government bonds is 6%. Both bonds are denominated in euros. Which country do investors believe is more likely to default...

See Answer

Q: Describe the important changes that have occurred in stock markets over the

Describe the important changes that have occurred in stock markets over the last decade.

See Answer

Q: Suppose the current zero-coupon yield curve for risk-free

Suppose the current zero-coupon yield curve for risk-free bonds is as follows: a. What is the price per $100 face value of a two-year, zero-coupon, risk-free bond? b. What is the price per $100 face...

See Answer

Q: In the Global Financial Crisis box in Section 6.1,

In the Global Financial Crisis box in Section 6.1, www.Bloomberg.com reported that the three month Treasury bill sold for a price of $100.002556 per $100 face value. What is the yield to maturity of t...

See Answer

Q: Suppose a 10-year, $1000 bond with an 8

Suppose a 10-year, $1000 bond with an 8% coupon rate and semiannual coupons is trading for a price of $1034.74. a. What is the bond’s yield to maturity (expressed as an APR with semiannual compounding...

See Answer

Q: The prices of several bonds with face values of $1000 are

The prices of several bonds with face values of $1000 are summarized in the following table: For each bond, state whether it trades at a discount, at par, or at a premium.

See Answer

Q: Explain why the yield of a bond that trades at a discount

Explain why the yield of a bond that trades at a discount exceeds the bond’s coupon rate.

See Answer

Q: Your brother wants to borrow $10,000 from you.

Your brother wants to borrow $10,000 from you. He has offered to pay you back $12,000 in a year. If the cost of capital of this investment opportunity is 10%, what is its NPV? Should you undertake the...

See Answer

Q: You have been offered a very long term investment opportunity to increase

You have been offered a very long term investment opportunity to increase your money one hundredfold. You can invest $1000 today and expect to receive $100,000 in 40 years. Your cost of capital for th...

See Answer

Q: How many IRRs are there in part (a) of Problem

How many IRRs are there in part (a) of Problem 5? Does the IRR rule give the right answer in this case? How many IRRs are there in part (b) of Problem 5? Does the IRR rule work in this case? Data fro...

See Answer

Q: You have 3 projects with the following cash flows:

You have 3 projects with the following cash flows: a. For which of these projects is the IRR rule reliable? b. Estimate the IRR for each project (to the nearest 1%). c. What is the NPV of each proje...

See Answer

Q: Your firm spends $500,000 per year in regular maintenance

Your firm spends $500,000 per year in regular maintenance of its equipment. Due to the economic downturn, the firm considers forgoing these maintenance expenses for the next three years. If it does so...

See Answer

Q: See Table 2.5 showing financial statement data and stock price

See Table 2.5 showing financial statement data and stock price data for Mydeco Corp. Suppose Mydeco’s costs and expenses had been the same fraction of revenues in 2013â€&#...

See Answer

Q: Your firm has been hired to develop new software for the university’s

Your firm has been hired to develop new software for the university’s class registration system. Under the contract, you will receive $500,000 as an upfront payment. You expect the development costs t...

See Answer

Q: You are considering investing in a start-up company. The

You are considering investing in a start-up company. The founder asked you for $200,000 today and you expect to get $1,000,000 in nine years. Given the riskiness of the investment opportunity, your co...

See Answer

Q: You are considering constructing a new plant in a remote wilderness area

You are considering constructing a new plant in a remote wilderness area to process the ore from a planned mining operation. You anticipate that the plant will take a year to build and cost $100 milli...

See Answer

Q: You are a real estate agent thinking of placing a sign advertising

You are a real estate agent thinking of placing a sign advertising your services at a local bus stop. The sign will cost $5000 and will be posted for one year. You expect that it will generate additio...

See Answer

Q: You have just started your summer internship, and your boss asks

You have just started your summer internship, and your boss asks you to review a recent analysis that was done to compare three alternative proposals to enhance the firm’s manufactur...

See Answer

Q: You work for an outdoor play structure manufacturing company and are trying

You work for an outdoor play structure manufacturing company and are trying to decide between two projects: You can undertake only one project. If your cost of capital is 8%, use the incremental IRR...

See Answer

Q: You are evaluating the following two projects: /

You are evaluating the following two projects: Use the incremental IRR to determine the range of discount rates for which each project is optimal to undertake. Note that you should also include the...

See Answer

Q: Consider two investment projects, both of which require an upfront investment

Consider two investment projects, both of which require an upfront investment of $10 million and pay a constant positive amount each year for the next 10 years. Under what conditions can you rank thes...

See Answer

Q: You are considering a safe investment opportunity that requires a $1000

You are considering a safe investment opportunity that requires a $1000 investment today, and will pay $500 two years from now and another $750 five years from now. a. What is the IRR of this investme...

See Answer

Q: Facebook is considering two proposals to overhaul its network infrastructure. They

Facebook is considering two proposals to overhaul its network infrastructure. They have received two bids. The first bid, from Huawei, will require a $20 million upfront investment and will generate $...

See Answer

Q: Explain how the bid-ask spread is determined in most markets

Explain how the bid-ask spread is determined in most markets today.

See Answer

Q: Natasha’s Flowers, a local florist, purchases fresh flowers each day

Natasha’s Flowers, a local florist, purchases fresh flowers each day at the local flower market. The buyer has a budget of $1000 per day to spend. Different flowers have different pr...

See Answer

Q: You own a car dealership and are trying to decide how to

You own a car dealership and are trying to decide how to configure the showroom floor. The floor has 2000 square feet of usable space. You have hired an analyst and asked her to estimate the NPV of pu...

See Answer

Q: Kaimalino Properties (KP) is evaluating six real estate investments.

Kaimalino Properties (KP) is evaluating six real estate investments. Management plans to buy the properties today and sell them five years from today. The following table summarizes the initial cost a...

See Answer

Q: Your firm is considering the launch of a new product, the

Your firm is considering the launch of a new product, the XJ5. The upfront development cost is $10 million, and you expect to earn a cash flow of $3 million per year for the next five years. Plot the...

See Answer

Q: Bill Clinton reportedly was paid $15 million to write his book

Bill Clinton reportedly was paid $15 million to write his book My Life. Suppose the book took three years to write. In the time he spent writing, Clinton could have been paid to make speeches. Given h...

See Answer

Q: Fast Track Bikes, Inc. is thinking of developing a new

Fast Track Bikes, Inc. is thinking of developing a new composite road bike. Development will take six years and the cost is $200,000 per year. Once in production, the bike is expected to make $300,000...

See Answer

Q: Open Seas, Inc. is evaluating the purchase of a new

Open Seas, Inc. is evaluating the purchase of a new cruise ship. The ship would cost $500 million, and would operate for 20 years. Open Seas expects annual cash flows from operating the ship to be $70...

See Answer

Q: You are CEO of Rivet Networks, maker of ultra-high

You are CEO of Rivet Networks, maker of ultra-high performance network cards for gaming computers, and you are considering whether to launch a new product. The product, the Killer X3000, will cost $90...

See Answer

Q: You are considering an investment in a clothes distributor. The company

You are considering an investment in a clothes distributor. The company needs $100,000 today and expects to repay you $120,000 in a year from now. What is the IRR of this investment opportunity? Given...

See Answer

Q: Using the assumptions in part (a) of Problem 5 (

Using the assumptions in part (a) of Problem 5 (assuming there is no cannibalization), a. Calculate Home Net’s net working capital requirements (that is, reproduce Table 8.4 under the assumptions in P...

See Answer

Q: The following quote on Yahoo! stock appeared on July 23,

The following quote on Yahoo! stock appeared on July 23, 2015, on Yahoo! Finance: If you wanted to buy Yahoo!, what price would you pay? How much would you receive if you wanted to sell Yahoo!?

See Answer

Q: A bicycle manufacturer currently produces 300,000 units a year and

A bicycle manufacturer currently produces 300,000 units a year and expects output levels to remain steady in the future. It buys chains from an outside supplier at a price of $2 a chain. The plant man...

See Answer

Q: Consider again the choice between outsourcing and in-house assembly of

Consider again the choice between outsourcing and in-house assembly of Home Net discussed in Section 8.3 and analyzed in Table 8.6. Suppose, however, that the upfront cost to set up for in-house produ...

See Answer

Q: Your firm is considering a project that would require purchasing $7

Your firm is considering a project that would require purchasing $7.5 million worth of new equipment. Determine the present value of the depreciation tax shield associated with this equipment if the f...

See Answer

Q: Arnold Inc. is considering a proposal to manufacture high-end

Arnold Inc. is considering a proposal to manufacture high-end protein bars used as food supplements by body builders. The project requires use of an existing warehouse, which the firm acquired three y...

See Answer

Q: Bay Properties is considering starting a commercial real estate division. It

Bay Properties is considering starting a commercial real estate division. It has prepared the following four-year forecast of free cash flows for this division: Assume cash flows after year 4 will g...

See Answer

Q: Your firm would like to evaluate a proposed new operating division.

Your firm would like to evaluate a proposed new operating division. You have forecasted cash flows for this division for the next five years, and have estimated that the cost of capital is 12%. You wo...

See Answer

Q: In September 2008, the IRS changed tax laws to allow banks

In September 2008, the IRS changed tax laws to allow banks to utilize the tax loss carry forwards of banks they acquire to shield their future income from taxes (prior law restricted the ability of ac...

See Answer

Q: Using the FCF projections in part (b) of Problem 11

Using the FCF projections in part (b) of Problem 11, calculate the NPV of the Home Net project assuming a cost of capital of a. 10%. b. 12%. c. 14%. What is the IRR of the project in this case?

See Answer

Q: For the assumptions in part (a) of Problem 5,

For the assumptions in part (a) of Problem 5, assuming a cost of capital of 12%, calculate the following: a. The break-even annual sales price decline. b. The break-even annual unit sales increase. D...

See Answer

Q: Hyperion, Inc. currently sells its latest high-speed color

Hyperion, Inc. currently sells its latest high-speed color printer, the Hyper 500, for $350. It plans to lower the price to $300 next year. Its cost of goods sold for the Hyper 500 is $200 per unit, a...

See Answer

Q: Suppose the following orders are received by an exchange for Cisco stock

Suppose the following orders are received by an exchange for Cisco stock: Limit Order: Buy 200 shares at $25 Limit Order: Sell 200 shares at $26 Limit Order: Sell 100 shares at $25.50 Limit Order:...

See Answer

Q: After looking at the projections of the Home Net project, you

After looking at the projections of the Home Net project, you decide that they are not realistic. It is unlikely that sales will be constant over the four-year life of the project. Furthermore, other...

See Answer

Q: Cellular Access, Inc. is a cellular telephone service provider that

Cellular Access, Inc. is a cellular telephone service provider that reported net income of $250 million for the most recent fiscal year. The firm had depreciation expenses of $100 million, capital exp...

See Answer

Q: Mersey Chemicals manufactures polypropylene that it ships to its customers via tank

Mersey Chemicals manufactures polypropylene that it ships to its customers via tank car. Currently, it plans to add two additional tank cars to its fleet four years from now. However, a proposed plant...

See Answer

Q: Assume Evco, Inc., has a current price of $50

Assume Evco, Inc., has a current price of $50 and will pay a $2 dividend in one year, and its equity cost of capital is 15%. What price must you expect it to sell for right after paying the dividend i...

See Answer

Q: DFB, Inc., expects earnings at the end of this year

DFB, Inc., expects earnings at the end of this year of $5 per share, and it plans to pay a $3 dividend at that time. DFB will retain $2 per share of its earnings to reinvest in new projects with an ex...

See Answer

Q: Procter and Gamble (PG) paid an annual dividend of $

Procter and Gamble (PG) paid an annual dividend of $1.72 in 2009. You expect PG to increase its dividends by 8% per year for the next five years (through 2014), and thereafter by 3% per year. If the a...

See Answer

Q: Colgate-Palmolive Company has just paid an annual dividend of $

Colgate-Palmolive Company has just paid an annual dividend of $1.50. Analysts are predicting dividends to grow by $0.12 per year over the next five years. After then, Colgate’s earnings are expected t...

See Answer

Q: What is the value of a firm with initial dividend Div,

What is the value of a firm with initial dividend Div, growing for n years (i.e., until year n + 1) at rate g1 and after that at rate g2 forever, when the equity cost of capital is r ?

See Answer

Q: Suppose Amazon.com Inc. pays no dividends but spent $

Suppose Amazon.com Inc. pays no dividends but spent $3 billion on share repurchases last year. If Amazon’s equity cost of capital is 8%, and if the amount spent on repurchases is expected to grow by 6...

See Answer

Q: Benchmark Metrics, Inc. (BMI), an all-equity

Benchmark Metrics, Inc. (BMI), an all-equity financed firm, reported EPS of $5.00 in 2008. Despite the economic downturn, BMI is confident regarding its current investment opportunities. But due to th...

See Answer

Q: What does the phrase limited liability mean in a corporate context?

What does the phrase limited liability mean in a corporate context?

See Answer

Q: See Table 2.5 showing financial statement data and stock price

See Table 2.5 showing financial statement data and stock price data for Mydeco Corp. a. From 2012 to 2016, what was the total cash flow from operations that Mydeco generated? b. What fraction of the t...

See Answer

Q: Anle Corporation has a current price of $20, is expected

Anle Corporation has a current price of $20, is expected to pay a dividend of $1 in one year, and its expected price right after paying that dividend is $22. a. What is Anle’s expected dividend yield?...

See Answer

Q: IDX Technologies is a privately held developer of advanced security systems based

IDX Technologies is a privately held developer of advanced security systems based in Chicago. As part of your business development strategy, in late 2008 you initiate discussions with IDX’s founder ab...

See Answer

Q: Consider the valuation of Kenneth Cole Productions. a. Suppose

Consider the valuation of Kenneth Cole Productions. a. Suppose you believe KCP’s initial revenue growth rate will be between 4% and 11% (with growth slowing in equal steps to 4% by year 2011). What ra...

See Answer

Q: Kenneth Cole Productions (KCP) was acquired in 2012 for a

Kenneth Cole Productions (KCP) was acquired in 2012 for a purchase price of $15.25 per share. KCP has 18.5 million shares outstanding, $45 million in cash, and no debt at the time of the acquisition....

See Answer

Q: You notice that PepsiCo (PEP) has a stock price of

You notice that PepsiCo (PEP) has a stock price of $72.62 and EPS of $3.80. Its competitor, the Coca-Cola Company (KO), has EPS of $1.89. Estimate the value of a share of Coca-Cola stock using only th...

See Answer

Q: Suppose that in January 2006, Kenneth Cole Productions had EPS of

Suppose that in January 2006, Kenneth Cole Productions had EPS of $1.65 and a book value of equity of $12.05 per share. a. Using the average P/E multiple in Table 9.1, estimate KCP’s...

See Answer

Q: Suppose that in January 2006, Kenneth Cole Productions had sales of

Suppose that in January 2006, Kenneth Cole Productions had sales of $518 million, EBITDA of $55.6 million, excess cash of $100 million, $3 million of debt, and 21 million shares outstanding. a. Using...

See Answer

Q: In addition to footwear, Kenneth Cole Productions designs and sells handbags

In addition to footwear, Kenneth Cole Productions designs and sells handbags, apparel, and other accessories. You decide, therefore, to consider comparable for KCP outside the footwear industry. a. Su...

See Answer

Q: Consider the following data for the airline industry for December 2015 (

Consider the following data for the airline industry for December 2015 (EV = enterprise value, Book = equity book value). Discuss the potential challenges of using multiples to value an airline.

See Answer

Q: Suppose Hawaiian Airlines (HA) has 53 million shares outstanding.

Suppose Hawaiian Airlines (HA) has 53 million shares outstanding. Estimate Hawaiian’s share value using each of the five valuation multiples in Problem 28, based on the median valuation multiple of th...

See Answer

Q: See Table 2.5 showing financial statement data and stock price

See Table 2.5 showing financial statement data and stock price data for Mydeco Corp. a. In what year was Mydeco’s net income the lowest? b. In what year did Mydeco need to reduce its...

See Answer

Q: In mid-2015, Coca-Cola Company (KO)

In mid-2015, Coca-Cola Company (KO) had a share price of $41, and had paid a dividend of $1.32 for the prior year. Suppose you expect Coca-Cola to raise this dividend by approximately 7% per year in p...

See Answer

Q: Krell Industries has a share price of $22 today. If

Krell Industries has a share price of $22 today. If Krell is expected to pay a dividend of $0.88 this year, and its stock price is expected to grow to $23.54 at the end of the year, what is Krell’s di...

See Answer

Q: No Growth Corporation currently pays a dividend of $2 per year

No Growth Corporation currently pays a dividend of $2 per year, and it will continue to pay this dividend forever. What is the price per share if its equity cost of capital is 15% per year?

See Answer

Q: Dorpac Corporation has a dividend yield of 1.5%. Dorpac’s

Dorpac Corporation has a dividend yield of 1.5%. Dorpac’s equity cost of capital is 8%, and its dividends are expected to grow at a constant rate. a. What is the expected growth rate of Dorpac’s divid...

See Answer

Q: Canadian-based mining company El Dorado Gold (EGO) suspended

Canadian-based mining company El Dorado Gold (EGO) suspended its dividend in March 2016 as a result of declining gold prices and delays in obtaining permits for its mines in Greece. Suppose you expect...

See Answer

Q: In 2006 and 2007, Kenneth Cole Productions (KCP) paid

In 2006 and 2007, Kenneth Cole Productions (KCP) paid annual dividends of $0.72. In 2008, KCP paid an annual dividend of $0.36, and then paid no further dividends through 2012. KCP was acquired at the...

See Answer

Q: The figure on page 351 shows the one-year return distribution

The figure on page 351 shows the one-year return distribution for RCS stock. Calculate a. The expected return. b. The standard deviation of the return.

See Answer

Q: Using the data in Table 10.2, a.

Using the data in Table 10.2, a. What was the average dividend yield for the SP500 from 2002–2014? b. What was the volatility of the dividend yield? c. What was the average annual re...

See Answer

Q: Consider an investment with the following returns over four years:

Consider an investment with the following returns over four years: a. What is the compound annual growth rate (CAGR) for this investment over the four years? b. What is the average annual return of...

See Answer

Q: Download the spreadsheet from that contains historical monthly prices and dividends (

Download the spreadsheet from that contains historical monthly prices and dividends (paid at the end of the month) for Ford Motor Company stock (Ticker: F) from August 1994 to August 1998. Calculate t...

See Answer

Q: See Table 2.5 showing financial statement data and stock price

See Table 2.5 showing financial statement data and stock price data for Mydeco Corp. Use the data from the balance sheet and cash flow statement in 2012 to determine the following: a. How much cash di...

See Answer

Q: Using the same data as in Problem 12, compute the

Using the same data as in Problem 12, compute the a. Average monthly return over this period. b. Monthly volatility (or standard deviation) over this period. Data from Problem 12: Download the sprea...

See Answer

Q: Explain the difference between the average return you calculated in Problem 13

Explain the difference between the average return you calculated in Problem 13(a) and the realized return you calculated in Problem 12. Are both numbers useful? If so, explain why. Data from Problem...

See Answer

Q: Compute the 95% confidence interval of the estimate of the average

Compute the 95% confidence interval of the estimate of the average monthly return you calculated in Problem 13(a). Data from Problem 13: Using the same data as in Problem 12, compute the a. Average...

See Answer

Q: How does the relationship between the average return and the historical volatility

How does the relationship between the average return and the historical volatility of individual stocks differ from the relationship between the average return and the historical volatility of large,...

See Answer

Q: Download the spreadsheet from containing the data for Figure 10.1

Download the spreadsheet from containing the data for Figure 10.1. a. Compute the average return for each of the assets from 1929 to 1940 (The Great Depression). b. Compute the variance and standard d...

See Answer

Q: Using the data from Problem 17, repeat your analysis over the

Using the data from Problem 17, repeat your analysis over the 1990s. a. Which asset was riskiest? b. Compare the standard deviations of the assets in the 1990s to their standard deviations in the Grea...

See Answer

Q: What if the last two and a half decades had been “

What if the last two and a half decades had been “normal”? Download the spreadsheet from containing the data for Figure 10.1. a. Calculate the arithmetic average re...

See Answer

Q: The following table shows the one-year return distribution of Startup

The following table shows the one-year return distribution of Startup, Inc. Calculate a. The expected return. b. The standard deviation of the return.

See Answer

Q: Consider two local banks. Bank A has 100 loans outstanding,

Consider two local banks. Bank A has 100 loans outstanding, each for $1 million, that it expects will be repaid today. Each loan has a 5% probability of default, in which case the bank is not repaid a...

See Answer

Q: Using the data in Problem 20, calculate a. The

Using the data in Problem 20, calculate a. The expected overall payoff of each bank. b. The standard deviation of the overall payoff of each bank. Data from Problem 20: Consider two local banks. Ban...

See Answer

Q: Can a firm with positive net income run out of cash?

Can a firm with positive net income run out of cash? Explain.

See Answer

Q: Consider the following two, completely separate, economies. The expected

Consider the following two, completely separate, economies. The expected return and volatility of all stocks in both economies is the same. In the first economy, all stocks move together—in good times...

See Answer

Q: Consider an economy with two types of firms, S and I

Consider an economy with two types of firms, S and I. S firms all move together. I firms move independently. For both types of firms, there is a 60% probability that the firms will have a 15% return a...

See Answer

Q: Using the data in Problem 23, plot the volatility as a

Using the data in Problem 23, plot the volatility as a function of the number of firms in the two portfolios. Data from Problem 23: Consider an economy with two types of firms, S and I. S firms all...

See Answer

Q: Explain why the risk premium of a stock does not depend on

Explain why the risk premium of a stock does not depend on its diversifiable risk.

See Answer

Q: Identify each of the following risks as most likely to be systematic

Identify each of the following risks as most likely to be systematic risk or diversifiable risk: a. The risk that your main production plant is shut down due to a tornado. b. The risk that the economy...

See Answer

Q: Suppose the risk-free interest rate is 5%, and the

Suppose the risk-free interest rate is 5%, and the stock market will return either 40% or -20% each year, with each outcome equally likely. Compare the following two investment strategies: (1) invest...

See Answer

Q: Download the spreadsheet from containing the realized return of the S&

Download the spreadsheet from containing the realized return of the S&P 500 from 1929–2008. Starting in 1929, divide the sample into four periods of 20 years each. For each 20-year period, calculate t...

See Answer

Q: Characterize the difference between the two stocks in Problems 1 and 2

Characterize the difference between the two stocks in Problems 1 and 2. What trade-offs would you face in choosing one to hold? Data from Problems 1: The figure on page 351 shows the one-year return...

See Answer

Q: What does the beta of a stock measure?

What does the beta of a stock measure?

See Answer

Q: You turn on the news and find out the stock market has

You turn on the news and find out the stock market has gone up 10%. Based on the data in Table 10.6, by how much do you expect each of the following stocks to have gone up or down: (1) Starbucks, (2)...

See Answer

Q: Suppose your firm receives a $5 million order on the last

Suppose your firm receives a $5 million order on the last day of the year. You fill the order with $2 million worth of inventory. The customer picks up the entire order the same day and pays $1 millio...

See Answer

Q: Based on the data in Table 10.6, estimate which

Based on the data in Table 10.6, estimate which of the following investments you expect to lose the most in the event of a severe market down turn: (1) A $2000 investment in Hershey, (2) a $1500 inves...

See Answer

Q: Suppose the market portfolio is equally likely to increase by 30%

Suppose the market portfolio is equally likely to increase by 30% or decrease by 10%. a. Calculate the beta of a firm that goes up on average by 43% when the market goes up and goes down by 17% when t...

See Answer

Q: Suppose the risk-free interest rate is 4%. a

Suppose the risk-free interest rate is 4%. a. i. Use the beta you calculated for the stock in Problem 33(a) to estimate its expected return. ii. How does this compare with the stock’s actual expected...

See Answer

Q: Suppose the market risk premium is 5% and the risk-

Suppose the market risk premium is 5% and the risk-free interest rate is 4%. Using the data in Table 10.6, calculate the expected return of investing in a. Starbucks’ stock. b. Hersh...

See Answer

Q: Given the results to Problem 35, why don’t all investors hold

Given the results to Problem 35, why don’t all investors hold Autodesk’s stock rather than Hershey’s stock? Data from Problem 35: Suppose the market risk premium is 5% and the risk-free interest rat...

See Answer

Q: Suppose the market risk premium is 6.5% and the

Suppose the market risk premium is 6.5% and the risk-free interest rate is 5%. Calculate the cost of capital of investing in a project with a beta of 1.2.

See Answer

Q: State whether each of the following is inconsistent with an efficient capital

State whether each of the following is inconsistent with an efficient capital market, the CAPM, or both: a. A security with only diversifiable risk has an expected return that exceeds the risk-free in...

See Answer

Q: You bought a stock one year ago for $50 per share

You bought a stock one year ago for $50 per share and sold it today for $55 per share. It paid a $1 per share dividend today. a. What was your realized return? b. How much of the return came from divi...

See Answer

Q: Repeat Problem 4 assuming that the stock fell $5 to $

Repeat Problem 4 assuming that the stock fell $5 to $45 instead. a. Is your capital gain different? Why or why not? b. Is your dividend yield different? Why or why not? Data from Problem 4: You boug...

See Answer

Q: Using the data in the following table, calculate the return for

Using the data in the following table, calculate the return for investing in Boeing stock (BA) from January 2, 2008, to January 2, 2009, and also from January 3, 2011, to January 3, 2012, assuming all...

See Answer

Q: Nokela Industries purchases a $40 million cyclo-converter. The

Nokela Industries purchases a $40 million cyclo-converter. The cyclo-converter will be depreciated by $10 million per year over four years, starting this year. Suppose Nokela’s tax rate is 40%. a. Wha...

See Answer

Q: Assume that historical returns and future returns are independently and identically distributed

Assume that historical returns and future returns are independently and identically distributed and drawn from the same distribution. a. Calculate the 95% confidence intervals for the expected annual...

See Answer

Q: Using the data in Table 10.2, a.

Using the data in Table 10.2, a. What was the average annual return of Microsoft stock from 2002–2014? b. What was the annual volatility for Microsoft stock from 2002â€...

See Answer

Q: Arbor Systems and Gencore stocks both have a volatility of 40%.

Arbor Systems and Gencore stocks both have a volatility of 40%. Compute the volatility of a portfolio with 50% invested in each stock if the correlation between the stocks is (a) +1, (b) 0.50, (c) 0,...

See Answer

Q: Suppose Wesley Publishing’s stock has a volatility of 60%, while Addison

Suppose Wesley Publishing’s stock has a volatility of 60%, while Addison Printing’s stock has a volatility of 30%. If the correlation between these stocks is 25%, what is the volatility of the followi...

See Answer

Q: Suppose Avon and Nova stocks have volatilities of 50% and 25

Suppose Avon and Nova stocks have volatilities of 50% and 25%, respectively, and they are perfectly negatively correlated. What portfolio of these two stocks has zero risk?

See Answer

Q: Suppose Tex stock has a volatility of 40%, and Mex stock

Suppose Tex stock has a volatility of 40%, and Mex stock has a volatility of 20%. If Tex and Mex are uncorrelated, a. What portfolio of the two stocks has the same volatility as Mex alone? b. What por...

See Answer

Q: Using the data in Table 11.1, a.

Using the data in Table 11.1, a. Compute the annual returns for a portfolio with 25% invested in North Air, 25% invested in West Air, and 50% invested in Tex Oil. b. What is the lowest annual return f...

See Answer

Q: Using the data from Table 11.3, what is the

Using the data from Table 11.3, what is the volatility of an equally weighted portfolio of Microsoft, Alaska Air, and Ford Motor stock? Table 11.3:

See Answer

Q: Suppose the average stock has a volatility of 50%, and the

Suppose the average stock has a volatility of 50%, and the correlation between pairs of stocks is 20%. Estimate the volatility of an equally weighted portfolio with (a) 1 stock, (b) 30 stocks, (c) 100...

See Answer

Q: What is the volatility (standard deviation) of an equally weighted

What is the volatility (standard deviation) of an equally weighted portfolio of stocks within an industry in which the stocks have a volatility of 50% and a correlation of 40% as the portfolio becomes...

See Answer

Q: See Table 2.5 showing financial statement data and stock price

See Table 2.5 showing financial statement data and stock price data for Mydeco Corp. a. What were Mydeco’s retained earnings each year? b. Using the data from 2012, what was Mydeco&a...

See Answer

Q: Consider an equally weighted portfolio of stocks in which each stock has

Consider an equally weighted portfolio of stocks in which each stock has a volatility of 40%, and the correlation between each pair of stocks is 20%. a. What is the volatility of the portfolio as the...

See Answer

Q: Stock A has a volatility of 65% and a correlation of

Stock A has a volatility of 65% and a correlation of 10% with your current portfolio. Stock B has a volatility of 30% and a correlation of 25% with your current portfolio. You currently hold both stoc...

See Answer

Q: You own three stocks: 600 shares of Apple Computer, 10

You own three stocks: 600 shares of Apple Computer, 10,000 shares of Cisco Systems, and 5000 shares of Colgate-Palmolive. The current share prices and expected returns of Apple, Cisco, and Colgate-Pal...

See Answer

Q: You currently hold a portfolio of three stocks, Delta, Gamma

You currently hold a portfolio of three stocks, Delta, Gamma, and Omega. Delta has a volatility of 60%, Gamma has a volatility of 30%, and Omega has a volatility of 20%. Suppose you invest 50% of your...

See Answer

Q: Suppose Ford Motor stock has an expected return of 20% and

Suppose Ford Motor stock has an expected return of 20% and a volatility of 40%, and Molson Coors Brewing has an expected return of 10% and a volatility of 30%. If the two stocks are uncorrelated, a. W...

See Answer

Q: Suppose Intel’s stock has an expected return of 26% and a

Suppose Intel’s stock has an expected return of 26% and a volatility of 50%, while Coca-Cola’s has an expected return of 6% and volatility of 25%. If these two stocks were perfectly negatively correla...

See Answer

Q: Calculate (a) the expected return and (b

Calculate (a) the expected return and (b) the volatility (standard deviation) of a portfolio that is equally invested in Johnson & Johnson’s and Walgreens’ stoc...

See Answer

Q: For the portfolio in Problem 23, if the correlation between Johnson

For the portfolio in Problem 23, if the correlation between Johnson & Johnson’s and Walgreens’ stock were to increase, a. Would the expected return of the portf...

See Answer

Q: Calculate (a) the expected return and (b

Calculate (a) the expected return and (b) the volatility (standard deviation) of a portfolio that consists of a long position of $10,000 in Johnson & Johnson and a short position of $2000 in Walgr...

See Answer

Q: Using the same data as for Problem 23, calculate the expected

Using the same data as for Problem 23, calculate the expected return and the volatility (standard deviation) of a portfolio consisting of Johnson & Johnson’s and Walgreensâ...

See Answer

Q: Find online the annual 10-K report for Costco Wholesale Corporation

Find online the annual 10-K report for Costco Wholesale Corporation (COST) for fiscal year 2015 (filed in October 2015). Answer the following questions from the notes to their financial statements: a....

See Answer

Q: A hedge fund has created a portfolio using just two stocks.

A hedge fund has created a portfolio using just two stocks. It has shorted $35,000,000 worth of Oracle stock and has purchased $85,000,000 of Intel stock. The correlation between Oracleâ€&#...

See Answer

Q: Consider the portfolio in Problem 27. Suppose the correlation between Intel

Consider the portfolio in Problem 27. Suppose the correlation between Intel and Oracle’s stock increases, but nothing else changes. Would the portfolio be more or less risky with thi...

See Answer

Q: Fred holds a portfolio with a 30% volatility. He decides

Fred holds a portfolio with a 30% volatility. He decides to short sell a small amount of stock with a 40% volatility and use the proceeds to invest more in his portfolio. If this transaction reduces t...

See Answer

Q: Consider a world that only consists of the three stocks shown in

Consider a world that only consists of the three stocks shown in the following table: a. Calculate the total value of all shares outstanding currently. b. What fraction of the total value outstandin...

See Answer

Q: Suppose Target’s stock has an expected return of 20% and a

Suppose Target’s stock has an expected return of 20% and a volatility of 40%, Hershey’s stock has an expected return of 12% and a volatility of 30%, and these two stocks are uncorrelated. a. What is t...

See Answer

Q: You have $10,000 to invest. You decide to

You have $10,000 to invest. You decide to invest $20,000 in Google and short sell $10,000 worth of Yahoo! Google’s expected return is 15% with a volatility of 30% and Yahoo!’s expected return is 12% w...

See Answer

Q: You expect HGH stock to have a 20% return next year

You expect HGH stock to have a 20% return next year and a 30% volatility. You have $25,000 to invest, but plan to invest a total of $50,000 in HGH, raising the additional $25,000 by shorting either KB...

See Answer

Q: Suppose you have $100,000 in cash, and you

Suppose you have $100,000 in cash, and you decide to borrow another $15,000 at a 4% interest rate to invest in the stock market. You invest the entire $115,000 in a portfolio J with a 15% expected ret...

See Answer

Q: You have $100,000 to invest. You choose to

You have $100,000 to invest. You choose to put $150,000 into the market by borrowing $50,000. a. If the risk-free interest rate is 5% and the market expected return is 10%, what is the expected return...

See Answer

Q: You currently have $100,000 invested in a portfolio that

You currently have $100,000 invested in a portfolio that has an expected return of 12% and a volatility of 8%. Suppose the risk-free rate is 5%, and there is another portfolio that has an expected ret...

See Answer

Q: See Table 2.5 showing financial statement data and stock price

See Table 2.5 showing financial statement data and stock price data for Mydeco Corp. a. What were Mydeco’s gross margins each year? b. Comparing Mydeco’s gross marg...

See Answer

Q: Assume the risk-free rate is 4%. You are a

Assume the risk-free rate is 4%. You are a financial advisor, and must choose one of the funds below to recommend to each of your clients. Whichever fund you recommend, your clients will then combine...

See Answer

Q: Assume all investors want to hold a portfolio that, for a

Assume all investors want to hold a portfolio that, for a given level of volatility, has the maximum possible expected return. Explain why, when a risk-free asset exists, all investors will choose to...

See Answer

Q: In addition to risk-free securities, you are currently invested

In addition to risk-free securities, you are currently invested in the Tanglewood Fund, a broad based fund of stocks and other securities with an expected return of 12% and a volatility of 25%. Curren...

See Answer

Q: You have noticed a market investment opportunity that, given your current

You have noticed a market investment opportunity that, given your current portfolio, has an expected return that exceeds your required return. What can you conclude about your current portfolio?

See Answer

Q: There are two ways to calculate the expected return of a portfolio

There are two ways to calculate the expected return of a portfolio: either calculate the expected return using the value and dividend stream of the portfolio as a whole, or calculate the weighted aver...

See Answer

Q: The Optima Mutual Fund has an expected return of 20%, and

The Optima Mutual Fund has an expected return of 20%, and a volatility of 20%. Optima claims that no other portfolio offers a higher Sharpe ratio. Suppose this claim is true, and the risk-free interes...

See Answer

Q: You are currently only invested in the Natasha Fund (aside from

You are currently only invested in the Natasha Fund (aside from risk-free securities). It has an expected return of 14% with a volatility of 20%. Currently, the risk-free rate of interest is 3.8%. You...

See Answer

Q: Calculate the Sharpe ratio of each of the three portfolios in Problem

Calculate the Sharpe ratio of each of the three portfolios in Problem 41. What portfolio weight in Hannah stock maximizes the Sharpe ratio? Data from Problem 41: You are currently only invested in t...

See Answer

Q: Returning to Problem 38, assume you follow your broker’s advice and

Returning to Problem 38, assume you follow your broker’s advice and put 50% of your money in the venture fund. a. What is the Sharpe ratio of the Tanglewood Fund? b. What is the Sharpe ratio of your n...

See Answer

Q: When the CAPM correctly prices risk, the market portfolio is an

When the CAPM correctly prices risk, the market portfolio is an efficient portfolio. Explain why.

See Answer

Q: For fiscal year end 2015, Wal-Mart Stores, Inc

For fiscal year end 2015, Wal-Mart Stores, Inc. (WMT, brand name Walmart) had revenues of $485.65 billion, gross profit of $120.57 billion, and net income of $16.36 billion. Costco Wholesale Corporati...

See Answer

Q: A big pharmaceutical company, DRIg, has just announced a potential

A big pharmaceutical company, DRIg, has just announced a potential cure for cancer. The stock price increased from $5 to $100 in one day. A friend calls to tell you that he owns DRIg. You proudly repl...

See Answer

Q: Your investment portfolio consists of $15,000 invested in only

Your investment portfolio consists of $15,000 invested in only one stock—Microsoft. Suppose the risk-free rate is 5%, Microsoft stock has an expected return of 12% and a volatility of 40%, and the mar...

See Answer

Q: Suppose you group all the stocks in the world into two mutually

Suppose you group all the stocks in the world into two mutually exclusive portfolios (each stock is in only one portfolio): growth stocks and value stocks. Suppose the two portfolios have equal size (...

See Answer

Q: Consider a portfolio consisting of the following three stocks:

Consider a portfolio consisting of the following three stocks: The volatility of the market portfolio is 10% and it has an expected return of 8%. The risk-free rate is 3%. a. Compute the beta and ex...

See Answer

Q: Using the data in the following table, estimate (a

Using the data in the following table, estimate (a) the average return and volatility for each stock, (b) the covariance between the stocks, and (c) the correlation between these two stocks.

See Answer

Q: Suppose Autodesk stock has a beta of 2.16, whereas

Suppose Autodesk stock has a beta of 2.16, whereas Costco stock has a beta of 0.69. If the risk-free interest rate is 4% and the expected return of the market portfolio is 10%, what is the expected re...

See Answer

Q: What is the risk premium of a zero-beta stock?

What is the risk premium of a zero-beta stock? Does this mean you can lower the volatility of a portfolio without changing the expected return by substituting out any zero-beta stock in a portfolio an...

See Answer

Q: Use the data in Problem 5, consider a portfolio that maintains

Use the data in Problem 5, consider a portfolio that maintains a 50% weight on stock A and a 50% weight on stock B. a. What is the return each year of this portfolio? b. Based on your results from par...

See Answer

Q: Using your estimates from Problem 5, calculate the volatility (standard

Using your estimates from Problem 5, calculate the volatility (standard deviation) of a portfolio that is 70% invested in stock A and 30% invested in stock B. Data from Problem 5: Using the data in...

See Answer

Q: Using the data from Table 11.3, what is the

Using the data from Table 11.3, what is the covariance between the stocks of Alaska Air and Southwest Airlines? Table 11.3:

See Answer

Q: Which organizational forms give their owners limited liability?

Which organizational forms give their owners limited liability?

See Answer

Q: At the end of 2015, Apple had cash and short-

At the end of 2015, Apple had cash and short-term investments of $41.60 billion, accounts receivable of $35.89 billion, current assets of $89.38 billion, and current liabilities of $80.61 billion. a....

See Answer

Q: Suppose two stocks have a correlation of 1. If the first

Suppose two stocks have a correlation of 1. If the first stock has an above average return this year, what is the probability that the second stock will have an above average return?

See Answer

Q: Suppose Pepsico’s stock has a beta of 0.57. If

Suppose Pepsico’s stock has a beta of 0.57. If the risk-free rate is 3% and the expected return of the market portfolio is 8%, what is Pepsico’s equity cost of capital?

See Answer

Q: You need to estimate the equity cost of capital for XYZ Corp

You need to estimate the equity cost of capital for XYZ Corp. You have the following data available regarding past returns: a. What was XYZ’s average historical return? b. Compute...

See Answer

Q: Go to Chapter Resources on and use the data in the spreadsheet

Go to Chapter Resources on and use the data in the spreadsheet provided to estimate the beta of Nike and Dell stock based on their monthly returns from 2011–2015.

See Answer

Q: Using the same data as in Problem 11, estimate the alpha

Using the same data as in Problem 11, estimate the alpha of Nike and Dell stock, expressed as % per month. Data from Problem 11: Go to Chapter Resources on and use the data in the spreadsheet provid...

See Answer

Q: Using the same data as in Problem 11, estimate the 95

Using the same data as in Problem 11, estimate the 95% confidence interval for the alpha and beta of Nike and Dell stock using Excel’s regression tool (from the data analysis menu) or the linest() fun...

See Answer

Q: In mid-2012, Ralston Purina had AA-rated,

In mid-2012, Ralston Purina had AA-rated, 10-year bonds outstanding with a yield to maturity of 2.05%. a. What is the highest expected return these bonds could have? b. At the time, similar maturity T...

See Answer

Q: In mid-2009, Rite Aid had CCC-rated,

In mid-2009, Rite Aid had CCC-rated, 6-year bonds outstanding with a yield to maturity of 17.3%. At the time, similar maturity Treasuries had a yield of 3%. Suppose the market risk premium is 5% and y...

See Answer

Q: During the recession in mid-2009, homebuilder KB Home had

During the recession in mid-2009, homebuilder KB Home had outstanding 6-year bonds with a yield to maturity of 8.5% and a BB rating. If corresponding risk-free rates were 3%, and the market risk premi...

See Answer

Q: The Dunley Corp. plans to issue 5-year bonds.

The Dunley Corp. plans to issue 5-year bonds. It believes the bonds will have a BBB rating. Suppose AAA bonds with the same maturity have a 4% yield. Assume the market risk premium is 5% and use the d...

See Answer

Q: See Table 2.5 showing financial statement data and stock price

See Table 2.5 showing financial statement data and stock price data for Mydeco Corp. a. How did Mydeco’s accounts receivable days change over this period? b. How did Mydecoâ...

See Answer

Q: Your firm is planning to invest in an automated packaging plant.

Your firm is planning to invest in an automated packaging plant. Harburtin Industries is an all-equity firm that specializes in this business. Suppose Harburtin’s equity beta is 0.85, the risk free ra...

See Answer

Q: Consider the setting of Problem 18. You decided to look for

Consider the setting of Problem 18. You decided to look for other comparables to reduce estimation error in your cost of capital estimate. You find a second firm, Thurbinar Design, which is also engag...

See Answer

Q: Suppose the market portfolio has an expected return of 10% and

Suppose the market portfolio has an expected return of 10% and a volatility of 20%, while Microsoft’s stock has a volatility of 30%. a. Given its higher volatility, should we expect Microsoft to have...

See Answer

Q: IDX Tech is looking to expand its investment in advanced security systems

IDX Tech is looking to expand its investment in advanced security systems. The project will be financed with equity. You are trying to assess the value of the investment, and must estimate its cost of...

See Answer

Q: In mid-2015, Cisco Systems had a market capitalization of

In mid-2015, Cisco Systems had a market capitalization of $130 billion. It had A-rated debt of $25 billion as well as cash and short-term investments of $60 billion, and its estimated equity beta at t...

See Answer

Q: Consider the following airline industry data from mid-2009:

Consider the following airline industry data from mid-2009: a. Use the estimates in Table 12.3 to estimate the debt beta for each firm (use an average if multiple ratings are listed). b. Estimate th...

See Answer

Q: Weston Enterprises is an all-equity firm with two divisions.

Weston Enterprises is an all-equity firm with two divisions. The soft drink division has an asset beta of 0.60, expects to generate free cash flow of $50 million this year, and anticipates a 3% perpet...

See Answer

Q: Harrison Holdings, Inc. (HHI) is publicly traded,

Harrison Holdings, Inc. (HHI) is publicly traded, with a current share price of $32 per share. HHI has 20 million shares outstanding, as well as $64 million in debt. The founder of HHI, Harry Harrison...

See Answer

Q: Your company operates a steel plant. On average, revenues from

Your company operates a steel plant. On average, revenues from the plant are $30 million per year. All of the plants costs are variable costs and are consistently 80% of revenues, including energy cos...

See Answer

Q: Unida Systems has 40 million shares outstanding trading for $10 per

Unida Systems has 40 million shares outstanding trading for $10 per share. In addition, Unida has $100 million in outstanding debt. Suppose Unida’s equity cost of capital is 15%, its debt cost of capi...

See Answer

Q: See Table 2.5 showing financial statement data and stock price

See Table 2.5 showing financial statement data and stock price data for Mydeco Corp. a. Compare Mydeco’s accounts payable days in 2012 and 2016. b. Did this change in accounts payabl...

See Answer

Q: You would like to estimate the weighted average cost of capital for

You would like to estimate the weighted average cost of capital for a new airline business. Based on its industry asset beta, you have already estimated an unlevered cost of capital for the firm of 9%...

See Answer

Q: Aluminum maker Alcoa has a beta of about 2.0,

Aluminum maker Alcoa has a beta of about 2.0, whereas Hormel Foods has a beta of 0.45. If the expected excess return of the marker portfolio is 5%, which of these firms has a higher equity cost of cap...

See Answer

Q: Suppose Best Buy stock is trading for $30 per share for

Suppose Best Buy stock is trading for $30 per share for a total market cap of $9 billion, and Walt Disney has 1.65 billion shares outstanding. If you hold the market portfolio, and as part of it hold...

See Answer

Q: Standard and Poor’s also publishes the S&P Equal Weight Index

Standard and Poor’s also publishes the S&P Equal Weight Index, which is an equally weighted version of the S&P 500. a. To maintain a portfolio that tracks this index, what trades would need to be made...

See Answer

Q: Suppose that in place of the S&P 500, you

Suppose that in place of the S&P 500, you wanted to use a broader market portfolio of all U.S. stocks and bonds as the market proxy. Could you use the same estimate for the market risk premium when ap...

See Answer

Q: From the start of 1999 to the start of 2009, the

From the start of 1999 to the start of 2009, the S&P 500 had a negative return. Does this mean the market risk premium we should have used in the CAPM was negative?

See Answer

Q: Assume that all investors have the same information and care only about

Assume that all investors have the same information and care only about expected return and volatility. If new information arrives about one stock, can this information affect the price and return of...

See Answer

Q: To put the turnover of Figure 13.3 into perspective,

To put the turnover of Figure 13.3 into perspective, let’s do a back of the envelope calculation of what an investor’s average turnover per stock would be were he t...

See Answer

Q: How does the disposition effect impact investors’ tax obligations?

How does the disposition effect impact investors’ tax obligations?

See Answer

Q: Consider the price paths of the following two stocks over six time

Consider the price paths of the following two stocks over six time periods: Neither stock pays dividends. Assume you are an investor with the disposition effect and you bought at time 1 and right no...

See Answer

Q: See Table 2.5 showing financial statement data and stock price

See Table 2.5 showing financial statement data and stock price data for Mydeco Corp. a. By how much did Mydeco increase its debt from 2012 to 2016? b. What was Mydeco’s EBITDA/Interest coverage ratio...

See Answer

Q: Suppose that all investors have the disposition effect. A new stock

Suppose that all investors have the disposition effect. A new stock has just been issued at a price of $50, so all investors in this stock purchased the stock today. A year from now the stock will be...

See Answer

Q: Davita Spencer is a manager at Half Dome Asset Management. She

Davita Spencer is a manager at Half Dome Asset Management. She can generate an alpha of 2% a year up to $100 million. After that her skills are spread too thin, so cannot add value and her alpha is ze...

See Answer

Q: Allison and Bill are both mutual fund managers, although Allison is

Allison and Bill are both mutual fund managers, although Allison is more skilled than Bill. Both have $100 million in assets under management and charge a fee of 1%/year. Allison is able to generate a...

See Answer

Q: Assume the economy consisted of three types of people. 50%

Assume the economy consisted of three types of people. 50% are fad followers, 45% are passive investors (they have read this book and so hold the market portfolio), and 5% are informed traders. The po...

See Answer

Q: Explain what the size effect is.

Explain what the size effect is.

See Answer

Q: Assume all firms have the same expected dividends. If they have

Assume all firms have the same expected dividends. If they have different expected returns, how will their market values and expected returns be related? What about the relation between their dividend...

See Answer

Q: Each of the six firms in the table below is expected to

Each of the six firms in the table below is expected to pay the listed dividend payment every year in perpetuity. a. Using the cost of capital in the table, calculate the market value of each firm....

See Answer

Q: Assume that the CAPM is a good description of stock price returns

Assume that the CAPM is a good description of stock price returns. The market expected return is 7% with 10% volatility and the risk-free rate is 3%. New news arrives that does not change any of these...

See Answer

Q: Consider the following stocks, all of which will pay a liquidating

Consider the following stocks, all of which will pay a liquidating dividend in a year and nothing in the interim: a. Calculate the expected return of each stock. b. What is the sign of correlation b...

See Answer

Q: In Problem 20, assume the risk-free rate is 3

In Problem 20, assume the risk-free rate is 3% and the market risk premium is 7%. a. What does the CAPM predict the expected return for each stock should be? b. Clearly, the CAPM predictions are not e...

See Answer

Q: See Table 2.5 showing financial statement data and stock price

See Table 2.5 showing financial statement data and stock price data for Mydeco Corp. a. How did Mydeco’s book debt-equity ratio change from 2012 to 2016? b. How did Mydecoâ ...

See Answer

Q: Explain how to construct a positive-alpha trading strategy if stocks

Explain how to construct a positive-alpha trading strategy if stocks that have had relatively high returns in the past tend to have positive alphas and stocks that have had relatively low returns in t...

See Answer

Q: If you can use past returns to construct a trading strategy that

If you can use past returns to construct a trading strategy that makes money (has a positive alpha), it is evidence that market portfolio is not efficient. Explain why.

See Answer

Q: Explain why you might expect stocks to have nonzero alphas if the

Explain why you might expect stocks to have nonzero alphas if the market proxy portfolio is not highly correlated with the true market portfolio, even if the true market portfolio is efficient.

See Answer

Q: Explain why if some investors are subject to systematic behavioral biases,

Explain why if some investors are subject to systematic behavioral biases, while others pick efficient portfolios, the market portfolio will not be efficient.

See Answer

Q: Explain why an employee who cares only about expected return and volatility

Explain why an employee who cares only about expected return and volatility will likely underweight the amount of money he invests in his own company’s stock relative to an investor who does not work...

See Answer

Q: Using the factor beta estimates in the table shown here and the

Using the factor beta estimates in the table shown here and the monthly expected return estimates in Table 13.1, calculate the risk premium of General Electric stock (ticker: GE) using the FFC factor...

See Answer

Q: You are currently considering an investment in a project in the energy

You are currently considering an investment in a project in the energy sector. The investment has the same riskiness as Exxon Mobil stock (ticker: XOM). Using the data in Table 13.1 and the table abov...

See Answer

Q: You work for Microsoft Corporation (ticker: MSFT), and you

You work for Microsoft Corporation (ticker: MSFT), and you are considering whether to develop a new software product. The risk of the investment is the same as the risk of the company. a. Using the da...

See Answer

Q: Suppose the CAPM equilibrium holds perfectly. Then the risk-free

Suppose the CAPM equilibrium holds perfectly. Then the risk-free interest rate increases, and nothing else changes. a. Is the market portfolio still efficient? b. If your answer to part a is yes, expl...

See Answer

Q: You know that there are informed traders in the stock market,

You know that there are informed traders in the stock market, but you are uninformed. Describe an investment strategy that guarantees you will not lose money to the informed traders and explain why it...

See Answer

Q: Use the data in Problem 8 to determine the change, from

Use the data in Problem 8 to determine the change, from 2012 to 2015, in GE’s a. book debt-equity ratio. b. market debt-equity ratio. Data from Problem 8: In early 2012, General Electric (GE) had a...

See Answer

Q: What are the only conditions under which the market portfolio might not

What are the only conditions under which the market portfolio might not be an efficient portfolio?

See Answer

Q: Explain what the following sentence means: The market portfolio is a

Explain what the following sentence means: The market portfolio is a fence that protects the sheep from the wolves, but nothing can protect the sheep from themselves.

See Answer

Q: You are trading in a market in which you know there are

You are trading in a market in which you know there are a few highly skilled traders who are better informed than you are. There are no transaction costs. Each day you randomly choose five stocks to b...

See Answer

Q: Why does the CAPM imply that investors should trade very rarely?

Why does the CAPM imply that investors should trade very rarely?

See Answer

Q: Your brother Joe is a surgeon who suffers badly from the overconfidence

Your brother Joe is a surgeon who suffers badly from the overconfidence bias. He loves to trade stocks and believes his predictions with 100% confidence. In fact, he is uninformed like most investors....

See Answer

Q: Explain what is wrong with the following argument: “If a

Explain what is wrong with the following argument: “If a firm issues debt that is risk free, because there is no possibility of default, the risk of the firm’s equity does not change. Therefore, risk-...

See Answer

Q: Consider the entrepreneur described in Section 14.1 (and referenced

Consider the entrepreneur described in Section 14.1 (and referenced in Tables 14.1–14.3). Suppose she funds the project by borrowing $750 rather than $500. a. According to MM Proposi...

See Answer

Q: Suppose Visa Inc. (V) has no debt and an

Suppose Visa Inc. (V) has no debt and an equity cost of capital of 9.2%. The average debt-to value ratio for the credit services industry is 13%. What would its cost of equity be if it took on the ave...

See Answer

Q: Global Pistons (GP) has common stock with a market value

Global Pistons (GP) has common stock with a market value of $200 million and debt with a value of $100 million. Investors expect a 15% return on the stock and a 6% return on the debt. Assume perfect c...

See Answer

Q: Hubbard Industries is an all-equity firm whose shares have an

Hubbard Industries is an all-equity firm whose shares have an expected return of 10%. Hubbard does a leveraged recapitalization, issuing debt and repurchasing stock, until its debt-equity ratio is 0.6...

See Answer

Q: You are analyzing the leverage of two firms and you note the

You are analyzing the leverage of two firms and you note the following (all values in millions of dollars): a. What is the market debt-to-equity ratio of each firm? b. What is the book debt-to-equit...

See Answer

Q: Hartford Mining has 50 million shares that are currently trading for $

Hartford Mining has 50 million shares that are currently trading for $4 per share and $200 million worth of debt. The debt is risk free and has an interest rate of 5%, and the expected return of Hartf...

See Answer

Q: Mercer Corp. has 10 million shares outstanding and $100 million

Mercer Corp. has 10 million shares outstanding and $100 million worth of debt outstanding. Its current share price is $75. Mercer’s equity cost of capital is 8.5%. Mercer has just announced that it wi...

See Answer

Q: In mid-2015 Qualcomm Inc. had $11 billion in

In mid-2015 Qualcomm Inc. had $11 billion in debt, total equity capitalization of $89 billion, and an equity beta of 1.43 (as reported on Yahoo! Finance). Included in Qualcomm’s assets was $21 billion...

See Answer

Q: Indell stock has a current market value of $120 million and

Indell stock has a current market value of $120 million and a beta of 1.50. Indell currently has risk-free debt as well. The firm decides to change its capital structure by issuing $30 million in addi...

See Answer

Q: Jim Campbell is founder and CEO of Open Start, an innovative

Jim Campbell is founder and CEO of Open Start, an innovative software company. The company is all equity financed, with 100 million shares outstanding. The shares are trading at a price of $1. Campbel...

See Answer

Q: Yerba Industries is an all-equity firm whose stock has a

Yerba Industries is an all-equity firm whose stock has a beta of 1.2 and an expected return of 12.5%. Suppose it issues new risk-free debt with a 5% yield and repurchases 40% of its stock. Assume perf...

See Answer

Q: You are CEO of a high-growth technology firm. You

You are CEO of a high-growth technology firm. You plan to raise $180 million to fund an expansion by issuing either new shares or new debt. With the expansion, you expect earnings next year of $24 mil...

See Answer

Q: Zelnor, Inc., is an all-equity firm with 100

Zelnor, Inc., is an all-equity firm with 100 million shares outstanding currently trading for $8.50 per share. Suppose Zelnor decides to grant a total of 10 million new shares to employees as part of...

See Answer

Q: Suppose Levered Bank is funded with 2% equity and 98%

Suppose Levered Bank is funded with 2% equity and 98% debt. Its current market capitalization is $10 billion, and its market to book ratio is 1. Levered Bank earns a 4.22% expected return on its asset...

See Answer

Q: Wolfrum Technology (WT) has no debt. Its assets will

Wolfrum Technology (WT) has no debt. Its assets will be worth $450 million in one year if the economy is strong, but only $200 million in one year if the economy is weak. Both events are equally likel...

See Answer

Q: See Table 2.5 showing financial statement data and stock price

See Table 2.5 showing financial statement data and stock price data for Mydeco Corp. a. Compute Mydeco’s PE ratio each year from 2012 to 2016. In which year was it the highest? b. Wh...

See Answer

Q: Suppose Alpha Industries and Omega Technology have identical assets that generate identical

Suppose Alpha Industries and Omega Technology have identical assets that generate identical cash flows. Alpha Industries is an all-equity firm, with 10 million shares outstanding that trade for a pric...

See Answer

Q: Cisoft is a highly profitable technology firm that currently has $5

Cisoft is a highly profitable technology firm that currently has $5 billion in cash. The firm has decided to use this cash to repurchase shares from investors, and it has already announced these plans...

See Answer

Q: Schwartz Industry is an industrial company with 100 million shares outstanding and

Schwartz Industry is an industrial company with 100 million shares outstanding and a market capitalization (equity value) of $4 billion. It has $2 billion of debt outstanding. Management have decided...

See Answer

Q: Zetatron is an all-equity firm with 100 million shares outstanding

Zetatron is an all-equity firm with 100 million shares outstanding, which are currently trading for $7.50 per share. A month ago, Zetatron announced it will change its capital structure by borrowing $...

See Answer

Q: Pelamed Pharmaceuticals has EBIT of $325 million in 2006. In

Pelamed Pharmaceuticals has EBIT of $325 million in 2006. In addition, Pelamed has interest expenses of $125 million and a corporate tax rate of 40%. a. What is Pelamed’s 2006 net income? b. What is t...

See Answer

Q: Rogot Instruments makes fine violins and cellos. It has $1

Rogot Instruments makes fine violins and cellos. It has $1 million in debt outstanding, equity valued at $2 million, and pays corporate income tax at rate of 35%. Its cost of equity is 12% and its cos...

See Answer

Q: Restex maintains a debt-equity ratio of 0.85,

Restex maintains a debt-equity ratio of 0.85, and has an equity cost of capital of 12% and a debt cost of capital of 7%. Restex’s corporate tax rate is 40%, and its market capitalization is $220 milli...

See Answer

Q: Acme Storage has a market capitalization of $100 million and debt

Acme Storage has a market capitalization of $100 million and debt outstanding of $40 million. Acme plans to maintain this same debt-equity ratio in the future. The firm pays an interest rate of 7.5% o...

See Answer

Q: Milton Industries expects free cash flow of $5 million each year

Milton Industries expects free cash flow of $5 million each year. Milton’s corporate tax rate is 35%, and its unlevered cost of capital is 15%. The firm also has outstanding debt of $19.05 million, an...

See Answer

Q: Suppose Microsoft has 8.75 billion shares outstanding and pays a

Suppose Microsoft has 8.75 billion shares outstanding and pays a marginal corporate tax rate of 35%. If Microsoft announces that it will pay out $50 billion in cash to investors through a combination...

See Answer

Q: In early-2015, United Airlines (UAL) had a

In early-2015, United Airlines (UAL) had a market capitalization of $24.8 billion, debt of $12.8 billion, and cash of $5.5 billion. United also had annual revenues of $38.9 billion. Southwest Airlines...

See Answer

Q: Rally, Inc., is an all-equity firm with assets

Rally, Inc., is an all-equity firm with assets worth $25 billion and 10 billion shares outstanding. Rally plans to borrow $10 billion and use these funds to repurchase shares. The firm’s corporate tax...

See Answer

Q: Suppose the corporate tax rate is 40%, and investors pay a

Suppose the corporate tax rate is 40%, and investors pay a tax rate of 15% on income from dividends or capital gains and a tax rate of 33.3% on interest income. Your firm decides to add debt so it wil...

See Answer

Q: Facebook, Inc. had no debt on its balance sheet in

Facebook, Inc. had no debt on its balance sheet in 2014, but paid $2 billion in taxes. Suppose Facebook were to issue sufficient debt to reduce its taxes by $250 million per year permanently. Assume F...

See Answer

Q: Markum Enterprises is considering permanently adding $100 million of debt to

Markum Enterprises is considering permanently adding $100 million of debt to its capital structure. Markum’s corporate tax rate is 35%. a. Absent personal taxes, what is the value of the interest tax...

See Answer

Q: Garnet Corporation is considering issuing risk-free debt or risk-

Garnet Corporation is considering issuing risk-free debt or risk-free preferred stock. The tax rate on interest income is 35%, and the tax rate on dividends or capital gains from preferred stock is 15...

See Answer

Q: Suppose the tax rate on interest income is 35%, and the

Suppose the tax rate on interest income is 35%, and the average tax rate on capital gains and dividend income is 10%. How high must the marginal corporate tax rate be for debt to offer a tax advantage...

See Answer

Q: With its current leverage, Impi Corporation will have net income next

With its current leverage, Impi Corporation will have net income next year of $4.5 million. If Impi’s corporate tax rate is 35% and it pays 8% interest on its debt, how much additional debt can Impi i...

See Answer

Q: Colt Systems will have EBIT this coming year of $15 million

Colt Systems will have EBIT this coming year of $15 million. It will also spend $6 million on total capital expenditures and increases in net working capital, and have $3 million in depreciation expen...

See Answer

Q: PMF, Inc., is equally likely to have EBIT this coming

PMF, Inc., is equally likely to have EBIT this coming year of $10 million, $15 million, or $20 million. Its corporate tax rate is 35%, and investors pay a 15% tax rate on income from equity and a 35%...

See Answer

Q: Braxton Enterprises currently has debt outstanding of $35 million and an

Braxton Enterprises currently has debt outstanding of $35 million and an interest rate of 8%. Braxton plans to reduce its debt by repaying $7 million in principal at the end of each year for the next...

See Answer

Q: See Table 2.5 showing financial statement data and stock price

See Table 2.5 showing financial statement data and stock price data for Mydeco Corp. a. Compute Mydeco’s ROE each year from 2012 to 2016. b. Compute Mydeco’s ROA ea...

See Answer

Q: Arnell Industries has just issued $10 million in debt (at

Arnell Industries has just issued $10 million in debt (at par). The firm will pay interest only on this debt. Arnell’s marginal tax rate is expected to be 35% for the foreseeable future. a. Suppose Ar...

See Answer

Q: Ten years have passed since Arnell issued $10 million in perpetual

Ten years have passed since Arnell issued $10 million in perpetual interest only debt with a 6% annual coupon, as in Problem 6. Tax rates have remained the same at 35% but interest rates have dropped,...

See Answer

Q: Bay Transport Systems (BTS) currently has $30 million in

Bay Transport Systems (BTS) currently has $30 million in debt outstanding. In addition to 6.5% interest, it plans to repay 5% of the remaining balance each year. If BTS has a marginal corporate tax ra...

See Answer

Q: Safeco Inc. has no debt, and maintains a policy of

Safeco Inc. has no debt, and maintains a policy of holding $10 million in excess cash reserves, invested in risk-free Treasury securities. If Safeco pays a corporate tax rate of 35%, what is the cost...

See Answer

Q: Gladstone Corporation is about to launch a new product. Depending on

Gladstone Corporation is about to launch a new product. Depending on the success of the new product, Gladstone may have one of four values next year: $150 million, $135 million, $95 million, or $80 mi...

See Answer

Q: You work for a large car manufacturer that is currently financially healthy

You work for a large car manufacturer that is currently financially healthy. Your manager feels that the firm should take on more debt because it can thereby reduce the expense of car warranties. To q...

See Answer

Q: Facebook, Inc. has no debt. As Problem 21 in

Facebook, Inc. has no debt. As Problem 21 in Chapter 15 makes clear, by issuing debt Facebook can generate a very large tax shield potentially worth nearly $2 billion. Given Facebook’s success, one wo...

See Answer

Q: Your firm is considering issuing one-year debt, and has

Your firm is considering issuing one-year debt, and has come up with the following estimates of the value of the interest tax shield and the probability of distress for different levels of debt: Sup...

See Answer

Q: Real estate purchases are often financed with at least 80% debt

Real estate purchases are often financed with at least 80% debt. Most corporations, however, have less than 50% debt financing. Provide an explanation for this difference using the trade-off theory.

See Answer

Q: On May 14, 2008, General Motors paid a dividend of

On May 14, 2008, General Motors paid a dividend of $0.25 per share. During the same quarter GM lost a staggering $15.5 billion or $27.33 per share. Seven months later the company asked for billions of...

See Answer

Q: What are the main advantages and disadvantages of organizing a firm as

What are the main advantages and disadvantages of organizing a firm as a corporation?

See Answer

Q: See Table 2.5 showing financial statement data and stock price

See Table 2.5 showing financial statement data and stock price data for Mydeco Corp. Was Mydeco able to improve its ROIC in 2016 relative to what it was in 2012? Table 2.5:

See Answer

Q: Dynron Corporation’s primary business is natural gas transportation using its vast gas

Dynron Corporation’s primary business is natural gas transportation using its vast gas pipeline network. Dynron’s assets currently have a market value of $150 million. The firm is exploring the possib...

See Answer

Q: Sarvon Systems has a debt-equity ratio of 1.2

Sarvon Systems has a debt-equity ratio of 1.2, an equity beta of 2.0, and a debt beta of 0.30. It currently is evaluating the following projects, none of which would change the firm’...

See Answer

Q: Baruk Industries has no cash and a debt obligation of $36

Baruk Industries has no cash and a debt obligation of $36 million that is now due. The market value of Baruk’s assets is $81 million, and the firm has no other liabilities. Assume perfect capital mark...

See Answer

Q: Petron Corporation’s management team is meeting to decide on a new corporate

Petron Corporation’s management team is meeting to decide on a new corporate strategy. There are four options, each with a different probability of success and total firm value in th...

See Answer

Q: Consider the setting of Problem 21, and suppose Petron Corp.

Consider the setting of Problem 21, and suppose Petron Corp. has debt with a face value of $40 million outstanding. For simplicity assume all risk is idiosyncratic, the risk-free interest rate is zero...

See Answer

Q: Consider the setting of Problems 21 and 22, and suppose Petron

Consider the setting of Problems 21 and 22, and suppose Petron Corp. must pay a 25% tax rate on the amount of the final payoff that is paid to equity holders. It pays no tax on payments to, or capital...

See Answer

Q: You own your own firm, and you want to raise $

You own your own firm, and you want to raise $30 million to fund an expansion. Currently, you own 100% of the firm’s equity, and the firm has no debt. To raise the $30 million solely through equity, y...

See Answer

Q: Ralston Enterprises has assets that will have a market value in one

Ralston Enterprises has assets that will have a market value in one year as follows: That is, there is a 1% chance the assets will be worth $70 million, a 6% chance the assets will be worth $80 mill...

See Answer

Q: Although the major benefit of debt financing is easy to observe—

Although the major benefit of debt financing is easy to observe—the tax shield—many of the indirect costs of debt financing can be quite subtle and difficult to observe. Describe some of these costs....

See Answer

Q: If it is managed efficiently, Remel Inc. will have assets

If it is managed efficiently, Remel Inc. will have assets with a market value of $50 million, $100 million, or $150 million next year, with each outcome being equally likely. However, managers may eng...

See Answer

Q: For fiscal year 2015, Costco Wholesale Corporation (COST) had

For fiscal year 2015, Costco Wholesale Corporation (COST) had a net profit margin of 2.05%, asset turnover of 3.48, and a book equity multiplier of 3.15. a. Use this data to compute Costco’s ROE using...

See Answer

Q: Which of the following industries have low optimal debt levels according to

Which of the following industries have low optimal debt levels according to the trade-off theory? Which have high optimal levels of debt? a. Tobacco firms b. Accounting firms c. Mature restaurant chai...

See Answer

Q: When a firm defaults on its debt, debt holders often receive

When a firm defaults on its debt, debt holders often receive less than 50% of the amount they are owed. Is the difference between the amount debt holders are owed and the amount they receive a cost of...

See Answer

Q: According to the managerial entrenchment theory, managers choose capital structure so

According to the managerial entrenchment theory, managers choose capital structure so as to preserve their control of the firm. On the one hand, debt is costly for managers because they risk losing co...

See Answer

Q: During the Internet boom of the late 1990s, the stock prices

During the Internet boom of the late 1990s, the stock prices of many Internet firms soared to extreme heights. As CEO of such a firm, if you believed your stock was significantly overvalued, would usi...

See Answer

Q: We R Toys” (WRT) is considering expanding into new

We R Toys” (WRT) is considering expanding into new geographic markets. The expansion will have the same business risk as WRT’s existing assets. The expansion will require an initial investment of $50...

See Answer

Q: Which type of firm is more likely to experience a loss of

Which type of firm is more likely to experience a loss of customers in the event of financial distress: a. Campbell Soup Company or Intuit, Inc. (a maker of accounting software)? b. Allstate Corporati...

See Answer

Q: Which type of asset is more likely to be liquidated for close

Which type of asset is more likely to be liquidated for close to its full market value in the event of financial distress: a. An office building or a brand name? b. Product inventory or raw materials?...

See Answer

Q: Suppose Tefco Corp. has a value of $100 million if

Suppose Tefco Corp. has a value of $100 million if it continues to operate, but has outstanding debt of $120 million that is now due. If the firm declares bankruptcy, bankruptcy costs will equal $20 m...

See Answer

Q: You have received two job offers. Firm A offers to pay

You have received two job offers. Firm A offers to pay you $85,000 per year for two years. Firm B offers to pay you $90,000 for two years. Both jobs are equivalent. Suppose that firm A’s contract is c...

See Answer

Q: As in Problem 1, Gladstone Corporation is about to launch a

As in Problem 1, Gladstone Corporation is about to launch a new product. Depending on the success of the new product, Gladstone may have one of four values next year: $150 million, $135 million, $95 m...

See Answer

Q: For fiscal year 2015, Wal-Mart Stores, Inc.

For fiscal year 2015, Wal-Mart Stores, Inc. (WMT) had total revenues of $485.65 billion, net income of $16.36 billion, total assets of $203.49 billion, and total shareholder’s equity of $81.39 billion...

See Answer

Q: What options does a firm have to spend its free cash flow

What options does a firm have to spend its free cash flow (after it has satisfied all interest obligations)?

See Answer

Q: Using Table 17.2, for each of the following years

Using Table 17.2, for each of the following years, state whether dividends were tax disadvantaged or not for individual investors with a one-year investment horizon: a. 1985 b. 1989 c. 1995 d. 1999 e....

See Answer

Q: What was the effective dividend tax rate for a U.S

What was the effective dividend tax rate for a U.S. investor in the highest tax bracket who planned to hold a stock for one year in 1981? How did the effective dividend tax rate change in 1982 when th...

See Answer

Q: The dividend tax cut passed in 2003 lowered the effective dividend tax

The dividend tax cut passed in 2003 lowered the effective dividend tax rate for a U.S. investor in the highest tax bracket to a historic low. During which other periods in the last 35 years was the ef...

See Answer

Q: Suppose that all capital gains are taxed at a 25% rate

Suppose that all capital gains are taxed at a 25% rate, and that the dividend tax rate is 50%. Arbuckle Corp. is currently trading for $30, and is about to pay a $6 special dividend. a. Absent any oth...

See Answer

Q: You purchased CSH stock for $40 one year ago and it

You purchased CSH stock for $40 one year ago and it is now selling for $50. The company has announced that it plans a $10 special dividend. You are considering whether to sell the stock now, or wait t...

See Answer

Q: On Monday, November 15, 2004, TheStreet.com reported

On Monday, November 15, 2004, TheStreet.com reported: “An experiment in the efficiency of financial markets will play out Monday following the expiration of a $3.08 dividend privilege for holders of M...

See Answer

Q: At current tax rates, which of the following investors are most

At current tax rates, which of the following investors are most likely to hold a stock that has a high dividend yield: a. individual investors, b. pension funds, c. mutual funds, d. corporations

See Answer

Q: Que Corporation pays a regular dividend of $1 per share.

Que Corporation pays a regular dividend of $1 per share. Typically, the stock price drops by $0.80 per share when the stock goes ex-dividend. Suppose the capital gains tax rate is 20%, but investors p...

See Answer

Q: ABC Corporation announced that it will pay a dividend to all shareholders

ABC Corporation announced that it will pay a dividend to all shareholders of record as of Monday, April 2, 2012. It takes three business days of a purchase for the new owners of a share of stock to be...

See Answer

Q: Consider a retailing firm with a net profit margin of 3.

Consider a retailing firm with a net profit margin of 3.5%, a total asset turnover of 1.8, total assets of $44 million, and a book value of equity of $18 million. a. What is the firm’s current ROE? b....

See Answer

Q: A stock that you know is held by long-term individual

A stock that you know is held by long-term individual investors paid a large one-time dividend. You notice that the price drop on the ex-dividend date is about the size of the dividend payment. You fi...

See Answer

Q: Clovix Corporation has $50 million in cash, 10 million shares

Clovix Corporation has $50 million in cash, 10 million shares outstanding, and a current share price of $30. Clovix is deciding whether to use the $50 million to pay an immediate special dividend of $...

See Answer

Q: Assume capital markets are perfect. Kay Industries currently has $100

Assume capital markets are perfect. Kay Industries currently has $100 million invested in short term Treasury securities paying 7%, and it pays out the interest payments on these securities each year...

See Answer

Q: Redo Problem 22, but assume that Kay must pay a corporate

Redo Problem 22, but assume that Kay must pay a corporate tax rate of 35%, and investors pay no taxes. Data from Problem 22: Assume capital markets are perfect. Kay Industries currently has $100 mil...

See Answer

Q: Harris Corporation has $250 million in cash, and 100 million

Harris Corporation has $250 million in cash, and 100 million shares outstanding. Suppose the corporate tax rate is 35%, and investors pay no taxes on dividends, capital gains, or interest income. Inve...

See Answer

Q: Redo Problem 22, but assume the following: a.

Redo Problem 22, but assume the following: a. Investors pay a 15% tax on dividends but no capital gains taxes or taxes on interest income, and Kay does not pay corporate taxes. b. Investors pay a 15%...

See Answer

Q: Raviv Industries has $100 million in cash that it can use

Raviv Industries has $100 million in cash that it can use for a share repurchase. Suppose instead Raviv invests the funds in an account paying 10% interest for one year. a. If the corporate tax rate i...

See Answer

Q: Use the data in Table 15.3 to calculate the tax

Use the data in Table 15.3 to calculate the tax disadvantage of retained cash in the following: a. 1998 b. 1976 Table 15.3:

See Answer

Q: Explain under which conditions an increase in the dividend payment can be

Explain under which conditions an increase in the dividend payment can be interpreted as a signal of the following: a. Good news b. Bad news

See Answer

Q: Why is an announcement of a share repurchase considered a positive signal

Why is an announcement of a share repurchase considered a positive signal?

See Answer

Q: Find online the annual 10-K report for Costco Wholesale Corporation

Find online the annual 10-K report for Costco Wholesale Corporation (COST) for fiscal year 2015 (filed in October 2015). a. Which auditing firm certified these financial statements? b. Which officers...

See Answer

Q: Describe the different mechanisms available to a firm to use to repurchase

Describe the different mechanisms available to a firm to use to repurchase shares.

See Answer

Q: AMC Corporation currently has an enterprise value of $400 million and

AMC Corporation currently has an enterprise value of $400 million and $100 million in excess cash. The firm has 10 million shares outstanding and no debt. Suppose AMC uses its excess cash to repurchas...

See Answer

Q: Explain why most companies choose to pay stock dividends (split their

Explain why most companies choose to pay stock dividends (split their stock).

See Answer

Q: When might it be advantageous to undertake a reverse stock split?

When might it be advantageous to undertake a reverse stock split?

See Answer

Q: RFC Corp. has announced a $1 dividend. If RFC’s

RFC Corp. has announced a $1 dividend. If RFC’s price last price cum-dividend is $50, what should its first ex-dividend price be (assuming perfect capital markets)?

See Answer

Q: KMS Corporation has assets with a market value of $500 million

KMS Corporation has assets with a market value of $500 million, $50 million of which are cash. It has debt of $200 million, and 10 million shares outstanding. Assume perfect capital markets. a. What i...

See Answer

Q: Suppose you work for Oracle Corporation, and part of your compensation

Suppose you work for Oracle Corporation, and part of your compensation takes the form of stock options. The value of the stock option is equal to the difference between Oracle’s stock price and an exe...

See Answer

Q: Explain whether each of the following projects is likely to have risk

Explain whether each of the following projects is likely to have risk similar to the average risk of the firm. a. The Clorox Company considers launching a new version of Armor All designed to clean an...

See Answer

Q: Consider Alcatel-Lucent’s project in Problem 6. a.

Consider Alcatel-Lucent’s project in Problem 6. a. What is Alcatel-Lucent’s unlevered cost of capital? b. What is the unlevered value of the project? c. What are th...

See Answer

Q: Consider Alcatel-Lucent’s project in Problem 6. a.

Consider Alcatel-Lucent’s project in Problem 6. a. What is the free cash flow to equity for this project? b. What is its NPV computed using the FTE method? How does it compare with t...

See Answer

Q: WorldCom reclassified $3.85 billion of operating expenses as capital

WorldCom reclassified $3.85 billion of operating expenses as capital expenditures. Explain the effect this reclassification would have on WorldCom’s cash flows. (Hint: Consider taxes.) WorldCom’s acti...

See Answer

Q: In year 1, AMC will earn $2000 before interest and

In year 1, AMC will earn $2000 before interest and taxes. The market expects these earnings to grow at a rate of 3% per year. The firm will make no net investments (i.e., capital expenditures will equ...

See Answer

Q: Prokter and Gramble (PKGR) has historically maintained a debt-

Prokter and Gramble (PKGR) has historically maintained a debt-equity ratio of approximately 0.20. Its current stock price is $50 per share, with 2.5 billion shares outstanding. The firm enjoys very st...

See Answer

Q: Amarindo, Inc. (AMR), is a newly public firm

Amarindo, Inc. (AMR), is a newly public firm with 10 million shares outstanding. You are doing a valuation analysis of AMR. You estimate its free cash flow in the coming year to be $15 million, and yo...

See Answer

Q: Remex (RMX) currently has no debt in its capital structure

Remex (RMX) currently has no debt in its capital structure. The beta of its equity is 1.50. For each year into the indefinite future, Remex’s free cash flow is expected to equal $25...

See Answer

Q: You are evaluating a project that requires an investment of $90

You are evaluating a project that requires an investment of $90 today and provides a single cash flow of $115 for sure one year from now. You decide to use 100% debt financing, that is, you will borro...

See Answer

Q: Tybo Corporation adjusts its debt so that its interest expenses are 20

Tybo Corporation adjusts its debt so that its interest expenses are 20% of its free cash flow. Tybo is considering an expansion that will generate free cash flows of $2.5 million this year and is expe...

See Answer

Q: You are on your way to an important budget meeting. In

You are on your way to an important budget meeting. In the elevator, you review the project valuation analysis you had your summer associate prepare for one of the projects to be discussed: Looking...

See Answer

Q: Your firm is considering building a $600 million plant to manufacture

Your firm is considering building a $600 million plant to manufacture HDTV circuitry. You expect operating profits (EBITDA) of $145 million per year for the next 10 years. The plant will be depreciate...

See Answer

Q: Suppose Caterpillar, Inc., has 665 million shares outstanding with a

Suppose Caterpillar, Inc., has 665 million shares outstanding with a share price of $74.77, and $25 billion in debt. If in three years, Caterpillar has 700 million shares outstanding trading for $83 p...

See Answer

Q: Parnassus Corporation plans to invest $150 million in a new generator

Parnassus Corporation plans to invest $150 million in a new generator that will produce free cash flows of $20 million per year in perpetuity. The firm is all equity financed, with an equity cost of c...

See Answer

Q: See Table 2.5 showing financial statement data and stock price

See Table 2.5 showing financial statement data and stock price data for Mydeco Corp. a. What is Mydeco’s market capitalization at the end of each year? b. What is Mydecoâ€...

See Answer

Q: DFS Corporation is currently an all-equity firm, with assets

DFS Corporation is currently an all-equity firm, with assets with a market value of $100 million and 4 million shares outstanding. DFS is considering a leveraged recapitalization to boost its share pr...

See Answer